^2?\r\n\r\n\r\n: /", "Solution_3": "You can express $ p$ as a linear combination of $ p^2$ and $ d$. In the other direction, it's easy to show that the squares are divisible by $ p$.", "Solution_4": "oh ok. I did that.\r\n\r\nI guess I need to show that
is a prime.\r\n\r\nif O/
is a domain then the ideal is prime. this is the part I'm having trouble with.",
"Solution_5": "There's no need to bring domains into this; just use the definition of a prime ideal directly. Multiply two elements and ask under what conditions their product can lie in this ideal.",
"Solution_6": "It may be useful to consider the norm $ N(a\\plus{}b\\sqrt{d})\\equal{}a^2\\minus{}b^2d$.",
"Solution_7": "Ok. I think i got it.\r\n\r\nSo, if we have two elemens, r and s.\r\n\r\nr= a+b* (sqrt(d)) and s=(c+e*sqrt(d))\r\n\r\nthen rs= (ac+bed) + (ae+bc) (sqrt(d))\r\n\r\nand for this to be in the ideal generated by p and sqrt(d), then p|(ac+bed) and p| (ae+bc)\r\n\r\nsince p|d, the p|a or p|c. \r\n\r\nIf p|a then on the left side p|b so that r is factor. And it works the same way for the other side.\r\n\r\ncorrect?",
"Solution_8": "I don't know what \"r is factor\" means. Do you mean to say $ r\\in \\langle p, \\sqrt{d}\\rangle$? These distinctions are super-duper important.",
"Solution_9": "sry. yes that's what i mean."
}
{
"Tag": [
"ratio",
"algebra",
"polynomial",
"induction",
"blogs",
"vector",
"function"
],
"Problem": "Prove Binet's Formula. (i.e. Find the formula for the Fibonacci Numbers)",
"Solution_1": "What do you mean the formula for the Fibonacci numbers? The golden ratio between them?",
"Solution_2": "Prove that $ F_n \\equal{} \\frac1{\\sqrt5}\\left(\\left(\\frac {1 \\plus{} \\sqrt5}2\\right)^n \\minus{} \\left(\\frac {1 \\minus{} \\sqrt5}2\\right)^n\\right)$, for all integers $ n\\in\\mathbb{Z}^ \\plus{}$.",
"Solution_3": "Is this supposed to be related to the [url=http://www.artofproblemsolving.com/Wiki/index.php/Binet%27s_formula]WoW?[/url] [i]Edit[/i]: an explanation (by me, lol) similar to nateharman1234's is at that link \r\n\r\nSince $ F_{n \\plus{} 1} \\equal{} F_{n} \\plus{} F_{n \\minus{} 1}$, we have a linear recurrence with characteristic polynomial $ x^2 \\minus{} x \\minus{} 1 \\equal{} 0 \\equal{} \\left(x \\minus{} \\frac {1 \\plus{} \\sqrt {5}}{2}\\right)\\left(x \\minus{} \\frac {1 \\minus{} \\sqrt {5}}{2}\\right)$. We know that $ F_0 \\equal{} 0$, $ F_1 \\equal{} 1$, and by the characteristic we have $ F_n \\equal{} c_1 \\left(\\frac {1 \\plus{} \\sqrt {5}}{2}\\right)^n \\plus{} c_2 \\left(\\frac {1 \\minus{} \\sqrt {5}}{2}\\right)^n$. So now we have a system of two equations (if we let $ n \\equal{} 0,1$), $ 0 \\equal{} c_1 \\plus{} c_2 \\Longrightarrow c_2 \\equal{} \\minus{} c_1$ and $ 1 \\equal{} c_1\\left(\\frac {1 \\plus{} \\sqrt {5}}{2}\\right) \\plus{} c_2\\left(\\frac {1 \\minus{} \\sqrt {5}}{2}\\right)$ $ \\Longrightarrow 1 \\equal{} c_1\\left(\\frac {1 \\plus{} \\sqrt {5}}{2} \\minus{} \\frac {1 \\minus{} \\sqrt {5}}{2}\\right)$ $ \\Longrightarrow c_1 \\equal{} \\frac {1}{\\sqrt {5}}$.\r\n\r\nThus, $ F_n \\equal{} \\frac {1}{\\sqrt {5}}\\left(\\frac {1 \\plus{} \\sqrt {5}}{2}\\right)^n \\minus{} \\frac {1}{\\sqrt {5}}\\left(\\frac {1 \\minus{} \\sqrt {5}}{2}\\right)^n$.",
"Solution_4": "Many people do the induction proof of this at some point in their life, and while it may be a good induction exercise, it gives no insight into why this works. Heres a proof I know, and for another you can read one of t0rajir0u's recent blog posts.\r\n\r\n[hide]\n\nConsider the set S of sequences of the form An= A(n-1)+A(n-2), A0= x and A1 = y for real x,y (real works here, but for different sequences you may need complex)\n\nNote that if a sequence An is in S and a sequence Bn is in S, then (An+Bn) is in S.\n\nAlso note that this addition of sequences is associative and commutative and all that nice stuff as real additon is\n\nOh and of course if k is a real number and An is in S than so is the sequence k*An, and this multiplication distributes over the sequence addition I talked about before. and note that 1*An=An\n\nOk then note the sequence An=0 is in S and for all An in S -An is in S too.\n\nWell if you haven't realized already I am trying to establish that S forms a vector space, I think thats just about everything if I forgot an axiom I appologize.\n\nAnyway, now that we have a nice vector space it is of course nice to find a basis for it. Well i hope it is painfully obvious that S is two dimensional.\n\nOk now lets see if there are any nicely behaved sequences in S that have a closed form, perhaps any geometric sequences...\n\nSolve X^n = X^(n-1)+X^(n-2), x =/= 0...\nOoh... two linearly independent geometric sequences, that would make a lovely basis, now to just express the fibbonacci numbers in terms of them...\n\nI'll stop there, because I don't feel like doing any of the actual computations.\n[/hide]",
"Solution_5": "The proof nateharman is talking about can be found [url=http://www.artofproblemsolving.com/Forum/weblog_entry.php?t=175257]here[/url]. \r\n\r\n[hide=\"Another solution by generating functions\"] Let $ G(x) \\equal{} \\sum_{k\\equal{}0}^{\\infty} F_k x^k$, where $ F_0 \\equal{} 0, F_1 \\equal{} 1$. The Fibonacci relation implies (where $ \\phi, \\varphi$ are the positive and negative roots of $ x^2 \\equal{} x \\plus{} 1$): \n\n$ x G(x) \\plus{} x^2 G(x) \\equal{} G(x) \\minus{} x \\Leftrightarrow$\n$ G(x) \\equal{} \\frac{x}{1 \\minus{} x \\minus{} x^2} \\equal{} \\frac{x}{(1 \\minus{} \\phi x)(1 \\minus{} \\varphi x)} \\equal{} \\frac{1}{\\phi \\minus{} \\varphi} \\left( \\frac{1}{1 \\minus{} \\phi x} \\minus{} \\frac{1}{1 \\minus{} \\varphi x} \\right)$\n\n(The last step is partial fraction decomposition.) Using the expansion $ \\frac{1}{1 \\minus{} ax} \\equal{} 1 \\plus{} ax \\plus{} a^2 x^2 \\plus{} ...$, we conclude that\n\n$ \\sum_{k\\equal{}0}^{\\infty} F_k x^k \\equal{} \\frac{1}{\\phi \\minus{} \\varphi} \\left( \\sum_{k\\equal{}0}^{\\infty} (\\phi^k \\minus{} \\varphi^k) x^k \\right) \\Leftrightarrow$\n$ F_k \\equal{} \\frac{\\phi^k \\minus{} \\varphi^k}{\\phi \\minus{} \\varphi}$\n\nas desired (this is the nice form of Binet's formula). [/hide]",
"Solution_6": "Let $ \\tau \\equal{} \\frac {(1\\plus{}\\sqrt{5})}{2}$ and $ \\sigma \\equal{} \\frac {1\\minus{}\\sqrt{5}}{2}$\r\n\r\nSince $ \\tau$ and $ \\sigma$ are the roots of $ x^2\\equal{}x\\plus{}1$ then $ \\tau^2\\equal{}\\tau\\plus{}1$ and similarly with $ \\sigma$.\r\n\r\nNow multiplying both by their corresponding n-powers, $ \\tau^{n\\plus{}2} \\equal{} \\tau^{n\\plus{}1}\\plus{}\\tau^n$, and $ \\sigma^{n\\plus{}2} \\equal{} \\sigma^{n\\plus{}1}\\plus{}\\sigma^n$\r\n\r\nSubtracting the first from the latter yeilds $ \\tau^{n\\plus{}2}\\minus{}\\sigma^{n\\plus{}2}\\equal{} \\tau^{n\\plus{}1}\\plus{}\\tau^n\\minus{}\\sigma^{n\\plus{}1}\\minus{}\\sigma^n$\r\n\r\nso $ (\\tau^{n\\plus{}2}\\minus{}\\sigma^{n\\plus{}2})\\equal{} (\\tau^{n\\plus{}1}\\minus{}\\sigma^{n\\plus{}1})\\plus{}(\\tau^n\\minus{}\\sigma^n)$.\r\n\r\nSince $ \\tau\\minus{}\\sigma \\equal{} \\sqrt{5}$, and $ \\tau^2\\minus{}\\sigma^2\\equal{}\\sqrt{5}$ we must divide by $ \\tau\\minus{}\\sigma$\r\n\r\nso $ \\frac {\\tau^{n\\plus{}2}\\minus{}\\sigma^{n\\plus{}2}}{\\tau\\minus{}\\sigma}\\equal{} \\frac{\\tau^{n\\plus{}1}\\minus{}\\sigma^{n\\plus{}1}}{\\tau\\minus{}\\sigma}\\plus{}\\frac{\\tau^n\\minus{}\\sigma^n}{\\tau\\minus{}\\sigma}$. \r\n\r\nIf you would like it in your form with the denomiator being the radical, \r\n\r\nso $ \\frac {\\tau^{n\\plus{}2}\\minus{}\\sigma^{n\\plus{}2}}{\\sqrt{5}}\\equal{} \\frac{\\tau^{n\\plus{}1}\\minus{}\\sigma^{n\\plus{}1}}{\\sqrt{5}}\\plus{}\\frac{\\tau^n\\minus{}\\sigma^n}{\\sqrt{5}}$. \r\n\r\nLet $ u_n \\equal{} \\frac{\\tau^n\\minus{}\\sigma^n}{\\tau\\minus{}\\sigma}$, then the sequence is found as $ u_{n\\plus{}2}\\equal{}u_{n\\plus{}1}\\plus{}u{n}$\r\n\r\nand this completes the proof."
}
{
"Tag": [],
"Problem": "when $ \\text{1,3,5,5\\minus{}tetramethyl cyclohexa\\minus{}1,3\\minus{}diene}$ is disssolved in cold conc. $ \\text{H}_2\\text{SO}_4$ then a lowering of Freezing point is observed that correspond to 2 particles for each molecule of diene.When the solution is treated with water the diene is regenerated.Describe what happens with relevant equations",
"Solution_1": "The part \"that correspond to 2 particles for each molecule of diene\" means that for each molecule of diene that reacts with H2SO4, two species are formed. Well, the diene has double bonds and H2SO4 is an acid: what do you think that might happen?\r\n\r\n[hide=\"Answer\"]A salt of the form cyclohexadiene+HSO4- is formed.[/hide]\r\n\r\n\r\n[b]Question[/b]: what would happen if the 1,3,5,5-tetramethylcyclohexa-1,3-diene had a C=O bond in carbon 6 and were treated with sulphuric acid?",
"Solution_2": "i got the salt but couldn't know how on adding water that get's converted to diene",
"Solution_3": "In the salt formation reaction, the diene acts as a base, accepting an H+ ion from sulphuric acid. When the solution is treated with water, the diene acts as an acid with a molecule of water acting as a base, regenerating the diene. These are simply acid-base reactions."
}
{
"Tag": [
"algebra",
"polynomial",
"induction",
"algebra unsolved"
],
"Problem": "Any polynomial $g(x)$ of degree d or less satisfies the recursion \r\n\r\n$\\sum_{i=0}^{d+1}(-1)^i\\binom{d+1}{i}g(x+i)=0$\r\n\r\nCan someone explain this to me?",
"Solution_1": "It sufficies to prove the identity for $g(x)=x^k$, $k\\leq d$.\r\nWe will use an induction.\r\n1. For $d=1$ the assertion is true.\r\n2. We have \\[\\sum_{i=0}^d(-1)^i\\binom{d+1}{i}(x+i)^k=\\sum_{i=0}^d(-1)^i\\binom{d+1}{i}(x+i)^{k-1}(x+i)=\\] \\[=x\\cdot \\sum_{i=0}^d(-1)^i\\binom{d+1}{i}(x+i)^{k-1}+\\sum_{i=0}^d(-1)^ii\\binom{d+1}{i}(x+i)^{k-1}.\\]\r\nWe know that $\\sum_{i=0}^d(-1)^i\\binom{d+1}{i}(x+i)^{k-1}=0$ using induction proposition for the pair $(d,k-1)$. Moreover, \\[\\sum_{i=0}^d(-1)^ii\\binom{d+1}{i}(x+i)^{k-1}=\\sum_{i=0}^{d-1}(d+1)(-1)^i\\binom{d}{i}((x+1)+i)^{k-1}=0\\] using induction proposition for pair $(d-1,k-1)$.",
"Solution_2": "If you're inducting on d how can you assume the pair (d,k-1) is true?",
"Solution_3": "Or like this... For polynomial $g(x)$ let $\\Delta g(x)=g(x+1)-g(x)$. Observe that $\\Delta g(x)$ is polynomial which degree is smaller than degree of $g(x)$. Also let $\\Delta_2 g(x)=\\Delta(\\Delta g(x))$ and $\\Delta_n g(x)=\\Delta (\\Delta_{n-1} g(x) )$. It is very easy to proove (by induction) that $\\Delta_n g(x)=\\sum_{i=1}^n (-1)^n {n \\choose i}g(x+i)$. We need to proove that $\\Delta_{d+1} g(x) =0$. But, this is now trivial, because $\\deg \\Delta g(x) \\leq d-1$, $\\deg \\Delta_2 g(x) \\leq d-2$, and so on, and $\\deg \\Delta_d g(x) =0$ so it is constant and that is why $\\Delta_{d+1} g(x) =0$",
"Solution_4": "How do you prove $\\Delta_n g(x)=\\sum_{i=1}^n (-1)^n {n \\choose i}g(x+i)?$\r\n\r\nFor n=1 wouldn't you have $\\Delta g(x)=-g(x+1)?$",
"Solution_5": "[quote=\"pilot\"]If you're inducting on d how can you assume the pair (d,k-1) is true?[/quote]\r\nSorry, I forgot to say that we use the double induction on pairs $(d,k)$ and for $k=0$ we get the well known equality $\\sum_{i=0}^{d+1}(-1)^i\\binom{d+1}{i}=0$.",
"Solution_6": "I typed it wrong. It should be $\\Delta_n g(x)=\\sum_{i=0}^n (-1)^{n-i} {n \\choose i}g(x+i)$. For $n=1$ it becomes $\\Delta g(x) = g(x+1)-g(x)$, for $n=2$ it should be $\\Delta_2 g(x)=(g(x+2)-g(x+1))-(g(x+1)-g(x))=g(x+2)-2g(x+1)+g(x)$ and so on. If it's true for $n$ then $\\Delta_{n+1} g(x) = \\Delta_n g(x+1) - \\Delta_n g(x)=\\sum_{i=0}^n (-1)^{n-i} {n \\choose i}g(x+1+i) - \\sum_{i=0}^n (-1)^{n-i} {n \\choose i}g(x+i)=g(x+n+1)-g(x+n)({n \\choose n-1}+{n \\choose n})+g(x+n-1)({n \\choose n-2}+{n \\choose n-1})+\\cdots =\\sum_{i=0}^{n+1} (-1)^{n+1-i} {n+1 \\choose i}g(x+i)$.\r\n\r\nI hope I didn't make any mistake again..."
}
{
"Tag": [
"quadratics",
"Pythagorean Theorem",
"geometry"
],
"Problem": "Quadrilateral ABCD is inscribed in a circle with diameter AD=4. If sides AB and BC each have length 1, then find CD.",
"Solution_1": "[hide=\"Solution\"]By Ptolemy's Theorem, $ AB \\cdot CD \\plus{} BC \\cdot AD \\equal{} CA \\cdot BD$, we get\n$ 4 \\plus{} CD \\equal{} CA \\cdot BD$.\nBecause $ AD$ is the diameter, we know that triangles $ ABD$ and $ DCA$ are right. We then use the Pythagorean Theorem to find the values of CA and BD.\n$ 4 \\plus{} CD \\equal{} \\sqrt {15} \\cdot \\sqrt {16 \\minus{} CD}$.\nThis is just a quadratic, so we solve and get $ CD \\equal{} \\frac {7}{2}$.[/hide]",
"Solution_2": "[hide=\"solution\"][img]http://i.imgur.com/063bPwZ.png[/img]\nAs $AB=BC=1$, $ \\overarc{AB} = \\overarc{BC} \\Rightarrow \\angle{BDA}=\\angle{BDC}=\\frac{1}{2}\\angle{CDA}$\n$\\frac{CD}{AD}=\\cos(CDA)=\\cos(2BDA)=\\cos^{2}(BDA)-\\sin^{2}(BDA)=1-2\\sin^{2}(BDA)=1-2(\\frac{AB}{AD})^2$\n$\\Rightarrow CD=\\frac{7}{2}$[/hide]",
"Solution_3": "[hide=Nice problem!] We use Pot Theorem to get AB*CD+BC*AD=CA*BD, therefore 4+CD=CA*BD. Because AD is diameter, we know triangles ABD and DCA are right. By Pythagorean Theorem, to find CA and BD. 4+CD=root(15)*root(16-CD). This is just a quadratic, so we solve and get CD=7/2. [/hide]\n\nOh no! Isn't this the same thing as sup3rcrash3r's solution? Oh well post count inflation is fun, so I won't delete.",
"Solution_4": "It should be 4+CD=root(15)*root(16-CD^2)."
}
{
"Tag": [
"email"
],
"Problem": "can anyone give an exact number of how many AoPS users participated at nationals?\r\nThere seems like an awful lot.",
"Solution_1": "[quote=\"236factorial\"]can anyone give an exact number of how many AoPS users participated at nationals?[/quote]No, I don't think anyone can give you an [b]exact[/b] number :)",
"Solution_2": "AoPS is getting really big. There were a lot of AoPS shirts (I saw about 7), plus you shouldn't forget the people like me who don't have AoPS shirts. I would say there were a good 20 there. Then again, I really have no sense for how many there are. Maybe more, maybe less.",
"Solution_3": "too many. Two off my team mates are here on aops and so is (was) my coach. then one of them (Secret Asian) went around asking everyone for their usrn.",
"Solution_4": "Hey! Not everyone....er....ok yes everyone. I just wanted to see who was who. It was mathgeek who decided to go around and also tell people his username.",
"Solution_5": "mathgeek? All he ever do is jump around in the halls and hit you. And through that you say he found time to ask people for their usrn?? \r\n\r\nI said Secret Asian cause I remebered at that moment you asking kingof21penguins.",
"Solution_6": "Ok, I asked: New Jersey(which one is treething/aopser), Entropywins(but he was wearing an aops shirt, hence me asking) the whole reason I started asking texas was because asdf4(something like that, he's from tenessee) asked [b]me[/b] if I was an aopser...I asked more people than that, but I can't remember all of them",
"Solution_7": "lol, nice signature secret asian",
"Solution_8": "thanks g-unit.",
"Solution_9": "That is hilarious signatur",
"Solution_10": "The only people I saw were Naga, Xantos. C. Guin, and Solafidefarms. :( :( :(",
"Solution_11": "You didn't see any of the TX or CA teams?",
"Solution_12": "Well yeah but I didnt know who was who.",
"Solution_13": "Yeah, a lot of people were running up to me asking \"Are you Treething?\"\r\n\r\nI'M A CELEBRITY :P !",
"Solution_14": "I saw jb05, xantos, xxreddevilxx, zepelinator, smiley, smartnerd666, treething, biffandoc, 09husbri, entropywins, Hexahedron, mathgeek2006, Secret Asian, Dnas(though I didn't know it was him), kchande, aznness, bob123, piggypi, cutiepi, nosoupforyou, falconwing64, shlomadapenguin, jaeminlee, natedawg, mdean09, topper(who was at Word Power nats too), rorrimimage, jasonho007, dengmi, MooGoesCow?, RC-7th, and seungsoo. Oh, yes, and rcv, naga, frost13, and rrusczyk for the adult AoPSersAt least those are the ones I know the usernames for. 35. I didn't meet asdf4 from Tennessee, or if I did, I didn't ask if he was a AoPSer. Would someone please PM me his real name?\r\n\r\nBilly",
"Solution_15": "[quote=\"solafidefarms\"]I saw jb05, xantos, xxreddevilxx, zepelinator, smiley, smartnerd666, treething, biffandoc, 09husbri, entropywins, Hexahedron, mathgeek2006, Secret Asian, Dnas(though I didn't know it was him), kchande, aznness, bob123, piggypi, cutiepi, nosoupforyou, falconwing64, shlomadapenguin, jaeminlee, natedawg, mdean09, topper(who was at Word Power nats too), rorrimimage, jasonho007, dengmi, MooGoesCow?, RC-7th, and seungsoo. Oh, yes, and rcv, naga, frost13, and rrusczyk for the adult AoPSersAt least those are the ones I know the usernames for. 35. I didn't meet asdf4 from Tennessee, or if I did, I didn't ask if he was a AoPSer. Would someone please PM me his real name?\n\nBilly[/quote]\r\n\r\nHoly Monkey!!\r\n\r\nThat's even more than I saw. Can you email me that list of everyone's emails and AoPS usernames?",
"Solution_16": "It was nice meeting people who I knew only by username before. Now I can see some faces while reading posts.",
"Solution_17": "Ohh, yes, Monkey, and Kingof21 penguins. Did I forget anymore???",
"Solution_18": "WOW 35 AoPSers including 3 adults\r\nI know mdean09 he's at my school. Doesn't post much though. (sry dean)\r\nLet's see...\r\n57*5=285\r\n285/38=7.5%!!!\r\nplus the undoubtable 10 poeple we missed or so,\r\nwe have about 10%!!!!!!!!!\r\nWoohoo!\r\n~1 out of 10 national people was an AoPSer!!!! :D :D :D :D",
"Solution_19": "Billy - since you do have my e-mail, could I just request that you not give it out to anybody unless you know for a fact that they were at nationals? Thanks!\r\n\r\nAnd everybody else, if you want my e-mail and you fit the above prerequisite, Billy has it!",
"Solution_20": "[quote=\"solafidefarms\"]I saw jb05, xantos, xxreddevilxx, zepelinator, smiley, smartnerd666, treething, biffandoc, 09husbri, entropywins, Hexahedron, mathgeek2006, Secret Asian, Dnas(though I didn't know it was him), kchande, aznness, bob123, piggypi, cutiepi, nosoupforyou, falconwing64, shlomadapenguin, jaeminlee, natedawg, mdean09, topper(who was at Word Power nats too), rorrimimage, jasonho007, dengmi, MooGoesCow?, RC-7th, and seungsoo. Oh, yes, and rcv, naga, frost13, and rrusczyk for the adult AoPSersAt least those are the ones I know the usernames for. 35. I didn't meet asdf4 from Tennessee, or if I did, I didn't ask if he was a AoPSer. Would someone please PM me his real name?\n\nBilly[/quote]\r\n\r\nYou saw me too, remember at Joe DUmars",
"Solution_21": "OK OK, I forgot some people. ... Does that make 36 now?",
"Solution_22": "I think you saw me... although I was being swarmed by people who wanted that extremely rare and valuable CT pin.",
"Solution_23": "[quote=\"Treething\"]Yeah, a lot of people were running up to me asking \"Are you Treething?\"\n\nI'M A CELEBRITY :P ![/quote]\r\n\r\n...I just wanted to know. Basically my rules were if you were wearing an Aops shirt you were fair game...or if I knew where you were from. I think Asod and mathgeek told people who I was though. I wanted to stay \"Secret\"",
"Solution_24": "hmmm. I've been thinking, and I don't think that I saw xxreddevilzxx, since he wasn't from Washington. hmmmm. Sorry!"
}
{
"Tag": [],
"Problem": "These question(s) are from [i]Gravity from the Ground up[/i] - I want to know if my answer(s) are in fact solutions.\r\n\r\nThe first question is: Suppose that at the $ n^{th}$ time-step $ t_n$, the vertical speed is $ v_n$ and the vertical distance above the ground is $ h_n$. Show that at the next time-step $ t_{n\\plus{}1}$ $ \\equal{}$ $ t_n$ $ \\plus{}$ $ \\Delta$$ t$, the vertical speed is $ v_{n\\plus{}1}$ $ \\equal{}$ $ v_n$ $ \\minus{}$ $ g$ $ \\Delta$ $ t$.\r\n\r\nMy answer:\r\n\r\n$ v_{n\\plus{}1}$ = $ v_n$ $ \\plus{}$ $ v_1$\r\n\r\n$ v_1 \\equal{} at$ \r\n\r\nIn this case $ a \\equal{} g$ (gravity)\r\n\r\nSo $ v_n$ + $ v_1$ $ \\equal{}$ $ v_n$ $ \\minus{}g$ $ \\Delta$ $ t$\r\n\r\n\r\nI'm not sure why, but my answer doesn't seem complete to me.\r\n\r\nI'll post the rest of the questions/my answers tomorrow :)",
"Solution_1": "Seems logical... :maybe:"
}
{
"Tag": [
"quadratics"
],
"Problem": "If the base $8$ representation of a perfect square is $ab3c$, where $a\\ne 0$, then $c$ equals \n\n$\\textbf{(A) } 0\\qquad \\textbf{(B) }1 \\qquad \\textbf{(C) } 3\\qquad \\textbf{(D) } 4\\qquad \\textbf{(E) } \\text{not uniquely determined}$",
"Solution_1": "[hide=\"NT Solution\"]Recall that a perfect square must be equivilant to $0$, $1$, or $4$ mod $8$. Thus, c=0,1, or 4.\n\n$c=0$ is impossible because we're left with $8(8^2a+8b+3)$, which cannot be a square because $8$ isn't a square.\n\nIf $c=4$, we are left with $4(8N +7)$, where N is $4\\cdot 8^2a+32b$. This is once again impossible, because numbers of the form $8N+7$ cannot be a perfect square. (8 isn't a quadratic residue mod 7)\n\nThus, the answer is $n=1$\n[/hide]",
"Solution_2": "[quote=\"4everwise\"]If the base $8$ representation of a perfect square is $ab3c$, where $a\\ngeq 0$, then $c$ equals \n\n$\\text{(A)} 0\\qquad \\text{(B)}1 \\qquad \\text{(C)} 3\\qquad \\text{(D)} 4\\qquad \\text{(E)} \\text{not uniquely determined}$[/quote]\r\n\r\nyou might want to edit the \r\n\r\n$a0$, $\\$$a\\neeq 0$\\$$ to \r\n$a\\ne 0$, $\\$$a\\ne 0$\\$$ i did not notice this until i looked at the code\r\n\r\nanyway, your solution is good, i was thinking mod 4, but that is not as easy, but also 25 is 31 base 8 so 1, answers are never \"not determined\"..."
}
{
"Tag": [
"puzzles"
],
"Problem": "There are five friends. Josh says that his birth date is 31st dec 1878. Rick says that his birth date is 14th nov 1902, Derik says his birth date is 12th july 1898, Usha says her birth date is 29th feb 1900 and Mike says his birth date is 15th May 1904.In which year will be the sum of their ages is equal to a prime number between 175 and 240.",
"Solution_1": "Considering that this is probably homework, I'll just give you a hint.\r\n\r\n[hide]Start from the beginning of 1905 and note that each year the sum of the ages increase by 5.\n\nBtw, I think there might be more than one solution, but I'll have to check that. [/hide]",
"Solution_2": "Never; February 29 1900 did not happen because 1900 is not a leap year.",
"Solution_3": "[quote=\"1=2\"]Never; February 29 1900 did not happen because 1900 is not a leap year.[/quote]\r\n\r\nUhh...... yes it is. Any year divisible by 4 is a leap year...",
"Solution_4": "Except for those divisible with 100 and not divisible with 400.\r\n\r\n[quote=\"Wikipedia\"]However, some exceptions to this rule are required since the duration of a solar year is slightly less than 365.25 days. Years that are evenly divisible by 100 are not leap years, unless they are also evenly divisible by 400, in which case they are leap years.[1][2] For example, 1600 and 2000 were leap years, but 1700, 1800 and 1900 were not. Similarly, 2100, 2200, 2300, 2500, 2600, 2700, 2900, and 3000 will not be leap years, but 2400 and 2800 will be. By this rule, the average number of days per year will be 365 + 1/4 \u2212 1/100 + 1/400 = 365.2425, which is 365 days, 5 hours, 49 minutes, and 12 seconds.[/quote]",
"Solution_5": "[quote=\"hsiljak\"]Except for those divisible with 100 and not divisible with 400.\n\n[quote=\"Wikipedia\"]However, some exceptions to this rule are required since the duration of a solar year is slightly less than 365.25 days. Years that are evenly divisible by 100 are not leap years, unless they are also evenly divisible by 400, in which case they are leap years.[1][2] For example, 1600 and 2000 were leap years, but 1700, 1800 and 1900 were not. Similarly, 2100, 2200, 2300, 2500, 2600, 2700, 2900, and 3000 will not be leap years, but 2400 and 2800 will be. By this rule, the average number of days per year will be 365 + 1/4 \u2212 1/100 + 1/400 = 365.2425, which is 365 days, 5 hours, 49 minutes, and 12 seconds.[/quote][/quote]\r\n\r\nOh, yes. You are right. I've never heard that before.... I wonder why....",
"Solution_6": "There are four solutions that I found, 1931, 1935, 1941, and 1943"
}
{
"Tag": [
"trigonometry",
"complex analysis",
"function",
"logarithms",
"real analysis",
"real analysis unsolved"
],
"Problem": "Let $ \\bold{f}: \\mathbb{R}^2\\to\\mathbb{R}^2$ be defined by $ \\bold{f}(x,y)\\equal{}(e^x\\cos y, e^x\\sin y)$\r\nShow that $ \\bold{f}$ is injective on the strip $ \\{(x,y): \\minus{}\\pi n$\n\nOr in other words, $p\\nmid \\frac{n!}{p}$\n\nNow Note that $p|\\frac{n!}{k} \\forall k \\in \\mathbb{N}, 1\\le k \\le n, k\\ne p$\n\nSo, ${p\\nmid \\frac{n!}{1}+\\frac{n!}{2}+\\cdots+\\frac{n!}{n}}$ but $p|n!$ \n\n$\\Longrightarrow \\frac{\\frac{n!}{1}+\\frac{n!}{2}+\\cdots+\\frac{n!}{n}}{n!}$ is never an integer.",
"Solution_4": "[quote=\"gouthamphilomath\"]By Bertrand's postulate, there exists a prime $p$ such that $\\left\\lfloor{\\frac{n}{2}}\\right\\rfloor 0 and q>0 because the minor arc MA is in the first quadrant. If T(u,v) then up+vq=0 because MX is perpendicular to MT.\r\nAfter a lot of computations one get\r\ntan(MTB-CTM) = a/h, which in fact does not depends on X(p,q).",
"Solution_2": "Denote E be the midpoint of BT, then the quadrilateral XMET inscribed the circle whose diameter is XT. Since ME || CT, \r\n$ \\angle MTB \\minus{} \\angle MTC \\equal{} \\angle MTE \\minus{} \\angle EMT \\equal{} \\angle MXE \\minus{} \\angle EXT \\equal{} \\angle MXE \\minus{} \\angle EXB \\equal{} \\angle MXB \\equal{} \\angle MAB$",
"Solution_3": "Let the perpendiculers from $ B$ and $ C$ to $ XM$ and $ TM$ respectively intersect at $ P$.\r\n$ \\angle BPC \\equal{} 90^0\\implies BM \\equal{} CM \\equal{} PM$. So $ P$ is the reflection of $ C$ at $ TM$ and of $ B$ at $ XM$.\r\n$ XT \\equal{} XB \\equal{} XP\\implies$ $ X$ is the circumcenter of $ TBP$. \r\n$ \\angle MTB \\minus{} \\angle CTM \\equal{} \\angle MTB \\minus{} \\angle PTM \\equal{} \\angle BTP \\equal{} \\frac {1}{2}\\angle BXP \\equal{}$\r\n$ \\equal{} \\angle BXM \\equal{} \\angle BAM\\ \\ \\text{(Const.)}$",
"Solution_4": "\u039dotice here that the condition AB=AC is absolutely redundant.... Hence the problem can be generalized to any triangle ABC,using the same method of proof as mr.danh's idea,which is also the same as mine.\n\nCheers,\nNick",
"Solution_5": "Sorry for the revival; why is XMET cyclic in mr.danh's solution?\n\n[quote=\"mr.danh\"]then the quadrilateral XMET inscribed the circle whose diameter is XT[/quote]",
"Solution_6": "[quote=\"AwesomeToad\"]Sorry for the revival; why is XMET cyclic in mr.danh's solution?\n[/quote]\n\n$TX=BX$\nso, $\\triangle TXB$ is isosceles\nsince $E$ is the midpoint\n$XE$ is the median as well as altitude of the triangle\nSo, $\\angle TEX=90^\\circ$\nit is given $\\angle TMX=90^\\circ$\nSo, $TEMX$ is cyclic with $TX$ being the diameter",
"Solution_7": "[hide=\"Solution\"]\nLet $N$ be the midpoint of $BT$. Since $M$ is the midpoint of $BC$, we have $MN||CT$. Therefore, $\\angle{CTM}=\\angle{TMN}$. \n\nSince $TX=BX$, $\\triangle{BXT}$ is isosceles, so $\\angle{TXN}=\\angle{BXN}$ and $XN\\perp BT$. Then $\\angle{TNX}=90=\\angle{TMX}$, so $XMNT$ is cyclic. Thus, $\\angle{MTB}=\\angle{MXN}$ and $\\angle{CTM}=\\angle{TMN}=\\angle{TXN}=\\angle{BXN}$. Finally, observe that from cyclic quadrilateral $ABMX$, \n\\[ \\angle{BAM}=\\angle{BXM}=\\angle{MXN}-\\angle{BXN}=\\angle{MTB}-\\angle{CTM}.\\]\nBecause $\\angle{BAM}$ is independent of $X$, $\\angle{MTB}-\\angle{CTM}$ is also not dependent on the choice of $X$. // [/hide]",
"Solution_8": "Let $C' $ be the symetrical of $C$ wrt $TM$ and $L=AM\\cap TC'$ and $Y$ the reflexion of $X$ wrt the midpoint of $AB$. and $E$ the midpoint of $TB$\nthen $ \\angle{MTB}-\\angle{CTM} = \\angle LTB$\nit is enough to show that $ATLB$ is cyclic to conclude that $\\angle{MTB}-\\angle{CTM} = \\angle MAB$\nwe have: $\\angle ALB=\\frac{\\pi}{2}-\\angle TMB - \\angle MTC = \\frac{\\pi }{2}- \\angle AYX -\\angle TME = \\frac{\\pi}{2}-\\angle AYX -\\frac{\\angle TXB}{2}=\\angle XBT -\\angle ABX =\\angle ABT $\nHence $ATBL$ is cyclic which leads to \n\\[ \\angle{MTB}-\\angle{CTM}= \\angle LTB = \\angle MAB \\]",
"Solution_9": "We will prove the result for an arbitrary $\\triangle ABC.$ Define $\\Gamma \\equiv \\odot(X, XB)$ and $\\omega \\equiv \\odot(ABM).$ Let $S$ be the reflection of $T$ in $M$ and let $MT$ meet $\\omega$ for a second time at $Y.$ \n[asy]\n /* Geogebra to Asymptote conversion, documentation at artofproblemsolving.com/Wiki, go to User:Azjps/geogebra */\nimport graph; size(15cm); \nreal labelscalefactor = 0.5; /* changes label-to-point distance */\npen dps = linewidth(0.7) + fontsize(10); defaultpen(dps); /* default pen style */ \npen dotstyle = black; /* point style */ \nreal xmin = 937.3756443093395, xmax = 1122.3822298074242, ymin = 958.746170568615, ymax = 1051.0863181805378; /* image dimensions */\npen qqwuqq = rgb(0.,0.39215686274509803,0.); \n\ndraw(arc((981.6398411686986,1019.9779690940252),4.894354113705946,-41.70511190447346,-22.045779161991184)--(981.6398411686986,1019.9779690940252)--cycle, qqwuqq); \ndraw(arc((1036.6505840120194,970.956380225791),4.894354113705946,138.29488809552655,157.95422083800884)--(1036.6505840120194,970.956380225791)--cycle, qqwuqq); \ndraw(arc((976.8060711531035,995.1907717416409),4.894354113705946,78.96519747630964,98.62453021878137)--(976.8060711531035,995.1907717416409)--cycle, qqwuqq); \nLabel laxis; laxis.p = fontsize(10); \nxaxis(xmin, xmax, Ticks(laxis, Step = 10., Size = 2, NoZero),EndArrow(6), above = true); \nyaxis(ymin, ymax, Ticks(laxis, Step = 10., Size = 2, NoZero),EndArrow(6), above = true); /* draws axes; NoZero hides '0' label */ \n /* draw figures */\ndraw((976.8060711531035,995.1907717416409)--(1041.4843540276145,995.7435775781753)); \ndraw(circle((992.8090020683957,1014.8258301910363), 25.330403040433268), linetype(\"2 2\") + blue); \ndraw(circle((1013.9145017344058,1000.8191459995926), 37.532841845441226), linetype(\"2 2\") + blue); \ndraw((1036.6505840120194,970.956380225791)--(971.7035024023855,1028.83251438248)); \ndraw((976.8060711531035,995.1907717416409)--(981.6398411686986,1019.9779690940252), red); \ndraw((1041.4843540276145,995.7435775781753)--(981.6398411686986,1019.9779690940252), red); \ndraw((976.8060711531035,995.1907717416409)--(1036.6505840120194,970.956380225791), red); \ndraw((1036.6505840120194,970.956380225791)--(1041.4843540276145,995.7435775781753), red); \ndraw((971.7035024023855,1028.83251438248)--(976.8060711531035,995.1907717416409)); \n /* dots and labels */\ndot((990.,1040.),linewidth(3.pt) + dotstyle); \nlabel(\"$A$\", (990.7241041487343,1041.9713196788784), NW * labelscalefactor); \ndot((976.8060711531035,995.1907717416409),linewidth(3.pt) + dotstyle); \nlabel(\"$B$\", (974.5727355735047,993.1697945016816), SW * labelscalefactor); \ndot((1041.4843540276145,995.7435775781753),linewidth(3.pt) + dotstyle); \nlabel(\"$C$\", (1042.4411126168939,993.1697945016816), SE * labelscalefactor); \ndot((1009.145212590359,995.4671746599081),linewidth(3.pt) + dotstyle); \nlabel(\"$M$\", (1008.5069240951992,992.1909236789404), S * labelscalefactor); \nlabel(\"$\\omega$\", (960,1010), NE * labelscalefactor,blue); \ndot((1013.9145017344058,1000.8191459995926),linewidth(3.pt) + dotstyle); \nlabel(\"$X$\", (1015.5221649915111,1000.1850353979937), E * labelscalefactor); \ndot((971.7035024023855,1028.83251438248),linewidth(3.pt) + dotstyle); \nlabel(\"$Y$\", (967.3520911855517,1029.8774503544778), NE * labelscalefactor); \nlabel(\"$\\Gamma$\", (1042,1032), NE * labelscalefactor,blue); \ndot((981.6398411686986,1019.9779690940252),linewidth(3.pt) + dotstyle); \nlabel(\"$T$\", (981.6513640015929,1022.2730164414478), N * labelscalefactor); \ndot((1036.6505840120194,970.956380225791),linewidth(3.pt) + dotstyle); \nlabel(\"$S$\", (1037.7099036403113,967.3928628361625), SE * labelscalefactor); \nclip((xmin,ymin)--(xmin,ymax)--(xmax,ymax)--(xmax,ymin)--cycle); \n /* end of picture */\n[/asy]\nNote that $BTCS$ is a parallelogram because its diagonals bisect one another. Hence, $\\measuredangle CTM = \\measuredangle BST.$ Meanwhile, $S \\in \\Gamma$ because $XM$ is the perpendicular bisector of $\\overline{ST}.$ Then since $\\measuredangle YBX = \\measuredangle YMX = 90^{\\circ}$, it follows that $YB$ is tangent to $\\Gamma.$ Thus, the Alternate Segment Theorem yields $\\measuredangle BST = \\measuredangle YBT.$ Hence,\n\\begin{align*}\n\\measuredangle MTB - \\measuredangle CTM = \\measuredangle MTB - \\measuredangle YBT = \\measuredangle BYT.\n\\end{align*}\nBut clearly $\\measuredangle BYT = \\measuredangle BYM = \\measuredangle BAM$ is fixed, as desired. $\\square$",
"Solution_10": "Here is an analytic solution:\n\nWe use complex numbers and set $(ABM)$ to be the unit circle. Furthermore, let $a=-1,b=1$. Then it is clear that $c=2m-1$. Now since $TM \\perp XM$, we have \\[\\frac{t-m}{x-m}=-\\frac{\\bar t - \\frac{1}{m}}{\\frac{1}{x}-\\frac{1}{m}} \\implies t-m=mx \\bar t - x \\implies \\bar t = \\frac{t-m+x}{mx}\\] Now since $XT=XB$, we have \\[(t-x)(\\bar t - \\tfrac{1}{x})=(1-x)(1-\\tfrac{1}{x}) \\implies t \\bar t - \\frac{t}{x}- \\bar t x + x +\\frac{1}{x} -1=0\\] Plugging in our first equation into our second, we get \\[t\\left(\\frac{t-m+x}{mx}\\right)-\\frac{t}{x}-\\left(\\frac{t-m+x}{m}\\right) + x + \\frac{1}{x} -1=0 \\implies t^2 - 2mt + (m-1)x^2+m=0\\] Now let $r$ be the root that lies within angle $AMB$ and call the other root $s$. It is clear that these roots are reflections about $M$ (and thus $r,m,s$ are collinear), so now consider the following complex number which has argument $\\angle MTB - \\angle CTM$ \\[\\frac{\\frac{s-r}{1-r}}{\\frac{2m-1-r}{s-r}}=-\\frac{(s-r)^2}{(1-r)(1-s)}=\\frac{4m^2-4((m-1)x^2+m)}\n{(m-1)(1-x^2)}=\\frac{4(m-x^2)}{1-x^2}\\] Now notice that since $|x|=1$, $x^2$ lies on the unit circle, and so we see that the argument of this complex number is $\\angle MXB=\\angle MAB$ which is independent of $X$, as desired. $\\Box$",
"Solution_11": "We do not use complex numbers\n\n[hide=Solution]\nLet $N$ be the midpoint of $BT$. Since $XB=XT$, $\\angle XNT=90^{\\circ}=\\angle XMT$, so $XMNT$ is cyclic, yielding $\\angle BTM=\\angle NTM=\\angle NXM$.\nNow $MN$ is a midline in $\\triangle BCT$, so \n\\[ \\angle MTC=\\angle BTC - \\angle MTC=\\angle BNM-\\angle MXN=\\angle MXT-\\angle MXN=\\angle NXT=\\angle NXB \\]\nHence, $\\angle BTM-\\angle CTM=\\angle BXN - \\angle MXN=\\angle BXM=\\angle BAM$ which is fixed.\n[/hide]\n\nWhen I started working on this problem, I misread it as $TB=TX$. If we characterize $T$ as such, can we find any interesting properties of this new configuration? All my progress so far for this configuration is rather trivial.",
"Solution_12": "[hide=Solution]\nLet $N$ be the midpoint of $BT$. Then $MN \\parallel CT$. Also, $XN \\perp NT$, so $XMNT$ is cyclic. Then \\[\\begin{aligned} \\angle MTB - \\angle CTM &= \\angle MTB - (\\angle CTB - \\angle MTB) \\\\ &= 2\\angle MTB - \\angle CTB \\\\ &= 2\\angle MTN - \\angle MNB \\\\ &= 2\\angle MXN - (\\pi - \\angle MNT) \\\\ &= 2\\angle MXN - \\angle MXT \\\\ &= 2\\angle MXN - (\\angle MXN + \\angle NXT) \\\\ &= \\angle MXN - \\angle NXT \\\\ &= (\\angle MXB + \\angle BXN) - \\angle NXT \\\\ &= (\\angle MXB + \\angle NXT) - \\angle NXT \\\\ &= \\angle MXB \\\\ &= \\angle MAB \\end{aligned}\\] which does not depend on $X$.\n[/hide]",
"Solution_13": "My solution:(similar MStang and mr.danh)\nLet $Q$ be the midpoint of $BT,$ then $\\angle XQT =90,$ also $\\angle XMT=90\\to XMQT$ is cyclic.\nAlso $MQ$ is the midline of $\\triangle CBT\\to MQ\\parallel CT.$\nThen $\\angle MTB-\\angle CTM=\\angle MXQ-\\angle QMT=\\angle MXQ-\\angle QXT=\\angle MXQ-\\angle BXQ=\\angle MXB=\\angle MAB.$ As desired.",
"Solution_14": "I generally don't like geometry, but the locus of $T$ as $X$ varies on $\\omega_{ABM}$ is (showed in black) a cute shape, which surprisingly is independent of $\\angle BAM$: \n\n[asy] /* Geogebra to Asymptote conversion, documentation at artofproblemsolving.com/Wiki, go to User:Azjps/geogebra */\nimport graph; size(27.1cm); \nreal labelscalefactor = 0.5; /* changes label-to-point distance */\npen dps = linewidth(0.7) + fontsize(10); defaultpen(dps); /* default pen style */ \npen dotstyle = black; /* point style */ \nreal xmin = -4.3, xmax = 22.8, ymin = -5.76, ymax = 6.3; /* image dimensions */\npen ffxfqq = rgb(1.,0.4980392156862745,0.); \n /* draw figures */\ndraw(circle((6.38,1.), 4.011234224026315), linewidth(2.) + blue); \ndraw((9.02,4.02)--(3.74,-2.02), linewidth(2.) + green); \ndraw((3.74,-2.02)--(14.408602941635356,-1.9233120679801736), linewidth(2.) + green); \ndraw((9.02,4.02)--(9.074301470817678,-1.9716560339900866), linewidth(2.) + green); \ndraw((xmin, -0.5299145299145296*xmin + 4.380854700854699)--(xmax, -0.5299145299145296*xmax + 4.380854700854699), linewidth(2.) + linetype(\"2 2\") + ffxfqq); /* line */\ndraw(circle((9.924343193909214,-0.8781989574561639), 6.288863996049319), linewidth(2.) + linetype(\"2 2\") + blue); /* locus construction */\n\ndraw((3.666532459319288,1.7158227254556473)^^(5.685493539125569,0.9999999930549354)--(5.6854935026018385,0.9999999861098701)--(5.685493429554378,0.9999999722197372)--(5.685493283459452,0.9999999444394625)--(5.685492991269605,0.9999998888788786)--(5.685492406889907,0.9999997777575703)--(5.685491238130497,0.9999995555143926)--(5.685488900611638,0.9999991110257929)--(5.685484225573751,0.9999982220396174)--(5.685474875497324,0.9999964440313623)--(5.685456175341818,0.9999928878712353)--(5.685418775020227,0.9999857749765105)--(5.685343974334768,0.9999715468891729)--(5.685194372795103,0.9999430815228921)--(5.6848951690436405,0.9998861140234769)--(5.684296758875018,0.999772031953789)--(5.683099928057738,0.9995432795034654)--(5.680706225965126,0.9990834211396069)--(5.675918671647202,0.9981542888147239)--(5.66634305611945,0.9962583479987558)--(5.647190547285527,0.9923156543167926)--(5.608886427386223,0.9838262051167292)--(5.532330009850903,0.9644252800129192)--(5.45593496758696,0.9417860264693532)--(5.379812586571335,0.9159008409444593)--(5.304074679375265,0.8867657406276426)--(5.228833469397423,0.8543803858449063)--(5.154201474284849,0.8187480987858757)--(5.080291388658231,0.7798758785249891)--(5.007215966259028,0.7377744123133211)--(4.935087901636658,0.6924580831212156)--(4.8640197114945805,0.6439449734156475)--(4.794123615814696,0.5922568651599956)--(4.725511418879817,0.5374192360276844)--(4.658294390314279,0.4794612518249427)--(4.592583146262918,0.41841575512172113)--(4.528487530828694,0.3543192500936243)--(4.466116497889135,0.2872118835815003)--(4.405577993411586,0.21713742237914313)--(4.346978838386987,0.14414322676336178)--(4.290424612501343,0.06828022028443304)--(4.236019538663634,-0.010397144161248928)--(4.183866368508116,-0.09183092195079112)--(4.134066268988286,-0.17595971801737553)--(4.086718710178719,-0.2627187317628561)--(4.041921354400151,-0.35203980556364733)--(3.999769946781858,-0.4438514768296642)--(3.9603582073742345,-0.5380790335723598)--(3.9237777249230215,-0.6346445734344237)--(3.8901178524153184,-0.7334670661300633)--(3.874410431075394,-0.783698508657428)--(3.8594656045055804,-0.8344624192415226)--(3.845293867670346,-0.8857478192080364)--(3.8319055569285347,-0.9375435473137967)--(3.8193108470060984,-0.9898382620819577)--(3.8075197480037692,-1.042620444186364)--(3.7965421024407617,-1.0958783988843006)--(3.786387582334997,-1.1496002584973328)--(3.777065686320849,-1.2037739849394553)--(3.768585736804999,-1.258387372292142)--(3.760956877161276,-1.3134280494255997)--(3.7541880689650657,-1.3688834826657588)--(3.7482880892683106,-1.4247409785061471)--(3.7432655279153932,-1.480987686364395)--(3.7391287849011388,-1.5376106013822852)--(3.735886067771019,-1.5945965672692508)--(3.733545389064971,-1.6519322791880227)--(3.732114563804804,-1.709604286682412)--(3.7316012070264213,-1.767598996646019)--(3.7320127313571203,-1.825902676331583)--(3.7333563446389872,-1.884501456399909)--(3.7356390475986863,-1.9433813340079813)--(3.7817729391770887,-2.0169625784216554)--(3.8406614889536357,-2.011884343848216)--(3.8991536309809525,-2.0059079212844435)--(3.9572363309402707,-1.9990420087628293)--(4.014896712864063,-1.9912954933108074)--(4.072122061911019,-1.9826774482532565)--(4.128899827096567,-1.9731971304735236)--(4.185217623978182,-1.9628639776339127)--(4.2410632372949255,-1.9516876053562344)--(4.296424623560574,-1.9396778043632736)--(4.351289913609769,-1.926844537581855)--(4.40564741509662,-1.913197937208281)--(4.459485614945187,-1.8987483017369464)--(4.512793181751267,-1.8835060929528697)--(4.565558968135011,-1.8674819328889105)--(4.617772013043776,-1.8506866007485507)--(4.66942154400475,-1.8331310297949206)--(4.7204969793267955,-1.8148263042070127)--(4.770987930251089,-1.7957836559037932)--(4.870175801073993,-1.755530238254017)--(4.96690598349996,-1.7124634643729049)--(5.061102520338164,-1.6666781772839392)--(5.152692729714776,-1.6182712687308367)--(5.241607261134152,-1.567341570728622)--(5.327780148019183,-1.5139897455534361)--(5.411148856686603,-1.458318174284143)--(5.491654331716694,-1.4004308440101652)--(5.569241037680472,-1.3404332338210294)--(5.643856997191119,-1.2784321996941987)--(5.715453825250149,-1.21453585839865)--(5.783986759862463,-1.1488534705325133)--(5.849414688898338,-1.0814953228136701)--(5.911700173184018,-1.01257260974289)--(5.970809465806485,-0.9421973147594456)--(6.026712527621715,-0.8704820910094742)--(6.079383038959575,-0.7975401418476298)--(6.128798407522286,-0.7234851011925337)--(6.1749397724772415,-0.6484309138566487)--(6.217792004748711,-0.5724917159708548)--(6.257343703516804,-0.4957817156239312)--(6.29358718893584,-0.41841507383657733)--(6.326518491088016,-0.3405057859892265)--(6.356137335192053,-0.26216756382215917)--(6.382447123090203,-0.18351371812575248)--(6.405454911040664,-0.1046570422378223)--(6.425171383846217,-0.025709696463980317)--(6.441610825353404,0.05321690646413829)--(6.45479108536028,0.1320122147790795)--(6.464733542974214,0.21056664980396)--(6.471463066464841,0.28877171670411605)--(6.475007969660631,0.366520113754667)--(6.475399964940974,0.44370584001635704)--(6.472674112879078,0.5202243013137076)--(6.46686876859422,0.5959724144114407)--(6.45802552487513,0.6708487092870767)--(6.446189152139483,0.7447534293997062)--(6.43140753529755,0.8175886298570284)--(6.4137316075910915,0.8892582733851142)--(6.393215281481533,0.9596683240076236)--(6.369915376664348,1.028726838343676)--(6.343891545289346,1.0963440544361516)--(6.315206194469325,1.1624324780247315)--(6.283924406162099,1.2269069661808207)--(6.250113854513555,1.2896848082241383)--(6.213844720751744,1.3506858038437304)--(6.1751896057244515,1.4098323383490716)--(6.134223440174887,1.4670494549799202)--(6.091023392852363,1.5222649242066486)--(6.045668776556834,1.5754093099560182)--(5.998240952218204,1.6264160327004318)--(5.948823231113102,1.6752214293520855)--(5.897500775323634,1.7217648099067209)--(5.84436049654423,1.7659885107850446)--(5.789490953344301,1.8078379448233224)--(5.732982246995787,1.8472616478681394)--(5.6749259159760195,1.884211321933829)--(5.615414829257601,1.9186418748845964)--(5.554543078497945,1.9505114566070063)--(5.492405869242279,1.979781491642084)--(5.429099411254618,2.006416708249943)--(5.364720808092082,2.0303851638835315)--(5.2993679460385,2.051658267051775)--(5.233139382513753,2.070210795556103)--(5.166134234075729,2.0860209110881094)--(5.098452064132063,2.0990701701797354)--(5.0301927704789,2.109343531501242)--(4.961456472784042,2.1168293595058554)--(4.892343400131779,2.1215194244237745)--(4.822953778746377,2.1234088986119635)--(4.753387720011027,2.1224963492698428)--(4.683745108898547,2.1187837275347285)--(4.614125492929577,2.1122763539745906)--(4.5446279717733296,2.10298290049932)--(4.475351087605282,2.090915368715359)--(4.406392716335074,2.0760890647522405)--(4.337849959817122,2.0585225705930683)--(4.269819039155097,2.0382377119446)--(4.2023951892103675,2.0152595226860868)--(4.1356725544229915,1.9896162059395197)--(4.069744086052536,1.961339091807299)--(4.0047014409442525,1.9304625918268503)--(3.9406348819246984,1.89702415019487)--(3.8776331799288593,1.861064191817328)--(3.815783517959146,1.8226260672444372)--(3.7551713969745366,1.7817559945530228)--(3.6958805438061364,1.7385029982417912)--(3.637992821193168,1.692918845208002)--(3.5815881400311973,1.6450579778770307)--(3.5267443739220026,1.5949774445591358)--(3.473537276112087,1.5427368271105628)--(3.422040398904174,1.4883981659788486)--(3.372325015623585,1.4320258827147827)--(3.324460045218516,1.3736867000360564)--(3.2785119795705544,1.3134495595300808)--(3.2345448135888564,1.2513855370858298)--(3.192619978158516,1.1875677561468216)--(3.152796276010604,1.1220712988795545)--(3.115129820578321,1.0549731153537794)--(3.0796739779005318,0.9863519308329873)--(3.046479311630807,0.9162881512753641)--(3.0155935312067754,0.8448637671472466)--(2.9870614432313367,0.7721622556527938)--(2.9609249061138847,0.6982684814851486)--(2.937222788016291,0.6232685962058302)--(2.915990928144988,0.5472499363604465)--(2.897262101426889,0.4703009204400513)--(2.8810659866035255,0.39251094479861454)--(2.867429137773995,0.313970278638062)--(2.8563749594139387,0.23476995817327012)--(2.8479236848940026,0.1550016800901659)--(2.84209235851762,0.07475769441076185)--(2.83889482109435,-0.005869303120494607)--(2.838341699061271,-0.08678627901423397)--(2.8404403971612093,-0.1678999702624202)--(2.845195094683023,-0.24911699269799303)--(2.852606745265287,-0.33034394937248246)--(2.862673080261134,-0.4114875388362289)--(2.8753886156582973,-0.4924546632059379)--(2.8907446625446607,-0.5731525359046548)--(2.908729341106011,-0.6534887889595333)--(2.92932759813894,-0.7333715797433715)--(2.9525212280582744,-0.8127096970464502)--(2.9782888973747674,-0.8914126663659339)--(3.006606172615117,-0.9693908543010348)--(3.037445551652917,-1.0465555719429918)--(3.0707764984155395,-1.1228191771500586)--(3.1065654809284435,-1.198095175598918)--(3.144776012654988,-1.272298320505147)--(3.1853686970863646,-1.345344710906856)--(3.228301275532933,-1.4171518884071133)--(3.273528678064902,-1.4876389322723633)--(3.321003077547001,-1.5567265527858622)--(3.370673946708675,-1.6243371827568658)--(3.4224881181880713,-1.6903950670883643)--(3.4763898474850774,-1.7548263503081132)--(3.532320878755609,-1.8175591619698683)--(3.5902205133764484,-1.8785236998339527)--(3.6500256812069916,-1.9376523107386412)--(3.7116710144714986,-1.9948795690761667)--(3.775088924182728,-2.0501423527897904)--(3.840209679025163,-2.103379916810797)--(3.906961486613503,-2.154533963857111)--(3.9752705770396304,-2.203548712517821)--(4.045061288618811,-2.2503709625508614)--(4.1162561557437085,-2.2949501573239077)--(4.188775998752504,-2.3372384433315023)--(4.262540015715336,-2.3771907267246117)--(4.3374658760412705,-2.414764726791693)--(4.41346981580614,-2.4499210263337146)--(4.490466734699709,-2.4826231188787085)--(4.568370294489033,-2.5128374526847153)--(4.647093018893145,-2.5405334714833425)--(4.726546394762882,-2.565683651919534)--(4.806640974458132,-2.5882635376465752)--(4.887286479313617,-2.608251770038825)--(4.968391904083121,-2.6256301154881836)--(5.049865622251042,-2.6403834892538685)--(5.131615492099235,-2.652499975838608)--(5.213548963416232,-2.661970845867995)--(5.2955731847353205,-2.668790569453413)--(5.377595110987289,-2.6729568260224825)--(5.459521611453228,-2.6744705106047943)--(5.541259577902418,-2.673335736564246)--(5.622716032800073,-2.669559834773075)--(5.703798237469634,-2.663153349226357)--(5.7844138000942795,-2.654130029099403)--(5.864470783442447,-2.642506817254245)--(5.943877812202401,-2.628303835205039)--(6.022544179811227,-2.6115443645559555)--(6.100379954664131,-2.5922548249287236)--(6.177296085590451,-2.570464748400741)--(6.253204506483588,-2.5462067504781927)--(6.328018239972782,-2.5195164976323614)--(6.401651500025676,-2.490432671430731)--(6.474019793371595,-2.458996929298212)--(6.5450400196366925,-2.4252538619472173)--(6.614630570083332,-2.389250947518787)--(6.682711424847496,-2.3510385024805487)--(6.749204248569482,-2.310669629330468)--(6.814032484314764,-2.268200161158872)--(6.877121445683596,-2.2236886031244434)--(6.938398407009734,-2.177196070903121)--(6.997792691550595,-2.128786226172008)--(7.055235757573147,-2.0785252091935535)--(7.11066128224197,-2.026481568568283)--(7.1640052432181225,-1.9727261882273293)--(7.21520599787973,-1.9173322117389469)--(7.2642043600776605,-1.8603749640060485)--(7.31094367434207,-1.80193187043449)--(7.355369887458228,-1.7420823736546087)--(7.397431617332653,-1.6809078478810122)--(7.4370802190733345,-1.6184915109982243)--(7.474269848210662,-1.5549183344621311)--(7.508957520988518,-1.4902749511095918)--(7.54110317165801,-1.4246495609707153)--(7.570669706709337,-1.3581318351805618)--(7.5976230559803755,-1.2908128180889764)--(7.621932220583759,-1.2227848276692956)--(7.643569317597443,-1.1541413543284293)--(7.662509621467036,-1.0849769582226687)--(7.678731602071531,-1.0153871651850954)--(7.692216959407448,-0.9454683613720967)--(7.702950654849841,-0.8753176867378674)--(7.710920938952139,-0.8050329274470911)--(7.716119375750251,-0.7347124073372292)--(7.718540863540035,-0.6644548785428993)--(7.718183652100688,-0.5943594113958361)--(7.715049356340395,-0.5245252837147606)--(7.70914296634409,-0.4550518696002611)--(7.700472853806974,-0.3860385278503894)--(7.6890507748410455,-0.31758449011322704)--(7.674891869145676,-0.24978874889302194)--(7.658014655536929,-0.182749945526834)--(7.638441023834116,-0.11656625824875011)--(7.616196223105752,-0.0513352904587574)--(7.591308846280891,0.012846040686644855)--(7.563810811135435,0.07588161524531412)--(7.533737337666901,0.13767622090523002)--(7.501126921874691,0.19813566272911132)--(7.466021305966696,0.2571668721123279)--(7.428465445016741,0.31467801483921676)--(7.388507470101018,0.37057859812373284)--(7.346198647945297,0.42477957652124854)--(7.301593337118333,0.47719345659930035)--(7.2057258562395825,0.576318326578249)--(7.145670669463989,0.6305820984210833)--(7.082824779499289,0.681924453783592)--(7.017299996009972,0.7302208882615004)--(6.949213262109829,0.775350986483673)--(6.878686505069395,0.8171986242525913)--(6.80584647928422,0.8556521651896496)--(6.730824601707877,0.8906046515942045)--(6.653756779964498,0.9219539892328772)--(6.574783233366126,0.9496031257836874)--(6.494048307070418,0.9734602226679547)--(6.411700279624169,0.993438820011705)--(6.3278911641477285,1.009457994487441)--(6.24277650342457,1.0214425097966837)--(6.156515159169266,1.0293229595634736)--(6.069269095755586,1.0330359024192304)--(5.981203158694583,1.032523989069822)--(5.892484848160368,1.0277360811464995)--(5.80328408786853,1.0186273616533879)--(5.713772989619189,1.00515943683559)--(5.691367083097633,1.0011074497453225)--(5.685764594958424,1.000051514851744)--(5.6855895118414,1.0000182399576059)--(5.6855019701664835,1.0000015962158717)--(5.685496498809231,1.0000005558426774)--(5.6854937631304905,1.000000035649933)--(5.685493592150569,1.0000000031377505)^^(3.666532459319288,1.7158227254556468), linewidth(2.)); \n /* dots and labels */\ndot((6.38,1.),dotstyle); \ndot((9.02,4.02),dotstyle); \ndot((9.074301470817678,-1.9716560339900866),dotstyle); \ndot((3.74,-2.02),dotstyle); \ndot((14.408602941635356,-1.9233120679801736),dotstyle); \ndot((9.924343193909214,-0.8781989574561639),dotstyle); \ndot((4.367476888693955,2.066465238469869),linewidth(4.pt) + dotstyle); \nclip((xmin,ymin)--(xmin,ymax)--(xmax,ymax)--(xmax,ymin)--cycle); \n /* end of picture */[/asy]",
"Solution_15": "Let $N$ be the midpoint of $BT$. Now since $M$ is the midpoint of $BC$, we hace $MN||CT$ so $\\angle CTM = \\angle TMN$. Also $\\angle TNX = 90 = \\angle XMT$, hence $\\omega_{TNMX}$ exists. So coupled with this, we get $\\angle CTM = \\angle TMN = \\angle TXN$ (call this $\\spadesuit$)\n\nNow, $\\angle BTM $ $$= \\angle BTX - \\angle XTM = \\angle BTX + \\angle TXM - 90 = \\angle TXM + (\\angle BTX - 90)$$ $$= \\angle TXM - \\angle TXN = \\angle NXM = \\angle NXB + \\angle BXM = \\angle TXN + \\angle BXM \\overset{\\spadesuit}{=} \\angle CTM + \\angle BAM$$, hence $\\angle BTM - \\angle CTM = \\angle BAM$, independent of $X$, as desired. ",
"Solution_16": "Let $N$ be [i]not[/i] the midpoint of $BT$. ;)\n\n[hide=Solution]Let $N$ be the reflection of $T$ across point $M$ and note that $BNCT$ is a parallelogram. Let $\\angle{TNX}=\\beta$ and $\\angle{BNT}=\\angle{NTC}=\\alpha$. Then since $XT=XN$ and $M$ is the midpoint of $TN$, $\\angle{XTM}=\\angle{XNM}=\\beta$. Since $XT=XB=XN$ is the radius of the circumcircle of $\\triangle{TBN}$, $\\angle{BXT}=2\\angle{BNT}=2\\alpha$. Since $\\triangle{BTX}$ is isosceles, $\\angle{BTX}=90^{\\circ}-\\alpha$ so $\\angle{BTN}=\\angle{BTX}-\\angle{NTX}=90^{\\circ}-\\alpha-\\beta$. So we want to find the value of $\\angle{BTN}-\\angle{CTN}=90^{\\circ}-2\\alpha-beta$.\n\nSince $\\triangle{XNT}$ is a right triangle, $\\angle{TXN}=90^{\\circ}-\\beta$ and $\\angle{BTX}=2\\alpha$ so $\\angle{BXM}=\\angle{TXN}-\\angle{BTX}=90^{\\circ}-\\beta-2\\alpha=\\angle{BAM}$ as $BMXA$ is cyclic. Therefore, $\\angle{BTM}-\\angle{CTM}=\\angle{BAM}$, which is independent of $X$.[/hide]",
"Solution_17": "[quote=delegat]Denote by $ M$ midpoint of side $ BC$ in an isosceles triangle $ \\triangle ABC$ with $ AC = AB$. Take a point $ X$ on a smaller arc $ \\overarc{MA}$ of circumcircle of triangle $ \\triangle ABM$. Denote by $ T$ point inside of angle $ BMA$ such that $ \\angle TMX = 90$ and $ TX = BX$. \n\nProve that $ \\angle MTB - \\angle CTM$ does not depend on choice of $ X$.\n\n[i]Author: Farzan Barekat, Canada[/i][/quote]\nLet $N$ be the midpoint of $AB.$ Define $\\omega,\\Omega$ to be the circles $(AMB), \\mathcal{C}(X,XB).$ Let $Y$ be the antipode of $X$ in $\\omega.$ Clearly $T \\in \\overline{MY}, T \\in \\Omega$ by the conditions. Also, $\\measuredangle XBY=\\pi/2$ implies that $YB$ is tangent to $\\Omega.$\n\n[color=#f00]Claim:[/color] $\\measuredangle YBT=\\measuredangle CTM.$\n[color=#00f]Proof:[/color] Let $L$ be the reflection of $T$ over $M.$ Then $TBLC$ is a parallelogram. Further, since $TM=ML$ and $\\measuredangle XMT=\\pi/2,$ hence $TX=XL.$\n\nSo, $L \\in \\Omega.$ Since $YB$ is tangent to this circle, hence $\\measuredangle YBT=\\measuredangle BLT=\\measuredangle CTL.$ $\\square$\n-------\nThus we have\n$$\\measuredangle MTB-\\measuredangle CTM=\\measuredangle MTB-\\measuredangle YBT=\\measuredangle TYB=\\measuredangle MAB$$\nwhich is independent of the position of $X.$ $\\blacksquare$\n[asy]\n /* Geogebra to Asymptote conversion, documentation at artofproblemsolving.com/Wiki go to User:Azjps/geogebra */\nimport graph; size(13cm); \nreal labelscalefactor = 0.5; /* changes label-to-point distance */\npen dps = linewidth(0.7) + fontsize(10); defaultpen(dps); /* default pen style */ \npen dotstyle = black; /* point style */ \nreal xmin = -1086.5975564873966, xmax = 1190.1243804115663, ymin = -879.117740078539, ymax = 608.3405920287947; /* image dimensions */\n\n\ndraw(arc((-586.1713229414271,-304.6162131768956),108.41533032852203,92.83654263451801,108.26645574818497)--(-586.1713229414271,-304.6162131768956)--cycle, linewidth(0.2) + red); \ndraw(arc((-603.0349055618302,35.73566225078713),108.41533032852203,-35.66215225285588,-20.232239139188955)--(-603.0349055618302,35.73566225078713)--cycle, linewidth(0.2) + red); \n /* draw figures */\ndraw(circle((-353.0915854617429,10.809060298607557), 392.19799485807476), linewidth(0.4)); \ndraw((-586.1713229414271,-304.6162131768956)--(-120.01184798205874,326.2343337741107), linewidth(0.4)); \ndraw((-120.01184798205874,326.2343337741107)--(337.5007033357593,-310.9150475089905), linewidth(0.2)); \ndraw((337.5007033357593,-310.9150475089905)--(-586.1713229414271,-304.6162131768956), linewidth(0.4)); \ndraw((-120.01184798205874,326.2343337741107)--(-124.3353098028339,-307.76563034294304), linewidth(0.4)); \ndraw((-728.1424189633996,125.50927590610821)--(21.959248039913632,-103.89115530889305), linewidth(0.2)); \ndraw((-586.1713229414271,-304.6162131768956)--(-728.1424189633996,125.50927590610821), linewidth(0.4)); \ndraw((-728.1424189633996,125.50927590610821)--(-124.3353098028339,-307.76563034294304), linewidth(0.4)); \ndraw(circle((21.959248039913632,-103.89115530889305), 640.4009214688908), linewidth(0.2) + linetype(\"2 2\")); \ndraw((-603.0349055618302,35.73566225078713)--(354.3642859561624,-651.2669229366732), linewidth(0.4)); \ndraw((337.5007033357593,-310.9150475089905)--(354.3642859561624,-651.2669229366732), linewidth(0.4)); \ndraw((-603.0349055618302,35.73566225078713)--(337.5007033357593,-310.9150475089905), linewidth(0.4)); \ndraw((-586.1713229414271,-304.6162131768956)--(354.3642859561624,-651.2669229366732), linewidth(0.4)); \ndraw((-603.0349055618302,35.73566225078713)--(-586.1713229414271,-304.6162131768956), linewidth(0.4)); \ndraw((-124.3353098028339,-307.76563034294304)--(337.5007033357593,-310.9150475089905), linewidth(0.2)); \ndraw((-603.0349055618302,35.73566225078713)--(-728.1424189633996,125.50927590610821), linewidth(0.2)); \n /* dots and labels */\ndot((-353.0915854617429,10.809060298607557),dotstyle); \nlabel(\"$N$\", (-377.5612961388625,42.4125677139054), NE * labelscalefactor); \ndot((-120.01184798205874,326.2343337741107),dotstyle); \nlabel(\"$A$\", (-110.85958353069825,348.1437992403399), NE * labelscalefactor); \ndot((-120.0118479820587,326.2343337741107),linewidth(4pt) + dotstyle); \ndot((-586.1713229414271,-304.6162131768956),linewidth(4pt) + dotstyle); \nlabel(\"$B$\", (-622.5799426813222,-360.89246110819965), NE * labelscalefactor); \ndot((-124.3353098028339,-307.76563034294304),dotstyle); \nlabel(\"$M$\", (-141.2158760226844,-365.2290743213406), NE * labelscalefactor); \ndot((337.5007033357593,-310.9150475089905),linewidth(4pt) + dotstyle); \nlabel(\"$C$\", (346.6531104556648,-293.6749563045155), NE * labelscalefactor); \ndot((21.959248039913632,-103.89115530889305),dotstyle); \nlabel(\"$X$\", (56.1000251752257,-98.52736171317432), NE * labelscalefactor); \ndot((21.95924803991367,-103.89115530889306),linewidth(4pt) + dotstyle); \ndot((-728.1424189633996,125.50927590610821),linewidth(4pt) + dotstyle); \nlabel(\"$Y$\", (-783.0346315675349,129.14483197672368), NE * labelscalefactor); \ndot((-603.0349055618302,35.73566225078713),linewidth(4pt) + dotstyle); \nlabel(\"$T$\", (-652.9362351733084,12.056275221919), NE * labelscalefactor); \ndot((354.3642859561624,-651.2669229366732),linewidth(4pt) + dotstyle); \nlabel(\"$L$\", (368.33617652136917,-696.9799851266205), NE * labelscalefactor); \nclip((xmin,ymin)--(xmin,ymax)--(xmax,ymax)--(xmax,ymin)--cycle); \n /* end of picture */\n[/asy]",
"Solution_18": "[hide=Solution]Spent Whole day solving $BX=BT$ :wallbash: \n[quote=ISL 2007 G2]Denote by $ M$ midpoint of side $ BC$ in an isosceles triangle $ \\triangle ABC$ with $ AC = AB$. Take a point $ X$ on a smaller arc $ \\overarc{MA}$ of circumcircle of triangle $ \\triangle ABM$. Denote by $ T$ point inside of angle $ BMA$ such that $ \\angle TMX = 90$ and $ TX = BX$. \n\nProve that $ \\angle MTB - \\angle CTM$ does not depend on choice of $ X$.\n[/quote]\n[b][color=#000]Solution:[/color][/b] Let $T'$ be reflection of $T$ over $M$ $\\implies$ $X$ is center of $\\odot (BTT')$. Hence, $$ \\angle BTM-\\angle CTM=\\angle BXM-\\tfrac{1}{2} \\angle BXT-\\angle BT'T=\\angle BXM=\\angle BAM \\qquad \\blacksquare$$\n[/hide]",
"Solution_19": "The structure of the most un-elegant solution in this topic.\n\nI will compute these step by step:\n\n1) Let $a,b,c$ be the side lengths of $\\Delta ABC$.We can compute $AM$ by Pythagoras.\n\n2) Let $AX=x$, We can compute $BX$ by Pythagoras.\n\n3) We can find $MX$ by Ptolemy over $ABMX$.\n\n4)We can compute $MT$ by Pythagoras over $\\Delta MTX$($TX=BX$ will be useful here.)\n\n5) Note that $\\angle XMA =\\angle TMB$. The side lengths of $\\Delta MXA$ is known, hence both the angles is known.\n\n6) By cosine rule we can find BT and TC and hence $\\angle BTM \\& \\angle CTM$.\n\n7) If we subtract these angles, we will get a relation in which there is not $x$ ! That means it is independent of $X$.\n\nI know it is near impossible to do but It is the first thing that came to my mind :-D (Geo is a lie without bash)",
"Solution_20": "[asy]\nsize(9cm);\ndefaultpen(fontsize(10pt));\ndefaultpen(linewidth(0.4));\n\npair A = dir(55), B = dir(235), M = dir(305), C = 2M-B, X = dir(355), D = dir(175), T = intersectionpoint(D--M,arc(X,abs(B-X),90,225)), E = 2M-T;\n\ndot(\"$A$\", A, dir(55));\ndot(\"$B$\", B, dir(235));\ndot(\"$M$\", M, dir(270));\ndot(\"$C$\", C, dir(20));\ndot(\"$X$\", X, dir(5));\ndot(\"$D$\", D, dir(175));\ndot(\"$T$\", T, dir(210));\ndot(\"$E$\", E, dir(280));\n\ndraw(A--B--C--A--M--X^^unitcircle, purple);\ndraw(T--X^^E--X^^B--X, blue);\ndraw(T--B--E--C--T, orange);\ndraw(X--A--D--2B-D^^D--E, heavygreen);\ndraw(arc(circumcenter(T,B,E),abs(circumcenter(T,B,E)-T),165,315), magenta);\n[/asy]\nLet $\\overline{MT}$ meet $(ABM)$ again at $D$, and $E$ be the reflection of $T$ over $M$. \n\n[b]Claim:[/b] $X$ is the circumcenter of $(TBE)$. \n\n[i]Proof.[/i] Note that $\\triangle XMT \\cong \\triangle XME$ by SAS, so $TX = EX$. Also we are given that $TX = BX$ so we are done. $\\square$\n\nIn addition, $\\angle DBX = \\angle DMX = 90^\\circ$, so $\\overline{DB}$ is tangent to $(TBE)$. It follows that \\begin{align*}\\angle MTB - \\angle CTM &= \\angle MTB - \\angle BEM = \\angle MTB - \\angle DBT \\\\ &= \\angle MDB = \\angle BAM,\\end{align*} which does not depend on $X$ as desired. $\\blacksquare$",
"Solution_21": "Solution from [i]Twitch Solves ISL[/i]:\n\nConstruct parallelogram $CTBS$ whose diagonals meet at $M$. Also, let $N$ be the midpoint of $\\overline{BT}$.\n[asy] pair A = dir(180); pair B = dir(0); pair M = dir(35); pair C = 2*M-B; pair X = dir(100);\n\nfilldraw(A--B--C--cycle, invisible, red); draw(A--M, red); filldraw(unitcircle, invisible, blue);\n\npair Y = -X; draw(CP(X, B), mediumgreen); pair T = IP(M--Y, CP(X, B)); pair S = 2*M-T; draw(S--T, mediumgreen); draw(B--X--M, mediumgreen);\n\npair N = midpoint(B--T); draw(B--T, deepgreen); draw(T--X--S, mediumgreen); draw(X--N, deepgreen);\n\nfilldraw(circumcircle(X, M, T), invisible, dotted+deepgreen);\n\ndot(\"$A$\", A, dir(A)); dot(\"$B$\", B, dir(B)); dot(\"$M$\", M, dir(M)); dot(\"$C$\", C, dir(C)); dot(\"$X$\", X, dir(X)); dot(\"$T$\", T, dir(T)); dot(\"$S$\", S, dir(S)); dot(\"$N$\", N, dir(N));\n\n/* TSQ Source:\n\nA = dir 180 B = dir 0 M = dir 35 C = 2*M-B X = dir 100\n\nA--B--C--cycle 0.1 lightred / red A--M red unitcircle 0.1 cyan / blue\n\nY := -X CP X B mediumgreen T = IP M--Y CP X B S = 2*M-T S--T mediumgreen B--X--M mediumgreen\n\nN = midpoint B--T B--T deepgreen T--X--S mediumgreen X--N deepgreen\n\ncircumcircle X M T 0.1 yellow / dotted deepgreen\n\n*/ [/asy]\nWe first eliminate $C$ from the diagram by noting that \\[ \\angle CTM = \\angle MSB = \\angle TSB = \\frac{1}{2} \\angle TXB = \\angle NXB. \\] Also, noting that $XMTN$ are cyclic (as $\\angle XMT = \\angle XNT = 90^{\\circ}$), we have \\[ \\angle MTB = \\angle MTN = \\angle NXM. \\] Thus $\\angle MTB - \\angle CTM = \\angle NXM - \\angle NXB = \\angle MXB$ which is fixed.",
"Solution_22": "Let $N$ be the midpoint of $BT$. Observe that since $\\angle XMT = \\angle XNT = 90$, the quadrilateral $XMNT$ is cyclic. Furthermore, $NM \\parallel CT$.\n\nOn the one hand, note that $$\\angle MTB = \\angle MTN = MXN.$$\n\nOn the other hand, note that $$\\angle CTM = \\angle NMT = \\angle NXT = \\angle NXB.$$ Now, note that $$\\angle MTB - \\angle CTM = \\angle MXN - \\angle NXB = \\angle BXM = \\angle BAM$$ which does not depend on $X$, as desired.\n",
"Solution_23": "[asy]\n\t\tunitsize(3cm);\n\t\tpair A, B, C, M, T, T1, X;\n\t\t\n\t\tA = dir(90);\n\t\tB = dir(210);\n\t\tC = dir(330);\n\t\tM = (B + C) / 2;\n\t\tX = rotate(20, (A + B) / 2) * M;\n\t\tT = IP(CP(X, B), Line(M, rotate(90, M) * X, 10), 0);\n\t\tT1 = 2 * M - T;\n\t\t\n\t\tdraw(A--B--C--A^^unitcircle, heavyred);\n\t\tdraw(circumcircle(A, B, M), heavygreen);\n\t\tdraw(CP(X, T), blue);\n\t\tdraw(X--T^^X--B^^X--M^^X--T1^^T--T1, orange);\n\t\t\n\t\tdot(\"$A$\", A, N);\n\t\tdot(\"$B$\", B, SW);\n\t\tdot(\"$C$\", C, SE);\n\t\tdot(\"$M$\", M, S);\n\t\tdot(\"$X$\", X, E);\n\t\tdot(\"$T$\", T, NW);\n\t\tdot(\"$T'$\", T1, SE);\n[/asy]\n\nLet $T'$ be the reflection of $T$ over $M$. Note that $TBT'C$ is a parallelogram. Additionally, as $\\angle TMX = 90^\\circ$, $\\triangle XTT'$ is isosceles, so we have $XT = XB = XT'$. Hence, $X$ is the circumcenter of $\\triangle BTT'$. Now, \n\\begin{align*}\n\t\\angle BTM - \\angle CTM &= \\angle BTT' - \\angle CTT' \\\\\n\t&= \\angle BTT' - \\angle BT'T \\\\\n\t&= \\frac{1}{2}(\\angle BXT' - \\angle BXT) \\\\\n\t&= \\frac{1}{2}((\\angle BXM + \\angle MXT) - (\\angle MXT - \\angle BXM)) \\\\\n\t&= \\frac{1}{2}(2\\angle BXM) \\\\\n\t&= \\angle BAM,\n\\end{align*}\nwhich is fixed, as desired. $\\Box$\n",
"Solution_24": "[quote=AMN300]We do not use complex numbers\n\n[hide=Solution]\nLet $N$ be the midpoint of $BT$. Since $XB=XT$, $\\angle XNT=90^{\\circ}=\\angle XMT$, so $XMNT$ is cyclic, yielding $\\angle BTM=\\angle NTM=\\angle NXM$.\nNow $MN$ is a midline in $\\triangle BCT$, so \n\\[ \\angle MTC=\\angle BTC - \\angle MTC=\\angle BNM-\\angle MXN=\\angle MXT-\\angle MXN=\\angle NXT=\\angle NXB \\]\nHence, $\\angle BTM-\\angle CTM=\\angle BXN - \\angle MXN=\\angle BXM=\\angle BAM$ which is fixed.\n[/hide] yes I think the locus of T is a circle passing through $A,M$ and midpoint of $AB$\n ;) \n\nWhen I started working on this problem, I misread it as $TB=TX$. If we characterize $T$ as such, can we find any interesting properties of this new configuration? All my progress so far for this configuration is rather trivial.[/quote]\n\n",
"Solution_25": "Dunno if this has been posted before.\n\nComplex bash with $a=-1, b=1$ so that $|m|=|x|=1$ and $c=2m-1$. Since the foot from $T$ to $\\overline{MX}$ is $M$, $$m=\\frac{1}{2}(t+m+x-mx\\overline{t}) \\implies \\overline{t}=\\frac{t-m+x}{mx}.$$ Since $\\overline{TX}=\\overline{BX}$ we should have $\\frac{t-x}{b-x}$ on the unit circle, and upon substituting our expression for $\\overline{t}$ $$\\frac{t-x}{b-x}=\\frac{\\frac{1}{b}-\\frac{1}{x}}{\\overline{t}-\\frac{1}{x}} \\implies t^2-2mt+x^2m+m-x^2.$$ Let $r$ be the unwanted root. By Vieta's, it follows that the argument of $\\angle{MTB}-\\angle{CTM}$ is (for $k \\in \\mathbb{R}$) $$\\frac{\\frac{m-t}{1-t}}{\\frac{2m-1-t}{m-t}}=k_1 \\cdot \\frac{\\frac{r-t}{1-t}}{\\frac{r-1}{r-t}}=k_1 \\cdot \\frac{(r-t)^2}{-(rt-r-t+1)} = k_1 \\cdot \\frac{4m^2-4m-4x^2m+4x^2}{-(x^2m-x^2-m+1)} = k_2 \\cdot \\frac{m-x^2}{1-x^2}$$ or $\\frac{m-x^2}{1-x^2}$. Since $x^2 \\in (ABM)$, this is equivalent to $\\angle{MXB}=\\angle{MAB}$ which does not depend on $X$, done.",
"Solution_26": "Construct $N$ the midpoint of $TB$. Since $\\overline{MN} \\parallel \\overline{TC}$, we can calculate \\begin{align*} \\angle MTB - \\angle MTC = \\angle MTN - \\angle TMN = \\angle MXN - \\angle NXB = \\angle MXB = \\angle MAB, \\end{align*} which is constant done yay kill me now\n\nmisread this problem for a good 50 minutes :D",
"Solution_27": "Solved with [b][url=https://artofproblemsolving.com/community/user/599649]MrOreoJuice[/url][/b] and [b][url=https://artofproblemsolving.com/community/user/785149]Fakesolver19[/url][/b].\n[asy]\n//meh asy, as always...\nsize(210);\npair A=(0,6.5),B=(-3,0),C=(3,0),M=(0,0),T=(-3.246,3.0711);\npair X=(1.828,1.9321), TT=(3.246,-3.0711);\ndraw(A--B--C--cycle, orange);\ndraw(circumcircle(A,B,M), orange);\ndraw(T--B--TT--C--cycle, magenta);\ndraw(X--T, red);\ndraw(X--B, red);\ndraw(X--TT, red);\ndraw(X--M, orange);\ndraw(circumcircle(B,T,TT), red);\ndraw(T--TT, magenta);\ndot(\"$A$\",A,N);\ndot(\"$B$\",B,W);\ndot(\"$C$\",C,E);\ndot(\"$M$\",M,S);\ndot(\"$T'$\",TT,S);\ndot(\"$X$\",X,NE);\ndot(\"$T$\",T,NW);\n[/asy]\nLet $T'$ be the reflection of $T$ over $M$. Note that $TBT'C$ is a parallelogram and $X$ is circumcenter of $(TBT')$.\n\\begin{align*}\n \\angle MTB - \\angle CTM &= \\angle T'TB - \\angle TT'B \\\\\n &= \\dfrac{\\angle T'XB - \\angle TXB}{2} \\\\\n &= \\dfrac{\\angle TXT' - 2\\angle TXB}{2} \\\\\n &= \\dfrac{2\\angle TXM - 2\\angle TXB}{2}\\\\\n &= \\dfrac{2\\angle BXM}{2}\\\\\n &= \\angle BAM\n\\end{align*}\nwhich does not depend on $X$ as desired. $\\blacksquare$",
"Solution_28": "Let $N$ be the midpoint of $BT$. We have that $\\angle XMT=\\angle XNT=90$ so $XMNT$ is cyclic. Also $CT\\parallel MN$ thus $\\angle CTM=\\angle TMN$. In turn $\\angle TMN=\\angle TXN=\\angle BXN$. Also $\\angle MTB=\\angle MXN$, so we can get that $$\\angle MTB-\\angle CTM= \\angle MXN-\\angle BXN = \\angle MXB=\\angle MAB$$ which doesn't depend on $X$. ",
"Solution_29": "Denote by $N$ midpoint of $BT.$ Clearly $CT\\parallel MN, XN\\perp BT,$ hence $$\\angle BTM-\\angle CTM=\\angle NTM-\\angle NMT=\\angle NXM-\\angle NXT=\\angle BXM=\\angle BAM.$$",
"Solution_30": "Let $N$ be midpoint of $TB$. $BXT$ is isosceles so $\\angle XNT = 90 = XMT$ so $XMNT$ is cyclic.\n$\\angle MTB - \\angle MTC = \\angle MXE - \\angle EXT = \\angle MXE - \\angle BXE = \\angle MXB = \\angle MAB$ and $\\angle MAB$ is fixed and independent from $X$.\nwe're Done.",
"Solution_31": "Let $N$ be the midpoint of $\\overline{BT}$ and note $MNTX$ is cyclic and $\\overline{MN}\\parallel\\overline{CT}.$ Hence, $\\angle BTM=\\angle BXM$ and $$\\angle MTC=\\angle NMB=\\angle TXN=\\angle NXB$$ so $$\\angle MTB-\\angle CTM=\\angle BXM=\\tfrac{1}{2}\\angle A.$$ $\\square$",
"Solution_32": "All angles are in degrees.\n\nLet $MT$ intersect the circle again at $U$. Note that $\\angle XBU = \\angle XMU = 90$, thus $AXBU$ is a rectangle. Since $\\angle XBM + \\angle TMB = \\angle XBM + \\angle UMB = \\angle XBM + \\angle ABX = \\angle MBA$, we have $$\\angle MTB = 180 - \\angle TBX - \\angle XBM - \\angle TMB = 180 - \\angle MBA - \\angle TBX.$$\n\nLet $N$ be the midpoint of $BT$. Notice that $\\angle XMT = \\angle XNT = 90$, so $XMNT$ is cyclic. Since $MN\\parallel CT$, we have $$\\angle MTC = \\angle TMN = \\angle TXN = 90 - \\angle XBT.$$ This implies that $\\angle BTM - \\angle MTC = 90 - \\angle MBA$, a constant value.",
"Solution_33": "[img width=50]https://media.discordapp.net/attachments/986049049834713139/986428810968825886/ISL2007G2.png[/img]Let $N$ denote the midpoint of $BT.$ Then, $\\angle TNX=\\angle TMX=90^\\circ$ so $TNMX$ is cyclic. Then, $\\angle MTB-\\angle CTM=\\angle NTM-\\angle TMN$ because $NM || TC.$ Now, $\\angle NTM-\\angle TMN=\\angle NXM-\\angle TXN=\\angle BXM=\\angle BAM$ which is fixed so we're done.",
"Solution_34": "Let $T_1$ be the reflection of $T$ in $M$, ray $MT$ meet $(ABM)$ again at $Y$, and the circle centered at $X$ with radius $XB = XT$ meet $(ABM)$ again at $Q$.\n\nBecause $XM \\perp TT_1$, we know $XT = XT_1$, so $BTQT_1$ is cyclic. Now, observe $$\\angle YBX = \\angle YMX = \\angle TMX = 90^{\\circ}$$ which implies $YB$ is tangent to $(BTQT_1)$. Similarly, we deduce that $YQ$ is tangent to $(BTQT_1)$, so $\\overline{TT_1Y}$ is the $T$-Symmedian of $BQT$. \n\nNow, since $M$ is the midpoint of $TT_1$, we know it's the $T$-Dumpty point of $BQT$, yielding $MBT \\overset{+}{\\sim} MTQ$. Thus, because $BTCT_1$ is clearly a parallelogram, $$\\angle MTB - \\angle CTM = \\angle MQT - \\angle BT_1T$$ $$= \\angle MQT - \\angle BQT = \\angle MQB = \\angle MAB$$ which finishes. $\\blacksquare$\n\n\n[b]Config Stuff:[/b] Let ray $XT$ meet $(ABM)$ again at $P$. The Incenter-Excenter Lemma implies $T$ is the incenter of $BPQ$. Moreover, since $\\angle XMT = 90^{\\circ}$, we know the $P$-Mixtilinear Incircle of $BPQ$ touches $(BPQ)$ at $M$. In fact, $MBT \\overset{+}{\\sim} MTQ$ is actually a well-known mixtilinear lemma.",
"Solution_35": "Let $D$ be the midpoint of $BT$. So $TDMX$ is cyclic and $MD\\parallel CT$. So the desired angle quantity is $\\angle BAM$ which is fixed.",
"Solution_36": "Let $E = \\overline{TM} \\cap (X)$. Notice that $\\angle MTB - \\angle CTM = \\angle MTB - \\angle BET = \\frac 12(\\widehat{BE} - \\widehat{TB})$ because $TCEB$ is a parallelogram. However, setting $D = \\overline{XM} \\cap (X)$ as the arc midpoint of $\\widehat{TE}$, we have $$\\frac 12(\\widehat{BE} - \\widehat{TB})= \\widehat{BD} = \\angle BXM = \\angle BAM$$ is fixed regardless of $X$, as needed.",
"Solution_37": "time to go through my writeup list :P\n\nLet $N$ be the midpoint of $BT$, with $XMNT$ cyclic (since $\\angle TNX=\\angle TMX=90^\\circ$) and $MN \\parallel TC$ (since $MN$ is the $B$-midsegment in $\\triangle BTC$). Now we angle chase:\n\\[\\angle MTC=\\angle NMT=\\angle TXN=\\angle NXB\\]\nand $\\angle BTM=\\angle NXM$, so we are done since $\\angle BTM - \\angle MTC = \\angle BXM = 90 - \\angle ACM$. $\\square$",
"Solution_38": "[quote]When I started working on this problem, I misread it as $TB=TX$. If we characterize $T$ as such, can we find any interesting properties of this new configuration? All my progress so far for this configuration is rather trivial.[/quote]\nLOL i made the exact same mistake on ggb, even though i READ TX=BX I didn't construct it properly and constructed it as on the circle with radius BX but with center B, so i was like \"why is this not working\"\n\nMy solution is the same as above, but without typos: @above I think it should be MTC=NMT=..., and it should also be BTM=NXM, hence BTM-MTC=BXM=BAM, which is independent.",
"Solution_39": "Let $\\overline{MT}$ intersect $(ABM)$ again at $X'$ (the $X$-antipode) and the circle $\\omega$ centered at $X$ passing through $B$ and $T$ at $D$. Then since $\\overline{XM} \\perp \\overline{TD}$, $M$ is the midpoint of chord $\\overline{DT}$, hence $BDCT$ is a parallelogram. On the other hand, since $\\angle X'BX=90^\\circ$, $\\overline{X'B}$ is tangent to $\\omega$, hence $\\angle X'BT=\\angle BDT=\\angle CTM$, hence $\\angle MTB-\\angle CTM=\\angle BX'M=\\angle BAM$ which is fixed. $\\blacksquare$",
"Solution_40": "Let $S$ be such that $SBTC$ is a parallelogram. Then\n\\[2(\\angle MTB-\\angle CTM)=\\angle BXS-\\angle BTX=(\\angle BXM+\\angle MXS)-(\\angle MXT-\\angle BXM)=2\\angle BXM=2\\angle BAM\\]\ndone. ",
"Solution_41": "Let Omega be circle with center X and radius XB.\nLet TM intersect Omega at Y.\nLet P be point on Omega(Arc TBY) such that Angle PTY=CTY\nAfter that observe TY||BP (Angle chase for this)\nWe are done"
}
{
"Tag": [
"combinatorics proposed",
"combinatorics"
],
"Problem": "Suppose that there are n people, and they each have their favourite numbers between 1 and n. Person #1 starts at 1 and moves towards n, and takes his favourite number. Person #2,... repeats the steps. If his favourite number is already taken, then he would move on and take the next available number. However, if all the numbers between the person's favourite number and n are taken, the person gets angry and leaves. Find the number of pairs of the (person, favourite number) so that no one gets angry and every number gets taken.\r\n\r\n[Edit] Note that, two people may have the same favourite number. For instance, the arrangement (n, f(n)), where f(n) is the favourite number of nth person, may be (1,5), (2,5), ..., (6,5), (7,5) (assuming n=10). Then this arrangement is not possible, because the 2nd person would take 6, 3rd would take 7, etc., and 6th taking 10. Then the 7th person has nothing to take, so he leaves for good.\r\n\r\nI hope I have made the problem more understandable.",
"Solution_1": "I've got $(n+1)^{n-1}$.",
"Solution_2": "Sorry, I haven't been checking on my problems lately.\r\n\r\nMaxal, do you want to share your solution? The reason I posted this problem is because it has an ingenious way of solving it, which I can't remember anymore (maybe only vaguely).\r\n\r\nAs soon as I remember mine, I'll post it as well, in maybe a week or so, so that others can try it as well."
}
{
"Tag": [],
"Problem": "A wooden raft (density=775 kg/m^3) is supporting a 72.7 kg man entirely above the water when he stands on it at Lake Travis. Find the minimum volume of the raft.",
"Solution_1": "I'm so happy to finally see an easy problem! :)"
}
{
"Tag": [],
"Problem": "It might be a little early for me to be thinking about this, but should I go straight to grad school after college? Is it all right to work for maybe 2 years, and then apply for grad? Or will it be more difficult that way?",
"Solution_1": "I have read some graduate stats for schools in the ivy league and such and it seems that quite a few people take time off before continuing on to graduate school. I would assume this is the same throughout most schools. What are you studying, by the way?",
"Solution_2": "I might be doing Physics and CS... \r\nCollege will (hopefully) determine which field I want to go into, though. I'm very indecisive.",
"Solution_3": "You have (at least) 4 years -- don't worry about it yet."
}
{
"Tag": [],
"Problem": "[quote=\"Vutang\"]Given $a, b, c$ are three real numbers such that : $a 2$\n$a+b=6-c$\n$ab+c(a+b)=9$ we see that $a,b$ satisfy\n$a+b=6-c$ and $ab=(c-3)^2$ so $a$ and $b$ are the roots of $f(x)=x^2+(c-6)x+(c-3)^2=0$ the discriminant is $D=-3c(c-4)$ $D>0$ when $c<4$\n$c$ is outside the roots of $f(x)=0$ so $f(c)>0$ but $f(c)=3(c-1)(c-3)$ so $c>3$\nAs $f(1)=(c-1)(c-4)$ we see that $f(1)<0$ so $a<10$ and $b>1$ we conclude that $a>0$ and as $a+c>3$ $b<3$\n[/hide]"
}
{
"Tag": [
"induction"
],
"Problem": "For each positive integer $k>1$, define the sequence ${a_{n}}$ by\r\n$a_{0}=1$ and $a_{n}=kn+(-1)^{n}a_{n-1}$ for each $n \\geq 1$.\r\n\r\nDetermine all values of $k$ for which 2000 is a term of the sequence.",
"Solution_1": "Is [hide]$\\ k=3,23,667,2001$[/hide] correct?",
"Solution_2": "I think that [hide=\"k can also equal\"]87[/hide] :maybe:",
"Solution_3": "You're right. I'll outline my solution:\r\n[hide]So you can prove by induction or whatever that if \n$\\ n \\equiv 0 \\mod 4 \\rightarrow a_{n}=nk+1$\n$\\ n \\equiv 1 \\mod 4 \\rightarrow a_{n}=k-1$\n$\\ n \\equiv 2 \\mod 4 \\rightarrow a_{n}= (n+1)k-1$\n$\\ n \\equiv 3 \\mod 4 \\rightarrow a_{n}= 1$\n\nSo, clearly when $\\ n \\equiv 3 \\mod 4$ $\\ a_{n}$ can never be 2001. \nAlso, if $\\ n \\equiv 0 \\mod 4$ then $\\ 2000=4mk+1$ however $\\ 4$ does not divide $\\ 1999$ so $\\ 2000$ will not appear when $\\ n$ is divisible by $\\ 4$.\nSo, when $\\ n \\equiv 1 \\mod 4$ then $\\ a_{n}= k-1$ so if $\\ k=2001$ $\\ 2000$ will appear in the series.\nWhen $\\ n$ is in the form $\\ 4m+3$ then we can right $\\ 2000=(4m+3)k-1$ or $\\ 2001=(4m+3)k$. Now the factors of $\\ 2001$ which appear in the form $\\ 4m+3$ are $\\ 3, 23, 87, 667$ and the corresponding $\\ k$ values are $\\ 667, 87, 23, 3$\nSo, $\\ k= 3, 23, 87, 667, 2001$\n [/hide]"
}
{
"Tag": [
"function",
"calculus",
"algebra",
"domain",
"linear algebra",
"calculus computations"
],
"Problem": "Let g: A - B, f: B - C, f o g: A - C be functions.\r\nIf f and f o g are one to one, does it follow that g is one to one?",
"Solution_1": "Suppose $g$ is not 1-1. Then there exists $x,y\\in A$ such that $g(x)=c=g(y)$ for some $c\\in B$. So $f(g(x))=f(c)$ and $f(g(y))=f(c)$, but this is a contradiction since $f\\circ g$ is 1-1. Therefore $g$ is 1-1.\r\n\r\n...I didn't use $f$ is 1-1. Do you only need $f\\circ g$ to be 1-1 to get $g$ 1-1?",
"Solution_2": "Yes, you only need $f \\circ g$ to be injective. \r\nProof: choose $a_{1}, a_{2}\\in A$. Suppose $g(a_{1}) = g(a_{2})$, we wish to show that $a_{1}= a_{2}$. Applying f to $g(a_{1}) = g(a_{2})$, and use the fact that this composition is injective, we get $a_{1}= a_{2}$, as it was required. So you did not need f to be injective.",
"Solution_3": "This is clearly not calculus. This is elementary \"functionspeak.\" Problems of this ilk also sometimes show up in the linear algebra forum. I'd move them, except I'm stumped as to where to move them to.",
"Solution_4": "At my university at least, the first time you would see this material and this terminology would be in a second year analysis course (it is also covered in a third-year abstract algebra course).\r\n\r\nI think it's fine where it is?",
"Solution_5": "[quote=\"blahblahblah\"]At my university at least, the first time you would see this material and this terminology would be in a second year analysis course (it is also covered in a third-year abstract algebra course).[/quote]\r\nAn abstract algebra course is a bit late in the game for this. I'm using some of this language right now teaching a (college sophomore level) linear algebra course; we also quite explicitly deal with it in a sophomore level \"transition\" course (discrete structures/introduction to proof and abstraction.) Some questions one might ask - say about counting certain kinds of functions between finite sets - would properly belong to the combinatorics forum here. Ideally, students should come to a basic understanding of this language, and be able to write short proofs and construct counterexamples, before they even start calculus - but in the real world, that's a lot to ask.\r\n\r\nIt's really a matter of vocabulary: relation, function, domain, codomain, range, injective (1-1), surjective (onto), composition, identity map, inverse function."
}
{
"Tag": [
"LaTeX",
"number theory proposed",
"number theory"
],
"Problem": "Let $ k_1,k_2,...,$k_304 be the positive integers satisfying $ k_1^3\\plus{}k_2^3\\plus{}...\\plus{}$(k_304)^3\u22642003.\r\nFinh the largest value of A=$ k_1\\plus{}k_2\\plus{}k_3\\plus{}...\\plus{}$k_ 304",
"Solution_1": ":blush: The answer is 304*3\u221a2003/304",
"Solution_2": "[quote=\"New player\"]:blush: The answer is 304*3\u221a2003/304[/quote]\r\nNew player, use $ LaTeX$\r\nAlso $ k_1, k_2, \\dots \\dots ,k_{304}$ are positive integers."
}
{
"Tag": [
"combinatorics unsolved",
"combinatorics"
],
"Problem": "Each of seven boys went to a cafe 3 times, such that every pair met there. Prove that at some moment 3 of the boys were at the cafe.",
"Solution_1": "Take boy 1.\r\nHe meets the 6 other boys in his 3 visits to the cafe.\r\nSo in at least one of his visits, he meets more than one of the boys. \r\nAnd at that time, there are 3 boys at the cafe. \r\n\r\nOr have i misread the question? Do you mean [b]exactly[/b] three boys in the cafe?",
"Solution_2": "You understood the problem well, but the thing is, I thought of the same solution and it's wrong. Namely, while one of the 3 boys is in the cafe, one of the other 2 might have come and left before the third one arrived.\r\n______ ___________\r\n ____________ Here it is visually, reperesented on time line, where easch of these 3 lines represents the time each of the three boys were present.",
"Solution_3": "Consider the intervals when boys are in cafe as line-segments on a line. Consider the 21 left ends of this segments. If two boys meet they also meet in some of this 21 points(not hard to prove). If some of this points coincide then we may think that there are 20 points with this condition. If not take the leftmost point. Obviously there is no meeting there so again we may think the points are 20. But there are at least $ \\binom{7}{2}= 21$ meetings so some two meetings coincide, i.e. some 3 boys are in the cafe at the same time."
}
{
"Tag": [
"summer program",
"PROMYS",
"Ross Mathematics Program"
],
"Problem": "Ive got into both PROMYS, SSP, and the ROSS (unfortunately no RSI).\r\nwhich one should it be?",
"Solution_1": "[quote=\"xxreddevilzxx\"]Ive got into [b]both[/b] PROMYS, SSP, and the ROSS (unfortunately no RSI).\nwhich one should it be?[/quote] \r\n\r\n :P \r\n\r\nPROMYS and ROSS are more math related than SSP, ( i think ;) ), \r\n\r\ngo to the one that gives you more money, so you have to pay less :lol:",
"Solution_2": "where are you going?",
"Solution_3": "Well Qiaochu will be at PROMYS, Edward at Ross, so maybe you'll see one of them there.",
"Solution_4": "well im still undecided to attend PROMYS or ROSS.\r\ni was wondering where Young will go?",
"Solution_5": "Young is going to PROMYS I believe. They've offered him a good chunk of money.\r\n\r\nYeah...I'm going somewhere else. You Washingtonians have fun!!! And say no to drugs! :ninja:",
"Solution_6": "Actually i might be going somewhere else. My schedule just totally changed 360 degrees due to some personal reasons.",
"Solution_7": "[quote=\"xxreddevilzxx\"]somewhere else. [/quote]\r\n\r\nas in? :P",
"Solution_8": "Ewww... I don't want anybody like Young or Amir to be at Ross with me. They're just icky. :P And we all know that QC's a loser for choosing PROMYS over SIMUW, since SIMUW rules over all else. :D",
"Solution_9": "[quote=\"Uh oh!\"]Ewww... I don't want anybody like Young or Amir to be at Ross with me. :P [/quote]\r\n\r\nerr...you're the one who made me apply in the first place!! :D"
}
{
"Tag": [
"induction",
"inequalities",
"strong induction",
"number theory unsolved",
"number theory"
],
"Problem": "the problem like following is far from my ability maybe masters would help :) \r\nPut $a_k=p_k^2-p_k,k\\geq 2$.($p_k$ is kth prime number)then there exists an absolute constant $A$ so that every even integer greater than $A$ is the sum of distinct $a_k$'s.",
"Solution_1": "Wow, that's too beautiful! Thanks for the dedication, but this problem seems far too much for my poor head.",
"Solution_2": "Here's a way to reduce the problem to a finite problem. Suppose we can find an even integer $A$ and an integer $k_0 \\ge 2$ with the following properties:\r\n\r\n(1) $A+2$, $A+4$, $A+6$, ..., $A + a_{k_0}$ are representable (as the sum of distinct $a_k$'s).\r\n\r\n(2) $2 a_k > a_{k+1} + A$ for every integer $k \\ge k_0$. (Presumably we can prove such a bound using the Prime Number Theorem or something similar.)\r\n\r\nThen I claim that every even integer $n$ bigger than $A$ is representable. The proof is by strong induction on $n$. The cases $n \\le a_{k_0} + A$ follow from property 1. So assume $n > a_{k_0} + A$. Let $j$ be the maximum integer such that $a_j + A < n$. We know that $j \\ge k_0$. By Property 2 and maximality, we have\r\n\\[\r\n 2 a_j > a_{j+1} + A \\ge n \\, .\r\n\\]\r\nBy induction, $n - a_j$ is representable. Because $2 a_j > n$, the representation of $n - a_j$ cannot use $a_j$. Thus $n = (n - a_j) + a_j$ is representable. That completes the inductive proof.\r\n\r\nAlas, I haven't done the computations needed to find $A$ and $k_0$.",
"Solution_3": "Okay, I did some computations on an Excel spreadsheet. The choice $A = 3106$ and $k_0 = 19$ seems to work. We have $p_{k_0} = 67$ and $a_{k_0} = 4422$. The spreadsheet says that 3108, 3110, 3112, ..., 7528 are representable, so Property 1 is true. The inequality $2 a_k > a_{k+1} + 3106$ is true for the first few $k \\ge 19$, and can presumably be shown for all $k \\ge 19$ because $p_{k+1} / p_k$ (and thus $a_{k+1}/a_k$) approaches 1.\r\n\r\nEdit: I found a way to weaken Property 2 (at the expense of strengthening Property 1). That should make it easier to prove Property 2. Here are the new properties.\r\n\r\n(1') The numbers $A+2$, $A+4$, $A+6$, $A + a_{k_0}$ are representable with the numbers $a_2$, ..., $a_{k_0 - 1}$.\r\n\r\n(2') $a_{k+1} \\le 2 a_k$ for all $k \\ge k_0$.\r\n\r\nAccording to my spreadsheet, the new Property 1' is true for $A = 3106$ and $k_0 = 17$. So we just need to prove Property 2' that $a_{k+1} \\le 2 a_k$ for all $k \\ge 17$. In fact, the inequality seems to be true for all $k \\ge 5$. We can probably prove it by using inequalities such as $p_{k+1} \\le 1.3 p_k$ for all sufficiently large $k$."
}
{
"Tag": [
"function",
"algebra",
"floor function",
"number theory open",
"number theory"
],
"Problem": "If $ x\\in R$ then find all solution of the given floor - function $ [\\frac {2x \\minus{} 1}{3}] \\plus{} [\\frac {4x \\plus{} 1}{6}] \\equal{} \\frac {5x \\minus{} 4}{3}$",
"Solution_1": "[quote=\"ertanrock\"]If $ x\\in R$ then find all solution of the given floor - function $ [\\frac {2x \\minus{} 1}{3}] \\plus{} [\\frac {4x \\plus{} 1}{6}] \\equal{} \\frac {5x \\minus{} 4}{3}$[/quote]\r\n\r\n$ 5x\\equal{}3 ([\\frac {2x \\minus{} 1}{3}] \\plus{} [\\frac {4x \\plus{} 1}{6}] ) \\plus{}4\\equal{}a \\in \\mathbb{Z} \\Rightarrow x\\equal{}\\frac{a}{5}$\r\n$ a\\equal{}3 ([\\frac {4a\\minus{}10}{30}] \\plus{} [\\frac {4a\\plus{}5}{30}]) \\plus{}4$\r\n\r\n$ a\\equal{}30m\\plus{}n$ with $ 0 \\le n < 30$\r\n$ 30m\\plus{}n\\equal{}3 ([\\frac {4(30m\\plus{}n)\\minus{}10}{30}] \\plus{} [\\frac {4(30m\\plus{}n)\\plus{}5}{30}]) \\plus{}4$\r\n$ 30m\\plus{}n\\equal{}24m\\plus{}3 ([\\frac {4n\\minus{}10}{30}] \\plus{} [\\frac {4n\\plus{}5}{30}]) \\plus{}4$\r\n$ 6m\\plus{}n\\equal{}3 ([\\frac {4n\\minus{}10}{30}] \\plus{} [\\frac {4n\\plus{}5}{30}]) \\plus{}4$\r\ntaking mod $ 3$ then $ n \\equiv 1 (3)$ therefore only solutions for $ n \\in \\{1,4,7, 10,28 \\}$ \r\nThus $ x \\in \\{\\frac{1}{5},\\frac{4}{5},\\frac{7}{5},2,\\minus{}\\frac{2}{5} \\}$",
"Solution_2": "[quote=\"mszew\"][quote=\"ertanrock\"]If $ x\\in R$ then find all solution of the given floor - function $ [\\frac {2x \\minus{} 1}{3}] \\plus{} [\\frac {4x \\plus{} 1}{6}] \\equal{} \\frac {5x \\minus{} 4}{3}$[/quote]\n\n$ 5x \\equal{} 3 ([\\frac {2x \\minus{} 1}{3}] \\plus{} [\\frac {4x \\plus{} 1}{6}] ) \\plus{} 4 \\equal{} a \\in \\mathbb{Z} \\Rightarrow x \\equal{} \\frac {a}{5}$\n$ a \\equal{} 3 ([\\frac {4a \\minus{} 10}{30}] \\plus{} [\\frac {4a \\plus{} 5}{30}]) \\plus{} 4$\n\n$ a \\equal{} 30m \\plus{} n$ with $ 0 \\le n < 30$\n$ 30m \\plus{} n \\equal{} 3 ([\\frac {4(30m \\plus{} n) \\minus{} 10}{30}] \\plus{} [\\frac {4(30m \\plus{} n) \\plus{} 5}{30}]) \\plus{} 4$\n$ 30m \\plus{} n \\equal{} 24m \\plus{} 3 ([\\frac {4n \\minus{} 10}{30}] \\plus{} [\\frac {4n \\plus{} 5}{30}]) \\plus{} 4$\n$ 6m \\plus{} n \\equal{} 3 ([\\frac {4n \\minus{} 10}{30}] \\plus{} [\\frac {4n \\plus{} 5}{30}]) \\plus{} 4$\ntaking mod $ 3$ then $ n \\equiv 1 (3)$ therefore only solutions for $ n \\in \\{1,4,7, 10,28 \\}$ \nThus $ x \\in \\{\\frac {1}{5},\\frac {4}{5},\\frac {7}{5},2, \\minus{} \\frac {2}{5} \\}$[/quote] great solution."
}
{
"Tag": [],
"Problem": "I'm taking Sequences and Series at nationals, so I was studying and I've found that certain number sequences appear like Pentagonal numbers, Catalan numbers, and Lucas numbers. If someone knows offhand several types of numbers and the formulas to find them, it'd be greatly appreciated. Or at least compiling a list of different types of numbers",
"Solution_1": "Try searching on [url=http://mathworld.wolfram.com/]MathWorld[/url]."
}
{
"Tag": [],
"Problem": "Topsoil costs $ \\$6$ per cubic foot. What is the cost, in dollars, of\n5 cubic yards of topsoil?",
"Solution_1": "Well there are 3x3x3=27 cubic feet in a cubic yard. Thus there are 5x27=135 cubic feet, so its 6x135=810."
}
{
"Tag": [
"geometry",
"trigonometry",
"function",
"algebra",
"polynomial",
"quadratics",
"analytic geometry"
],
"Problem": "Hey guys,\r\ni'm 8th grade and about preparing for the euclid,\r\ni have these books:\r\n\"AoPS 2\"\r\n\"Challenging Problems in Geometry\"\r\nAlfred S. Posamentier\r\n\"Contest Problem Books (ahsme) 1-4\"\r\nHow to Solve It\"\r\nG. Polya\r\n\"Five Hundred Mathematical Challenges\"\r\nEdward J. Barbeau\r\n\"Mathematics of Choice\"\r\nIvan Morton Niven\r\n\"Introduction to Geometry\"\r\nCoxeter\r\n\r\nAre they helpful?\r\n\r\nAnd of course i did the old tests and my marks ranged from 90-100 and an 86. (I know on the real contest, i'll do crap :( )\r\n \r\nAnyway, thanks for helping :)",
"Solution_1": "i'm sure thats enough..lol..u'll probably do fine if u can tackle most of the questions in past euclid contests",
"Solution_2": "especially since youre in grade 8!",
"Solution_3": "I found out that I can write the euclid as well as the hypatia :D\r\n\r\nBut what would you guys think the best topics to review would be, theorems that kinda thing?",
"Solution_4": "For Euclid? This is the official list of topics (and it's pretty thorough IMO):\r\n\r\n-Euclidean and Analytic Geometry \r\n-Trigonometry, including functions, graphs, identities, sine and cosine laws\r\n-Exponential and logarithmic functions\r\n-Functional notation\r\n-Systems of equations\r\n-Polynomials, including relationships involving the roots of quadratic and cubic equations, the remainder theorem.\r\n-Sequences and series\r\n-Simple counting problems\r\n-Properties of numbers\r\n\r\nBasically anything you'd see in the high school basics forum extending into a bit of intermediate forum stuff.",
"Solution_5": "[quote=\"max_tm\"]For Euclid? This is the official list of topics (and it's pretty thorough IMO):\n\n-Euclidean and Analytic Geometry \n-Trigonometry, including functions, graphs, identities, sine and cosine laws\n-Exponential and logarithmic functions\n-Functional notation\n-Systems of equations\n-Polynomials, including relationships involving the roots of quadratic and cubic equations, the remainder theorem.\n-Sequences and series\n-Simple counting problems\n-Properties of numbers\n\nBasically anything you'd see in the high school basics forum extending into a bit of intermediate forum stuff.[/quote]\r\ni dont get anything by looking at the topics :rotfl:",
"Solution_6": "i already knew the topics and saw through old contests and most of the difficulty of the topics were kind of basic.\r\n\r\nthanks anyway"
}
{
"Tag": [],
"Problem": "A clock chimes two times 15 minutes after the hour, four times 30 minutes after the hour and 6 times 45 minutes after the hour. The clock also chimes eight times on each hour in addition to chiming the number of times equal to the hour. (So at 2:00 p.m. the clock chimes 8+2=10 times). Starting at 12:05 a.m., how many times does the clock chime in a 24 hour period?",
"Solution_1": "[quote=\"myspacesucks\"]A clock chimes two times 15 minutes after the hour, four times 30 minutes after the hour and 6 times 45 minutes after the hour. The clock also chimes eight times on each hour in addition to chiming the number of times equal to the hour. (So at 2:00 p.m. the clock chimes 8+2=10 times). Starting at 12:05 a.m., how many times does the clock chime in a 24 hour period?[/quote]\r\n[hide=\"There is probably a shorter way...\"]For every hour (x:00-x:59), the number of chimes is $2+4+6+8+x=20+x$\nThen you solve for each value (1-12)\nThe values are then 21,22, 23, 24, 25, 26, 27, 28, 29, 30, 31, 32\n$21+22+23+24+25+26+27+28+29+30+31+32=6(21+32)=6(53)$\nMultiply by 2 for am/pm to get a final answer of $318*2=\\boxed{636}$[/hide]\r\nThat's an annoying clock!",
"Solution_2": "[quote=\"Quevvy\"][quote=\"myspacesucks\"]A clock chimes two times 15 minutes after the hour, four times 30 minutes after the hour and 6 times 45 minutes after the hour. The clock also chimes eight times on each hour in addition to chiming the number of times equal to the hour. (So at 2:00 p.m. the clock chimes 8+2=10 times). Starting at 12:05 a.m., how many times does the clock chime in a 24 hour period?[/quote]\n[hide=\"There is probably a shorter way...\"]For every hour (x:00-x:59), the number of chimes is $2+4+6+8+x=20+x$\nThen you solve for each value (1-12)\nThe values are then 21,22, 23, 24, 25, 26, 27, 28, 29, 30, 31, 32\n$21+22+23+24+25+26+27+28+29+30+31+32=6(21+32)=6(53)$\nMultiply by 2 for am/pm to get a final answer of $318*2=\\boxed{636}$[/hide]\nThat's an annoying clock![/quote]\n[hide=\"I have a much easier way\"]Each hour, excluding the clock chiming for the number of hours in the time, there are $2+4+6+8=20$ chimes. Since there are $24$ hours, there are $480$ chimes. Next we account for the clock chiming for the number of hours in the time. It is not hard to check that it is: $2(1+2+...+12)=156$. Add these two up to get a final answer of $636$. [/hide]",
"Solution_3": "You do realize that your ways are pretty much the same, right?",
"Solution_4": "[quote=\"Ubemaya\"]You do realize that your ways are pretty much the same, right?[/quote]\r\n\r\nI'm just curious, but: Ubemaya, which method would you use?\r\n\r\nI realize that they are practically the same ways, but I'm more of a \"casework\" kind of person.",
"Solution_5": "Uhh... well if I didn't read your solutions I probably couldn't have solved this.",
"Solution_6": "You guys are right, the answer is 636, Vishalarul, your method is pretty much the same with a minute difference in the middle when you add them together. Nice job you guys."
}
{
"Tag": [
"AMC",
"USA(J)MO",
"USAMO",
"email",
"AIME",
"AMC 10",
"MATHCOUNTS"
],
"Problem": "I write to correct a number of false statements and incorrect\r\nunderstandings in this forum about the time it takes the AMC office\r\nto complete the USAMO selection process.\r\n\r\nIn the 2009 USAMO Teachers' Manual, on page 4, Section I, paragraph 6\r\nit says: \"The selection of the USAMO participants will be made on or\r\nbefore April 10th. All students ... will receive information...no\r\nlater than April 10.\" We published the USAMO invitation list and sent\r\nthe email invitations on April 10. We met our announced date. We\r\nwere not late or delayed in making the invitations.\r\n\r\nThe 2009 USAMO Teachers' Manual was written and printed months ago.\r\nAfter more than 30 years of administering the USAMO, we know our\r\nbusiness and we know how long it takes to complete our business. We\r\nmet our announced date. We were not late or delayed in making our\r\ninvitations. \r\n\r\nThere are two primary reasons that it takes us approximately 9 days\r\nfrom the alternate AIME date to the announcement of USAMO\r\ninvitations.\r\n (1) We do not receive the AIME II score sheets from some schools \r\n until as late as 1 week after the AIME II. On Friday, April 3\r\n we had received from about 60% of participating\r\n schools. Even on Wednesday, April 8 the AMC office still\r\n received AIME score sheets from some schools. This is in spite\r\n of requiring schools to send by some kind of express mail or\r\n package delivery. In some cases, it still takes 5 calendar days\r\n or more for the answer forms to reach us.\r\n\r\n (2) Second, and far more time consuming for us is that fact that\r\n many students incorrectly fill out the answer sheets. The most\r\n common error is to not use the same name when taking the AMC\r\n contest and the AIME contest, or when taking the AMC contests on\r\n both the A-date and the B-date. If the student uses the same\r\n name each time, our scoring processes automatically match the\r\n names. If a student did not use the same name, or incorrectly\r\n filled out the answer from, we have to laboriously match the\r\n student names and scores. This is tedious and time-consuming,\r\n particularly if the students took the contest at different\r\n locations. If students correctly and consistently filled out\r\n the answer forms, we might be able to save 2 to 3 days on the\r\n selection process.\r\n\r\n (3) A third time consuming effort is when students take the AMC 10\r\n or AMC 12 at more than one location, but request that scores and\r\n affiliations be reported from, say, their high school. It takes a\r\n lot of searching, matching, cross-checking and validation to make\r\n sure that we have reported the scores correctly.\r\n\r\nOne potential solution to the second and third problems would be to allow\r\nstudents to take only one AMC contest on the A-date or the B-date but\r\nnot both. Of course, a simpler solution would be for all students to\r\nconsistently and correctly enter their information on the answer\r\nforms. The AMC office continues to explore technological solutions\r\nto the first problem that would easily adapt to all schools nationally\r\nand internationally. \r\n\r\nSteve Dunbar\r\nMAA Director, American Mathematics Competitions",
"Solution_1": "Thanks for the time and effort you spend responding to concerns on this forum.",
"Solution_2": "Yes, I agree with worthawholebean, thank you.",
"Solution_3": "And thanks for all the effort on sorting out tests and making sure that the results were on time. Just a thought, but maybe the AMC office was too nice last year with the preliminary list? :D",
"Solution_4": "Thank you for your efforts. Sometimes people are just a bit eager to find out whether they made USAMO =p but we appreciate your work.",
"Solution_5": "[quote](3) A third time consuming effort is when students take the AMC 10\nor AMC 12 at more than one location, but request that scores and\naffiliations be reported from, say, their high school. It takes a\nlot of searching, matching, cross-checking and validation to make\nsure that we have reported the scores correctly. [/quote]\r\nI'm sure this is a massive problem and a thankless one to try to overcome. And very, very, very hard to get everyone on the first pass. Do not be surprised if the list of USAMO qualifiers grows slightly as more multiple-site names are matched up. (In fact, I personally know of one such case, and I will inform the AMC office of what I know.)",
"Solution_6": "[quote=\"AMCDirector\"]\n (2) Second, and far more time consuming for us is that fact that\n many students incorrectly fill out the answer sheets. The most\n common error is to not use the same name when taking the AMC\n contest and the AIME contest, or when taking the AMC contests on\n both the A-date and the B-date. If the student uses the same\n name each time, our scoring processes automatically match the\n names. If a student did not use the same name, or incorrectly\n filled out the answer from, we have to laboriously match the\n student names and scores. This is tedious and time-consuming,\n particularly if the students took the contest at different\n locations. If students correctly and consistently filled out\n the answer forms, we might be able to save 2 to 3 days on the\n selection process.\n\n (3) A third time consuming effort is when students take the AMC 10\n or AMC 12 at more than one location, but request that scores and\n affiliations be reported from, say, their high school. It takes a\n lot of searching, matching, cross-checking and validation to make\n sure that we have reported the scores correctly.\n[/quote]\r\n\r\nDear AMCDirector,\r\nHave you thought about assigning an unique number for each student (like using SSN)? I know many students took AMC 10 / AMC 12 at more than one location, unique number for each student may help to identify the correct score for each student.",
"Solution_7": "[quote=\"AMCDirector\"]One potential solution to the second and third problems would be to allow\nstudents to take only one AMC contest on the A-date or the B-date but\nnot both. [/quote]\r\n\r\nA better solution is to modifying the answer sheets for students to write in their CEEB# and scores earned at the specific CEEB location:\r\n\r\nAMC 10/12A: CEEB#, AMC10 or AMC 12, your score\r\nAMC 10/12B: CEEB#, AMC 10 or AMC 12, your score\r\n\r\nIt will make the automatic matching easier.",
"Solution_8": "[quote=\"shtsxc12\"][quote=\"AMCDirector\"]\nDear AMCDirector,\nHave you thought about assigning an unique number for each student (like using SSN)? I know many students took AMC 10 / AMC 12 at more than one location, unique number for each student may help to identify the correct score for each student.[/quote][/quote]\r\n\r\nThat will not work as AMC/AIME exams are international. Students in China, Taiwan, Singapore, etc. do not have SSN!",
"Solution_9": "[quote=\"zmli\"]A better solution is to modifying the answer sheets for students to write in their CEEB# and scores earned at the specific CEEB location:\n\nAMC 10/12A: CEEB#, AMC10 or AMC 12, your score\nAMC 10/12B: CEEB#, AMC 10 or AMC 12, your score\n\nIt will make the automatic matching easier.[/quote]\r\n\r\nProblem is, people from International locations don't have a CEEB. (Or, could you explain what a CEEB is, if I'm wrong?)",
"Solution_10": "[quote=\"zmli\"][quote=\"shtsxc12\"][quote=\"AMCDirector\"]\nDear AMCDirector,\nHave you thought about assigning an unique number for each student (like using SSN)? I know many students took AMC 10 / AMC 12 at more than one location, unique number for each student may help to identify the correct score for each student.[/quote]\n\nThat will not work as AMC/AIME exams are international. Students in China, Taiwan, Singapore, etc. do not have SSN![/quote][/quote]\r\n\r\nI think he means a special identification number just for AMC, not the actual SSN itself.",
"Solution_11": "[quote=\"not_trig\"]I think he means a special identification number just for AMC, not the actual SSN itself.[/quote]\r\n\r\nSo something along the lines of what they do with USAMO student numbers? The problem is that the numbers would have to be centrally managed, and that takes a long time to administer. An [i]extremely[/i] long time.",
"Solution_12": "The problems with an AMC-assigned special identification number:\r\n\r\nIt would have to have enough digits to distinguish several hundred thousand individuals - so it would have to be a 6 or 7 digit number, not the easiest thing to remember.\r\n\r\nThe onus would be on the individual student to remember/bring that number to the B date. There would have to be a mechanism for giving each student a piece of paper or sticker with that number on it (extra printing cost), and even if you did that, many hundreds of students would forget to bring that sticker to the B exam, in which case you're no better off than before, perhaps worst off.\r\n\r\nThere are a dozen different reasons why the AMC shouldn't and won't use Social Security numbers. I won't elaborate, other than to say this is a complete non-starter.\r\n\r\nThe only scheme I can see with a slight amount of hope: phone numbers. Put a place for a 10 - digit number, and ask student to fill in a number of their own choosing that they will remember, with the suggestion that it be a phone number (but the student is allowed to invent some other number). There would be some duplicates, both among siblings and among people that invented numbers, and there would still be students forgetting which number it was they had used the first time.\r\n\r\nI still think that there really isn't a better scheme than name-matching with address-matching as a backup. Unless the AMC wants to prohibit students from taking more than one AMC, or from taking AMC and AIME at different locations (which actions would ignite their own protests), then this problem is going to be there.",
"Solution_13": "Personally, I really like the idea of restricting the AMC to one taking.",
"Solution_14": "I would like to continue having the opportunity to take two AMCs. Apart from the mathematical side, AMC is very error-prone; by that, I include scantron error and teacher error. Being able to take two time greatly reduces such problems.\r\n\r\nA possible modification: if one choose to take the second AMC, then he or she automatically forfeits the first AMC score. It would be best if the first AMC score comes out before this decision is made.",
"Solution_15": "I can only imagine the logistical challenge facing the AMC every year! Kudos to them for managing as well as they do.\r\n\r\nAccording to the published statistics, thousands of students don't even bother to bubble in their grade level on the AMC exams, and hundreds of students don't bubble in their grade levels on the AIME. Also, students with common names present challenges as well. (At our state MATHCOUNTS competition, there were ~200 students competing, and when the emcee read off the list of the top twelve students to call them down for Countdown, 14 students materialized at the front of the room, due to multiplicity of names.)\r\n\r\nSo any suggestions we can make can only mitigate the logistical challenge, not make it go away. I completely agree with Kent that the idea of SSN's is totally a non-starter.\r\n\r\n[quote]The only scheme I can see with a slight amount of hope: phone numbers. Put a place for a 10 - digit number, and ask student to fill in a number of their own choosing that they will remember, with the suggestion that it be a phone number (but the student is allowed to invent some other number).[/quote]\r\n\r\nHow about including a field for date of birth month/day/year on the AMC and AIME answer sheets, as a supplement to the phone number?\r\n\r\nThe date of birth should at least be easy for students to remember and replicate consistently each time. The phone number is a good idea because there may be some occasional instances when the easiest way for the AMC to get some piece of information is just to call and ask.\r\n\r\nAlso, how about designing the AIME answer form to ask the student about both their A date and B date scores, along with a place for the student to bubble in a field to designate if one of the AMC scores comes from a different location than the place where s/he is taking the AIME? \r\n\r\nAgain, these suggestions are only designed to make life a little bit easier for the AMC. I am completely in awe of their ability to manage the process as quickly as they do.\r\n\r\nOne more thought: perhaps there could be a special incentive for complete, consistent, and correctly filled out answer sheets. \r\n\r\nEach year, students whose AMC and AIME forms provide clear, consistent, and correct information that cause no problems for the AMC staff could be eligible for a drawing for a small prize, for example, a subscription to the MAA publication, [url=http://www.maa.org/mathhorizons/]Math Horizons[/url]. Students whose AIME forms wound up in the \"problem pile\" would not be eligible for the drawing.\r\n\r\nAlternatively, the incentive could operate at a school level. Schools that had no students winding up in the \"problem pile\" could be eligible for a drawing for a bulk subscription of Math Horizon magazines.\r\n\r\nThe drawing could be done in late summer, when the AMC is not quite so busy as at other times of year. The winning school could get recognized on the AMC website and in the MAA Messenger as a way to remind contest managers and students to be careful with their data entry.\r\n\r\nI have previously donated to the MAA to recognize the hard work done by the AMC Lincoln staff--there's a brick in the [url=http://www.maa.org/development/riverofbricks.html]MAA headquarters \"River of Bricks\" walkway[/url]celebrating the AMC staff, with a special shout-out to Donita Bowers, who retired a couple years ago.\r\n\r\nI would be happy to make an additional donation to fund subscriptions to a great magazine as a small incentive to students who do their part to make the AMC staff's work just a little bit easier.",
"Solution_16": "[quote=\"timwu\"]A possible modification: if one choose to take the second AMC, then he or she automatically forfeits the first AMC score. It would be best if the first AMC score comes out before this decision is made.[/quote]\nCorrection: this should only be applicable if the first score comes out before the second test. You need the information first. What if you got better on the first one and didn't know it until after you did worse on the second one? It's too risky to do it without the information (and isn't in the spirit of the AMC. I don't think it will promote dishonesty either).\n\n[quote=\"sophia\"]The date of birth should at least be easy for students to remember and replicate consistently each time. The phone number is a good idea because there may be some occasional instances when the easiest way for the AMC to get some piece of information is just to call and ask. [/quote]\r\nThis is an extremely good idea. They do this on the Euclid contest (not sure about other CEMC contests, but alright) as well.",
"Solution_17": "[quote=\"sdkudrgn88\"][quote=\"zmli\"]A better solution is to modifying the answer sheets for students to write in their CEEB# and scores earned at the specific CEEB location:\n\nAMC 10/12A: CEEB#, AMC10 or AMC 12, your score\nAMC 10/12B: CEEB#, AMC 10 or AMC 12, your score\n\nIt will make the automatic matching easier.[/quote]\n\nProblem is, people from International locations don't have a CEEB. (Or, could you explain what a CEEB is, if I'm wrong?)[/quote]\r\n\r\nThis is not a problem. AMC office automatically assign a test site a CEEB number if they don't have one. So, every test site has a CEEB number as far as AMC is concerned.",
"Solution_18": "The problem is that the CEEB number of the location where the student took a previous exam is not something that most students carry around in their heads. The diligent ones will write it down and carry a note with them, but if you could trust everyone to do that, you could go ahead an assign a unique identifier to each individual. \r\n\r\nStudents don't always remember their previous scores correctly, either. I'm sure the AMC policy about that is \"Thanks for the hint, but if you don't mind, we'll look that up ourselves.\"",
"Solution_19": "For starters, why not just print a warning message?\r\n\r\n\"If you are also taking the AMC 10/12B, or if you qualify for the AIME, note that [b]you must give exactly the same name on each contest that you participate in[/b].\"\r\n\r\nAnd similarly for each contest.",
"Solution_20": "[quote=\"worthawholebean\"]Personally, I really like the idea of restricting the AMC to one taking.[/quote]\r\n\r\nI certainly agree with you relative to taking 2 AMC-12s or 2 AMC-10s. Getting two bites at the apple here doesn't seem to make things more fair for anyone.\r\n\r\nHowever, given all the fuss on these forums about whether underclassmen should take the AMC-12 or the AMC-10 (maximizing AIME chances (and taking a more challenging test) vs. potentially maximizing USAMO chances), I think a great solution there is to have the gifted underclassmen take 1 AMC-12 test and and 1 AMC-10 test. I realize this won't work for everyone (e.g. school doesn't/won't offer both test dates, money concerns, etc.) and some will have to choose (or have their school choose for them), but it does present a remedy for a large number of the borderline cases. I wouldn't want to give this up unless there is a good solution to the other problem that doesn't compare AMC-10 scores as equal to AMC-12 scores for the underclassmen USAMO index calculation.\r\n\r\nP.S. On a 'rabbit-trail' side note, I went through the system in the early 90s where we had 1 ASHME and 1 AIME (no alternate date tests, and no AMC-10), and consideration to underclassmen wasn't given until MOP selection. I find it a bit ironic that the process has certainly bent over backwards to afford underclassmen a lot of opportunity (a whole AMC-10 test, a generous portion of 'underclassmen-only' slots in the USAMO, and the large number of MOP spots reserved for freshmen and underclassmen), yet we spend time on the forums debating how the process isn't fair to those underclassmen. I'm not suggesting the debate isn't warranted, but there is a level of irony here that is quite inescapable.",
"Solution_21": "I'm pretty sure we're debating on how it's not fair to the upperclassmen, not the other way around.",
"Solution_22": "No, I thought everyone was saying that some underclassmen are kicked out of the second cut because of large amount of underclassmen who take the 10 instead of the 12. The upperclassmen actually get the advantage of not having to compete with many underclassmen.",
"Solution_23": "[quote]\ncould you explain what a CEEB is\n[\\quote]\n\nCEEB stands for College Entrance Exam Board, now just known as the College Board. They are the folks behind the SAT, the AP Exams and several other things. They have a 6-digit code for each high school in the US, the CEEB number, that helps identify their information. In the 1980s and 1990s when the AMC went to centralized scoring, we adopted the numbers as a way to uniquely identify high schools. With many home schools, math circles, colleges and universities and schools outside the US now administering the AMC contests, we have created many new numbers, so the \"CEEB\"s that many of you use do not actually exist at the CB, just at the AMC.\n\n[/quote][quote]\nperhaps there could be a special incentive for complete, consistent, and correctly filled out answer sheets. \n[\\quote]\n\nInteresting idea. We'll explore it. Philosophically, I like positive incentives rather than negative enforcements. Might be worth considering.\n\n[quote]I have previously donated to the MAA to recognize the hard work done by the AMC Lincoln staff--there's a brick in the MAA headquarters \"River of Bricks\" walkway celebrating the AMC staff, with a special shout-out to Donita Bowers, who retired a couple years ago. \n[\\quote]\n\nThanks! Next time I'm in Washington at MAA HQ, I'll take a picture of it, and we'll post it at the website.\n\n\n\n[quote]\nfield for date of birth month/day/year on the AMC and AIME answer sheets, as a supplement to the phone number\n[/quote]\n\nThe date of birth and the phone number are both possibly feasible ideas. We will consider using them. I wonder about students outside the US, where phone numbers can be variable length.....\n\nOne thing to balance here is that we try to collect as little personal information as reasonably possible about each student. That's better for you, and and better for us. This maybe a place where we reasonably have to increase what we collect.\n\n[quote]For starters, why not just print a warning message?\n\n\"If you are also taking the AMC 10/12B, or if you qualify for the AIME, note that you must give exactly the same name on each contest that you participate in.\" \n[/quote][/quote][/quote][quote][/quote]\r\n\r\nWe will consider adding some kind of warning like that, perhaps in the instructions in the Teachers' Manual.\r\n\r\nOur experience is that such warning messages are ignored. Most students seem to be focused on solving the problems and getting the responses bubbled in, not on studying detailed instructions on the answer sheet. And that's okay, I understand that. \r\n\r\nIn any case, the students who did not follow the instructions to use the same name each time would still write or call us, and ask that we match their scores. In that case, we might not have a significant increase in efficiency. \r\n\r\nWe work about 12-24 months in advance planning for contests. We might be able to make these changes for 2010, more likely for 2011.\r\n\r\nSteve Dunbar\r\nMAA Director, American Mathematics Competitions",
"Solution_24": "I think the date of birth/phone number method is the best idea here so far. I don't think people will mind having that extra personal information collected as long as they know it's only being used to match up tests. And international phone numbers shouldn't be a problem--just allow the phone number to have variable length.\r\n\r\nIt still leaves room for ambiguity, though--you'd have to remember if you put your home or cell number. And international students, I suppose, would have to remember if they included country code and all that.",
"Solution_25": "[quote=\"m1sterzer0\"]P.S. On a 'rabbit-trail' side note, I went through the system in the early 90s where we had 1 ASHME and 1 AIME (no alternate date tests, and no AMC-10), and consideration to underclassmen wasn't given until MOP selection. [/quote]\r\nIn the late 90's, there was only 1 ASHME and 1 AIME, but there was definitely a multi-tiered selection system. A main selection (\"first cut\") with everyone eligible, followed by a few more 11th grade and below, followed by a few more 10th grade and below, followed by a few more 9th grade and below.\r\n\r\nI still find that to be an attractive scheme, provided there is a single basis of selection for all of these tiers, such as \"M.M. index.\"\r\n\r\nOf course, when [i]I[/i] went through the system (1969-71), there was no AIME and no USAMO. Just the AHSME. And the U.S. did not participate in the IMO - that was something that communist countries in Eastern Europe did.",
"Solution_26": "I think the change might be at 1996- that's when the one qualifier per state rule was introduced, at least. Since then, the AMC has had various systems, all of which made it easier to qualify as an underclassman."
}
{
"Tag": [
"Stanford",
"college"
],
"Problem": "So an interesting idea hit me the other day. What if we as members of AOPS got together and formed a global secret society where we aspire to take over the world....muhuhahaha. Think about it for a second; AOPS consists of perhaps some of the most intelligent and gifted young students in the world, and we could do some great things. The society could consist of different chapters in various regions, with some sort of a hierarchy in which each region would have a head person and some sub-leaders. We could have virtual meetings online via webcam/webchat (perhaps Skype?) where all of the leaders would meet each week online to discuss the \"operations\" of the society and to see how each region is doing. It could be quite the organization. We could even design some sort of a stock market operation for investments and whatnot. What do you guys think of the idea?",
"Solution_1": "hey neato I call dibs on ultimate world leader okay",
"Solution_2": "hm.we probably could talk about random stuff for 5 seconds and then get bored and then go do the same on FTW...",
"Solution_3": "how much time did you spend planning this thread and thinking about all the e-props that you were going to get for it?",
"Solution_4": "imo you need a life\r\n\r\nthe fact that you posted it in round table kinda implies that you're taking this seriously\r\n\r\nwhich... is a scary thought",
"Solution_5": "Haha, no not at all. I just found it to be quite an amusing idea. Believe me, I definitely have a life ;)",
"Solution_6": "[quote=\"Lazarus\"]imo you need a life\n\nthe fact that you posted it in round table kinda implies that you're taking this seriously\n\nwhich... is a scary thought[/quote]\r\n\r\nThis isn't round table. This is Games & Fun Factory.",
"Solution_7": "It was originally posted in Round Table.",
"Solution_8": "Yeah, that was my bad. I was unsure of which one I should post it in. Obviously I chose the wrong one though.",
"Solution_9": "[quote=\"MysticTerminator\"]hey neato I call dibs on ultimate world leader okay[/quote]\r\n\r\nHow did I know that MysticTerminator was going to be the first person to say this?",
"Solution_10": "lol I'll join the ring... I call Planner!",
"Solution_11": "I'll join ring... I call [i][b]Whatever[/b][/i].",
"Solution_12": "I'll join, I'll ask people in EPGY OHS (gifted online high school run by stanford) to join :)",
"Solution_13": "EHEM! this is [size=200][u][i][b][color=darkred]AOPS[/color][/b][/i][/u][/size] secret society!!! :mad:\r\n\r\nI join! I can be the entering platform to Latin America",
"Solution_14": "I call snacks provider. Anyone care for a cookie?",
"Solution_15": "[quote=\"mathblitz\"]i join as prez advisor me is so not smart! o and lol googlism is so funny!id join but my parents wont let me[/quote]\r\n\r\nJust join it secretly.",
"Solution_16": "um... wait so what was the original purpose of this discussion again???\r\n\r\nanyways i is ok with revealing who i really am to the rest of the secret society. like who i am other than an anime-obsessed computer freak.",
"Solution_17": "I was showing how great a religion Googlism was :)",
"Solution_18": "yes, i completely agree.",
"Solution_19": "hm...well, I guess I'll be the person who pretends not to be a part of this, therefore allowing me to retrieve outside information from certain connections... =D \r\n\r\njk, I'll do whatever.",
"Solution_20": "An AOPS Secret Society FAILED!!!!!!!!\r\n\r\nIt is supposed to be SECRET. This is NO LONGER SECRET. So THIS FAILED!!! QED\r\n\r\nSorry",
"Solution_21": "How do YOU know it's not secret...\r\n\r\nYou aren't in the society...",
"Solution_22": "anyway, i also proclaim myself church co-minister (with chenhsi) and that the official religion is Googlism. Usually im an atheist, but googlism sounds good. :D",
"Solution_23": "No one else wants to join?",
"Solution_24": "i think if we got any more members it REALLY wouldnt be secret any more...",
"Solution_25": "I join! But considering the fact that I don't exist, this secret organization is still a secret, right? :roll:",
"Solution_26": "[quote=\"MysticTerminator\"]hey neato I call dibs on ultimate world leader okay[/quote]\r\n\r\nOMG THAT IS NOT ARNAV HE IS NOT POSTING IN CAPS OMG HELP ARNAV!\r\n\r\nAlso, this is a dumb idea, since some people *cough*me*cough* live in the middle of nowhere, and the nearest user isn't even in their county.",
"Solution_27": "um actually same here, don't get depressed\r\nunless you count all the ppl in seattle\r\nbut there still in a different county",
"Solution_28": "[quote=\"splatyango\"]um actually same here, don't get depressed\nunless you count all the ppl in seattle\nbut there still in a different county[/quote]\r\nHey! What about me?!! (I'm her brother)\r\nyeah........",
"Solution_29": "oh sry lol :blush: \r\n\r\nbut u dont come on often, excluding today\r\n\r\nso yeah.\r\n\r\nEDIT: putting subjects is ok, but most ppl dont, so unless you have a really important point, i would advise you not to..."
}
{
"Tag": [
"search"
],
"Problem": "What is the sum of the digits of the sum of the digits of the sum of the digits of $ 4444^{4444}$",
"Solution_1": "Let $ A=4444^{4444}< 4444^{10000}$\r\nThen $ A$ has less than $ 4444*\\lg10000 = 17776$ digits.\r\n$ \\implies \\text{the sum of digits of A}< 9*17776 = 159984$\r\nSo the sum of the digits of the sum of digits A is less than $ 9*6 = 54$\r\nThen X is the sum of the digits of the sum of the digits of the sum of digits A $ \\leq 4+9 = 13$\r\nWe know that $ X \\equiv A \\equiv (-2)^{4444}\\equiv 2^{3*1481+1}\\equiv-2 (mod 9)$\r\nThat means $ X=7$\r\n\r\nHope I don't make any mistakes.",
"Solution_2": "http://www.artofproblemsolving.com/Forum/viewtopic.php?search_id=1299623712&t=133303\r\n\r\nPlease, if you're going to insist on posting so many questions, at least check to make sure they haven't been posted before.",
"Solution_3": "[quote=\"bibobeo\"]Let $ A = 4444^{4444}< 4444^{10000}$\nThen $ A$ has less than $ 4444*\\lg10000 = 17776$ digits.\n$ \\implies\\text{the sum of digits of A}< 9*17776 = 159984$\nSo the sum of the digits of the sum of digits A is less than $ 9*6 = 54$\nThen X is the sum of the digits of the sum of the digits of the sum of digits A $ \\leq 4+9 = 13$\nWe know that $ X\\equiv A\\equiv (-2)^{4444}\\equiv 2^{3*1481+1}\\equiv-2 (mod 9)$\nThat means $ X = 7$\n\nHope I don't make any mistakes.[/quote]\r\n\r\nWhere did you get $ X\\equiv A\\equiv (-2)^{4444}$ from?",
"Solution_4": "Since $ 4446$ is a multiple of $ 9$, $ 4444$ must be equivalent to $ 7$, or $ \\minus{}2\\text{ mod }9$. Raising both $ 4444$ and $ \\minus{}2$ to the power of $ 4444$, we must obtain that the results would still be congruent $ \\text{ mod }9$. So $ 4444^{4444}\\equiv (\\minus{}2)^{4444}\\text{ mod }9$. \r\n\r\nConsidering the numbers mod 9 in these types of problems is very helpful, because if you keep taking the sum of digits till there is only one digit left, you get the [i]digital root[/i], which is constant mod 9."
}
{
"Tag": [
"\\/closed"
],
"Problem": "[url]http://www.artofproblemsolving.com/Forum/profile.php?mode=viewprofile&u=66008[/url]\r\n\r\nIt seems like this person is an impersonator of the user r15s11z55....(something) or some existing user's multi. Suspicions/thoughts?",
"Solution_1": "Please pm an admin directly about these.\r\n\r\nThis user is just bored and has nothing better to do. Please ignore him."
}
{
"Tag": [
"function",
"trigonometry",
"algebra",
"functional equation",
"real analysis",
"real analysis unsolved"
],
"Problem": "Prove or disprove the following:\r\nlet f,g be real functions s.t g(x-y)=g(x)g(y)+f(x)f(y) and g(0)=1 then g'(x)=-f(x)f'(0) & f'(x)=g(x)f'(0)",
"Solution_1": "$f(x)=\\sin x$ \r\n$g(x)=\\cos x$",
"Solution_2": "Yes, but can that be concluded from the functional equation (assuming continuity or measurability), for instance by constructing an additive function from $f$ and $g$? Some trigonometric equations are available, such as $f^2 + g^2 = 1$ and the angle-doubling formula for $g(x)$, but it is not immediately clear whether e.g. $f + ig$ is a homomorphism.",
"Solution_3": "Let $\\alpha\\left(x\\right)$ be any function satisfies $\\alpha\\left(x+y\\right)=\\alpha\\left(x\\right)+\\alpha\\left(y\\right)$, then $f\\left(x\\right)=\\sin\\left(\\alpha\\left(x\\right)\\right),\\ g\\left(x\\right)=\\cos\\left(\\alpha\\left(x\\right)\\right)$ is a solution, which is not necessarily continous.",
"Solution_4": "Yes, but given $f$ and $g$ can one construct a solution of $\\alpha(x+y) = \\alpha(x) + \\alpha(y)$?\r\nThis is the point mentioned above. \r\n\r\nThis problem is relatively easy if also given the functional equation for $f(x+y)$. Given only the equation for $g(x+y)$, is it possible?",
"Solution_5": "You're so greedy. The Captain wrote f'(0) in his post, so f'(0)=k exists, we're able show $f\\left(x\\right)=\\sin\\left(k x\\right),\\ g\\left(x\\right)=\\cos\\left(k x\\right)$.",
"Solution_6": "I interpreted the problem as to prove or disprove that the functional equation implies the differentiability and the formulas for $f', g'$. Nowhere is it stated that $f', g'$ are assumed to exist, it makes things easier to assume that, but it makes things more interesting to not assume that.",
"Solution_7": "fleeting_guest is right. \r\nGiven g(x-y)=g(x)g(y)+f(x)+f(y) where f,g are real functions can we say that \r\n(( g(0)=1 )) & ((there exists an a s.t f(a)=1 & g(a)=0)) are equivalent ?\r\nI think this will help a lot.",
"Solution_8": "The condition on $g(0)$ or $f(0)$ is kinda redundant, we can derive this from $g(x-y)=g(x)g(y)+f(x)f(y)$ except for some trivial case, in which $f,\\ g$ are constant functions.\r\nSince we can find solution $g(x)\\equiv 1,\\ f(x)\\equiv 0$, thus $g(0)=1$ and $\\exists a$ s.t $f(a)=1,\\ g(a)=0$ are not equivalent.\r\nIn order to get the differentiability, we need some extra conditions, for example, the existence of $f'(0)$.",
"Solution_9": "[quote=\"fleeting_guest\"]Yes, but can that be concluded from the functional equation (assuming continuity or measurability), for instance by constructing an additive function from $f$ and $g$? Some trigonometric equations are available, such as $f^2 + g^2 = 1$ and the angle-doubling formula for $g(x)$, but it is not immediately clear whether e.g. $f + ig$ is a homomorphism.[/quote]\r\nI have a basic question.\r\nwhat is the f+ig ? :?:",
"Solution_10": "$f+ig = f + g\\sqrt{-1}$ is a homomorphism (from $\\mathbb{R}^\\ast \\to \\mathbb{C}$) if $ (f(x) + ig(x))(f(y) + ig(y)) = f(x+y) + ig(x+y)$. This would be true if $f = \\cos (kx), g = \\sin (kx)$.",
"Solution_11": "[quote=\"fleeting_guest\"]$f+ig = f + g\\sqrt{-1}$ is a homomorphism (from $\\mathbb{R}^\\ast \\to \\mathbb{C}$) if $ (f(x) + ig(x))(f(y) + ig(y)) = f(x+y) + ig(x+y)$. This would be true if $f = \\cos (kx), g = \\sin (kx)$.[/quote]\r\n\r\nRight ;but the captain said that f & g are real functions :!: :huh:",
"Solution_12": "If $f$ and $g$ are real functions, $f+ig$ is a complex function. This complex function is often a useful thing to consider when solving problems about real $f,g$ satisfying functional equations similar to those of $\\sin$ and $\\cos$.",
"Solution_13": "[quote=\"Buffalo\"]Since we can find solution $g(x)\\equiv 1,\\ f(x)\\equiv 0$, thus $g(0)=1$ and $\\exists a$ s.t $f(a)=1,\\ g(a)=0$ are not equivalent.\r\nBuffalo!Is'nt it fairly clear that in the case of constant functions the question is trivial?"
}
{
"Tag": [],
"Problem": "cho da thuc$P(x)$ he so nguyen thoa man :\r\n $P(2006)=2006!$\r\n $xP(x-1)=(x-2006)P(x)$ voi moi x thuoc $R$\r\n CMR: $G(x)=P(x)^2 +1$ bat kha qui trong$Z[x]$",
"Solution_1": "De dang chung minh duoc rang:$ P(0)\\equal{}P(1)\\equal{}P(2)\\equal{}...\\equal{}P(2005)\\equal{}0$.\r\nDat $ P(x)\\equal{}x(x\\minus{}1)(x\\minus{}2)...(x\\minus{}2005)Q(x)$, thay vao gia thiet ta co $ Q(x)\\equal{}Q(x\\minus{}1)$\r\nsuy ra $ Q(x)\\equal{}k$ voi k la hang so.\r\nMat khac $ P(2006)\\equal{}2006!$ nen suy ra $ P(x)\\equal{}x(x\\minus{}1)(x\\minus{}2)...(x\\minus{}2005)$\r\nTa da dua ve bai toan quen thuoc:\r\n\"Chung minh rang: $ R(x)\\equal{}x^2(x\\minus{}1)^2(x\\minus{}2)^2...(x\\minus{}n)^2\\plus{}1$ bat kha quy tren $ Z[x]$ voi moi n nguyen duong\""
}
{
"Tag": [
"AMC",
"AIME",
"USA(J)MO",
"USAMO",
"AMC 10"
],
"Problem": "I'm just wondering...\r\ncoz there's just a \"small\" little chance for me to get enroll...\r\nlol...\r\nI can't say no chance...but I know the truth :P",
"Solution_1": "There have been lots of discussion on this topic. Please check the American Mathematics Competitions board on this website.\r\n\r\nIf you're in 11th or 12th grade, then you will need a good index to qualify for the USAMO. Your index score is AMC + 10*AIME, and you qualify for the AIME by scoring at least 100 on the AMC 12 (AMC 10 = 120). It is in your best interest to do the best you can on the AMC 12.\r\n\r\nHowever, if you're in 10th grade or below, then just getting a 100 on the AMC 12 or something will do. Just make sure you do your best on the AIME.\r\n\r\nI believe I've answered your question?\r\n\r\nBottom line, do well on the AMCs.",
"Solution_2": "Does she mean if the AMC is good for CMO?\r\n\r\nIf that is what she asks I want to know the answer too.",
"Solution_3": "I mean I may never can get enroll to the Olympiad\r\nso...what's the point of doing the AMC??",
"Solution_4": "Sigh, alright. I'll give you some reasons.\r\n\r\nIdealistically: You enjoy doing Math and you want the opportunity to show all you've got, but also have fun and get a free pretty coloured pamphlet after the test.\r\n\r\nRealistically: A good AMC score can get the attention of college adcoms. Making the AIME would be ideal.\r\n\r\nSuper Idealistically: You never know, you just might make the USAMO. I know a kid who went from a 61 on the COMC last year to USAMO qualifier that same spring. Huge improvements can be made in a short amount of time; it just depends on how well you prepare I guess. And evidently, Canadians are known very much so for improving vastly in very short amounts of time.\r\n\r\nAs for the COMC, if you didn't score about a 69 or a 68 this time...strive for a better future! :)",
"Solution_5": "I wish it can be anything to canadian universities....",
"Solution_6": "since I've registered :P \r\n\r\ngoal for this yr is 152 hope it's not too hard for me :huh:",
"Solution_7": "[quote=\"british8985\"]since I've registered :P \n\ngoal for this yr is 152 hope it's not too hard for me :huh:[/quote]Oh 152....",
"Solution_8": "too low??\r\nthat's the best I can do I think..",
"Solution_9": "[quote=\"british8985\"]too low??\nthat's the best I can do I think..[/quote]Uh, no.\r\nIsn't 150 the perfect score? :?",
"Solution_10": "I think she means 152 index, AMC + 10*AIME score. That's not a bad goal.",
"Solution_11": "for me, i am wondering what is the right goal for a scarcely educated 12th grader, gone through little math study, first time writing AMC? :P",
"Solution_12": "maybe...130~140\r\n\r\ncoz I'm in gr11..so..."
}
{
"Tag": [
"geometry",
"trapezoid"
],
"Problem": "Find the area of trapezoid ABCD if AB || CD, AB = 24, CD = 12, the measure of angle A is 30 and the measure of angle B is 60.",
"Solution_1": "[hide=\"ans\"]\nwe need to find the height of the trapezoid and in order to do that we need to find the side length of one of our 30-60-90 triangles\n\nlet x and y equal the top lengths of the two triangles, x is for the angle 30, and y the angle 60\n\n$x+y = 12$ and $x\\tan30 = y\\tan60$\nthus,\n\n$x\\tan30 = (12-x)\\tan60$\n\n$x\\tan30 = 12\\tan60-x\\tan60$\n\n$x(\\tan30+\\tan60) = 12\\tan60$\n\n$x = \\frac{12\\tan60}{\\tan30+\\tan60}= 9$\n\nNow we can find the height of our trapezoid, using the 30-60-90 theory\n\nsince x = 9 and this side is $\\sqrt{3}$ in the theorem, then it has been mutliplied by $3\\sqrt{3}$, thus the height of the trapezoid is $3\\sqrt{3}$\n\nOR, we use the fact that $x\\tan30$ = the height of the trapezoid which comes out to 5.196 which is exactly the same as $3\\sqrt{3}$\n\ncomputing the area is now easy\n\n$area = 3\\sqrt{3}* (\\frac{24+12}{2})= 93.53$\n[/hide]"
}
{
"Tag": [
"LaTeX"
],
"Problem": "Jim has a certain amount of people in front of him. First he calculates the number of ways he can take six people from the group in front of him. Then he calculates the number of ways he can take ten people from the group in front of him. He then realizes that the number of ways he can take six or ten people are exactly the same. How many people are in front of him?",
"Solution_1": "[hide]$16!/10!/6!=16!/6!/10!$\n\n$16?$[/hide]",
"Solution_2": "[hide=\"hint\"]\nRemember that $\\binom nr=\\binom{n}{n-r}$[/hide]",
"Solution_3": "eheehee is right\r\n[hide=\"solution\"]Take the mean of 10 and 6\n8 is the center number. Doubling it brings [b]16[/b][/hide]",
"Solution_4": "The correct answer is...\r\n\r\n[hide=\"Solution\"]\n(n r) = (n n-r)\n\n(n 6) = (n 10)\n(n 6) = (n n - 10)\n6 = n - 10\nn = [b]16[/b]\n[/hide]\r\n\r\n(sorry about the lack of latex)"
}
{
"Tag": [],
"Problem": "Solve for $ x: 3^{2x} \\plus{} 19 \\equal{} 10^x$.",
"Solution_1": "look at it. x=2. simplest way to do it.",
"Solution_2": "[quote=\"nikeballa96\"]look at it. x=2. simplest way to do it.[/quote]\r\n\r\nI seriously do not see the simplicity of brute force/guessing here. How would you know to try x=2?",
"Solution_3": "$ 3^{2x}\\plus{}19\\equal{}10^x\\implies9^x\\plus{}19\\equal{}10^x$",
"Solution_4": "try multiples of 10. 10 gives a negative value for 3^2x. 10^2 gives an answer. also try math's method.",
"Solution_5": "/bump can anyone explain how to get this? Logically.\n",
"Solution_6": "[hide=One Solution]\nWe have $3^{2x}+19=10^x\\implies 9^x+19=10^x$. Suppose $x=2y$. The equation becomes $9^{2y}+19=10^{2y}$. Subtracting $9^{2y}$ over and factoring, we get $19=10^{2y}-9^{2y} \\implies 19=(10^y-9^y)(10^y+9^y)$. Since $19$ is prime, the only two positive integers that multiply to equal $19$ are $1$ and $19$. Since $1<19$, we have $10^y-9^y=1$ and $10^y+9^y=19$. It is clear that $y=1$ satisfies both of these, and since $x=2y=2(1)=\\boxed{2}$. \nWe can check this by plugging in $x=2$ to the original equation: $3^4+19=10^2 \\implies 81+19=100$, which is true. [/hide]"
}
{
"Tag": [
"percent",
"geometry",
"geometric transformation",
"reflection",
"vector",
"email",
"search"
],
"Problem": "just about 4 days left for the exam\r\n\r\ni m a bit confused abt cut offs\r\n\r\nLAst yr total marks 300 (part A-180,PART B-120)\r\nthis yr total marks 240 (part A-180,PART B-60)\r\n\r\nsome 1 told me cut offs were 120-130 and 150 safe zone\r\nwell was it out of 180 :P , 240 or 300 :o",
"Solution_1": "What about NSEJS?",
"Solution_2": "check on hbcse",
"Solution_3": "do u think HBCSE wud publish cutoffs?? :( \r\n\r\nofc i forgot NSEJS and NSEA :P",
"Solution_4": "i presume it would",
"Solution_5": "i had a doubt regarding NSEP \r\nis it that we have $ 2$ and a $ \\frac {1}{2}$ hours for both the sections A and B :( or only section A",
"Solution_6": "u r giving the exam?? :oops: \r\n\r\nsorry to disappoint u but we hav just 2 hrs ....for both section A and B",
"Solution_7": "and that too difficult questions",
"Solution_8": "best of luck to everybody giving the exam :) \r\n\r\n@rt:these exams are for students of class 12th,not for 11thies :rotfl:",
"Solution_9": "wat ??????????\r\nNSEP AND NSEC ARE FOR XII?????????????/",
"Solution_10": "ofc eleventhies lik u and skand will get thru the 1st round :)",
"Solution_11": "all the best to all for the test going on...................... :P :rotfl: \r\n\r\nsrry for the late wishes :maybe:",
"Solution_12": "well how many of you wrote NSEA????\r\nand what are you expecting???\r\nMine expected score is anything between 100 - 120",
"Solution_13": "thats really great.. :thumbup: \r\n\r\nit became bad 2 worse frm me :(",
"Solution_14": ",i answered 35+\r\n\r\nin end it all went in hurry\r\n\r\n@ritu,NSEP and NSEC were meant 4 timepass only 4 me\r\n\r\nhow did you all performed in them",
"Solution_15": "congrats to those who are selected :) \r\n\r\nas expected I am not selected\r\n\r\nnever mind,i have next chance :D\r\nand one in INMO too :rotfl:",
"Solution_16": "@anand...\r\nnice to see u dude\r\nthanks and grats to u\r\ni lost ur phone no :(\r\nand can u mail it to me?\r\nand i have ur arihant organice chem...didnt do a page :((\r\nbtw..why din ya write the last base exam??",
"Solution_17": "[quote=\"sumanth_d\"]@anand...\n\nand i have ur arihant organice chem...[b]didnt do a page[/b] :((\n[/quote]\r\n\r\n\r\neven i havent done a page.......and i dont plan to do it also :P",
"Solution_18": "Here is the site for chem papers. They are Icho but icho and incho are not very different.\r\n[url=http://nhb.topcities.com/Chemclub/Icho.html]IChO[/url]\r\n\r\n[hide]Made NSEC, didnt give astro, and failed in nsep[/hide]\n\n[hide=\"@Da Vinci\"]Don't give up on chem da, especially organic chem. I am sure you also are aware of your talent in chem. [/hide]",
"Solution_19": "@ritu\r\nDo not get confused betn ICO, INChO and INCO. The first one stands for International Chemistry Olympiad and rest two stands for Indian Chemistry Olympiad though INChO is more frequently used. Got it????",
"Solution_20": "i not getting confused neways :P ...there r millions of exceptions in chem :rotfl: \r\n\r\nbut just cant believe how i got thru Chem while other JOI'ians didn't :maybe:",
"Solution_21": "Can any1 plz tell the following details of the INCho paper ???? \r\n1. Total marks\r\n2. Subjective or objective\r\n3. Any negative marking process followed\r\nPlz. give me at least some idea. I'm totally blank!!!!",
"Solution_22": "Do the same for INPhO :D :)",
"Solution_23": "Is any1 interested in buying the book/s provided by HBCSE with contains the theory problems of past years INPho and INCho ? Btw what are your views regarding open tests held by various institutes like Fiitjee, Resonance, etc. for IITJEE???",
"Solution_24": "I just got my NSEP certificate! :rotfl: Statewise 1% thing.\r\nAnyone getting it so late?\r\nEdit: do they send 11th and 12th certificates separately?",
"Solution_25": "i got to know that i have cleared neep/c/a only after the results of ino's were out[the equivalent ino concept] :rotfl: i don't think there can b anything funnier than this",
"Solution_26": "[quote=\"VP_91\"]i got to know that i have cleared neep/c/a only after the results of ino's were out[the equivalent ino concept] :rotfl: i don't think there can b anything funnier than this[/quote]\r\nOh yeah! In comparison this is nothing....",
"Solution_27": "I havent got NSEP certificate yet :rotfl: \r\n\r\nand there was 1 more funnier thing\r\nTuhin got 71.5 in INChO(initially)\r\nHe didnt send the paper 4 reevaluation\r\nThe cutoff was written 72.5\r\nStill his name was there on the list as selected :rotfl:",
"Solution_28": "@rituraj\r\nall papers were rechecked due to mistake in q7.8 soln\r\n\r\neven i got a new performance card without sending for rechecking\r\n\r\ni got also nsec certificate\r\n\r\n[hide]\ni want to tear it to pieces burn it and drink the ashes \nolympiads are a big lie :furious: \nbeing top 1% in chem olympiad doesn't guarantee being in even top 10% in jee chem \n[/hide]",
"Solution_29": "dont say u r not in top 10% in JEE chem :mad: \r\n\r\n3.98 lakhs sat for JEE\r\nthat means top 10% implies being in top 40,000.. :rotfl:"
}
{
"Tag": [],
"Problem": "Without dividing, determine the remainder when f(x) = x^3 + 5x^2 - x -21 is divided by x^2 + 4x - 10.",
"Solution_1": "[quote=\"king_23\"]Without dividing, determine the remainder when \n $f(x) = x^3 + 5x^2 - x -21$ \nis divided by \n$x^2 + 4x - 10$ [/quote]\r\nsorry but i made it clearlier",
"Solution_2": "[quote=\"inom\"][quote=\"king_23\"]Without dividing, determine the remainder when \n $f(x) = x^3 + 5x^2 - x -21$ \nis divided by \n$x^2 + 4x - 10$ [/quote]\nsorry but i made it clearlier[/quote]\n\nsolution,\n$x^2+4x^2-10=10$\n$x^2=10-4x$\n$F(x^2)=69-26x$ so\n$R(x)=69-26x$[/quote]",
"Solution_3": "Sorry, I don't quite understand how you did solution. Your remainder is possible, but I don't understand how you derived it.",
"Solution_4": "By division (as a check), I got 5x-1.\r\n\r\nBut I am intriqued by your method! More info?"
}
{
"Tag": [
"calculus",
"integration"
],
"Problem": "Solve the following simultaneous equation for all positive integer $a,b,c$ \r\n\r\n$a+b+c=38$\r\n\r\n$a^2+b^2+c^2=722$",
"Solution_1": "We see that $a^2+b^2+c^2=2ab+2ac+2bc$",
"Solution_2": "I got that much, but where do you go from there?",
"Solution_3": "Well from brute force I got 4,9,25 as a solution. Iono if its coincidence or not that those are all perfect squares themselves.",
"Solution_4": "[quote=\"silouan\"]We see that $a^2+b^2+c^2=2ab+2ac+2bc$[/quote]\n\nya , silouan , can you show your solution ? ( I believe you wouldnt brute force it out ...)\n\n[quote=\"krustyteklown\"]\nWell from brute force I got 4,9,25 as a solution. Iono if its coincidence or not that those are all perfect squares themselves.[/quote]\r\n\r\nWell , I think it can be shown that they are all perfect square . :)",
"Solution_5": "[quote=\"silouan\"]We see that $a^2+b^2+c^2=2ab+2ac+2bc$[/quote]\r\n\r\n$ab+bc+ca=361$\r\n\r\n$\\Rightarrow ab + (a + b)( - a - b + 38) = 361$\r\n\r\n$\\Rightarrow (a + b)^2 - 38(a + b) + 19^2 = ab \\Rightarrow (a + b - 19)^2 = ab$\r\n\r\n$\\Rightarrow (19 - c)^2 = ab$\r\n\r\nThis gives $(a - b)^2 = c(76 - 3c)$\r\n\r\nThen I tried for values of $c$ which make $c(76 - 3c)$ a perfect square.\r\n\r\nI found $c= 4, 9, 25$ (25 is maximum for c)are the only numbers which satisfy.\r\n\r\nHence the solution set is any permutation of the numbers $4, 9, 25$ and are $6$ in number.",
"Solution_6": "well done [b]vidyamanohar[/b] :) I have a method to show that $a,b,c$ must be prefect square \r\n\r\nWLOG let $a\\geq b\\geq c$ . Since we know\r\n\r\n$a^2+b^2+c^2=2(ab+ac+bc)$ \r\n :arrow: $c^2-2c(a+b)+(a-b)^2$ which mean \r\n\r\n$c=(a+b)-2\\sqrt{ab}$ *we take minus sign here since $c0$ prove that\r\n$(2(b-c)^{2}+2a^{2}+bc)(2(c-a)^{2}+2b^{2}+ca)(2(a-b)^{2}+2c^{2}+ab)\\ge a^{2}+b^{2}+c^{2}\\ge ab+bc+ca$",
"Solution_1": "[quote=\"skywalker\"]Given $a,b,c>0$ prove that\n$(2(b-c)^{2}+2a^{2}+bc)(2(c-a)^{2}+2b^{2}+ca)(2(a-b)^{2}+2c^{2}+ab)\\ge a^{2}+b^{2}+c^{2}\\ge ab+bc+ca$[/quote]\r\nWhat is this?",
"Solution_2": "He must mean + on the left hand side which was left out in the latex, I'm guessing :wink:",
"Solution_3": "Sorry it isn't homogeneous it must be \r\nGiven $a,b,c>0$ prove that\r\n$(2(b-c)^{2}+2a^{2}+bc)(2(c-a)^{2}+2b^{2}+ca)(2(a-b)^{2}+2c^{2}+ab)\\ge (a^{2}+b^{2}+c^{2})^{3}\\ge (ab+bc+ca)^{3}$\r\n :)",
"Solution_4": "[quote=\"skywalker\"]\nGiven $a,b,c>0$ prove that\n$(2(b-c)^{2}+2a^{2}+bc)(2(c-a)^{2}+2b^{2}+ca)(2(a-b)^{2}+2c^{2}+ab)\\ge (a^{2}+b^{2}+c^{2})^{3}$\n [/quote]\r\nLet $a^{2}+b^{2}+c^{2}=1.$ Then your inequality equivalent to the following inequality:\r\n$(2-3ab)(2-3ac)(2-3bc)\\geq1,$ which was posted many time ago by Vasc or hungkhtn.\r\nMy ugly proof:$(2-3ab)(2-3ac)(2-3bc)\\geq1\\Leftrightarrow$\r\n$\\Leftrightarrow2\\cdot\\sum_{cyc}(a-b)^{6}+\\frac{3}{2}\\cdot\\sum_{cyc}(a-b)^{2}(a^{4}+2a^{3}b-3a^{2}b^{2}+2ab^{3}+b^{4})+$\r\n$+6\\cdot\\sum_{cyc}abc(a^{3}-a^{2}b-a^{2}c+abc)+\\sum_{cyc}(a^{3}b^{3}-a^{2}b^{2}c^{2})\\geq0.$",
"Solution_5": "Yes thus you recognized it, I am finding a beautiful solution, I known solution of Vasc but it is same of you (about beauty) :D",
"Solution_6": "[quote=\"skywalker\"]I am finding a beautiful solution...[/quote]\r\nIt's very interesting.",
"Solution_7": "I found a solution but perhaps it's ugly as well (but it's fast for me 'cause I'm familar with such kind of ways using Schur Inequality); let\r\n$x=a+b+c,\\ y=ab+bc+ca,\\ z=abc$, and let\r\n$a^{2}+b^{2}+c^{2}=x^{2}-2y=1$\r\nthen we need to prove\r\n$(2-3ab)(2-3bc)(2-3ca)\\geq0\\Longleftrightarrow 7-12y+3\\cdot 3z(2x-3z)\\geq0$\r\nAnd we see that if\r\n$f(t)=t(u-t)\\ \\left(t\\leq\\frac{u}{2}\\right)$\r\nthen\r\n$f'(t)=u-2t\\geq0$\r\nSo with\r\n$x-3z=(a+b+c)(a^{2}+b^{2}+c^{2})-3abc\\geq6abc\\geq0$\r\nand with Schur Inequality\r\n$a^{3}+b^{3}+c^{3}+3abc\\geq ab(a+b)+bc(b+c)+ca(c+a)\\Longleftrightarrow z\\geq\\frac{4xy-x^{3}}{9}$\r\n(notice that $x^{2}=1+2y$) we have\r\n$7-12y+3\\cdot 3z(2x-3z) \\geq 7-12y+3\\cdot\\frac{4xy-x^{3}}{3}\\left(2x-\\frac{4xy-x^{3}}{3}\\right)=7-12y+\\frac{1}{3}(1+2y)(2y-1)(7-2y)=\\frac{1}{3}(1-y)(4y^{2}-10y+7)\\geq0$\r\n(because\r\n$y=ab+bc+ca\\leq a^{2}+b^{2}+c^{2}=1$)\r\nThen it's done.",
"Solution_8": "I think, it's better:\r\n$(2-3ab)(2-3ac)(2-3bc)\\geq1\\Leftrightarrow$\r\n$\\Leftrightarrow7-12(ab+ac+bc)+18(a+b+c)abc-27a^{2}b^{2}c^{2}\\geq0.$\r\n$3(a+b+c)abc\\geq27a^{2}b^{2}c^{2}.$ Hence, remain to prove that\r\n$7-12(ab+ac+bc)+15(a+b+c)abc\\geq0,$ which is easily killed by SOS. :wink:",
"Solution_9": "[quote=\"arqady\"][quote=\"skywalker\"]\nGiven $a,b,c>0$ prove that\n$(2(b-c)^{2}+2a^{2}+bc)(2(c-a)^{2}+2b^{2}+ca)(2(a-b)^{2}+2c^{2}+ab)\\ge (a^{2}+b^{2}+c^{2})^{3}$\n [/quote]\nLet $a^{2}+b^{2}+c^{2}=1.$ Then your inequality equivalent to the following inequality:\n$(2-3ab)(2-3ac)(2-3bc)\\geq1,$ which was posted many time ago by Vasc or hungkhtn.\nMy ugly proof:$(2-3ab)(2-3ac)(2-3bc)\\geq1\\Leftrightarrow$\n$\\Leftrightarrow2\\cdot\\sum_{cyc}(a-b)^{6}+\\frac{3}{2}\\cdot\\sum_{cyc}(a-b)^{2}(a^{4}+2a^{3}b-3a^{2}b^{2}+2ab^{3}+b^{4})+$\n$+6\\cdot\\sum_{cyc}abc(a^{3}-a^{2}b-a^{2}c+abc)+\\sum_{cyc}(a^{3}b^{3}-a^{2}b^{2}c^{2})\\geq0.$[/quote]\r\n\r\nYou are right, Arqady. More exactly, this problem is of VASC and in Hungkhtn's book (vietnamese), following nice solution are show:\r\n\r\nLet $a,b,c \\ge 0$ verify that $a^{2}+b^{2}+c^{2}=3$. Prove that\r\n\\[(2-ab)(2-bc)(2-ca) \\ge 1. \\]\r\nProof.\r\nLet $x=2-ab,y=2-bc,z=2-ca$. \r\nWe need to prove $xyz \\ge 1$ if $x,y,z$ are three positive numbers satisfying that\r\n\\[\\frac{(2-x)(2-y)}{2-z}+\\frac{(2-y)(2-z)}{2-x}+\\frac{(2-z)(2-x)}{2-y}=3. \\]\r\nThe above condition can be rewritten as follow\r\n\\[(2-x)(2-y)+(2-y)(2-z)+(2-z)(2-x)\\&=3(2-x)(2-y)(2-z)\\]\r\n\\[\\Leftrightarrow 8(x+y+z)+3xyz\\&=12+5(xy+yz+zx).\\]\r\nAssume that it's false with some numbers $x,y,z$, or $xyz <1$. We will prove\r\n\\[8(x+y+z)+3xyz <12+5(xy+yz+zx). \\]\r\nThis last one is easy by Mixing variable or SOS or Schur. Notice that we can assume $xyz=1$.",
"Solution_10": "[quote=\"GacKiem\"]\n\\]\nAssume that it's false with some numbers $ x,y,z$, or $ xyz < 1$. We will prove\n\\[ 8(x \\plus{} y \\plus{} z) \\plus{} 3xyz < 12 \\plus{} 5(xy \\plus{} yz \\plus{} zx).\n\\]\nThis last one is easy by Mixing variable or SOS or Schur. Notice that we can assume $ xyz \\equal{} 1$.[/quote]\r\n\r\nI think it is wrong. :P",
"Solution_11": "[quote=\"arqady\"][quote=\"skywalker\"]\nGiven $ a,b,c > 0$ prove that\n$ (2(b \\minus{} c)^{2} \\plus{} 2a^{2} \\plus{} bc)(2(c \\minus{} a)^{2} \\plus{} 2b^{2} \\plus{} ca)(2(a \\minus{} b)^{2} \\plus{} 2c^{2} \\plus{} ab)\\ge (a^{2} \\plus{} b^{2} \\plus{} c^{2})^{3}$\n [/quote]\nLet $ a^{2} \\plus{} b^{2} \\plus{} c^{2} \\equal{} 1.$ Then your inequality equivalent to the following inequality:\n$ (2 \\minus{} 3ab)(2 \\minus{} 3ac)(2 \\minus{} 3bc)\\geq1,$ which was posted many time ago by Vasc or hungkhtn.\n[/quote]\r\npqr kill it easily :maybe:"
}
{
"Tag": [
"Divisibility Theory"
],
"Problem": "Let $a$ and $b$ be positive integers. When $a^{2}+b^{2}$ is divided by $a+b,$ the quotient is $q$ and the remainder is $r.$ Find all pairs $(a,b)$ such that $q^{2}+r=1977$.",
"Solution_1": "$ \\frac{a\\plus{}b}{2}\\le [\\frac{a^2\\plus{}b^2}{a\\plus{}b}]\\equal{}q0$ :)",
"Solution_3": "The sequence $a_{n}$ is (up to a constant multiple) the sine coefficients of the Fourier series for the function $f(x)$ such that $f(x)=\\frac1{\\sqrt{x}}$ for $0 n-2,$ contradiction$.$\n\nFinally, assume $0 k>p,$ contradiction$. \\blacksquare$",
"Solution_18": "[hide = rewording]Let $m$ and $s$ be positive integers with $2 \\leq s \\leq 3m^2$. Define a sequence $a_1, a_2, \\ldots$ recursively by $a_1 = s$ and\\[a_{n+1} = 2n + a_n \\text{ }(\\text{for } n = 1, 2, \\ldots).\\]Prove that if the numbers $a_1, a_2, \\ldots, a_m$ are prime, then $a_{s-1}$ is also prime.[/hide]\n\nNote that in general, we may write $a_n = n^2 - n + s$. \n\nLet $t$ be the smallest positive integer for which $a_t$ is not prime; we wish to prove $t \\geq s$. Let $p$ be the smallest prime dividing $a_t$. We have\\[p \\leq \\sqrt{a_t} = \\sqrt{t^2 - t + s} \\leq \\sqrt{4t(t - 1)} < 2t - 1\\]since clearly $s = 3m^2 \\leq 3t(t-1)$ as $t \\geq m + 1$. Hence $p \\in [0, 2t - 2]$. Next, we observe that \\[a_t \\equiv 0 \\pmod p \\implies a_{t-p} \\equiv 0 \\pmod p.\\]So in the case $p \\leq t-1$ then we can subtract sufficiently many copies of $p$ from $t$ to obtain an index $t - \\ell p = i$ for which $1 \\leq i \\leq m$ and $a_i \\equiv 0 \\pmod p$ is prime. Then, $a_i$ must be $p$ and then we have $a_t \\geq p^2 = a_i^2 \\geq a_1^2 = s^2 = a_s \\implies t \\geq s$ as desired. \n\nIn the case that $p \\in [t+1, 2t - 2]$, we can let $p = t + r$ where $r \\in [1, t-2]$ so that\\[a_t \\equiv a_{t - p} = a_{-r} = r^2 + r + s \\equiv a_{r-1} \\equiv 0 \\pmod{p}\\]and clearly $r - 1 \\leq t - 1 \\leq m$ so $a_{r-1}$ is prime, so it follows that\\[a_t \\geq p^2 = a_{r-1}^2 \\geq a_1^2 = s^2 = a_s \\implies t \\geq s\\]as desired. \n\nLastly, if $p = t$, then $a_t = a_p = p^2 - p + s$ is disivible by $p$ hence $p \\mid s$. However, we have $s = a_1$ is prime so $p = s \\implies t = s$ which is what we wanted to prove.\n\nWe have exhausted all three cases so we are done; indeed, $t \\geq s$ always. $\\blacksquare$",
"Solution_19": "Let $F(X)=X^2+X+n$ and define $p$ as the smallest prime factor of the numbers $F(0),F(1),.....$\nFor the sake of contradiction assume the opposite and proceed.\nSuppose for $X=k > , something that is unknow to the most ... preolympiad 'ers.Use very elementary theorems.You ' ll need to drow some auxiliary but very basic segments!!!\r\n\r\n Babis",
"Solution_3": "Because the circumcircle of the triangle $ABC$ is the incircle of the triangle $UVW$, where $U,\\ V,\\ W$ are the centers of the circles $k,\\ l,\\ m$ respectively, I offer a equivalent enunciation of this problem:\r\n\r\n\"The incircle of the triangle $ABC$ touchs the its sides in the points $D\\in BC,\\ E\\in CA,\\ F\\in AB$. I note the second intersection $L$ between the line $DF$ with the circle $w=C(C,CD)$. Prove that $EL\\perp EF.\\\"$\r\n\r\n[b]A solution.[/b]\r\n\r\n$m(\\widehat {EFL})=m(\\widehat {EDL})=\\frac 12 (A+B),\\ m(\\widehat {FLE})=m(\\widehat {DLE})=\\frac 12 m(\\widehat {DCE})$ $=\\frac 12 C\\Longrightarrow$\r\n\r\n$m(\\widehat {LEF})=180^{\\circ} -\\frac 12 (A+B+C)=90^{\\circ}\\Longrightarrow EL\\perp EF.$",
"Solution_4": "Perfect !\r\n\r\nThis is exactly my solution , too!\r\n\r\n Babis",
"Solution_5": "in my solution , u can see( arc AC=arc CD),without using homothecy.\r\n\r\ni said homothecy there to make it clear .",
"Solution_6": "in my solution , u can see( arc AC=arc CD),without using homothecy.\r\n\r\ni said homothecy there to make it clear . ;)"
}
{
"Tag": [
"geometry",
"rhombus",
"incenter",
"geometric transformation",
"homothety",
"similar triangles",
"geometry unsolved"
],
"Problem": "Let ABCD be a rhombus. A tangent line to its incircle intersects BC at E and CD at F. The incircle touches AD at T. Prove that AE and TF are parallel.",
"Solution_1": "Let $ O$ be incenter, $ D_1,D_2,D_3,D_4$ toucing point with $ BC,CD,EF,AB$.\r\nLet $ X$ and $ Y$ be polar points for $ TF$ and $ AE$.\r\n\r\nBy Pascal theorem for degenerate hexagon $ TTD_2D_3D_1D_4$ \r\nwe get $ X,Y,O$ are collinear. \r\n\r\nThen $ AE\\parallel{}TF$",
"Solution_2": "I am sorry\r\nI cannot figure out...\r\nI have found on wikipedia what \"Pascal theorem\" is\r\nIs the conclusion of this theorem to the hexagon $ TTD_2D_3D_1D_4$ the following, that the three intersections: $ TT, D_3D_1$, $ TD_2, D_1D_4$ and $ D_2D_3, D_4T$ are on the same line(on the projective plane)?\r\n\r\n[quote=\"Number1\"]Let $ O$ be incenter, $ D_1,D_2,D_3,D_4$ toucing point with $ BC,CD,EF,AB$.\nLet $ X$ and $ Y$ be polar points for $ TF$ and $ AE$.\n\nBy Pascal theorem for degenerate hexagon $ TTD_2D_3D_1D_4$ \nwe get $ X,Y,O$ are collinear. \n\nThen $ AE\\parallel{}TF$[/quote]",
"Solution_3": "[quote=\"LaurT\"]Let ABCD be a rhombus. A tangent line to its incircle intersects BC at E and CD at F. The incircle touches AD at T. Prove that AE and TF are parallel.[/quote]\r\n\r\nProbably this will help:\r\n\r\n I have seen a solution(elementary) in Crux ( issues 6 , page 351 ) of this year , using homothecy.\r\n\r\n Babis",
"Solution_4": "[quote=\"LaurT\"]Let ABCD be a rhombus. A tangent line to its incircle intersects BC at E and CD at F. The incircle touches AD at T. Prove that AE and TF are parallel.[/quote]\r\nDear [b]stergiu[/b], is this the solution you were speaking of?\r\n[hide=\"Solution\"]\nApply Brianchon's theorem to $ ATDFEB$, so $ BD$, $ AF$, and $ ET$ concur. Furthermore, we have that $ AB$ and $ FD$ are parallel as are $ BE$ and $ TD$. Combining this with the concurrence, we have that a homothety maps triangle $ ABE$ to triangle $ FDT$ and thus $ FT$ and $ AE$ are parallel. [/hide]",
"Solution_5": "Let us to present an elementary approach.\r\n\r\nWe denote the point $ P\\equiv BD\\cap ET$ and because of $ BC\\parallel AD,$ we have that $ \\frac {BE}{DT} \\equal{} \\frac {PE}{PT} \\equal{} \\frac {PB}{PD}$ $ ,(1)$\r\n\r\nLet $ T'$ be, the tangency point of the incircle $ (O)$ of $ ABCD,$ to its side-segment $ CD.$\r\n\r\nIt is easy to prove ( by an angle changing in the configuration of the triangle $ \\bigtriangleup CEF,$ with the circle $ (O)$ as its $ C$-excircke ) that $ \\angle BOE \\equal{} \\angle DFO$ and so, from the similar triangles $ \\bigtriangleup BEO,\\ \\bigtriangleup DOF,$ we have $ \\frac {BE}{DO} \\equal{} \\frac {BO}{DF}$ $ \\Longrightarrow$ $ (BE)\\cdot (DF) \\equal{} (DO)^{2}$ \r\n\r\n$ \\Longrightarrow$ $ (BE)\\cdot (DF) \\equal{} (DT')\\cdot (DC)$ $ ,(2)$ $ ($ because of $ (DO)^{2} \\equal{} (DT')\\cdot (DC)$ from the right triangle $ \\bigtriangleup OCD$ $ ).$\r\n\r\nFrom $ (2)$ $ \\Longrightarrow$ $ \\frac {BE}{DT'} \\equal{} \\frac {DC}{DF}$ $ \\Longrightarrow$ $ \\frac {BE}{DT} \\equal{} \\frac {AB}{DF}$ $ ,(3)$\r\n\r\nFrom $ (1),$ $ (3)$ $ \\Longrightarrow$ $ \\frac {AB}{DF} \\equal{} \\frac {PB}{PD}$ $ ,(4)$\r\n\r\nFrom $ (4)$ and because OF $ AB\\parallel DF,$ we conclude the collinearity of the points $ A,\\ P,\\ F$ and so, we have that $ \\frac {PB}{PD} \\equal{} \\frac {PA}{PF}$ $ ,(5)$\r\n\r\nHence, from $ (1),$ $ (5)$ $ \\Longrightarrow$ $ \\frac {PE}{PT} \\equal{} \\frac {PA}{PF}$ $ \\Longrightarrow$ $ AE\\parallel TF$ and the proof is completed.\r\n\r\nKostas Vittas.",
"Solution_6": "Hi everyone. \r\nI know that Vittasko,Number 1, Quatto Master and Babis have given brilliant proofs for this problem. However I was just wondering if an inversive solution is feasible here. Since we know that in triangle $ ABE$ and $ TFD$ we have $ AB$ parallel to $ DF$ and $ BE$ parallel to $ TD$ when we invert the whole figure with respect to the incircle of the rhombus we would get a family of circles passing through $ O$(the center of the incircle of the rhombus.) many of which are tangent circles in pairs. Now we can use this to prove that under the inversion lines $ AE$ and $ TF$ are transformed into tangent circles the point of tangency being $ O$ the center of incircle of the rhombus, since this would mean that lines $ AE$ and $ TF$ are parallel.\r\nCould anyone help me with a rigorous proof using this approach or inversion in general.\r\n\r\nThanks in advance.",
"Solution_7": "Hi everyone,\r\nCan someone please help me or otherwise give an inversive solution to this problem.",
"Solution_8": "Here is my solution:\r\nLet U,V,W,M be the points of tangency of the circle to BC, CD, AB, and EF, O the center of the circle with radius r, let $ WB \\equal{} BU \\equal{} VD \\equal{} DT \\equal{} x$, and let $ \\angle WOB \\equal{} \\angle BOU \\equal{} \\alpha$. Then, we have to show $ \\triangle ABE$ and $ \\triangle TFD$ are similar, or $ \\frac {x}{x \\plus{} VF} \\equal{} \\frac {x \\plus{} UE}{x \\plus{} AW}$, or $ x \\cdot AW \\equal{} x(VF \\plus{} UE) \\plus{} VF \\cdot UE$. But note that $ \\triangle AOB$ is a right triangle with altitude OW, so $ x \\cdot AW \\equal{} r^2$. So we have to show $ r^2 \\equal{} x(VF \\plus{} UE) \\plus{} VF \\cdot UE$.\r\nNote that $ \\angle WOV \\equal{} 180$, so $ \\angle UOV \\equal{} 180 \\minus{} 2 \\alpha$. But $ \\angle UOE \\equal{} \\angle EOM$ and $ \\angle MOF \\equal{} \\angle FOV$, so $ \\angle EOF \\equal{} (1/2) \\angle UOV \\equal{} 90 \\minus{} \\alpha$. Thus, $ \\frac {r}{x} \\equal{} cot \\alpha \\equal{} tan(90 \\minus{} \\alpha) \\equal{} tan EOF \\equal{} tan(EOM \\plus{} MOF) \\equal{} \\frac {\\frac {UE}{r} \\plus{} \\frac {VF}{r}}{1 \\minus{} \\frac {UE \\cdot VF}{r^2}}$. But\r\n$ \\frac {r}{x} \\equal{} \\frac {\\frac {UE}{r} \\plus{} \\frac {VF}{r}}{1 \\minus{} \\frac {UE \\cdot VF}{r^2}}$ is equivalent to what we wanted to show."
}
{
"Tag": [],
"Problem": "given that $a^{2}+b^{2}=ab$, find $\\frac{a^{5}}{b^{5}}$\r\n\r\nwait a minute...I don't think I'm doing anything wrong, but plugging the answer back into the equation gives an incorrect eequation ($2a^{2}=-a^{2}???$) :blush: \r\nI started with $a^{3}+b^{3}=0$, then divided by $(a+b)$ to get $a^{2}+b^{2}-ab=0$, which can be rearranged into the original equation. this means that $\\frac{a^{5}}{b^{5}}=\\frac{a^{3}}{b^{3}}=-1$\r\n[b]WHAT AM I DOING WRONG?????[/b]",
"Solution_1": "Unless I am wrong:\r\n\r\n$\\frac{a^{5}}{b^{5}}= \\frac{1}{2}\\pm \\frac{\\sqrt{3}}{2}i$",
"Solution_2": "ok I figured out what I did wrong :D If you want, I will leave it as a challenge to try and figure out by yourself\r\n\r\nif you just want to know, \r\n[hide]going through my solution backwards is fine, but dividing by $(a-b)$ is equal to dividing by zero when $a=-b$. This invalidates the equation[/hide]",
"Solution_3": "Isn't it when $a=b$?\r\n\r\n[hide]Shouldn't the answer be $1,-1$?[/hide]",
"Solution_4": "he meant $a+b$"
}
{
"Tag": [],
"Problem": "When not on the moving sidewalk, Kelsey can walk the length of the sidewalk in 3 minutes. If she stands on the sidewalk as it moves, she can travel the length in 2 minutes. If Kelsey walks on the sidewalk as it moves, how many minutes will it take her to travel the same distance? Assume she always walks at the same speed, and express your answer as a decimal to the nearest tenth.",
"Solution_1": "[hide=\"answer\"]k = speed of Kelsey\ns = speed of sidewalk\nl = length of sidewalk\n$k = \\frac{l}{3}$\n$s = \\frac{l}{2}$\n$k+s = \\frac{l}{3} + \\frac{l}{2}$\n$k+s = \\frac{5l}{6} = \\frac{l}{\\frac{6}{5}}$\nSo she can walk down the sidewalk in 6/5 minutes, or [color=green]1.2 minutes[/color][/hide]",
"Solution_2": "[hide]\nWhen walking not on the sidewalk, she goes $\\frac 13$ in a minute.\nWhen standing, or the speed of the sidewalk is $\\frac 12$ per minute.\nThus she can do $\\frac 13 + \\frac 12= \\frac 56$ per minute when walking on the sidewalk moving.\nIt then takes her $\\frac 65$ minutes = $1.2$[/hide]",
"Solution_3": "[hide]$d=K*3 \\implies K=d/3$\n$d=S*2 \\implies S=d/2$\n\n$d=(K+S)*x$, solve for x.\n$d=(d/3+d/2)*x$\n$d=(5d)/6 *x$\n$x=(6d)/(5d)=6/5=1.2$ minutes[/hide]",
"Solution_4": "I agree with those answers. \r\nA moving sidewalk is just an escalator but flat. \r\nI was on one at the airport. They move really slow.",
"Solution_5": "While waiting for our flight from Detroit to Atlanta, I wanted to know how long it would take me to walk backwards on the moving sidewalk without actually doing it. So I used the same process being the genius penguin I am. Then I decided to check my answer by being an idiot. My teacher said that you would never need to do this kind of problems in real life.",
"Solution_6": "they could at least say 'moving flat escalator' in the question, i'm like that would be pretty weird if you were standing on the street and the sidewalk started moving. :)",
"Solution_7": "It would if you lived in LA. Of course everything would move with it.",
"Solution_8": "[quote=\"ritchjp\"]It would if you lived in LA. Of course everything would move with it.[/quote]\r\n\r\nHaha that's where I live. :rotfl:"
}
{
"Tag": [
"algebra",
"polynomial",
"AoPS Books",
"abstract algebra"
],
"Problem": "Hi,\r\n\r\nI've recently gotten into problem solving as a hobby. Its really fun (though I'm kinda late to the game, junior in high school :( ). Anyways, I encountered this problem and found the answer (C), but I kinda brute forced it. Does anyone know of a more, how should I put this, elegant way of obtaining a solution? \r\n\r\n10. Which of the following divides $ n^3 \\plus{} 11n$ for all positive integers $ n$?\r\nA. 5 B. 7 C. 6 D. 4 E. 9\r\n\r\n\r\nThanks!",
"Solution_1": "I'm not sure what you define as brute-forcing because I just played around with mods and parity to arrive at a solution. First after factoring the expression I obtained $ n(n^2 \\plus{} 11)$. If n is even then the expression always divisible by 2 because the n on the outside is divisible by 2. If n is odd then $ n^2 \\plus{} 11$ is always divisible by 2. Then I considered the residues of $ n modulo 3$. By plugging in the possible residues of 0,1,and 2 modulo 3 into $ n$ and their respective squares into $ n^2$ I found by considering each of the three cases that the expression is always divisible 3, which gives C as an answer. I don't think there is any more elegant way than factoring and using modular arithmetic(which really didn't take me too long). But I could be wrong.",
"Solution_2": "Plug in $ n \\equal{} 1$ to eliminate A, B, and E, then $ n \\equal{} 2$ to eliminate D. :D",
"Solution_3": "[hide=\"Simple strategy\"] Test out a few values to see what the biggest number [i]could[/i] be. Once you think you've got it, try to prove it. Then you know it can't be a bigger number. [/hide]\n[hide=\"Complicated strategy\"] A polynomial of degree $ k$ has at most $ k$ roots $ \\bmod p$ for $ p$ prime. You might not know what this means, but essentially it means that since the expression you're given has degree $ 3$, the only primes involved in the answer are $ 2$ or $ 3$. [/hide]",
"Solution_4": "[quote] Simple strategy\nTest out a few values to see what the biggest number could be. Once you think you've got it, try to prove it. Then you know it can't be a bigger number.\n\nComplicated strategy\nA polynomial of degree k has at most k roots \\bmod p for p prime. You might not know what this means, but essentially it means that since the expression you're given has degree 3, the only primes involved in the answer are 2 or 3. [/quote]\r\n\r\nThanks! \r\n\r\n@Brut3Forc3 and phi-unit\r\nBy brute forcing, I meant what Brut3Forc3 said (haha how ironic :rotfl: ). I was just wondering if there was some way to do it like t0rajir0u posted. I like to see the principles behind solutions, I know that plugging in numbers will give me the same answers, but this is just cooler :P . I don't really know how the complicated solution works but sounds interesting. See, I'm planning to get into problem solving as best as I can but as I said, I am late to the game. How much will the AoPS books help with problems like this? I've borrowed Vol. 1 and Vol. 2 from a friend who bought them but never used them. Do they cover stuff like this? If not, what books do?\r\n\r\nThanks again!",
"Solution_5": "@t0rajir0u:\r\n\r\nI considered your strategy, it's pretty interesting. My interpretation for this is: we can always find a congruent polynomial of the original one $ \\bmod p$, suppose it's $ (n \\minus{} x_1)(n \\minus{} x_2)...(n \\minus{} x_k)$, which can only take at most $ k$ distinct roots $ \\bmod p$, i.e. be a multiple of $ p$. Therefore, to make it divisible by $ p$ for all $ n$, we must have $ p\\leq k$. Is this correct?",
"Solution_6": "Essentially. The theorem is called [url=http://en.wikipedia.org/wiki/Lagrange%27s_theorem_%28number_theory%29]Lagrange's theorem[/url]."
}
{
"Tag": [
"function",
"LaTeX",
"parameterization",
"trigonometry",
"calculus",
"derivative",
"calculus computations"
],
"Problem": "Suppose a smooth curve in $ \\mathbb{R}^2$ passes through the origin. Let $ s$ denote arclength as measured along the curve, with $ s > 0$ to the right of the origin and $ s < 0$ to the left.\r\n\r\nFix a constant $ k > 0.$ Suppose that the curve satisfies the property that $ y \\equal{} ks^2.$\r\n\r\nCan you find a more conventional representation (such as parametric form) for this curve? And does the curve have some familiar name?\r\n\r\nAs a possible intermediate step: Show that there is a particular number (which depends on $ k$) such that the arclength of the entire curve cannot exceed this number. In other words, this is a curve of finite length and bounded extent.",
"Solution_1": "The total length of the curve is 1/2k.\r\n\r\ny=ks^2 gives s as a function of y. Differentiating with respect to x, we get an equation in dy/dx \r\n\r\nThe equation is, \r\n\r\n{dy/dx}^2 = 1/{1/4ky - 1}. This puts a bound on y: y<1/4k. this means, s^2 < 1/4k^2. i.e., s<1/2k.\r\n\r\nthe above equation is solved by the substituting, y= 1/{t^2 + 4k}. \r\n\r\nThe final solution turns out to be,\r\n\r\n x= sqr.root{y/4k - y^2 } - (1/4k)tan^-1 {sqr.root(1/4ky - 1)} + pi/8k\r\n\r\nI am not able to get this in the form, y=f(x).",
"Solution_2": "One small comment: we actually have $ \\minus{} \\frac1{2k}\\le s\\le\\frac1{2k},$ with the curve extending on both sides of the origin, so the total length of the curve is $ \\frac1k.$\r\n\r\nIt's possible to get a nicer looking form than that. Not $ y \\equal{} f(x)$ but rather a pair of parametric equations, $ x \\equal{} x(t),y \\equal{} y(t).$",
"Solution_3": "Please learn a bit of [[LaTeX]]. (See also [url=http://www.artofproblemsolving.com/Forum/viewtopic.php?t=123690]here[/url].) \r\n\r\nWe see that taking $ y$ as a function of $ s$, $ y'(s) \\equal{} 2k s$, so $ |2ks| \\leq 1$. This suggests the parametrization $ s \\equal{} \\frac {\\sin t}{2k}$, so $ y \\equal{} \\frac {\\sin^2 t}{4k}$ and $ \\frac {dy}{dt} \\equal{} \\frac {\\sin t \\cos t}{2k}$. We have (taking all derivatives w.r.t. $ t$) that $ s'(t)^2 \\equal{} y'(t)^2 \\plus{} x'(t)^2$, so $ \\frac {\\cos^2 t}{4k^2} \\equal{} \\frac {\\sin^2 t \\cos^2 t}{4k^2} \\plus{} x'(t)^2$, and this gives $ x'(t)^2 \\equal{} \\frac {\\cos^4 t}{4k^2}$. Then (because I don't want to think about signs right now) we conclude from this that $ x'(t) \\equal{} \\frac {\\cos^2 t}{2k}$, and from here we can integrate to get the formula for $ x$. A few details to be patched up, but is this the sort of thing you were looking for?\r\n\r\n(Admission: I actually didn't think of the parametrization for $ s$ in the way I described in the second sentence, but rather because I started by parametrizing by arclength, which gives $ y'(s) \\equal{} 2k s$ and so $ x'(s) \\equal{} \\sqrt {1 \\minus{} 4k^2 s^2}$. This spits out an arcsine when you integrate, and [i]that's[/i] what got me to the substitution I used above.)",
"Solution_4": "It is a pretty famous curve. It got quite a bit of attention throughout the 17th century.",
"Solution_5": "substituting cot^2 t = 1/4ky - 1 in the solution mentioned in my previous post, the parametric equations turn out to be:\r\n x = (1-cos t)/8k; y = (t+ sin t)/8k",
"Solution_6": "[quote=\"bharathhm\"] x = (1-cos t)/8k; y = (t+ sin t)/8k[/quote]\r\nNow, this is starting to look familiar. Does anyone recognize the curve from that description?",
"Solution_7": "Cycloid. :lol:\r\n[url=http://images.google.co.jp/imgres?imgurl=http://www.scitechantiques.com/cycloidhtml/images/cycloid_versus_simple.jpg&imgrefurl=http://www.scitechantiques.com/cycloidhtml/&usg=__RTEEfWBr5aBU6aWbt8DoA6lccFI=&h=317&w=400&sz=43&hl=ja&start=10&sig2=y-0AsdEAAARDz6ZkCDje5A&um=1&tbnid=bkaJwLsjllHL9M:&tbnh=98&tbnw=124&prev=/images%3Fq%3DCycloid%26hl%3Dja%26rlz%3D1T4ADBR_jaJP295JP295%26sa%3DX%26um%3D1&ei=VQ3ESeHDONKAkQWCkOzXDA]Galileo's Pendlum.[/url]",
"Solution_8": "Or, the [url=http://en.wikipedia.org/wiki/Tautochrone_curve]tautochrone[/url].",
"Solution_9": "This is the cycloid oriented so its cusps point upward. \r\n\r\nAthough the cycloid has many uses and appears in many problems, expressing it in exactly the form I started with here emphasizes the tautochrone property. (Had jmerry not already posted that particular link, I was planning to do so myself.)\r\n\r\nWhat is the motion of a bead sliding on this wire without friction? Gravitational potential energy is proportional to $ y$ and $ y$ is in turn proportional to $ s^2.$ This is precisely the setup needed to have $ \\frac{d^2s}{dt^2}\\equal{}\\minus{}cs,$ which is the equation of simple harmonic motion. And in simple harmonic motion, frequency is independent of amplitude, which is simply another way to state the tautochrone property."
}
{
"Tag": [],
"Problem": "hi,\r\n\r\ni am currently doing some coursework of which requires me to need to find out what it means, can you help. the table below is what i have for information.\r\n\r\n[img]http://www.redihot.com/images/english_03.gif[/img]\r\n\r\nthanks alot, pleae click it to see it better.",
"Solution_1": "thanks alot for not helping, although i found out, the 100 represents the average so the mirror is less meaning it is less popular than the average newspaper",
"Solution_2": "Errr, if you wanted help, youd really have to give more information. What the hell are we supposed to be getting the mean of exactly?",
"Solution_3": "just so you know, what you found is stated on the top left corner where it says \r\n\r\n\"index: 100=UK national average\"\r\n\r\nthat's about all i see that you're gonna get from the table, unless if there's some more information you have, i.e specifically what you're supposed to do."
}
{
"Tag": [
"geometry",
"inequalities",
"geometry unsolved"
],
"Problem": "How many triangles $ ABC$ are there if radius of the inscribed circle of the triangle $ ABC$ is $ 1$ and all sides are integer?",
"Solution_1": "Let $ BC\\equal{}a,AB\\equal{}c,AC\\equal{}b$ where $ a\\leq{b}\\leq{c}$ are integers. If $ r\\equal{}1$ then $ a,b,c$ satisfy $ (b\\plus{}c\\minus{}a)(a\\plus{}b\\minus{}c)(a\\plus{}c\\minus{}b)\\equal{}4(a\\plus{}b\\plus{}c)$. \r\nMoreover $ a\\leq{12}$. \r\nProof: $ 4(a\\plus{}b\\plus{}c)\\leq{1}2c$ and $ (b\\plus{}c\\minus{}a)(a\\plus{}b\\minus{}c)(a\\plus{}c\\minus{}b)\\geq{ac}$. Thus $ ac\\leq{12c}$ and $ a\\leq{12}$.\r\nFor $ a\\leq{b}\\leq{c}\\leq{3000}$ there is only one solution: $ 3,4,5$.",
"Solution_2": "Another note is that when this occurs(inradius as 1)\r\n\r\nK=s where s is the semiperimeter of the triangle",
"Solution_3": "Moreover $ c\\leq{b}\\plus{}12$. \r\nProof: $ c\\leq{a}\\plus{}b\\leq{b}\\plus{}12$.\r\nFor $ a\\leq{b}\\leq{c}\\leq{1}0^5$ there is only one solution: $ 3,4,5$.",
"Solution_4": "[quote=\"loup blanc\"]Let $ BC \\equal{} a,AB \\equal{} c,AC \\equal{} b$ where $ a\\leq{b}\\leq{c}$ are integers. If $ r \\equal{} 1$ then $ a,b,c$ satisfy $ (b \\plus{} c \\minus{} a)(a \\plus{} b \\minus{} c)(a \\plus{} c \\minus{} b) \\equal{} 4(a \\plus{} b \\plus{} c)$. \nMoreover $ a\\leq{12}$. \nProof: $ 4(a \\plus{} b \\plus{} c)\\leq{1}2c$ and $ (b \\plus{} c \\minus{} a)(a \\plus{} b \\minus{} c)(a \\plus{} c \\minus{} b)\\geq{ac}$. Thus $ ac\\leq{12c}$ and $ a\\leq{12}$.\nFor $ a\\leq{b}\\leq{c}\\leq{3000}$ there is only one solution: $ 3,4,5$.[/quote]\r\nJust let $ x \\equal{} b \\plus{} c \\minus{} a,y \\equal{} a \\plus{} b \\minus{} c,z \\equal{} a \\plus{} c \\minus{} b$ then $ xyz \\equal{} 4(x \\plus{} y \\plus{} z)$.\r\nWlog $ x \\le y \\le z$. If $ x \\ge 4$ then: $ 4(yz \\minus{} 1) \\le x(yz \\minus{} 1) \\equal{} 4(y \\plus{} z) \\iff yz \\minus{} 1 \\le y \\plus{} z \\iff (y \\minus{} 1)(z \\minus{} 1) \\le 2$ which is impossible. So $ x \\in \\{1,2,3\\}$.\r\nSince $ a \\plus{} b \\minus{} c \\equiv a \\minus{} b \\plus{} c \\equiv \\minus{} a \\plus{} b \\plus{} c \\bmod 2$ we see $ x \\equiv y \\equiv z \\bmod 2$. Since $ 4 \\mid xyz$ we see $ 2 \\mid x,y,z$ and therefore $ x \\equal{} 2$. Inserting:\r\n$ 2yz \\equal{} 4(y \\plus{} z \\plus{} 2) \\iff yz \\equal{} 2(y \\plus{} z \\plus{} 2) \\iff (y \\minus{} 2)(z \\minus{} 2) \\equal{} 8$, and therefore $ y \\equal{} 4,z \\equal{} 6$, which in turn gives $ a \\equal{} 5,c \\equal{} 4,b \\equal{} 3$. So the only triangle is the $ 3,4,5$ triangle.",
"Solution_5": "Hi Mathias,\r\nyou have not $ x(yz\\minus{}1)\\equal{}4(y\\plus{}z)$.",
"Solution_6": "[quote=\"loup blanc\"]Hi Mathias,\nyou have not $ x(yz \\minus{} 1) \\equal{} 4(y \\plus{} z)$.[/quote]\r\nI'm sorry for my silly mistake! It should be $ x(yz\\minus{}4) \\equal{} 4(y\\plus{}z)$. The inequality then becomes $ (y\\minus{}1)(z\\minus{}1) \\le 5$ which is impossible too :)",
"Solution_7": "Pretty proof Mathias.",
"Solution_8": "[quote=\"Mathias_DK\"][quote=\"loup blanc\"]Hi Mathias,\nyou have not $ x(yz \\minus{} 1) \\equal{} 4(y \\plus{} z)$.[/quote]\nI'm sorry for my silly mistake! It should be $ x(yz \\minus{} 4) \\equal{} 4(y \\plus{} z)$. The inequality then becomes $ (y \\minus{} 1)(z \\minus{} 1) \\le 5$ which is impossible too :)[/quote]\n[quote]Couldnt understand this. $ 4(yz \\minus{} 1) \\le x(yz \\minus{} 4)$[/quote]",
"Solution_9": "Hi ertanrock, reread posts #6,#7.\r\n$ 4(yz\\minus{}4)\\leq{x}(yz\\minus{}4)\\equal{}4(y\\plus{}z)$ implies $ (y\\minus{}1)(z\\minus{}1)\\leq{5}$ that is impossible because $ 4\\leq{y}\\leq{z}$.",
"Solution_10": "[quote=\"loup blanc\"]Hi ertanrock, reread posts #6,#7.\n$ 4(yz \\minus{} 4)\\leq{x}(yz \\minus{} 4) \\equal{} 4(y \\plus{} z)$ implies $ (y \\minus{} 1)(z \\minus{} 1)\\leq{5}$ that is impossible because $ 4\\leq{y}\\leq{z}$.[/quote]\r\n\r\nok now. Got it.\r\nI thought like that. $ 4(yz \\minus{} 1)\\leq{x}(yz \\minus{} 4)$"
}
{
"Tag": [],
"Problem": "Heron triangles are triangles with integer sides and integer area.They are Primitive if their sides taken together have no common factors(not necessarily pairwise) \r\n1) Prove there are infinitely many primitive Heron Triangles two of whose sides have a common factor of 5.\r\n(example:(9,65,70),)\r\n2)determine a primitive heron triangle two of whose sides have a common factor of 65\r\n3)prove or disprove the existence of a primitive heron triangle each of whose sides is a square? :D",
"Solution_1": "any one has a solution?",
"Solution_2": "[quote=\"anorithdialga\"]any one has a solution?[/quote]\r\n\r\nyes, anyone has a solution",
"Solution_3": "coincidentally, i m reading the same only,ill respond in 4 long days!!\r\nedit:wastage of post i think",
"Solution_4": "Ok this is the first one:\r\n[hide=\"2\"]Every right triangle is a heron triangle, so their sum or difference should be also a heron triangle, thus forming acute and obtuse heron triangles. Here, we have to get two right triangles with sides multiple of 65. A right triangle with hypotenuse 65 gives options as (63,16,65) and (33,56,65). Choosing any one and creating an isosceles triangle with it does the job.[/hide]"
}
{
"Tag": [],
"Problem": "$ \\frac{a}{b}\\equal{}\\sqrt{2}\\plus{}1$. Then the following expression value is ?\r\n\r\n$ \\frac{a^{3}\\plus{}b^{3}}{a^{3}\\minus{}a^{2}.b\\plus{}a.b^{2}}$",
"Solution_1": "$ a^3 \\plus{} b^3 \\equal{} (a \\plus{} b)(a^2 \\minus{} ab \\plus{} b^2)$\r\n\r\n$ a^3 \\minus{} a^2b \\plus{} ab^2 \\equal{} a^3 \\minus{} ab(a \\minus{} b) \\equal{} a\\left(a^2 \\minus{} ab \\plus{} b^2\\right)$\r\n\r\n$ \\frac {a^{3} \\plus{} b^{3}}{a^{3} \\minus{} a^{2}.b \\plus{} a.b^{2}} \\equal{} \\frac {(a \\plus{} b)(a^2 \\minus{} ab \\plus{} b^2)}{a\\left(a^2 \\minus{} ab \\plus{} b^2\\right)}$\r\n\r\n$ \\equal{} \\frac {a \\plus{} b}{a}$\r\n\r\n$ \\equal{} b\\left(\\frac {\\frac {a}{b} \\plus{} 1}{a}\\right)$\r\n\r\n$ \\equal{} \\left(\\frac {\\frac {a}{b} \\plus{} 1}{\\frac {a}{b}}\\right)$\r\n\r\n$ \\equal{} \\frac {\\sqrt {2} \\plus{} 2}{\\sqrt {2} \\plus{} 1}$",
"Solution_2": "You can simplify the result.",
"Solution_3": "$ \\frac{\\sqrt{2}\\plus{}2}{\\sqrt{2}\\plus{}1} \\equal{}\\frac{(\\sqrt{2}\\plus{}2)(\\sqrt{2}\\minus{}1)}{(\\sqrt{2}\\plus{}1)(\\sqrt{2}\\minus{}1)}$\r\n\r\n$ \\equal{}2\\minus{}\\sqrt{2}\\plus{}2\\sqrt{2}\\minus{}2$\r\n\r\n$ \\equal{}\\sqrt{2}$",
"Solution_4": "O.K. :lol:",
"Solution_5": "Thanks, but both of the answers are correct ?",
"Solution_6": "[quote=\"luiseduardo\"]$ \\frac {a}{b} \\equal{} \\sqrt {2} \\plus{} 1$. Then the following expression value is ?\n\n$ \\frac {a^{3} \\plus{} b^{3}}{a^{3} \\minus{} a^{2}.b \\plus{} a.b^{2}}$[/quote]\r\n\r\nHere is my solution:\r\n\r\n$ \\frac {a^{3} \\plus{} b^{3}}{a^{3} \\minus{} a^{2}.b \\plus{} a.b^{2}}\\equal{}\\frac{(a\\plus{}b)(a^2\\minus{}ab\\plus{}b^2)}{a(a^2\\minus{}ab\\plus{}b^2)}\\equal{}1\\plus{}\\frac{b}{a}$\r\n\r\n$ \\equal{}1\\plus{}\\frac{1}{\\sqrt{2}\\plus{}1}\\equal{}1\\plus{}\\sqrt{2}\\minus{}1\\equal{}\\boxed{\\sqrt{2}}$.",
"Solution_7": "Ah ok :blush: \r\nThanks.",
"Solution_8": ":)"
}
{
"Tag": [
"geometry",
"perimeter",
"geometric transformation",
"dilation",
"trigonometry",
"number theory",
"relatively prime"
],
"Problem": "The perimeter of triangle $APM$ is $152,$ and the angle $PAM$ is a right angle. A circle of radius $19$ with center $O$ on $\\overline{AP}$ is drawn so that it is tangent to $\\overline{AM}$ and $\\overline{PM}.$ Given that $OP=m/n,$ where $m$ and $n$ are relatively prime positive integers, find $m+n.$",
"Solution_1": "is the answer:\r\n\r\n[hide]\n\n$\\frac{m}{n}= \\frac{7}{8}\\implies m+n = 15$?\n\nwhere is this from?\n\n[/hide]",
"Solution_2": "this problem was more: \"can you simplify algebra\" rather than any cool geometry...\r\n\r\nanyway, denote $OP=x$, $PT=y$, dialate by 1/19, denote the circle tangent to $PM$ at $T$, so $OT=OA=1$, and $p=8$\r\n\r\n$\\triangle OTP\\sim\\triangle MAP$, so\r\n\r\n$\\frac{1}{MA}=\\frac{x}{MP}=\\frac{y}{x+1}=\\frac{1+x+y}{8}$\r\n\r\nso $(7-x)y=(x+1)^{2}$, and $1+y^{2}=x^{2}$, so $(x-7)^{2}[(x-1)(x+1)]=(x+1)^{4}$, since this is easily expanded, and solved, we obtain $x=\\frac{5}{3}$, then undo the dialation, so $OP=\\frac{95}{3}\\implies m+n=\\boxed{098}$",
"Solution_3": "[quote=\"Altheman\"]this problem was more: \"can you simplify algebra\" rather than any cool geometry...\n[/quote]\r\n[color=red]A cool problem [/color]:D\r\n [hide]\nLet restate the problem. Will just change the letters and will dilate by 19 (see Altheman solution). Triangle $ABC$ is right ($A=90$) with perimeter of 8. Circle with center $D$ on $AC$ touches $AB$ and $BC$ and has radius 1. Find $DC$.\n Construct $GF$ parallel to $AC$ and tangent to the circle. \nLet $AB=x$. The perimeter of $\\Delta BFG=BF+FE+EG+BG=BF+FH+GA+BG=2x$ as $\\Delta BFG \\sim \\Delta BAC$ we have $\\frac{BG}{BA}=\\frac{2x}{8}$ \n$\\frac{x-1}{x}=\\frac{2x}{8}\\Rightarrow x^{2}-4x+4=0$ or $x=AB=2$.\n$\\tan{DAB}=\\frac{DA}{BA}=\\frac12$. As $CD$ is angle bisector $\\tan{ABC}= \\frac43$ (by double angle formula).\nSo $\\frac43=\\frac{AC}{AB}$, $AC=\\frac83$, $DC=\\frac53$ [/hide]",
"Solution_4": "excuse me, but what does the p mean in your solution?\r\ndenote the circle tangent to PM at T, so OT=OA=1, and p=8",
"Solution_5": "In Altheman's solution, $ p$ represents the [i]perimeter[/i] of $ \\triangle APM$.",
"Solution_6": "[hide=Solution]We see that the circle can only be tangent at $A.$ Suppose it it tangent at $X$ instead. Then $OX \\perp AM,$ but $OA\\perp AM.$ Thus, $OA \\parallel OX.$ But they meet at $O,$ so this is a contradiction. Therefore, circle $O$ is tangent to $AM$ at $A,$ and $OA=19.$\n\nCall the point where $O$ meets $PM,$ $Y.$ Note that $OY=19.$ Then, let $x=AM=MY$ ($AM=MY$ because they are tangents from $M$), and $OP=a.$ By Pythagoras, $PY = \\sqrt{a^2-19^2}.$\n\n$\\triangle POY \\sim \\triangle PMA$ by AA similarity because they share $\\angle APM$ and $\\angle PAM = \\angle PYO = 90.$ Hence, $AP/AM=PY/OY,$ or $\\frac{a+19}{x}=\\frac{\\sqrt{a^2-19^2}}{19}.$ Solving for $x,$ we have $x=\\frac{19a+19^2}{\\sqrt{a^2-19^2}}.$\n\nWe know that the perimeter is $152,$ which means that $a+19+2x+\\sqrt{a^2-19^2}=152.$ Substituting our value for $x,$$$a+19+\\frac{38a+2\\cdot19^2}{\\sqrt{a^2-19^2}}+\\sqrt{a^2-19^2}=152.$$Multiplying by $\\sqrt{a^2-19^2},$$$(a+19)\\sqrt{a^2-19^2}+38a+2\\cdot 19^2+a^2-19^2=152\\sqrt{a^2-19^2}.$$Moving the square roots to the LHS and everything else to the LHS and factoring,$$(a+19-152)\\sqrt{a^2-19^2}=-(a+19)^2.$$Squaring and using difference of squares,$$(a-7 \\cdot 19)^2 (a+19)(a-19)=(a+19)(a+19)^3.$$Canceling and expanding (and moving to one side),$$9a^2-570a+19^2\\cdot 50 =0.$$Using the quadratic formula, the only solution is $\\frac{95}{3}\\implies \\boxed{098}.$ [/hide]",
"Solution_7": "[hide=Solution]Scale down by $19$; then the radius is $1$ and the perimeter is $8$. Let $T$ be the tangency point between the circle and $\\overline{PM}$. Let $x=OP$. Since $\\overline{OM}$ bisects $\\angle PMA$, we can use the angle-bisector theorem to get $AM=k$ and $PM=kx$. Additionally, $TM=AM=k$.\n\nBecause the perimeter is $8$, we have $kx+k+x+1=8 \\implies k(x+1)=7-x \\implies k = \\frac{7-x}{x+1}$.\n\nSince $PT=kx-k$, we use the Pythagorean Theorem on $\\triangle PTO$ to get:\n$$(kx-k)^2+1=x^2$$\n$$k^2(x-1)^2 = x^2-1$$\n$$k^2(x-1) = x+1$$\n$$\\left(\\frac{7-x}{x+1}\\right)^2(x-1)=x+1$$\n$$(x-7)^2(x-1)=(x+1)^3$$\n$$x^3-15x^2+63x-49=x^3+3x^2+3x+1$$\n$$18x^2-60x+50=0$$\n$$9x^2-30x+25=0$$\n$$(3x-5)^2=0$$\n$$x=\\frac53$$\nScaling up, we get the answer $\\frac{95}{3}\\implies\\boxed{098}$.",
"Solution_8": "Scale down by $19$, and reflect $M$ across $AP$ to $M'$ so that said circle is the incircle of $\\triangle MM'P$. Let $(p,m):=(PA,MA)$.\n[list=1]\n[*] The perimeter condition becomes $p+m+\\sqrt{p^2+m^2}=8$. [/*]\n[*] The radius condition becomes, by $A=rs$, \n\\[pm=1(AM+PM)=\\text{Perimeter}(\\triangle APM)-AP=8-p.\\] [/*]\n[/list]\nEquation (1) becomes\n\\begin{align*}\np^2+m^2&=(8-p-m)^2\\\\\n&=(p+m)^2-16(p+m)+64\\\\\n\\implies32&=8(p+m)-pm.\n\\end{align*}\nSubstituting $pm=8-p$ and $m=\\tfrac{8-p}{p}$ from Equation (2),\n\\begin{align*}\n8(p+\\tfrac{8-p}{p})-(8-p)&=32\\\\\n8(p^2+(8-p))-p(8-p)&=32(8-p)\\\\\n9p^2-48p+64&=0.\n\\end{align*}\nSo $p=\\tfrac{8}{3}$ or $OP=\\tfrac{5}{3}$, giving $\\tfrac{95}{3}$."
}
{
"Tag": [],
"Problem": "In a room, each 22 liters of air weighs 29 grams. Each liter is 100 cubic centimeters. How much does a cubic meter of air in that room weigh? Express your answer to the nearest whole gram.",
"Solution_1": "[quote=\"mr. math\"]In a room, each 22 liters of air weighs 29 grams. Each liter is 100 cubic centimeters. How much does a cubic meter of air in that room weigh? Express your answer to the nearest whole gram.[/quote]\r\nEach liter by the way, it 1000 cubic centimeters since 1 cubic centimeter is 1 milliliter. \r\n[hide] $1 m^{3}=(100)^{3}cm^{3}=1000000 cm^{3}$. We also know that 1 Liter=1000 cc (cc is cubic centimeters), so 1 cubic meter is (1000)(1000) cc=1000 liters. Thus, since each liter weighs 29 grams, so 1000 liters weighs 29 kilograms, which is our answer. [/hide]"
}
{
"Tag": [
"trigonometry",
"AMC",
"AIME"
],
"Problem": "Given that $z$ is a complex number such that $z+\\frac 1z=2\\cos 3^\\circ,$ find the least integer that is greater than $z^{2000}+\\frac 1{z^{2000}}.$",
"Solution_1": "[hide]\n\n\n\nLet , then , so , so , so then the given equation evaluates to , and the answer is .\n\n\n\n[/hide]",
"Solution_2": "I got that exact same answer, but I didn't know AIME answers can be 0, or can they? :)",
"Solution_3": "[quote=\"1234567890\"]I got that exact same answer, but I didn't know AIME answers can be 0, or can they? :)[/quote]\r\n\r\nYeah, they can. It's 000-999. 1000 choice MC test? :P",
"Solution_4": "[hide=\"Answer\"]We have $z=e^{i\\theta}$, so $e^{i\\theta}+\\frac{1}{e^{i\\theta}}=\\frac{\\cos \\theta}{\\cos ^2\\theta +\\sin ^2\\theta}+\\cos \\theta +i\\sin \\theta - \\frac{i\\sin \\theta}{\\cos ^2\\theta +\\sin ^2\\theta}=2\\cos \\theta$. Therefore, $\\theta =3^\\circ=\\frac{\\pi}{60}$. From there, $z^{2000}+\\frac{1}{z^{2000}}=e^{\\frac{100\\pi (i)}{3}}+e^{\\frac{-100\\pi (i)}{3}}=2\\cos \\frac{4\\pi}{3}=-1$, so our answer is $\\boxed{0}$.[/hide]",
"Solution_5": "the question ask for \"the least integer that is GREATER than......\", not \"not less than\", so -1 can greater than -1, the answer is 0",
"Solution_6": "The question is amazingly easy (like 10 sec) if you know that the answer can ONLY be -1, 0 or 1 (AIME only 000 or 001), no matter what inteter power z is taken to and no matter the angle degree (in this situation its 0)...\r\n\r\nthere is a generalized formula in AoPS vol 2 for these types of problems",
"Solution_7": "[quote=Elemennop][hide]\n\n\n\nLet , then , so , so , so then the given equation evaluates to , and the answer is .\n\n\n\n[/hide][/quote]\nhuh?\n",
"Solution_8": "w0W that revive.\n\nProbably displaying issues from so old. That's why it looks so clipped and stutter-y.",
"Solution_9": "[quote=JesusFreak197][hide=\"Answer\"]We have $z=e^{i\\theta}$, so $e^{i\\theta}+\\frac{1}{e^{i\\theta}}=\\frac{\\cos \\theta}{\\cos ^2\\theta +\\sin ^2\\theta}+\\cos \\theta +i\\sin \\theta - \\frac{i\\sin \\theta}{\\cos ^2\\theta +\\sin ^2\\theta}=2\\cos \\theta$. Therefore, $\\theta =3^\\circ=\\frac{\\pi}{60}$. From there, $z^{2000}+\\frac{1}{z^{2000}}=e^{\\frac{100\\pi (i)}{3}}+e^{\\frac{-100\\pi (i)}{3}}=2\\cos \\frac{4\\pi}{3}=-1$, so our answer is $\\boxed{0}$.[/hide][/quote]\n\n \nIsn't Euler identity excluded from most content sets?",
"Solution_10": "I just realized I bumped a 9 year old topic oops",
"Solution_11": "I think this is much nicer to look at.\n\n[hide]\nLet $z=r(\\cos\\theta+i\\sin\\theta).$ The crucial claim is that $r=1$ (and the rest of the problem follows pretty easily from here).\n\nProof: Notice that $z+\\frac{1}{z}=r(\\cos\\theta+i\\sin\\theta)+\\frac{1}{r}(\\cos\\theta+i\\sin\\theta)=\\cos\\theta(r+\\frac{1}{r})+i\\sin\\theta(r-\\frac{1}{r}).$ Since $2\\cos 3^{\\circ}$ has no imaginary part, $r-\\frac{1}{r}=0\\to r=\\frac{1}{r}\\to r^2=1\\to r=1$ (as $r$ is a positive number).\n\nNow the rest of the problem follows.\n\nPlugging $r=1$ into the equation we see that $2\\cos\\theta=2\\cos 3^{\\circ},$ so $z=cis 3^{\\circ}.$\n\nThus $z^{2000}+z^{-2000}=cis(3\\cdot 2000)+cis(3\\cdot -2000)=-1,$ so the answer is $0.$\n[/hide]",
"Solution_12": "Whaaat I didn\u2019t know $0$ could be an answer!\n\n$z + \\frac{1}{z} = 2 \\cos 3^\\circ \\implies z^2 - (2 \\cos 3 )z + 1 = 0.$ So, $z = \\frac{2\\cos 3 \\pm \\sqrt{4\\cos^2 3 - 4}}{2} = \\cos 3 + i\\sin 3.$ By De Moivre's Theorem, $z^{2000} = \\cos 6000^\\circ + i\\sin 6000^\\circ = -\\frac{1}{2} - \\frac{\\sqrt{3}}{2}i$ Thus, $z^{2000}+\\frac{1}{z^{2000}} = 2\\cos 240^\\circ = -1.$ Therefore the answer is the least integer greater than $-1,$ which is $\\boxed{0}.$",
"Solution_13": "Let's denote $z = e^{i \\theta}$. Then we have $e^{i\\theta} + e^{-i\\theta} = 2 \\cos 3^\\circ$, and want to find $e^{i2000\\theta} + e^{-i2000\\theta}$. From the first equation, we have $\\cos \\theta + i \\sin \\theta + \\cos (-\\theta) + i \\sin (-\\theta) = 2 \\cos \\theta = 2 \\cos 3^\\circ$, so $\\cos \\theta = \\cos 3^\\circ \\implies \\theta = 3^\\circ$. We want to find $\\cos 2000\\theta + i \\sin 2000\\theta + \\cos (-2000\\theta) + i \\sin (-2000\\theta) = 2\\cos(2000\\theta) = 2 \\cos(6000^\\circ)$. This evaluates to $-1$, so the answer is $\\boxed{0}$."
}
{
"Tag": [],
"Problem": "What exactly is a convex hull ?? :?",
"Solution_1": "http://en.wikipedia.org/wiki/Convex_hull\r\nhttp://mathworld.wolfram.com/ConvexHull.html\r\n\r\n:)",
"Solution_2": "Thanks..... :) ......Just by the way , are there any proofs for that expression on mathworld ?? :?"
}
{
"Tag": [
"topology"
],
"Problem": "Suppose that metric space (X,d) is connected. Prove that for every two points a,b $ \\in$ X, and for every positive number $ \\delta$ exists finite sequence of points $ c_1, ... , c_n \\in$ X which satisfy conditions: $ a \\equal{} c_1, b \\equal{} c_n$, and for every k $ \\in$ {1,...,n}: $ d(c_k,c_{k \\plus{} 1})\\le \\delta$",
"Solution_1": "Let $ S(a)$ denote the set of points $ b$ reachable in this way. Let $ T(a)$ denote the set of points not reachable. \r\n\r\nThen $ S(a)$ is open, since each $ b \\in S(a)$ has a $ \\delta$ neighborhood that is also reachable. $ T(a)$ is also open. For if $ d \\in T(a)$ is not reachable \r\nin finitely many $ \\delta$-steps, then nothing in a $ \\delta$-neighborhood of $ d$ can be reachable either. Otherwise we may extend a path to such a point to $ d$. \r\n\r\nThen $ X \\equal{} S(a) \\cup T(a)$, while $ S(a) \\cap T(a) \\equal{} \\emptyset$ and each set $ S(a), T(a)$ is open. Therefore either $ S(a) \\equal{} \\emptyset$ or $ T(a) \\equal{} \\emptyset$. Since $ a \\in S(a)$, we conclude $ T(a) \\equal{} \\emptyset$ and so $ S(a) \\equal{} X$."
}
{
"Tag": [
"Ring Theory",
"MATHCOUNTS"
],
"Problem": "when is nats ths year?",
"Solution_1": "I think the date is between May 6 and May 9!! :P",
"Solution_2": "sometime in may :P",
"Solution_3": "May 7th to May 10th, as found [url=http://mathcounts.org/Page.aspx?pid=294]here[/url].\r\n\r\nIf you require some extra ...uh... \"motivation,\" look at the [url=http://www.swandolphin.com/home.html]hotel[/url]. Don't spend all your time looking at it though; go practice.",
"Solution_4": "[quote=\"xpmath\"]If you require some extra ...uh... \"motivation,\" look at the [url=http://www.swandolphin.com/home.html]hotel[/url]. [/quote]\r\n\r\nWhere did you read that? I didn't see that on the mathcounts website... I guess I am still in awe.",
"Solution_5": "At [url=http://mathcounts.org/Page.aspx?pid=1336]this page[/url], it's stated that \"The 2009 Raytheon MATHCOUNTS National Competition will be held at Walt Disney World's Swan and Dolphin Resort in Orlando, Florida!\"\r\n\r\nSearching \"Swan and Dolphin Resort\" on Google gives the website."
}
{
"Tag": [
"number theory solved",
"number theory"
],
"Problem": "Given integers a>b>0 let\r\n\r\nx= \\sqrt a+ \\sqrt b y= \\sqrt a- \\sqrt b\r\n\r\nIf a-b is twice an odd integer prove that both x and y are irrational.",
"Solution_1": "Sorry for my typing ; naturally x and y are distinct numbers\r\n\r\nx= \\sqrt a+ \\sqrt b\r\n\r\ny= \\sqrt a- \\sqrt b",
"Solution_2": "Suppose x is rational. Then xy=a-b is rational, so y is rational.\r\nThen 2 \\sqrt a=x+y is rational, so \\sqrt a si rational.\r\nSimilarly \\sqrt b is rational.\r\nSince a,b are integers and \\sqrt a, \\sqrt b are rational, it follows that a and b are squares.\r\nLet a=A 2 , b= B2.\r\nSo A 2 -B 2 =4k+2, which is impossible mod 4 (2 divides A-B iff 2 divides A+B)."
}
{
"Tag": [
"number theory open",
"number theory"
],
"Problem": "For sufficiently large $A, n$, can it be said that $A^n + 1$ is always squarefree?",
"Solution_1": "Here is a topic concerning a closely related problem.\r\n[url=http://www.mathlinks.ro/Forum/viewtopic.php?t=30001]http://www.mathlinks.ro/Forum/viewtopic.php?t=30001[/url]",
"Solution_2": "Suppose that for all sufficently large $A$, and $n$, $A^n + 1$ is squarefree.\r\n\r\nSince for all $A,n$ [b]sufficently large[/b] $A^n + 1$ is squarefree, we can take $A$ to be even and not divisible by $3$ (we just take an $A$ being a bit larger ;)). Hence $A^n + 1 \\equiv 1 ( \\mbox{mod } 2 )$ for all $n$. Take $n = 2 k$, for some positive integer $k$. Then $A^n + 1 \\equiv A^{2 k} + 1 \\equiv 2 ( \\mbox{mod } 3 )$. This means that we can take sufficently large $A, n$ such that $2 \\nmid A^n + 1$ and $3 \\nmid A^n + 1$ and $A^n + 1$ squarefree. Hence $A^n + 1 = \\prod_{j = 1}^m p_j$, with $p_j \\neq 2, 3$ for all $j = 1, \\ldots, m$. Hence $A^n + 1 \\equiv \\pm 1 (\\mbox{mod } 6 )$, but $A^n + 1 \\equiv A^{2 k} + 1 \\equiv 2 ( \\mbox{mod } 6 )$. Contradiction.\r\n\r\nIf $A$ is given, then remark that if there is some positive integer $k_0$ such that $A^{k_0} + 1 = a \\cdot b^2$ is not squarefree and $A^n + 1$ is squarefree for all sufficently large $n$ then $A^{k_0 \\cdot n} + 1$ is also squarefree for a sufficently large $n$. But $( A^{k_0 \\cdot n} + 1 ; A^{k_0} + 1 ) = A^{k_0} + 1$. Hence $a \\cdot b^2 = A^{k_0} + 1| A^{k_0 \\cdot n} + 1$ and $A^{k_0 \\cdot n} + 1$ is not squarefree. Contradiction.\r\n\r\nAlso I doubt there is some $A$ s.t for all $n$, $A^n + 1$ is squarefree. Hence that would imply that there is no integer $A$ s.t for all sufficently large $n$, $A^n + 1$ is squarefree.\r\n\r\nI hope it is clear.",
"Solution_3": "Just take any $A \\equiv -1 \\mod 4$ and any odd $n$ and you get big numbers such that $4|A^n+1$..."
}
{
"Tag": [
"geometry",
"ratio"
],
"Problem": "Prove that the perpendicular drawn from the point $ (4,1)$ on the join of $ (2,\\minus{}1)$ and $ (6,5)$ divides it in the ratio of $ 5: 13$ . I got its answer $ 5: 8$. I think there is some problem in calculation.So please show your solution with calculation. Thanks a lot.",
"Solution_1": "hello, let $ A(2;\\minus{}1)$, $ B(6;5)$ and $ C(4;1)$ then the line say $ g$ through $ A,B$ is $ y\\equal{}3/2x\\minus{}4$, the line which is perpendicular to $ g$ is $ h$: $ y\\equal{}\\minus{}2/3x\\plus{}11/3$ hence the intersection point $ D(46/13;17/13)$ so we get\r\n$ \\overline{AD}\\equal{}10/13\\sqrt{13}$\r\n$ \\overline{DB}\\equal{}16/13\\sqrt{13}$\r\nand we get\r\n$ \\frac{\\overline{AD}}{\\overline{DB}}\\equal{}5/8$.\r\nSonnhard.",
"Solution_2": "Thanks. I got the same.It means that it is written wrong in the book.Thanks again. :-)"
}
{
"Tag": [
"inequalities",
"inequalities unsolved"
],
"Problem": "Let $ a,b,c > 0$, prove that $ \\sum_{cyc}\\frac {a^2}b\\ge3\\sqrt [6]{\\frac{a^6 \\plus{} b^6 \\plus{} c^6}3}$",
"Solution_1": "[quote=\"akai\"]Let $ a,b,c > 0$, prove that $ \\sum_{cyc}\\frac {a^2}b\\ge3\\sqrt [6]{\\frac {a^6 \\plus{} b^6 \\plus{} c^6}3}$[/quote]\r\n\r\nPlease help me, Can_hang2007, arqady, dduclam, ... or nother one ??? Thanks !",
"Solution_2": "[quote=\"akai\"]Let $ a,b,c > 0$, prove that $ \\sum_{cyc}\\frac {a^2}b\\ge3\\sqrt [6]{\\frac {a^6 \\plus{} b^6 \\plus{} c^6}3}$[/quote]\r\nSee my hint here: http://www.mathlinks.ro/viewtopic.php?t=184602 :)",
"Solution_3": "We have a more weak inequality be\r\n$ \\sum_{cyc}\\frac {a^2}b\\ge\\sum\\sqrt [6]{\\frac {a^6 \\plus{} b^6 }2}$"
}
{
"Tag": [
"geometry",
"circumcircle",
"trigonometry",
"geometric transformation",
"homothety",
"rotation"
],
"Problem": "Let $\\omega$ be the nine-point circle of a triangle $ABC$. Prove that there exist precisely three points $P$, $Q$ and $R$ on the circumcircle of triangle $ABC$ whose Simson lines with respect to the triangle $ABC$ are tangent to $\\omega$. Prove also that the triangle $PQR$ formed by these three points is homothetic to the Morley triangle of the triangle $ABC$ (and the triangle with points of touching as the vertices is also homothetic to the Morley triangle).",
"Solution_1": "WLOG, assume that the relative size of the angles $\\alpha = \\angle CAB$, $\\beta = \\angle ABC$, $\\gamma = \\angle BCA$ is $\\beta < \\alpha < \\gamma$ (that's how I drew the triangle). Denote the Morley equilateral triangle $\\triangle DEF$, where D is the intersection of the appropriate trisectors of the angles $\\beta$,$\\gamma$, E the intersection of the appropriate trisectors of the angles $\\gamma$, $\\alpha$ and F the intersection of the appropriate trisectors of the angles $\\alpha$, $\\beta$. The angles formed by 2 trisectors from the same vertex of the $\\triangle ABC$ with the sides of the equilateral triangle $\\triangle DEF$ are:\r\n\r\n$\\angle CDE = \\frac{\\beta + \\pi}{3}$, $\\angle CED = \\frac{\\alpha + \\pi}{3}$\r\n\r\n$\\angle AEF = \\frac{\\gamma + \\pi}{3}$, $\\angle AFE = \\frac{\\beta + \\pi}{3}$\r\n\r\n$\\angle BFD = \\frac{\\alpha + \\pi}{3}$, $\\angle BDF = \\frac{\\gamma + \\pi}{3}$\r\n\r\nAn easy proof is to draw an arbitrary equilateral triangle $\\triangle DEF$, take the angles $\\alpha + \\beta + \\gamma = \\pi$, build the above angles on the sides of this equilateral triangle and show, for example, that the angles $\\angle FAB = \\frac{\\alpha}{3}$, $\\angle FBA = \\frac{\\beta}{3}$. Using sine theorem for the triangles $\\triangle AEF$, $\\triangle BDE$,\r\n\r\n$\\frac{EF}{AF} = \\frac {\\sin \\left ( \\frac{\\alpha}{3} \\right )}{\\sin \\left ( \\frac{\\gamma + \\pi}{3} \\right )}$, $\\frac{DE}{BF} = \\frac {\\sin \\left ( \\frac{\\beta}{3} \\right )}{\\sin \\left ( \\frac{\\gamma + \\pi}{3} \\right )}$, $\\frac{AF}{BF} = \\frac {\\sin \\left ( \\frac{\\beta}{3} \\right )}{\\sin \\left ( \\frac{\\alpha}{3} \\right )}$\r\n\r\nSince in addition, $\\angle AFB = 2\\pi - \\frac{\\pi}{3} - \\frac{\\alpha + \\pi}{3} - \\frac{\\beta + \\pi}{3} = \\pi - \\frac{\\alpha + \\beta}{3}$, from the sine theorem for the triangle $\\triangle ABF$, it follows that $\\angle FAB = \\frac{\\alpha}{3}$, $\\angle FBA = \\frac{\\beta}{3}$.\r\n\r\nWe are going to \"construct\" the Simson line perpendicular to the side EF of the Morley triangle $\\triangle DEF$. Simple angle chase reveals the this direction forms with the side AB the angle\r\n\r\n$\\pi - \\frac{\\beta + \\pi}{3} + \\frac{\\alpha}{3} - \\frac{\\pi}{2} = \\frac{\\pi}{6} + \\frac{\\alpha - \\beta}{3}$\r\n\r\nLet a line through the vertex A and forming this angle with the side AB intersect the circumcircle $(O)$ of the triangle $\\triangle ABC$ at a point K and let a normal from the point K to the side BC intersect the circumcircle at another point L. The point L is the pole of the desired Simson line. The line KL perpendicular to the side BC forms the angle $\\frac{\\pi}{2} - \\beta$ with the side AB and consequently, the angle $\\angle AKL$ is equal to\r\n\r\n$\\angle AKL = \\frac{\\pi}{2} - \\beta - \\left ( \\frac{\\pi}{6} + \\frac{\\alpha - \\beta}{3} \\right ) = \\frac{\\pi - (2\\beta + \\alpha)}{3} = \\frac{\\gamma - \\beta}{3}$\r\n\r\nContinuing the angle chase around the circumcircle, the angle $\\angle AKB = \\angle ACB = \\gamma$ (spans the same circumcircle arc AB) and\r\n\r\n$\\angle KBC = \\frac{\\pi}{2} - (\\gamma - \\angle AKL) = \\frac{\\pi}{2} - \\left ( \\gamma - \\frac{\\gamma - \\beta}{3} \\right ) = \\frac{\\pi}{2} - \\frac{2\\gamma + \\beta}{3}$\r\n\r\n$\\angle KBA = \\beta + \\angle KBC = \\beta + \\frac{\\pi}{2} - \\frac{2\\gamma + \\beta}{3} = \\frac{\\pi}{2} - \\frac{2(\\gamma - \\beta)}{3}$\r\n\r\nThe angles $\\angle KLA = \\angle KBA$ are equal, because they span the same circumcircle arc KA, and the angle $\\angle AOL = 2\\angle AKL$ is the central angle of the circumcircle arc AL. Hence\r\n\r\n$\\angle KLO = \\angle OLA - \\angle KLA = \\frac{\\pi - \\angle AOL}{2} - \\angle KLA = \\frac{\\pi}{2} - \\angle AKL - \\angle KBA =$\r\n\r\n$= \\frac{\\pi}{2} - \\frac{\\gamma - \\beta}{3} - \\left ( \\frac{\\pi}{2} - \\frac{2(\\gamma - \\beta)}{3} \\right ) = \\frac{\\gamma - \\beta}{3} = \\angle KLA$\r\n\r\nConsequently, the line AK (having the same direction as the Simson line perpendicular to the Morley triangle side EF) and the line LO (from the pole of this Simson line through the circumcenter O) form the same angles with their transversal KL and they are therefore parallel. As a result, we know that the three Simsons lines perpendicular to the three sides DE, EF, FD of the Morley triangle are parallel to the lines from their poles through the circumcenter O.\r\n\r\nAccording to Honsberger's Episodes (Chapter 11):\r\n[b]1. [/b]The orthopole of a line lies on the Simson line perpendicular to it.\r\n[b]2. [/b]If a line intersects the circumcircle, the two Simson lines corresponding to the two intersections meet at the orthopole of the line. \r\n[b]3. [/b]The orthopole of a line passing through the circumcenter lies on the 9-point circle.\r\n\r\nLet the line LO intersect the circumcircle $(O)$ at a point M diametrally opposite to L. The two Simson lines having poles L and M intersect at the orthopole of this line, which lies on the 9-point circle, and they are perpendicular to each other. Since the Simson line with the pole L is parallel to LM, the Simson line with the pole M is perpendicular to it. Altogether, there are three Simson lines perpendicular to the sides DE, EF, FD of the Morley triangle, respectively, and they mutually form angles $\\frac {\\pi}{3}$. Likewise, there are three Simson lines perpendicular to first three, which meet them on the 9-point circle. All that remains is to show one of the first three Simson lines passes through the center N of the 9-point circle. The Simson line perpendicular to it will then be a tangent to the 9-point circle and similarly for the other two pairs of Simson lines. For this, another theorem from Honsberger's Episodes (Chapter 5) comes handy:\r\n\r\n[b]4. [/b]A Simson line passes through the midpoint of the segment from its pole to the orthocenter and this midpoint lies on the 9-point circle.\r\n\r\nThe 9-point circle is the circumcircle of the medial triangle, hence, its radius is half of the circumradius. In addition, the orthocenter H is the external homothety center of the circumcircle $(O)$ and the 9-point circle $(N)$ (the centroid G is their internal homothety center). Therefore, the midpoint S of the segment HL and the pole L are centrally similar with the same homothety center H and the same coefficient $\\frac 1 2$. Since the line LM and the Simson line perpendicular the EF are parallel and since they pass through the corresponding points L, S, they are themselves centrally similar with the same homothety center and coefficient. Since the line LM passes through the circumcenter O, the parallel Simson line must pass through the center N of the 9-point circle $(N)$.\r\n\r\nQ.E.D.\r\n\r\nEdit: Honsberger's name and some other spelling errors fixed. Thanks.",
"Solution_2": ";)",
"Solution_3": "to yetti. Nice solution. I had some similar ideas concerning Simson lines passing through the center of nine point circle. And my solution also uses the same well-known facts about the homothety with center in the orthocenter that maps the circumcircle to the nine point circle. But anyway, I suppose that our solutions are different. So, I will post my solution soon.\r\n\r\nOne more solution is in the article of L. Emelyanov and T. Emelyanova \"The Morley theorem. 100 years later.\" In this article the authors give two open questions:\r\n\r\n1. It is easy to prove that the Morley triangle and the original triangle are perspective. What is the center of the perspective?\r\n\r\n2. What is the center of homothety of the triangle PQR (which vertices have Simson lines touching to the nine point circle) and the Morley triangle?\r\n\r\nThese two questions are still open. So, if someone have any ideas...",
"Solution_4": "So, here my solution is.\r\n\r\nWe will use the following well-known facts.\r\n\r\n(1) Let $ABC$ and $A'B'C'$ be triangles such that $AB \\parallel A'B'$, $BC \\parallel B'C'$ and $CA \\parallel C'A'$. Then the triangles $ABC$ and $A'B'C'$ are either homothetic or coincide by parallel translation.\r\n\r\n(2) Let $ABC$ be an arbitrary triangle, $H$ be its orthocenter, $\\omega$ be its nine point circle, $\\Omega$ be its circumcircle with center $O$. Then the homothety with center $H$ and coefficient $1/2$ maps $\\Omega$ to $\\omega$. In particular, the center of $\\omega$ is a midpoint of the segment $OH$.\r\n\r\n(3) Let $A'B'C'$ be the Morley triangle of a triangle $ABC$. Then $\\angle{C'B'A}=60^{\\circ}+\\angle{C}/3$.\r\n\r\n(4) Let $A'B'C'$ be the Morley triangle of a triangle $ABC$. Then the angle between the lines $BC$ and $B'C'$ is equal to $|\\angle{C}/3-\\angle{B}/3|$.\r\n\r\n(5) Let $PQ$ be a chord of $\\Omega$ such that $PQ \\perp AC$. Then the Simson line of the point $P$ is parallel to the line $BQ$.\r\n\r\n(6) If $X$ and $Y$ are diametrically opposite points then the Simson lines of these points are perpendicular to each other and $\\omega$ passes through its point of intersection.\r\n\r\n(7) If $P \\in \\Omega$ then the Simson line of $P$ passes through the midpoint of the segment $PH$.\r\n\r\nBack to the original problem, we have to show that there exist precisely three points which Simson lines tangent to $\\omega$. Let $P$ be one of such points and $l_P$ be its Simson line. Then, according to (6), for the diametrically opposite point $P'$ we have: $l_{P'} \\perp l_P$ and $l_P \\cap l_{P'} \\in \\omega$, that is $l_{P'}$ passes through the center of $\\omega$. Observe that the converse is also true, that is if the Simson line of some point passes through the center of $\\omega$ then the Simson line of the diametrically opposite point tangents to $\\omega$. Therefore it is sufficient to prove that there exist precisely three points which Simson lines pass through the center of $\\omega$.\r\n\r\nLet $O$ be the center of circumcircle $\\Omega$ of the triangle $ABC$ and $H$ be the orthocenter.\r\n\r\n[b]Lemma.[/b] Let $P \\in \\Omega$. Then the Simson line of $P$ passes through the center of $\\omega$ if and only if $PO \\parallel QB$, where $Q$ is a point at $\\Omega$ such that $PQ \\perp AC$.\r\n\r\n[i]Proof.[/i] According to (7), the Simson line of the point $P$ passes through the midpoint of the segment $PH$. If it, in addition, passes through the midpoint of $OH$ (that is, according to (2), through the center of $\\omega$), then it contains the medial line of the triangle $OHP$, and hence is parallel to $PO$. Conversely, if it's parallel to $PO$, then it contains the medial line of the triangle $OHP$, and hence passes through the midpoint of the segment $OH$, that is through the center of $\\omega$.\r\n\r\nTo complete the proof it is sufficient to apply the claim (5). $\\square$\r\n\r\nAnd now let $P$ be a point running over the circle $\\Omega$. We are going to investigate the behaviour of the point $Q$ (such that $PQ \\perp AC$). Let $P$ ran from the point $P_1$ to the point $P_2$ the arc with angular measure $\\varphi$, that is the line $OP$ rotated about the point $O$ by the angle $\\varphi$. Let $Q_1$ and $Q_2$ be the corresponding to $P_1$ and $P_2$ positions of $Q$. Then $P_1Q_1 \\parallel P_2Q_2$, and hence the arcs $P_1P_2$ and $Q_1Q_2$ are equal. But it is easy to see that the line $BQ$ rotated about $B$ by the angle $\\varphi/2$, in opposite direction (for instance, if the line $OP$ rotates clockwise, then the line $BQ$ rotates counter-clockwise).\r\n\r\nSo, during the point $P$ was running all the circle $\\Omega$, the line $OP$ rotated by $360^{\\circ}$. During the same time the line $BQ$ rotated by $180^{\\circ}$ (when the points $B$ and $Q$ were the same, we had to consider the tangent to $\\Omega$ at $B$). The question is: \"How many times was there a position when these lines ($PO$ and $BQ$) were parallel to each other?\". The answer is: 3 times. Indeed, observe that the centers of rotations don't matter. Therefore, without loss of generality, we may assume that the centers are the same. Then, instead of rotation both lines in opposite directions, we may consider rotation only one line by the angle $360^{\\circ}+180^{\\circ}=3 \\cdot 180^{\\circ}$. And now it is extremely obvious that the position with $PO \\parallel BQ$ occurs exactly three times. So, we have proved that there are exactly three points which Simson lines tangent to $\\omega$.\r\n\r\nNow let's prove the second part of the problem (about the homotheticness with the Morley triangle). For this goal we will find the location of the points which existance we have just proven.\r\n\r\nWithout loss of generality we may assume $\\angle{BCA} \\geq \\angle{CAB}$. Let the point $B$ be a start position of the point $P$. In this start position the angle between the lines $OP$ and $BQ$ is equal to $\\angle{OBH}=\\gamma-\\alpha$, where $\\alpha=\\angle{CAB}$, $\\beta=\\angle{ABC}$ and $\\gamma=\\angle{BCA}$. During the rotation of the line $OP$ counter-clockwise by the angle $\\varphi$, the angle between the lines $OP$ and $BQ$ increases by $3\\varphi/2$. Therefore by the first desired position of the point $P$, the line $OP$ have to rotate by the angle $2(\\gamma/3-\\alpha/3)$. Call this point $P_1$. Then to obtain the second desired position, the line $OP$ have to\r\nrotate more by the angle $2 \\cdot 180^{\\circ}/3=120^{\\circ}$. And then more by $120^{\\circ}$ to obtain the third desired position. Denote these points by $P_2$ and $P_3$. Clearly, the triangle $P_1P_2P_3$ is equilateral. The rest is to prove that its sides are parallel to the sides of the Morley triangle. But it is trivial. Indeed, the angle between the lines $P_1P_3$ and $BC$ is equal to $\\smile{P_3C}/2-\\smile{BP_1}/2=(\\smile{BA}/2+\\smile{AC}/2-\\smile{BP_1}/2-\\smile{P_1P_3}/2)-\\smile{BP_1}/2=(\\gamma+\\beta-\\gamma/3+\\alpha/3-120^{\\circ})-(\\gamma/3-\\alpha/3)=\\ldots=\\beta/3-\\gamma/3$, that is equal to the angle between the lines $B'C'$ and $BC$ (see the claim (4)), where $A'B'C'$ is the Morley triangle. Consequently, $P_1P_3 \\parallel B'C'$. Similarly we obtain that $P_1P_2 \\parallel A'B'$ and $P_2P_3 \\parallel C'A'$. So, we are done.\r\n\r\nWe have considered the triangle $P_1P_2P_3$ which vertices have Simson lines passing through the center of the nine point circle. And now let us consider the triangle $P_1'P_2'P_3'$ which vertices have Simson lines touching $\\omega$ (this triangle was called $PRQ$ in the problem setting). We have proved earlier that the central symmetry with center $O$ maps $P_1'P_2'P_3'$ to $P_1P_2P_3$. It means that we also have $P_1'P_3' \\parallel B'C'$, $P_1'P_2' \\parallel A'B'$ and $P_2'P_3' \\parallel C'A'$. Applying now the property (1), we can easily obtain desired fact (that is the triangle $P_1'P_2'P_3'$ is homothetic to the triangle $A'B'C'$).\r\n\r\nThe rest is to prove that the triangle with vertices in the points of touching is also homothetic to the triangle $A'B'C'$. Consider a homothety with center $H$ and coefficient $1/2$. According to (2), it maps $\\Omega$ to $\\omega$. Therefore the point $P_1'$ coincide with some point $X_1 \\in \\omega$. On the other hand, by the claim (7), $X_1$ lies at the Simson line of $P_1'$. But the Simson line of $P_1'$ tangents to $\\omega$, and hence $X_1$ is a point of touching.\r\n\r\nSimilarly, we can prove that the points of touching $X_2$ and $X_3$ are the images of the points $P_2'$ and $P_3'$ through our homothety. So, the triangle with vertices in the points of touching is homothetic to the triangle $P_1'P_2'P_3'$, which is homothetic to the Morley triangle as proven earlier.\r\n\r\nSo, all the parts of the problem are proven now.",
"Solution_5": "[quote=\"yetti\"]WLOG, assume that the relative size of the angles $\\alpha = \\angle CAB$, $\\beta = \\angle ABC$, $\\gamma = \\angle BCA$ is $\\beta < \\alpha < \\gamma$ (that's how I draw the triangle). Denote the Morley equilateral triangle $\\triangle DEF$, where D is the intersection of the appropriate trisectors of the angles $\\beta$,$\\gamma$, E the intersection of the appropriate trisectors of the angles $\\gamma$, $\\alpha$ and F the intersection of the appropriate trisectors of the angles $\\alpha$, $\\beta$. The angles formed by 2 trisectors from the same vertex of the $\\triangle ABC$ with the sides of the equilateral triangle $\\triangle ABC$ are:\n\n$\\angle CDE = \\frac{\\beta + \\pi}{3}$, $\\angle CED = \\frac{\\alpha + \\pi}{3}$\n\n$\\angle AEF = \\frac{\\gamma + \\pi}{3}$, $\\angle AFE = \\frac{\\beta + \\pi}{3}$\n\n$\\angle BFD = \\frac{\\alpha + \\pi}{3}$, $\\angle BDF = \\frac{\\gamma + \\pi}{3}$\n\nAn easy proof is to draw an arbitrary equilateral triangle $\\triangle DEF$, take the angles $\\alpha + \\beta + \\gamma = \\pi$, build the above angles on the sides of this equilateral triangle and show, for example, that the angles $\\angle FAB = \\frac{\\alpha}{3}$, $\\angle FBA = \\frac{\\beta}{3}$. Using sine theorem for the triangles $\\triangle AEF$, $\\triangle BDE$, $\\triangle ABF$,\n\n$\\frac{EF}{AF} = \\frac {\\sin \\left ( \\frac{\\alpha}{3} \\right )}{\\sin \\left ( \\frac{\\gamma + \\pi}{3} \\right )}$, $\\frac{DE}{BF} = \\frac {\\sin \\left ( \\frac{\\beta}{3} \\right )}{\\sin \\left ( \\frac{\\gamma + \\pi}{3} \\right )}$, $\\frac{AF}{BF} = \\frac {\\sin \\left ( \\frac{\\beta}{3} \\right )}{\\sin \\left ( \\frac{\\alpha}{3} \\right )}$\n\nSince in addition, $\\angle AFB = 2\\pi - \\frac{\\pi}{3} - \\frac{\\alpha + \\pi}{3} - \\frac{\\beta + \\pi}{3} = \\pi - \\frac{\\alpha + \\beta}{3}$, it follows that $\\angle FAB = \\frac{\\alpha}{3}$, $\\angle FBA = \\frac{\\beta}{3}$.[/quote]\n\nAt first, Yetti, thanks a lot for doing this work. The above quoted passage from your solution of the problem actually contains a nice proof of Morley's theorem, so you have done even more than was asked for.\n\nThe only complaint I have to your solution is that \"Hosenberger\" should be \"Honsberger\" throughout the text. :mrgreen:\n\nNow, to Pestich's post:\n\n[quote=\"pestich\"]Mathworld (ref. to Well's book) claims that the envelope\n of Simson lines is a deltoid (oriented as Morley's triangle) and it touches \n the sides of the original triangle in points isotomic to feet of the altitudes.\n\n Is it obvious?[/quote]\n\nWhat? That the envelope is a deltoid? By no means. A proof can be found in\n\n[url=http://racefyn.insde.es/Publicaciones/racsam/art%C3%ADculos/racsam%2095_1/Guzm%C3%A1n.pdf]M. de Guzm\u00e1n, [i]The envelope of the Wallace-Simson lines of a triangle. A simple proof of the Steiner theorem on the deltoid.[/i], Rev. R. Acad. Cien. Serie A. Mat. Geometr\u00eda y Topolog\u00eda 95 (2001) 1, p. 57-64[/url].\n\nI would call the proof anything but simple, but it is probably much simpler than all proofs of this fact that were known before.\n\n[quote=\"pestich\"]What triangle center is the radical point of the 3 circles\n constructed on these isotomic points as diameters?[/quote]\n\nYou mean the three circles whose diameters are the segments joining the feet of the altitudes of triangle ABC with the reflections of these feet in the midpoints of the respective sides of triangle ABC ? The radical center of these three circles is X(185), the Nagel point of the orthic triangle of triangle ABC. This is, by the way, quite easy to prove synthetically.\n\n[quote=\"pestich\"] It also seems that the 3 distances from the midpoints of the original triangle sides\n to the nearby touching points of Simson lines with the 9 PC (which seem to be\n equal to the distances from same midpoints to the intersections of sides with\n Simson lines tangent to 9 PC) \n are linked so that one of the distances is equal to the sum of the other two.[/quote]\n\nIf I correctly understand your assertion, then, unfortunately, it is not true. However, since the points where the Simson lines touch the nine-point circle form an equilateral triangle, for any point P on the nine-point circle, the distance from P to one of these points equals the sum of the distances from P to the other two.\n\nTo the post by Remike:\n\n[quote=\"Remike\"]One more solution is in the article of L. Emelyanov and T. Emelyanova \"The Morley theorem. 100 years later.\"[/quote]\n\nWhere can I find this article? Has it already appeared? I would be very delighted to see it, since I really like Emelyanov's articles!\n\n[quote=\"Remike\"]In this article the authors give two open questions:\n\n1. It is easy to prove that the Morley triangle and the original triangle are perspective. What is the center of the perspective?\n\n2. What is the center of homothety of the triangle PQR (which vertices have Simson lines touching to the nine point circle) and the Morley triangle?[/quote]\r\n\r\nWell, what do you mean by \"What is the center\"? Actually, none of these two centers is one of [i]the[/i] four famous triangle centers ;) . The center of perspective of the Morley triangle and the original triangle is X(357) in Clark Kimberling's ETC; the center of homothety of the triangle PQR and the Morley triangle is, if I'm not mistaken, not in the ETC yet.\r\n\r\nLast but not least, I wanted to say that I also have a solution to the initial problem of this thread. It is different from both Yetti's and Remike's solutions, although, maybe from the advanced viewpoint all three solutions are more or less the same. Whether I will post it here... well... that depends from weather ;) .\r\n\r\n darij",
"Solution_6": "[quote=\"darij grinberg\"]\nAt first, Yetti, thanks a lot for doing this work. The above quoted passage from your solution of the problem actually contains a nice proof of Morley's theorem, so you have done even more than was asked for.\n\nThe only complaint I have to your solution is that \"Hosenberger\" should be \"Honsberger\" throughout the text. :D \n[/quote]\nI have seen this proof at the Cut-The-Knot, with a comment that direct proofs are too complicated. I also fixed Honsberger's name. :blush: Thanks.\n\n\n[quote=\"Remike\"]\nNice solution. I had some similar ideas concerning Simson lines passing through the center of nine point circle. And my solution also uses the same well-known facts about the homothety with center in the orthocenter that maps the circumcircle to the nine point circle. But anyway, I suppose that our solutions are different. So, I will post my solution soon. \n[/quote]\r\nThanks. Please, give me some time to go through the details of your solution.\r\n\r\nyetti",
"Solution_7": "For other deepenings on Morley's theorem I suggest the following site http://shk.ans.hive.no/\r\n\r\nLeon",
"Solution_8": "Reading your proof, I realized that I did not really show that only 3 Simson lines are tangent to the 9-point circle (or only 3 Simson lines pass through the center of the 9-point circle, which is the same thing). It seemed clear to me that at most 3 such Simson lines can exist, because when the pole moves from one vertex of the triangle $\\triangle ABC$ to another, the corresponding Simson line moves from one altitude to another, but I should have said something. Otherwise, the ideas of both proofs are similar, even though I did not see that the homothety of the circumcircle and the 9-point circle WRT centroid $G$ can be used as well. Thanks for posting your solution.\r\n\r\nYou may consider fixing some typing errors:\r\n\r\n[quote=\"Remike\"](3) Let $A'B'C'$ be the Morley triangle of a triangle $ABC$. Then $\\angle{C'B'A}=60^{\\circ}+\\angle{C}/2$.\n...\n[b]Lemma.[/b] Let $P \\in \\Omega$. Then the Simson line of $P$ passes through the center of $\\omega$ if and only if $PO \\perp QB$, where $Q$ is a point at $\\Omega$ such that $PQ \\perp AC$.\n...\nWe have proved earlier that the central symmetry with center $O$ maps $P_1'P_2'P_3'$ to $P_1P_2P_3$.[/quote]\r\n\r\nIn lemma 2, the angle $\\angle{C'B'A}=60^{\\circ}+\\angle{C}/3$.\r\nIn lemma 7, the Simson line of $P$ passes through the center of $\\omega$ if and only if $PO \\parallel QB$, as you proved.\r\nFinally, the homothety center of the triangles $P_1'P_2'P_3'$ and $P_1P_2P_3$ is the centroid $G$ of the triangle $\\triangle ABC$, as you showed.",
"Solution_9": "[quote=\"yetti\"]even though I did not see that the homothety of the circumcircle and the 9-point circle WRT centroid $G$ can be used as well. Thanks for posting your solution.[/quote]\nWhat do you mean by \"centroid\"? Is it a common point of all three altitudes (that is the point that I called orthocenter)?\n\n[quote=\"yetti\"]You may consider fixing some typing errors:\n\n[quote=\"Remike\"](3) Let $A'B'C'$ be the Morley triangle of a triangle $ABC$. Then $\\angle{C'B'A}=60^{\\circ}+\\angle{C}/2$.\n...\n[b]Lemma.[/b] Let $P \\in \\Omega$. Then the Simson line of $P$ passes through the center of $\\omega$ if and only if $PO \\perp QB$, where $Q$ is a point at $\\Omega$ such that $PQ \\perp AC$.\n...\nWe have proved earlier that the central symmetry with center $O$ maps $P_1'P_2'P_3'$ to $P_1P_2P_3$.[/quote]\n\nIn lemma 2, the angle $\\angle{C'B'A}=60^{\\circ}+\\angle{C}/3$.\nIn lemma 7, the Simson line of $P$ passes through the center of $\\omega$ if and only if $PO \\parallel QB$, as you proved.[/quote]\nI have edited my post. Thank you.\n\n[quote=\"yetti\"]Finally, the homothety center of the triangles $P_1'P_2'P_3'$ and $P_1P_2P_3$ is the centroid $G$ of the triangle $\\triangle ABC$, as you showed.[/quote]\r\nIf I understand you correctly then it is not true. I meant here that $P_1$, $P_2$ and $P_3$ are the points for which the Simson lines pass through the center of $\\omega$; and $P_1'$, $P_2'$ and $P_3'$ are the points for which the Simson lines tangent $\\omega$. I showed that $P_i$ is diametrically opposite to $P_i'$. So, the central symmetry with center $O$ maps $P_1P_2P_3$ to $P_1'P_2'P_3'$.",
"Solution_10": "The intersection of medians is called centroid in English geometry textbooks and usually denoted $G$.\r\n\r\nWhen reading the problem, I thought that $P, Q, R$ were the the tangency points of the Simson lines with the 9-point circle, not their poles on the circumcircle. Similarly when reading your proof, for the points $P_1', P_2', P_3'$ (originally denoted $P, Q, R$) are mentioned at the end, when everything becomes clear. But of course, you denoted the tangency points $X_1, X_2, X_3$. So, the triangles $\\triangle P_1P_2P_3 \\sim \\triangle P_1'P_2'P_3'$ are centrally similar with homothety center at the circumcenter $O$ and coefficient $-1$, while the triangles $\\triangle P_1P_2P_3 \\sim \\triangle X_1X_2X_3$ are centrally similar with homothety center at the centroid $G$ and coefficient $-\\frac 1 2$.",
"Solution_11": "[quote=\"darij grinberg\"][quote=\"Remike\"]One more solution is in the article of L. Emelyanov and T. Emelyanova \"The Morley theorem. 100 years later.\"[/quote]\n\nWhere can I find this article? Has it already appeared? I would be very delighted to see it, since I really like Emelyanov's articles![/quote]\nWell, I don't know where you can find it. It was published in Russian journal \"Mathematics in school\" (\"\u041c\u0430\u0442\u0435\u043c\u0430\u0442\u0438\u043a\u0430 \u0432 \u0448\u043a\u043e\u043b\u0435\"), \u2116 9, 2004. If you know Russian, you can see here a content of this journal:\nhttp://www.schoolpress.ru/shop/aposoft.z/StoreFront-Main .\nAnd here the abstract of this paper is:\nhttp://www.schoolpress.ru/shop/SPStoreFront/Share/public/jornal/matematika/short_text/9_2004/08_9_04_62.html .\n\nBy the way, some time ago another article of L. Emelyanov \"Geometrical proof of the Ponsele theorem\" (\"\u0413\u0435\u043e\u043c\u0435\u0442\u0440\u0438\u0447\u0435?\u043a\u043e\u0435 \u0434\u043e\u043a\u0430\u0437\u0430\u0442\u0435\u043b\u044c?\u0442\u0432\u043e \u0442\u0435\u043e\u0440\u0435\u043c\u044b \u041f\u043e\u043d?\u0435\u043b\u0435\") was published in this journal (\u2116 3, 2004). Here the abstract is: http://www.schoolpress.ru/shop/SPStoreFront/Share/public/jornal/matematika/short_text/3_2004/08_3_04_73.html .\n\n[quote=\"darij grinberg\"][quote=\"Remike\"]In this article the authors give two open questions:\n\n1. It is easy to prove that the Morley triangle and the original triangle are perspective. What is the center of the perspective?\n\n2. What is the center of homothety of the triangle PQR (which vertices have Simson lines touching to the nine point circle) and the Morley triangle?[/quote]\n\nWell, what do you mean by \"What is the center\"? Actually, none of these two centers is one of [i]the[/i] four famous triangle centers ;) . The center of perspective of the Morley triangle and the original triangle is X(357) in Clark Kimberling's ETC; the center of homothety of the triangle PQR and the Morley triangle is, if I'm not mistaken, not in the ETC yet.[/quote]\r\nWell, I can quote from the original text of the article (in Russian), but I'm afraid it is out of law. But I suppose I may retell it in English. So, here my translation of the final part of the article is.\r\n\r\n\"It seems that we can finish our consideration of the Morley theorem. However, we want to think that this material will have become a beginning of an investigation of the properties of the Morley triangle, because every fundamental theorem generates more questions than answers. For instance, it is easy to prove that the original triangle and its Morley triangle are perspective, that is the lines AD, BE and CF have a common point. What is that point? Is it a well-known point for the triangles ABC and DEF? Does it lie at any well-known line? And there is one more natural question: what is the center of homothety of the triangles PQR and DEF?\r\n\r\nDream, ask questions, put forward ideas --- modern computer provides an opportunity to verify many and many geometrical conjectures. Fortunately, it (computer) can't prove these conjectures, since we need geometrical intuition and mind to prove it. But it can help us to develop both these things.\"",
"Solution_12": "[quote=\"yetti\"]The intersection of medians is called centroid in English geometry textbooks and usually denoted $G$.[/quote]\nYes, in Russian too.\n\n[quote=\"yetti\"]When reading the problem, I thought that $P, Q, R$ were the the tangency points of the Simson lines with the 9-point circle, not their poles on the circumcircle. Similarly when reading your proof, for the points $P_1', P_2', P_3'$ (originally denoted $P, Q, R$) are mentioned at the end, when everything becomes clear. But of course, you denoted the tangency points $X_1, X_2, X_3$. So, the triangles $\\triangle P_1P_2P_3 \\sim \\triangle P_1'P_2'P_3'$ are centrally similar with homothety center at the circumcenter $O$ and coefficient $-1$, while the triangles $\\triangle P_1P_2P_3 \\sim \\triangle X_1X_2X_3$ are centrally similar with homothety center at the centroid $G$ and coefficient $-\\frac 1 2$.[/quote]\r\nI understand you now. There are two homotheties that maps $\\Omega$ to $\\omega$: with positive coefficient and with negative coefficient. You consider a homothety with negative coefficient and with center at $G$. And I consider a homothety with positive coefficient and with center at $H$. You claim that the triangles $P_1P_2P_3$ and $X_1X_2X_3$ are homothetic with coefficient $-1/2$ and center at $G$. Well, it's true. And I claim that the triangles $P_1'P_2'P_3'$ and $X_1X_2X_3$ are homothetic with coefficient $1/2$ and center at $H$. Well, it's also true."
}
{
"Tag": [],
"Problem": "Simplify and express your answer as a common fraction.\n\n\\[ \\dfrac{\\frac{1}{1\\plus{}\\frac{2}{1\\plus{}3}}}{\\frac{3}{2\\plus{}\\frac{1}{1\\plus{}1}}}\\]",
"Solution_1": "$ \\dfrac{\\frac{1}{1\\plus{}\\frac{2}{1\\plus{}3}}}{\\frac{3}{2\\plus{}\\frac{1}{1\\plus{}1}}}\\equal{}\\dfrac{\\frac{1}{1\\plus{}\\frac{2}{4}}}{\\frac{3}{2\\plus{}\\frac{1}{2}}}\\equal{}\\dfrac{\\frac{1}{1\\plus{}\\frac{1}{2}}}{\\frac{3}{2\\plus{}\\frac{1}{2}}}\\equal{}\\dfrac{\\frac{1}{\\frac{3}{2}}}{\\frac{3}{\\frac{5}{2}}}\\equal{}\\dfrac{\\frac{2}{3}}{\\frac{6}{5}}\\equal{}\\frac{10}{18}\\equal{}\\frac{5}{9}$"
}
{
"Tag": [
"linear algebra",
"matrix"
],
"Problem": "Find the inverse of the $n$x$n$ \"lower\" triangular matrix. Prove that the matrix you give is indeed the inverse.\r\n\r\nBy lower triangular matrix I mean the matrix with a 1 in every entry below or on the main diagonal, and a 0 in every other entry.",
"Solution_1": "perform the following row operations on the matrix:\r\n\r\n$R_{n}\\mapsto R_{n}-R_{n-1}$,\r\n$R_{n-1}\\mapsto R_{n-1}-R_{n-2}$,\r\n...\r\n$R_{2}\\mapsto R_{2}-R_{1}$.\r\n\r\nthe result is $I_{n}$. performing these same operations on $I_{n}$ now gives the inverse of the original matrix: the result is a main diagonal of 1's, the diagonal below that of all -1's, and everything else 0."
}
{
"Tag": [
"linear algebra",
"matrix",
"vector"
],
"Problem": "Prove that the geometric multiplicity of an eigenvalue cannot exceed its algebraic multiplicity.",
"Solution_1": "If algebraic and geometric multiplicities are defined via the Jordan form of a matrix, as some authors do, your argument is obvious.\r\n\r\nHere is another treatment: \r\nLet $ k$ be the dimension of the eigenspace $ V_{\\lambda}$ corresponding to the eigenvalue $ \\lambda$. Then extend $ V_\\lambda$ to a basis $ \\mathcal B$ for your vector space $ V$. Clearly the matrix of your operator, say $ \\tau$, with respect to $ \\mathcal B$ has the form:\r\n$ \\begin{pmatrix}\\lambda I_k & A\\\\ 0& B\\end{pmatrix}$.\r\nBut, $ c_{\\tau}(x)=\\det(xI-[\\tau]_{\\mathcal B})=(x-\\lambda)^k\\det(xI_{n-k}-B).$ Here $ n$ is the dimension of $ V$. Hence, the result follows."
}
{
"Tag": [
"geometry",
"circumcircle",
"geometric transformation",
"reflection",
"analytic geometry",
"trigonometry",
"parameterization"
],
"Problem": "Let $ \\Gamma$ be the circle with radius $ r$ and center $ O$ and $ \\Gamma'$ be the circle with radius $ r'$ and center $ O'$. Let $ T$ be a variable point of $ \\Gamma$ and the tangent line $ t$ passing by $ T$ intersect the circle $ \\Gamma'$ at two points $ A$ and $ B$. Find the locus of the circumcenter of the triangle $ BOA$ when $ T$ varies on $ \\Gamma$. I know, making a figure, that the locus is a circle of center $ O'$ with a radius depending of $ OO'$, $ r$ and $ r'$. But how to prove it? Thanks in advance.",
"Solution_1": "First, invert the figure in the circle $ \\Gamma$ with center $ O.$ Tangent of $ \\Gamma$ at $ T$ goes to a circle $ \\mathcal T^*$ with diameter $ OT.$ Arbitrary circle $ \\Gamma'$ with center $ O'$ (not entirely inside of $ \\Gamma$) goes to a circle $ \\Gamma'^*$, intersecting the circle $ \\mathcal T^*$ at $ A^*, B^*.$ Circumcircle $ \\mathcal C$ of the $ \\triangle OAB$ goes to the line $ A^*B^*$ and its circumcenter $ C$ into the reflection $ C^*$ of $ O$ in $ A^*B^*.$ Let $ E \\in OO'$ be the center of $ \\Gamma'^*$ and $ s$ its radius. Denote $ d \\equal{} OO',\\ e \\equal{} OE.$ Since $ O$ is the similarity center of $ \\Gamma'^*$ and $ \\Gamma'$ with similarity cefficient $ k \\equal{} \\frac {r^2}{d^2 \\minus{} r'^2}$ (power of inversion divided by power of the inversion center to the original circle), we have $ e \\equal{} kd, s \\equal{} kr'.$ \r\n\r\nLet $ O$ be the coordinate origin and $ OO'$ the x-axis. Equations of the circle $ \\mathcal T^*$ with fixed diameter $ r \\equal{} OT$ and passing through the origin $ O$ is\r\n\r\n$ \\mathcal T^*: \\ \\ x^2 \\minus{} xr \\cos \\theta \\plus{} y^2 \\minus{} yr \\sin \\theta \\equal{} 0,$\r\n\r\nwhile equation of the circle $ \\Gamma'^*$ centered on the x-axis at $ E \\equal{} (e, 0)$ and with radius $ s$ is\r\n\r\n$ \\Gamma'^*: \\ \\ x^2 \\minus{} 2ex \\plus{} e^2 \\plus{} y^2 \\equal{} s^2.$\r\n\r\nSubtracting equations of the 2 circles from each other gives equation of their radical axis $ A^*B^*$:\r\n\r\n$ A^*B^*: \\ \\ F(x, y, \\theta ) \\equal{} x(r \\cos \\theta \\minus{} 2e) \\plus{} yr \\sin \\theta \\plus{} e^2 \\minus{} s^2 \\equal{} 0.$\r\n\r\nTransforming the last equation to polar coordinates $ (\\varrho, \\phi)$ by $ x \\equal{} \\varrho \\cos \\phi,\\ y \\equal{} \\varrho \\sin \\phi,$\r\n\r\n$ A^*B^*: \\ \\ \\Phi(\\varrho, \\phi, \\theta ) \\equal{} \\varrho r \\cos (\\phi \\minus{} \\theta) \\minus{} 2 \\varrho e \\cos \\phi \\plus{} e^2 \\minus{} s^2 \\equal{} 0.$\r\n\r\nThis is equation of a family of lines depending on the parameter $ \\theta.$ Equation of their envelope $ \\mathcal K$ (if any) is obtained by eliminating this parameter from the equations $ \\Phi(\\varrho, \\phi, \\theta ) \\equal{} 0$ and\r\n\r\n$ \\frac {\\partial \\Phi}{\\partial \\theta} \\equal{} \\varrho r \\sin (\\phi \\minus{} \\theta) \\equal{} 0.$\r\n\r\nThe 2nd equation yields $ \\phi \\minus{} \\theta \\equal{} n \\pi,\\ \\cos(\\phi \\minus{} \\theta) \\equal{} \\pm 1.$ Substituting this to the polar equation $ \\Phi(\\varrho, \\phi, \\theta) \\equal{} 0$ of the line $ A^*B^*,$ the envelope equation comes out as\r\n\r\n$ \\mathcal K: \\ \\ \\pm \\varrho r \\minus{} 2 \\varrho e \\cos \\phi \\plus{} e^2 \\minus{} s^2 \\equal{} 0,$ or\r\n\r\n$ \\varrho \\equal{} \\frac {e^2 \\minus{} s^2}{r} \\cdot \\frac {1}{\\frac {2e}{r} \\cos \\phi \\pm 1} \\equal{} \\frac {L}{\\epsilon \\cos \\phi \\pm 1}.$\r\n\r\nThis is polar equation of a conic with focus $ O,$ major axis line $ \\phi \\equal{} 0$ (i.e., the line $ OO'$), excentricity $ \\epsilon \\equal{} \\frac {2e}{r}$ and semilatus rectum $ |L| \\equal{} \\frac {|e^2 \\minus{} s^2|}{r}.$ The correct sign, obtained by demanding $ \\varrho > 0,$ depends on radii $ r, r'$ and center distance $ d \\equal{} OO'$ of the circles $ \\Gamma, \\Gamma'.$ For an ellipse $ (\\epsilon < 1$), the correct sign is the same as the sign of $ e^2 \\minus{} s^2 \\equal{} k^2(d^2 \\minus{} r'^2)$ or $ d \\minus{} r'$ ($ O$ outside or inside of $ \\Gamma'$), for a hyperbola ($ \\epsilon > 1$) one sign holds for each branch.\r\n\r\nSince $ A^*B^*$ is tangent to this conic, the foot $ F$ of perpendicular from the focus $ O$ to $ A^*B^*$ lies on its pedal circle $ \\mathcal P$ centered at $ P \\in OO'$ on the conic major axis. Since $ F$ is midpoint of $ OC^*,$ the image $ C^*$ of the circumcenter $ C$ lies on a circle $ \\mathcal Q.$ centrally similar to $ \\mathcal P$ with similarity center $ O$ and coefficient 2, also centered on $ OO'.$ As a result, $ C$ lies on the inversion image $ \\mathcal Q^*$ of $ \\mathcal Q,$ also centered on $ OO'.$ The center $ Q$ of $ \\mathcal Q$ is the other conic focus\r\n\r\nAssume $ \\epsilon > 1$ (hyperbola) and $ d > r'$ ($ O$ outside $ \\Gamma'$). Substituting $ \\phi \\equal{} 0$ into the hyperbola equation, $ q \\equal{} OQ \\equal{} \\frac {L}{\\epsilon \\plus{} 1} \\plus{} \\frac {L}{\\epsilon \\minus{} 1} \\equal{} \\frac {2\\epsilon L}{\\epsilon^2 \\minus{} 1}.$ Radius of $ \\mathcal Q$ is equal to the conic major axis length, $ t \\equal{} \\frac {OQ}{\\epsilon} \\equal{} \\frac {2L}{\\epsilon^2 \\minus{} 1}$ for a hyperbola. Again, $ O$ is the similarity center of $ \\mathcal Q^*, \\mathcal Q$ with similarity coefficient $ m \\equal{} \\frac {r^2}{q^2 \\minus{} t^2} \\equal{} r^2 \\cdot \\frac {\\epsilon^2 \\minus{} 1}{4L^2},$ so that distance of the center of $ \\mathcal Q^*$ from $ O$ is\r\n\r\n$ mq \\equal{} r^2 \\cdot \\frac {\\epsilon^2 \\minus{} 1}{4L^2} \\cdot \\frac {2\\epsilon L}{\\epsilon^2 \\minus{} 1} \\equal{} r^2 \\cdot \\frac {\\epsilon}{2L} \\equal{} r^2 \\cdot \\frac {2e}{r} \\cdot \\frac {r}{2(e^2 \\minus{} s^2)} \\equal{} r^2 \\cdot \\frac {kd}{k^2(d^2 \\minus{} r'^2)} \\equal{} d.$\r\n\r\nSince $ d \\equal{} OO',$ the center of $ \\mathcal Q^*$ containing the circumcenter $ C$ is identical with the center $ O'$ of $ \\Gamma'.$"
}
{
"Tag": [
"algebra",
"polynomial",
"algebra unsolved"
],
"Problem": "Let $P(x)$ be a polynomial with integer coefficents. And let $Q(x) = \\prod \\limits^n_{k=1} P(x^k) + 1.$ Determine whether $Q(x)$ has integer roots dependently on $n$.",
"Solution_1": "Is it right that we have to define all natural $n$ s.t. there is $P(x)\\in Z[x]$ that $Q(x)$ has integer root?"
}
{
"Tag": [
"AMC",
"USA(J)MO",
"USAMO",
"AIME",
"inequalities",
"pigeonhole principle",
"AMC 10"
],
"Problem": "I'd have to say mine is about 3",
"Solution_1": "mine is maybe 1 problem right.. so 7/42 :blush:",
"Solution_2": "I'm going to be taking the USAMO unofficially, I'm so mad I didn't qualify because of 1 error, I think I could easily get like a 14 if I made it :mad:",
"Solution_3": "lol i clicked 42 just for the heck of it.",
"Solution_4": "Hmmm... I clicked 8-14. Since I voted 0-9 in another topic, I guess my score will be 8 or 9.",
"Solution_5": "Well that depends on who's doing the expecting. If you ask Delong, I'm getting a 42. If you ask me, I'm getting just enough to make red mop, or 14+ if both #1 and #4 are really easy (read: not geometry).",
"Solution_6": "I am so getting a binary number",
"Solution_7": "[quote=\"Ignite168\"]I'm going to be taking the USAMO unofficially, I'm so mad I didn't qualify because of 1 error, I think I could easily get like a 14 if I made it :mad:[/quote]\r\n\r\nTake it from the guy who scored a 3 twice in a row Ignite, even if you think you could solve two problems, getting a 14 is far more difficult than you realize. \r\n\r\n--the bubala",
"Solution_8": "I'm hoping for a decent improvement, so I said 15-21 (I got 10 last year.) Given the fact that I had a 42-point increase between my first AMC-10 and my second, a 5-point increase between my first AIME and my second, and a 20-point increase between my first AMC-12 and my second, I'm thinking that that might be a reasonable goal.",
"Solution_9": "[quote=\"bubala\"][quote=\"Ignite168\"]I'm going to be taking the USAMO unofficially, I'm so mad I didn't qualify because of 1 error, I think I could easily get like a 14 if I made it :mad:[/quote]\n\nTake it from the guy who scored a 3 twice in a row Ignite, even if you think you could solve two problems, getting a 14 is far more difficult than you realize. \n\n--the bubala[/quote]\r\n\r\nHe was joking anyway; he qualified.",
"Solution_10": "does he ever not joke?\r\n\r\nhmm...i put 15-21 b/c i hope that that will be enough for blue MOP (at least, the upper end of that range) which is a goal that i have worked EXTREMELY hard for.\r\n\r\nplus i scored 7 last year (2 points from red MOP)",
"Solution_11": "uhhh\r\n\r\nhopefully enough to get into red mop\r\n\r\nmeaning i'll be spending my 4.5 hours bashing the living crap out of inequalities with my multivariable calc knowledge and gross misapplications of pigeonhole, CRT, and muirhead\r\n\r\n\r\nbe prepared, usamo graders",
"Solution_12": "[quote=\"Treething\"]uhhh\n\nhopefully enough to get into red mop\n\nmeaning i'll be spending my 4.5 hours bashing the living crap out of inequalities with my multivariable calc knowledge and gross misapplications of pigeonhole, CRT, and muirhead\n\n\nbe prepared, usamo graders[/quote]\r\n\r\nJust a tip. If you actually do find yourself writing a proof with Lagrange Multipliers, make sure that you do all the arithmetic (solving the n equations for n variables) in your proof, or else you will get very few points.",
"Solution_13": "Zero, since I didn't make it. I'll take it unofficially just to see how I do. Zero, probably, but then again I can''t really know whether I'd get a 0 or a 1 on any of the problems.",
"Solution_14": "[quote=\"Treething\"]uhhh\n\nhopefully enough to get into red mop\n\nmeaning i'll be spending my 4.5 hours bashing the living crap out of inequalities with my multivariable calc knowledge and gross misapplications of pigeonhole, CRT, and muirhead\n\n\nbe prepared, usamo graders[/quote]\r\n\r\nIf there are inequalities...hm\r\n\r\nAnd tim's facebook says you don't know the CRT",
"Solution_15": "42. obviously. since Delong says so. :lol: \r\nIn reality, hoping for 15-21 so that i at least improved....",
"Solution_16": "any positive score would satisfy me",
"Solution_17": "[quote=\"undefined117\"]mine is maybe 1 problem right.. so 7/42 :blush:[/quote]\r\n\r\nAs an 8th grader with your AMC10 and AIME scores, solving one problem fully would be very *very* impressive. Unless you've taken practice tests before and *can* solve one problem on each one...?\r\n\r\nAnyways my goal is a non-negative score, although I'd be happy to get a negative score also.",
"Solution_18": "Hm I said 13-31 in the last topic. I guess expected score is then $\\frac{13+31}{2}= 22$ assuming a linear probability distribution or symmetric about the median or whatever. I'll be happy if I do in fact get a $22$.",
"Solution_19": "[quote=\"probability1.01\"]Hmmm... I clicked 8-14. Since I voted 0-9 in another topic, I guess my score will be 8 or 9.[/quote]\r\n\r\ndude, come on.. there are only 6 problems. Even you can't pull off an 8 :P",
"Solution_20": "Good luck to everyone taking it tomorrow",
"Solution_21": "I'd say 1-7. I don't think I'll draw blanks on every single question and be able to write nothing. If I get lucky, I'll see a question that I can solve, and maybe get a 7 on that problem."
}
{
"Tag": [
"\\/closed"
],
"Problem": "When I move my mouse over the private messages it says that 1 unread message.\r\n\r\nBut I've read all messages.\r\n\r\nWhat is this? :?",
"Solution_1": "Looks like a bug :? . What style are you using? Also please tell me if refreshing the window / log in/out clears it ...",
"Solution_2": "This is bit weird.\r\n\r\nI had AoPS as my board style and when I changed to MathLinks, I have \"no new private messages.\"\r\n\r\n? :?",
"Solution_3": "[quote=\"Silverfalcon\"]This is bit weird.[/quote]I agree :) ... it might have been a cookie issue, or a private message that has been sent to you by someone, and then deleted during the time you spoted the link and when you clicked it. \r\n\r\nDoes returning to AoPS style cause the same problem? ...",
"Solution_4": "Thanks Valentin. :D \r\n\r\nBack to ages where I used MathLink Board Style :D :D :D"
}
{
"Tag": [
"superior algebra",
"superior algebra unsolved"
],
"Problem": "Let $I$ be an ideal of the ring $R$ and $f$ be nonidentity permutation of the set $\\{\\ 1,\\dots ,k\\}$.suppose that for every $0\\neq a\\in R,aI\\neq 0,Ia\\neq 0$ hold;for any elements of $x_1,x_2,\\dots,x_k\\in I$,$x_1x_2\\dots=x_{f1}\\dots x_{fk}$holdsprove that $R$ is commutative.",
"Solution_1": "This one has been lying here for quite some time, huh? :)\r\n\r\nIf $f1=1$, the problem is reduced to smaller $k$'s, so we can assume this is not the case. We replace $x_1$ with $ax_1$ for an arbitrary $a\\in R$, and we get $ax_1\\ldots x_k=x_{f1}\\ldots ax_{ff^{-1}1}\\ldots x_{fk}$ on the one hand, and $ax_1\\ldots x_k=ax_{f1}\\ldots x_{fk}$ on the other hand. Equating those, we get $x_{f1}\\ldots ax_{ff^{-1}1}\\ldots x_{fk}=ax_{f1}\\ldots x_{fk}$. In this relation, we can eliminate $x_{ff^{-1}1},\\ldots,x_{fk}$ one by one because of the fact that $tI=0\\rightarrow t=0$, and we're left with a relation of the type $ax_1\\ldots x_u=x_1\\ldots x_ua,\\ \\forall x_i\\in I,\\ \\forall a\\in R$ for some $1\\le u\\le k-1$. If we take $a$ from $I$, this is a relation just like the one we started with, except that the permutation $f$ is a cycle here. This means that from now on we can assume $f$ to be a cycle.\r\n\r\nNow, if $f$ is a cycle and $k\\ge 3$, then either $fk\\ne 1$ or $f^2k\\ne 1$, and if we pull the same trick as before, with the arbitrary $a$ and the elimination of $x_{ff^{-1}1},\\ldots,x_{fk}$, we'll eliminate at least two $x_i$'s. This leaves us with an analogous problem, but for smaller $k$, and we can use induction. This means that it suffices to prove the whole thing for $k=2$ (and $f$ being a transposition).\r\n\r\nWe have $x_1x_2=x_2x_1,\\ \\forall x_i\\in I$. This means that for all $a\\in R$, we have $x_2ax_1=x_1x_2a=x_2x_1a$, so $I(ax_1-x_1a)=0,\\ \\forall x_1\\in I,\\ \\forall a\\in R$. We thus get $I\\subset Z(R)$ (the center of $R$). Finally, for $a,b\\in R,\\ x_1\\in I$, we have $ax_1b=x_1ab=x_1ba\\Rightarrow I(ab-ba)=0$, so $ab=ba$, and we're done.",
"Solution_2": "From $ ax_1...x_u\\equal{}x_1...x_ua$ you can finish it faster:\r\n$ abx_1...x_u \\equal{} a(bx_1...x_u) \\equal{} bx_1...x_ua \\equal{} bax_1...x_u$, so $ (ab\\minus{}ba)$ is killed by $ I^u$ which means (by hypothesis) that $ ab\\minus{}ba\\equal{}0$"
}
{
"Tag": [
"AoPSwiki",
"articles",
"Alcumus",
"videos",
"Pascal\\u0027s Triangle"
],
"Problem": "Pascal's Triangle and the Hockey Stick Theorem are very useful in mathematics, so I heard.\r\n\r\nI want to learn it but the AOPSWiki article confused me.\r\n\r\nCan you guys help?",
"Solution_1": "There are Alcumus videos on those things. I find them very useful. Now, everybody can access all the videos, so watch them now before you can't!",
"Solution_2": "Hmm.. Did you Google them?",
"Solution_3": "[quote=\"Cliu0301\"]Pascal's Triangle and the Hockey Stick Theorem are very useful in mathematics, so I heard.\n\nI want to learn it but the AOPSWiki article confused me.\n\nCan you guys help?[/quote]\r\n\r\nTry intro. to counting&probability book (AOPS), that gives you a great understanding in pascal's triangle. You get to derive the proofs, and all.\r\nIts great fun!"
}
{
"Tag": [
"percent",
"MATHCOUNTS"
],
"Problem": "A shape that includes the point (3,8) is reflected across the line y = x + 2. At what point does the point (3,8) land?\r\n\r\nA store puts everything on sale for 20% off. If the sales tax is 8%, what percent of the original marked price is the final cost including tax? Round answers to the nearest tenth.\r\n\r\nCHALLENGE: SIG FIG MANIA\r\n\r\nAnswer the following in significant figures.\r\n\r\n1. 2.0 x 3.625\r\n2. 0.0000003 / 1.0\r\n\r\nHow many sig figs do each of these numbers have?\r\n\r\n1. 1.204\r\n2. 0.200000\r\n3. 0.036230\r\n4. 100.0",
"Solution_1": "hmm, these look suspiciously like homework problems as opposed to challenge problems (who gives sig figs in a challenge problem?)",
"Solution_2": "SIF FIG\r\n\r\nsounds like a sciences class and the problems are too easy to not be hw",
"Solution_3": "lol, nice try buddy\r\n\r\nu n00b",
"Solution_4": "What's bad about science?\r\n\r\nWhat's a significant figure anyway?\r\n\r\n(I grew up learning in foreign languages; I might know it already)",
"Solution_5": "[quote=\"soulzmischief\"]lol, nice try buddy\n\nu n00b[/quote]\r\n\r\n\r\ngoodness, person. i bet you're in math counts and you're in my science class. you're either T-guy, Ma-guy, L-guy, or Mi-guy. Most likely you're T-guy.",
"Solution_6": "it has been a day, and we did sig figs three weeks ago.\r\n\r\nanswers: (6,5), 86.4, 7.3, .0000003, 4, 6, 5, 4\r\n\r\nT-guy, shut up. and dont even THINK about talking to me during school",
"Solution_7": "[quote=\"pakagawa\"]What's bad about science?\n\nWhat's a significant figure anyway?\n\n(I grew up learning in foreign languages; I might know it already)[/quote]\r\n\r\nThe number of significant figures a number has tells you how accurate that number is. The number of sig. figures in 2.0021 is 5. I ll tell you more if you want but you might already know it so I ll just wait. It's not very hard to learn so anyone can just google it and learn it quickly.",
"Solution_8": "[quote=\"soulzmischief\"]lol, nice try buddy\n\nu n00b[/quote]\r\n\r\nare u from hopkins?",
"Solution_9": "hey buddy its yuchen.. ur mathcounts tutor\r\n\r\nhave a nice day... and i'll make sure thursday is pretty painful for you \r\n\r\n:)",
"Solution_10": "[quote=\"soulzmischief\"]... and i'll make sure thursday is pretty painful for you \n\n:)[/quote]\r\n\r\ni cant wait. har har har",
"Solution_11": "btw\r\n\r\nim sure that you know we're past sig figs and all that velocity ****. except for the fact that theres a test this friday. im ready to die.\r\n\r\nand you know all the science hw we get through your sister."
}
{
"Tag": [
"inequalities proposed",
"inequalities"
],
"Problem": "a)If $ a,b,c > 0$ such that $ abc \\equal{} 1$ then\r\n\\[ \\frac {a^4}{a^3 \\plus{} b^3} \\plus{} \\frac {b^4}{b^3 \\plus{} c^3} \\plus{} \\frac {c^4}{c^3 \\plus{} a^3}\\geq\\frac {9}{2(a \\plus{} b \\plus{} c)}\r\n\\]\r\nb) If $ a,b,c>0$ such that $ \\frac{1}{a}\\plus{}\\frac{1}{b}\\plus{}\\frac{1}{c}\\equal{}1$, then \\[ \\frac {a^4}{a^3 \\plus{} b^3} \\plus{} \\frac {b^4}{b^3 \\plus{} c^3} \\plus{} \\frac {c^4}{c^3 \\plus{} a^3}\\geq\\frac {9}{2}\\]",
"Solution_1": "[quote=\"Marius Mainea\"]Let $ a,b,c > 0$. Prove that:\n\n a) $ \\frac {a^4}{a^3 \\plus{} b^3} \\plus{} \\frac {b^4}{b^3 \\plus{} c^3} \\plus{} \\frac {c^4}{c^3 \\plus{} a^3}\\leq\\frac {a \\plus{} b \\plus{} c}{2}$\n[/quote]\r\nMaybe\r\n$ \\frac {a^4}{a^3 \\plus{} b^3} \\plus{} \\frac {b^4}{b^3 \\plus{} c^3} \\plus{} \\frac {c^4}{c^3 \\plus{} a^3}\\geq\\frac {a \\plus{} b \\plus{} c}{2}$ $ ?$ :wink:",
"Solution_2": "[quote=\"arqady\"][quote=\"Marius Mainea\"]\n[/quote]\nMaybe\n$ \\frac {a^4}{a^3 \\plus{} b^3} \\plus{} \\frac {b^4}{b^3 \\plus{} c^3} \\plus{} \\frac {c^4}{c^3 \\plus{} a^3}\\geq\\frac {a \\plus{} b \\plus{} c}{2}$ $ ?$ :wink:[/quote]\r\n\r\n Yes ,you are right. :|",
"Solution_3": "[quote=\"Marius Mainea\"]Let $ a,b,c>0$. Prove that: \na)$ \\frac {a^4}{a^3 \\plus{} b^3} \\plus{} \\frac {b^4}{b^3 \\plus{} c^3} \\plus{} \\frac {c^4}{c^3 \\plus{} a^3}\\geq\\frac {a \\plus{} b \\plus{} c}{2}$[/quote]\r\nSimilar problem posted previously.\r\nUse $ H\\ddot{o}lder$ on $ L.H.S.$. Followed by some generalizations. :wink:",
"Solution_4": "[quote=\"Sunkern_sunflora\"][quote=\"Marius Mainea\"]Let $ a,b,c > 0$. Prove that: \na)$ \\frac {a^4}{a^3 \\plus{} b^3} \\plus{} \\frac {b^4}{b^3 \\plus{} c^3} \\plus{} \\frac {c^4}{c^3 \\plus{} a^3}\\geq\\frac {a \\plus{} b \\plus{} c}{2}$[/quote]\nSimilar problem posted previously.\nUse $ H\\ddot{o}lder$ on $ L.H.S.$. [/quote]\r\nHow we can use Holder? I don't see it. :maybe: Explain please. Thank you!",
"Solution_5": "[quote=\"arqady\"][quote=\"Sunkern_sunflora\"][quote=\"Marius Mainea\"]Let $ a,b,c > 0$. Prove that: \na)$ \\frac {a^4}{a^3 \\plus{} b^3} \\plus{} \\frac {b^4}{b^3 \\plus{} c^3} \\plus{} \\frac {c^4}{c^3 \\plus{} a^3}\\geq\\frac {a \\plus{} b \\plus{} c}{2}$[/quote]\nSimilar problem posted previously.\nUse $ H\\ddot{o}lder$ on $ L.H.S.$. [/quote]\nHow we can use Holder? I don't see it. :maybe: Explain please. Thank you![/quote]\r\nIt 's here\r\nhttp://www.mathlinks.ro/viewtopic.php?t=216009&sid=eee3567d29fe4d8663712a426107f404\r\nHi arqady; Can U post your proof ?"
}
{
"Tag": [
"inequalities",
"inequalities unsolved"
],
"Problem": "Let a,b,c are positive real numbers such that $ \\left\\{ \\begin{array}{l}0 < a < b \\\\b^2 \\minus{} 4ac \\le 0 \\\\ \r\n \\end{array} \\right.$\r\n\r\nFind the min value of $ S \\equal{} \\frac{{a \\plus{} b \\plus{} c}}{{b \\minus{} a}}$",
"Solution_1": "hello, prove that $ \\frac{a\\plus{}b\\plus{}c}{b\\minus{}a}\\geq3$.\r\nSonnhard.",
"Solution_2": "[quote=\"Dr Sonnhard Graubner\"]hello, prove that $ \\frac {a \\plus{} b \\plus{} c}{b \\minus{} a}\\geq3$.\nSonnhard.[/quote]\r\n\r\nHello!\r\nThis is problem 5 in Gifted school examination of my city. Can u solve this problem detailly? I need a detail solution! :)\r\nThanks so much! ;)\r\n\r\nL_Euler.",
"Solution_3": "The answer of [b]Dr Sonnhard Graubner[/b] is right. Now I will solve it in detail.\r\nConsider $ P(x) \\equal{} ax^2 \\minus{} bx \\plus{} c$. Because $ b^2 \\minus{} 4ac\\le 0$ and $ a > 0$ therefore $ P(x)\\ge 0$ for all real number $ x$. Put $ x \\equal{} 2$, we will get $ 4a \\minus{} 2b \\plus{} c\\ge 0$, which is also means that $ a \\plus{} b \\plus{} c\\ge 3(b \\minus{} a)$, or $ \\frac {a \\plus{} b \\plus{} c}{b \\minus{} a}\\ge 3$. Then you will get the result :lol:",
"Solution_4": "equality holds if $ a\\equal{}1,b\\equal{}4,c\\equal{}4$"
}
{
"Tag": [
"quadratics"
],
"Problem": "I was recently doing a problem that related to quadratic equations.\r\n\r\nI am asked to complete the square and solve.\r\n\r\nSolve: $x^{2}-2x-9 = 0$\r\n\r\nMy answers:\r\n$x$ = $\\sqrt{10}-1$\r\n$x$ = $-\\sqrt{10}-1$\r\n\r\nHowever, those were said to be incorrect.\r\n\r\nThe correct answers are\r\n\r\n$x = 1+\\sqrt{10}$\r\nand\r\n$x = 1-\\sqrt{10}$\r\n\r\nWhat did I do wrong?\r\n\r\n--------\r\n\r\n1) $x^{2}-2x = 9$\r\n\r\n2) $\\frac{1}{2x}*-2x$\r\n\r\n3) $x^{2}-2x+1 = 9+1$\r\n\r\n4) $(x-1)(x-1) = 10$\r\n\r\n5) $\\sqrt{x-1}^{2}= \\sqrt{10}$\r\n\r\n6) $x-1 = \\sqrt{10}$\r\n\r\nFrom there, I started solving.",
"Solution_1": "You are mostly correct, but I I have a few comments:\r\n\r\n$\\sqrt[]{{x-1}^{2}}= \\pm \\sqrt[]{10}$, not just $\\sqrt[]{10}$\r\n\r\nIf $x-1 = \\pm \\sqrt[]{10}$, then $x = 1 \\pm \\sqrt[]{10}$, giving the correct answers. Maybe you just made a mistake in solving for x in the last step?",
"Solution_2": "[hide=\"hint\"]in step 5,just remeber that\n$\\sqrt{(x-1)^{2}}=|x-1|$\nso \n$x-1=\\sqrt{10}$\nand\n$x-1=-\\sqrt{10}$[/hide]",
"Solution_3": "Why is the $1$ put before the $\\sqrt{10}$? \r\nThat is what I do not understand.\r\n\r\nIf I am isolating $x$, then I need to add the one to both sides...\r\n\r\nAh. I subtracted instead of added...\r\n\r\n*Points finger to head*\r\n*Bang*",
"Solution_4": "\\begin{eqnarray*}x^{2}-2x-9 &=& 0 \\hfill \\\\ (x^{2}-2x+1) &=& 10 \\hfill \\\\ (x-1) &=& \\pm \\sqrt{10}\\hfill \\\\ x_{1}&=& 1+\\sqrt{10}\\hfill \\\\ x_{2}&=& 1-\\sqrt{10}\\hfill \\\\ \\end{eqnarray*}"
}
{
"Tag": [
"geometry",
"3D geometry",
"geometric transformation",
"rotation"
],
"Problem": "There are six uncoloured wooden cubes of equal size and three different colours. On each of the cubes exactly three sides shall be completely covered with one colour. On each of the cubes all three colours shall be used.\r\n\r\nIs it possible, to colour the six cubes as mentioned above, such that each cube can be distinguished from each of the other ones ?\r\n\r\nHint: Two coloured cubes are not to be distinguished if they can be transformed into each other by means of rotations and/or translations.",
"Solution_1": "Yes; paint as follows, letting the three colors be red, blue, and green for simplicity. The locations of the colors don't even matter...once again, this seems a bit easy: (spoiler)\n\n\n\n[hide]3 red, 2 blue, 1 green\n\n3 red, 2 green, 1 blue\n\n3 blue, 2 red, 1 green\n\n3 blue, 2 green, 1 red\n\n3 green, 2 red, 1 blue\n\n3 green, 2 blue, 1 red[/hide]"
}
{
"Tag": [
"AMC",
"AIME"
],
"Problem": "A stone is dropped into a well and the report of the stone striking the bottom is heard $ 7.7$ seconds after it is dropped. Assume that the stone falls $ 16t^2$ feet in $ t$ seconds and that the velocity of sound is $ 1120$ feet per second. The depth of the well is:\r\n\r\n$ \\textbf{(A)}\\ 784 \\text{ ft.} \\qquad\\textbf{(B)}\\ 342 \\text{ ft.} \\qquad\\textbf{(C)}\\ 1568 \\text{ ft.} \\qquad\\textbf{(D)}\\ 156.8 \\text{ ft.} \\qquad\\textbf{(E)}\\ \\text{none of these}$",
"Solution_1": "...is AHSME equiv to AMC or AIME?\r\n\r\n[hide=\"solution\"]\n$ d \\equal{} 16t_1^2 \\qquad(1)$\n$ d \\equal{} 1120t_2 \\qquad(2)$\n\n$ t_1 \\plus{} t_2 \\equal{} \\frac {d}{1120} \\plus{} \\frac {\\sqrt {d}}{4} \\equal{} 7.7$\n\nLet $ d \\equal{} a^2$\n\n$ a^2 \\plus{} 280a \\minus{} 8624 \\equal{} 0 \\Leftrightarrow (a \\minus{} 28)(a \\plus{} 308)$\n$ d \\equal{} 28^2 \\equal{} 784$\n\nAnswer $ \\rightarrow \\boxed{A}$\n\nAre you allowed calculators on this exam?[/hide]",
"Solution_2": "AHSME was the equivalent of AMC and AIME together until about 1984, when it was \"downgraded\" to only AMC because of the introduction of the AIME.\r\nAnd the first pocket calculator was invented in 1967, so I don't think calculators were allowed. :D",
"Solution_3": "We just have to solve for $ t$ in $ 1120(7.7 \\minus{} t) \\equal{} 16t^2$ and then plug that into $ 16t^2$ to get the answer. As a side note, the acceleration of the stone is taken to be $ 32$ feet per second.\r\n\r\nBy the way, how many problems are there on the AHSME?\r\n\r\nEDIT: Sorry, I messed up and have corrected it; I have no idea why I integrated instead of differentiating...",
"Solution_4": "mathwizard, the physics equation is $ \\delta x \\equal{} 1/2at^2 \\plus{} v_0 t$ so the acceleration is $ 32$ feet per second, which is commonly used as the Earth's acceleration due to gravity.",
"Solution_5": "[quote=\"mathwizarddude\"]We just have to solve for $ t$ in $ 1120(7.7 \\minus{} t) \\equal{} 16t^2$ and then plug that into $ 16t^2$ to get the answer. As a side note, the acceleration of the stone is taken to be $ 32$ feet per second.\n\nBy the way, how many problems are there on the AHSME?\n\nEDIT: Sorry, I messed up and have corrected it; I have no idea why I integrated instead of differentiating...[/quote]\n[quote=\"facis\"]mathwizard, the physics equation is $ \\delta x \\equal{} 1/2at^2 \\plus{} v_0 t$ so the acceleration is $ 32$ feet per second, which is commonly used as the Earth's acceleration due to gravity.[/quote]For the sake of accuracy, acceleration is feet per second [b]squared[/b] or $ \\text{ft}/\\text{sec}^2$",
"Solution_6": "oops... stupidity at its best, I guess. ft/s^2 sounds better... And while I'm at it, I meant to use a $ \\Delta$ not a $ \\delta$."
}
{
"Tag": [
"function",
"limit",
"real analysis",
"real analysis unsolved"
],
"Problem": "Let $f: X\\rightarrow Y$ be a continuous function and let $E$ be dense in $X$. Prove that $f\\left(E\\right)$ is dense in $f\\left(X\\right)$.",
"Solution_1": "Fix $y\\in f(Y)$. Exist $x\\in X$ such that $f(x)=y$ and sequence $\\{e_{n}\\}\\subset E$ such that $\\lim{e_{n}}=x$. Final $\\{f(e_{n})\\}\\subset f(E)$ and $\\lim{f(e_{n})}=f(x)=y$.",
"Solution_2": "If we're in a general topological space, note that $f^{-1}(\\overline{f(E)})$ is a closed subset containing $E$, hence containing $\\overline{E}= X$, so $f(X) \\subset \\overline{f(E)}$."
}
{
"Tag": [
"topology",
"real analysis",
"real analysis unsolved"
],
"Problem": "Family {$V_ {\\alpha}$} of open subsets of $X$ is called [i]basis[/i] for $X$, if for arbitrary $x \\in X$ and arbitrary $G \\subset X$ such that $x \\in G$ there exists $V_ {\\alpha}$ such that $x \\in V _{\\alpha} \\subset G$. In other words: every open subset of $X$ is sum of some sets of {$V_ {\\alpha}$}.\r\n\r\nProve that every metric space which contains countable dense subset has countable basis.",
"Solution_1": "Take the open balls with radii $\\frac 1n$ centered at the countably many points in the dense countable set.",
"Solution_2": ":first: perfect solution :) - it is THAT easy though I like it"
}
{
"Tag": [
"calculus",
"integration",
"geometry",
"trigonometry",
"calculus computations"
],
"Problem": "In the $ x \\minus{} y$ plane, find the area of the region bounded by the parameterized curve as follows.\r\n\r\n$ \\left\\{\\begin{array}{ll} x \\equal{} \\cos 2t & \\quad \\\\\r\ny \\equal{} t\\sin t & \\quad \\end{array} \\right.\\ (0\\leq t\\leq 2\\pi)$",
"Solution_1": "Can anyone solve the problem? :)",
"Solution_2": "[quote=\"kunny\"]In the $ x \\minus{} y$ plane, find the area of the region bounded by the parameterized curve as follows.\n\n$ \\left\\{\\begin{array}{ll} x \\equal{} \\cos 2t & \\quad \\\\\ny \\equal{} t\\sin t & \\quad \\end{array} \\right.\\ (0\\leq t\\leq 2\\pi)$[/quote]\r\n\r\nThe curve is not simple, it intersects itself at the point (1,0). So break it up into\r\n\r\n$ C_1: \\vec{r}(t)\\equal{}< \\cos 2t, t\\sin t>,\\mbox{ for }0\\leq t\\leq \\pi$ (Negatively orientated)\r\n\r\n\r\n$ C_2: \\vec{r}(t)\\equal{}< \\cos 2t, t\\sin t>,\\mbox{ for }\\pi\\leq t\\leq 2\\pi$ (Positively orientated)\r\n\r\nGreen's Theorem gives the area bounded by the curve as\r\n\r\n\\[ A\\equal{}\\minus{}\\oint_{\\minus{}C_1\\cup C_2} y\\, dx \\equal{} \\minus{}2\\int_{0}^{\\pi}t\\sin t \\sin 2t\\, dt\\plus{}2\\int_{\\pi}^{2\\pi}t\\sin t \\sin 2t\\, dt\\]\r\n\r\nreplace $ t$ with $ t\\minus{}\\pi$ in the second integral to get \r\n\r\n\\[ A\\equal{} \\minus{}2\\int_{0}^{\\pi}t\\sin t \\sin 2t\\, dt\\minus{}2\\int_{0}^{\\pi}(t\\minus{}\\pi)\\sin t \\sin 2t\\, dt\\]\r\n\\[ \\equal{} \\minus{}4\\int_{0}^{\\pi}t\\sin t \\sin 2t\\, dt\\plus{}2\\pi\\int_{0}^{\\pi}\\sin t \\sin 2t\\, dt\\]\r\n\\[ \\equal{} 2\\int_{0}^{\\pi}(\\pi \\minus{}2t)\\sin t \\sin 2t\\, dt\\equal{} 4\\int_{0}^{\\pi}(\\pi \\minus{}2t)\\sin^2 t \\cos t\\, dt\\]\r\n\r\nnow by parts with $ u\\equal{}\\pi\\minus{}2t$\r\n\r\n\\[ \\equal{} \\left[ \\frac{4}{3}(\\pi \\minus{}2t)\\sin^3 t\\right]_{t\\equal{}0}^{\\pi}\\plus{}\\frac{8}{3} \\int_{0}^{\\pi}\\sin^3 t\\, dt\\]\r\n\\[ \\equal{}\\frac{8}{3} \\int_{0}^{\\pi}\\left(\\sin t\\minus{}\\sin t\\cos^2 t\\right)\\, dt\\]\r\n\\[ \\equal{}\\frac{8}{3}\\left[\\minus{}\\cos t\\plus{}\\frac{1}{3}\\cos^3 t \\right]_{t\\equal{}0}^{\\pi}\\equal{}\\boxed{\\frac{32}{9}}\\]",
"Solution_3": "That's correct answer.\r\n\r\nThis problem seems to be difficult for applicants for Tokyo University, because Japanese high school students don't study Green's theorem."
}
{
"Tag": [
"MATHCOUNTS"
],
"Problem": "Discuss our plans to win the Spirit Award and win the Team Round... MUAHAHAHAHA :rotfl: :rotfl: :rotfl:",
"Solution_1": "You just lost the Spirit Award. Thank you, have a good nats trip.",
"Solution_2": "The four MO team people please post your heights here.",
"Solution_3": "shhhhhhhhhhhhhhhhhhhhhhhhhhhhhhhhhhh............................\r\nwe don't want to hint at our plan....\r\n\r\nAlthough asking for heights doesn't give away much...",
"Solution_4": "Just letting you guys know that I am going to be 10-15 minutes late for Mathcounts Practice at Mewto's house tomorrow night...",
"Solution_5": "Okay. I'll have little RomanianGenius go tell mom.",
"Solution_6": "Something tells me you guys will win the Spirit Award...\r\n\r\nUnless the CA team puts on some epic rickroll...*COUGH MILLER4MATH COUGH*",
"Solution_7": "Although there has been a horrendous amount of spam and useless posts/threads in this forum, I would like to say congratulations to everyone on the Missouri national MC team, being a former Missouri competitor myself. As everyone knows, Missouri's best rank was second as a team; since all four of you are from Ladue, please, [b]please[/b] make Missouri proud (as well as St. Louis) by at least getting first :D . After all, you all live right next to each other, so there's no reason to not get enough practice in, and overall, do your best, congratulations, and make sure at least half the team gets into the Countdown Round.",
"Solution_8": "Thanks Michael, we will try to make you proud by being the first team to be kicked out of National MC.\r\n\r\nIs that good?",
"Solution_9": "Mewto's response = WIN.",
"Solution_10": "I don't think that's what he meant by first...\r\n\r\n...and I hope you were joking.",
"Solution_11": "Oh dang I totally just remembered. My first ever official coutndown was when ZzZzZ...Z wiped the floor with me and Sam at chapter. He buzzed in like .5 sec and the fail judge guy read the whole problem before calling on him to give the answer :huh:",
"Solution_12": "[size=200]EPIC WIN....KANSAS GOT FIRST ONE YEAR AT NATS\n\nMISSOURI = EPIC UBER PHAILURE\n\nKANSAS = EPIC UBER WIN[/size]",
"Solution_13": "So that makes the mean place over your 25 years of MC what, 57x24+1 /25=54.76",
"Solution_14": "No Mewto. We definitely beat the Virgin Islands and Guam at least 3 times.",
"Solution_15": "Oh. Well then. It would appear my maths is flawed.",
"Solution_16": "Perhaps your math is just too beastly for mankind to grasp.\r\n\r\nSO BEASTLY MMM",
"Solution_17": "This would explain why I am currently failing pre-algebra...",
"Solution_18": "This would explain why I am currently failing pre-algebra...",
"Solution_19": "[quote=\"Mewto55555\"]Oh dang I totally just remembered. My first ever official coutndown was when ZzZzZ...Z wiped the floor with me and Sam at chapter. He buzzed in like .5 sec and the fail judge guy read the whole problem before calling on him to give the answer :huh:[/quote]\r\n\r\nAnd then I believe I lost to Shuyang... Strange how being up against Ladue kids gives you motivation to do well, but then with no more strong competition, the mind just goes blank...",
"Solution_20": "[quote=\"ZzZzZzZzZzZz\"][quote=\"Mewto55555\"]Oh dang I totally just remembered. My first ever official coutndown was when ZzZzZ...Z wiped the floor with me and Sam at chapter. He buzzed in like .5 sec and the fail judge guy read the whole problem before calling on him to give the answer :huh:[/quote]\n\nAnd then I believe I lost to Shuyang... Strange how being up against Ladue kids gives you motivation to do well, but then with no more strong competition, the mind just goes blank...[/quote]\r\n\r\nYeah and then you lost first round at state too, and then the same thing happened at nationals :rotfl:",
"Solution_21": "[quote=\"Mewto55555\"][quote=\"ZzZzZzZzZzZz\"][quote=\"Mewto55555\"]Oh dang I totally just remembered. My first ever official coutndown was when ZzZzZ...Z wiped the floor with me and Sam at chapter. He buzzed in like .5 sec and the fail judge guy read the whole problem before calling on him to give the answer :huh:[/quote]\n\nAnd then I believe I lost to Shuyang... Strange how being up against Ladue kids gives you motivation to do well, but then with no more strong competition, the mind just goes blank...[/quote]\n\nYeah and then you lost first round at state too, and then the same thing happened at nationals :rotfl:[/quote]\r\n\r\nAwful times... Awful times..."
}
{
"Tag": [
"function",
"algebra proposed",
"algebra"
],
"Problem": "Let a differentiable function $f: \\mathbb{R} \\rightarrow \\mathbb{R}$ that follows the conditions:\r\na) $f(0) = 0$ and $f(2) = 2$.\r\nb) For all $a \\in \\mathbb{R} - \\{0\\}$ the tangent line to $f$ on $P(a,f(a))$ meets x-axis on $A$ and y-axis on $B$ such that $A$ is the line-bissector of $\\overline{BP}$.\r\n\r\nFind $f(3)$.",
"Solution_1": "Solve the differetial equation $f'(x)=\\frac{2}{x}f(x)$ and $f(2)=2,$ yielding $f(x)=\\frac{1}{2}x^2.$\r\nThus $f(3)=\\frac{9}{2}.$"
}
{
"Tag": [],
"Problem": "Be a,b,c roots of $P(x)x^3+px^2+qx+r=0$ . If $S_n=a^n+b^n+c^n$, n is integer and n>3, being\r\n $K=S_n+pS{}_n{}_-{}_1+qS{}_n{}_-{}_2$ Find K",
"Solution_1": "what are you asking? $K$ in terms of $S$? if thats the case figure out $S_1, S_2, S_3$ using relationships btwn roots and coefficients.\r\n\r\nthen write $x^3=-px^2-qx-r$\r\nso $x^4=-px^3-qx^2-rx$.\r\n\r\nso then $S_4=-pS_3-qS_2-rS_1$ and continue like this so you have\r\n\r\n$S_n=-pS_{n-1}-qS_{n-2}-rS_{n-3}$, so $S_n+pS_{n-1}+qS_{n-2}=-rS_{n-3}=K$."
}
{
"Tag": [
"probability"
],
"Problem": "Is 1.9999999999999999999... recurring a rational number?\r\n\r\n[hide=\"working\"]let\nx=1.999...\n10x=19.9999\n9x=18\nx=2\n2 is rational\nTherefore as x=2, x is rational[/hide]\n\n[hide=\"ambiguity of the solution\"]In this case we proved that 2 is rational and when x is 2, x is rational, but in the original assumption, we said x=1.999...\nso in a way we did not prove that 1.999... is rational at the end.\nif this is the case, we can never prove that it is rational\ntherefore it is not rational (?)\n\nreasoning seems flawless but logic doesn't seem to tie up for either cases???[/hide]",
"Solution_1": "It is very well known that $ 0.9999999...\\equal{}1$.\r\nIt is not smaller than 1 by some infinitesimal value; this is a common misconception.\r\n\r\n[hide=\"Proof\"]\nLet $ x\\equal{}0.99999...$\nThen $ 10x\\equal{}9.99999...$\nAnd $ 10x\\minus{}x\\equal{}9x\\equal{}9$.\nDividing both sides by $ 9$, we get $ x\\equal{}1$.\n\nTherefore, $ 0.99999...\\equal{}1$\n[/hide]\r\n\r\nTherefore, $ 1.99999...\\equal{}2$, which is obviously rational.",
"Solution_2": "There exists the following remarkable result:\r\n\r\n$ 0.aaaaa...\\equal{}\\frac{a}{9}$, where $ a$ $ \\in$ $ \\{ 0,1,2,3,4,5,6,7,8,9 \\}$.",
"Solution_3": "I don't know if I'd call it \"remarkable\" :wink:\r\n\r\nI think you can generalize it for all $ n\\in\\mathbb Z$.\r\nIf $ n \\equal{} \\overline{a_0a_1\\cdots a_m}$, and we want the fractional form of the recurring decimal expansion $ 0.\\overline{a_0a_1\\cdots a_m}$, we let $ x$ be that value, multiply $ x$ by $ 10^{m \\plus{} 1}$, and subtract $ x$ away to get: $ x(10^{m\\plus{}1}\\minus{}1)\\equal{}n$\r\n\r\nAnd therefore, the fractional form is $ \\frac {n}{10^{m \\plus{} 1} \\minus{} 1}$.\r\n\r\nIf $ n \\equal{} a$, $ m \\equal{} 0$, and the fraction is $ \\frac {a}{9}$, as expected.",
"Solution_4": "I want to address a different step in your logic.\r\n\r\nWe can never prove that it is rational\r\nTherefore, it is not rational.\r\n\r\nThe above two steps rely on the false assumption that if something is true, it can be proven. See [url=http://en.wikipedia.org/wiki/Godel%27s_incompleteness_theorem]Godel's Incompleteness Theorem[/url].\r\n\r\nIn other words, the provability of a statement does not imply anything about that statement's truth."
}
{
"Tag": [
"modular arithmetic"
],
"Problem": "What is the smallest possible value of $\\left|12^{m}-5^{n}\\right|$, where $m$ and $n$ are positive integers?",
"Solution_1": "The value can't be zero, by the fundamental theorem of arithmetic. Also, note that $|12^{m}-5^{n}|$ can't be divisible by $2,3$ or $5$, hence the expression can't be equal to $2,3,4,5,6$. Assume it can be equal to $1$. Then either $12^{m}-1=5^{n}$ or $5^{n}-1=12^{m}$. The first case is impossible as LHS is divisible by $12-1=11$ and RHS is not. The second case, for modulo $6$, gives $(-1)^{n}-1\\equiv 0\\pmod{6}\\implies n=2k$. Hence $25^{k}-1=12^{m}$. Since the unit digit of LHS is $4$, that gives $m=4l+2$, hence $25^{k}-1=12^{4l+2}$. Now RHS is divisible by $9$, hence $25^{k}\\equiv(-2)^{k}\\equiv 1\\pmod{9}$, which gives $k=3s$. Therefore $25^{3s}-1=12^{4l+2}$. However, the LHS is divisible by $25^{3}-1=(25-1)(625+25+1)=24\\cdot 651=2^{3}\\cdot 3^{2}\\cdot 7\\cdot 31$, and RHS isn't divisible by either $7$ or $31$. Therefore the initial expression can't be equal to $1$. It can be equal to $7$ for $m=n=1$, thus we conclude that $7$ is its minimal value."
}
{
"Tag": [
"superior algebra",
"superior algebra unsolved"
],
"Problem": "Let $p$ be a prime number. Show that $\\sigma\\in S_n$ is of order $p$ if and only if $\\sigma$ can be written as products of commuting $p$-cycles.",
"Solution_1": "just look at cycle structure."
}
{
"Tag": [
"geometry",
"rectangle",
"perimeter",
"symmetry",
"geometry solved"
],
"Problem": "A rectangle has a perimeter of 15cm. What is it`s maximum volume if the rectangle starts spinning around one of it`s sides shaping a cylinder?\r\n\r\nI assume one side is: x\r\nI assume one side is: x\r\nThe other side is : (15-2x)/2\r\nThe other side is : (15-2x)/2\r\n\r\nA cylinders volume is : \u03c0*r^2*h\r\nWhere \u03c0=3.14159265359\r\nr: is half of one side as one whole side becomes the cylinders diametre when it starts spinning.\r\nand height is a full opposite site.\r\n\r\nI assume it starts spinning on the side (15-2x)/2 which becomes the cylinders diametre and x its hight:\r\nBut the forumla for a cylinders volume needs the radius which is: ((15-2x/2)/2) = (15-2x)/4\r\n\r\nThe formula for the cylinders volume then is: \u03c0*x((15-2x)/4)^2\r\n\r\nthe Above calculation results in a volume for the cylinder as :\r\n(225\u03c0x) - (60\u03c0x^2)+(4\u03c0x^3) /16\r\n\r\nAs we want to know the maximum volume we derive(typo) it:\r\n\r\nV`=37,699x^2 - 376,991x + 706,858\r\nThis is a simple 2nd degree equation resulting in x1=(7,5) x2=2,5\r\n\r\nNow to get the maximum volume we put 2,5 into the Volume formula since this is a maximum:\r\n\r\n(225\u03c0*2,5) - (60\u03c0*2,5^2) - (4\u03c02,5^3) / 16 = 49,08\r\n\r\nBUT HERE is my problem: The answer ought to be 196 and therefor (225\u03c0*2,5) - (60\u03c0*2,5^2) - (4\u03c02,5^3) / [b]4[/b] \r\nis the correct volume formula for the cylinder.\r\n\r\nBut since one side of the original rectangle is (15-2x/2) then the raidus for the cylinder is ((15-2x/2)/2) \r\n\r\nMy fault accurs as I get the forumla for the cylinder to be split by 16, it should be 4, can anyone see where in the transitiation/calculation from rectangle to cylinder and coverting diametre to a radius I do wrong since the above calculation is correct except the volume is to be split by 4 and not as I get 16.\r\n\r\nThanks in advance, Stephan",
"Solution_1": "[quote=\"gossen19\"]r: is half of one side as one whole side becomes the cylinders diametre when it starts spinning...\n[/quote]\r\nNo, cylinder radius is one whole side. The rectangle is not spinning around one of its symmetry axes, but around one of its sides.",
"Solution_2": "Ah right, ofcourse! thanks :>"
}
{
"Tag": [],
"Problem": "http://www.youtube.com/watch?v=VjgidAICoQI\r\nWhat are your opinions?",
"Solution_1": "Neal Adams is a comic book artist, not a scientist.\r\n\r\nhttp://en.wikipedia.org/wiki/Expanding_earth_theory",
"Solution_2": "A takedown of Neal Adams is available [url=http://scienceblogs.com/goodmath/2006/12/wacky_physics_it_must_be_right_1.php]here[/url]."
}
{
"Tag": [
"Princeton",
"college"
],
"Problem": "I just took the SAT 1. \r\nhope to get a 800 in math. does any one remember a mathquestion where a stupid mistake could be done?\r\n\r\nin the past one i got 1 question wrong.... i always do silly mistakes.. :?",
"Solution_1": "To be honest, I didn't see any today that may have caused stupid mistakes? I mean...I suppose there was that one question with the chart and it asked for the number of books sold in a certain week...(this better not count as revealing questions) I mean you could've read the chart wrong...but otherwise...eh...",
"Solution_2": "true,\r\ni think the answer to this oen was 46,000 if i'm not mistaken...\r\ni think i was careful to read the chart well (i am the kind of person that could have read it wrong)",
"Solution_3": "[quote=\"manuel\"]true,\ni think the answer to this oen was 46,000 if i'm not mistaken...\ni think i was careful to read the chart well (i am the kind of person that could have read it wrong)[/quote]\r\n\r\nDifferent problem.",
"Solution_4": "What about SAT II, how'd it go for ya!!!",
"Solution_5": "i almost missed the $m^2-n^2=12$, find values of $(m-n)$ question. then i realized 1 didn't work.",
"Solution_6": "what answer did u selected inthis one..\r\n2 right?",
"Solution_7": "[quote=\"Rushil\"]What about SAT II, how'd it go for ya!!![/quote]\r\n\r\nMath IIC was a piece of cake.\r\n\r\nPhysics... was another story. I left 4 blank, which might kill my 800. I hope not, though, there's just too much material to study and formulas to memorize. The problems were easier than the AP test, but they don't give you formulas which is stupid; I forgot some of the waves/sound/optics ones.",
"Solution_8": "[quote=\"manuel\"]what answer did u selected inthis one..\n2 right?[/quote]\r\n :( now i realize only 1 is right. yes i did put 2 right. there goes the 800.",
"Solution_9": "oh... so u got 790? that's good too,\r\nbut wait i don't get it, what were the possible answers to this question?\r\n\r\nbtw, i wrote the best essay i've done in my life!!! so i hope toget a good writing score too",
"Solution_10": "Edit: Oops. Forgot that you can't reveal questions. ;)\r\n\r\nThat retarded essay! Stopped in mid-sentence on my third paragraph!",
"Solution_11": "[quote=\"Forsaken\"]Edit: Oops. Forgot that you can't reveal questions. [/quote]\r\ndon't worry. we won't tell anyone. ;)\r\n\r\nand i wonder how they will like an essay that isn't divided into paragraphs. i just couldn't find a spot where my ideas transitioned where paragraphs were needed. o well.",
"Solution_12": "[quote=\"paladin8\"]\n\nPhysics... was another story. I left 4 blank, which might kill my 800. I hope not, though, there's just too much material to study and formulas to memorize. The problems were easier than the AP test, but they don't give you formulas which is stupid; I forgot some of the waves/sound/optics ones.[/quote]\r\n\r\nit has a nice curve. i got an 800 after guessing on 4 or 5, and im sure i missed a few more than that.",
"Solution_13": "Ugh. Essay. :wallbash_red: I don't GET IT. I wrote for the entire 25 minutes, right? And all I got was a page and a half in normal writing. But the girl next to me? (I glanced over after we were closing the tests up) She wrote a FULL TWO PAGES... and her writing was [size=59]tiny[/size]. HOW? How do people [i]do that????[/i]",
"Solution_14": "I wrote two full pages because i did an essay in the same topic for school...that's good luck :D",
"Solution_15": "writing two pages shouldn't be that difficult. i spent the first 10 minutes trying to decide which side to argue and then come up with details. so i wrote a page and a quarter in around 15 minutes. 2 pages isn't out of the question at all.",
"Solution_16": "If I didn't even start my conclusion, what's the max score I can get? If I had finished, \r\nI think I would have gotten a 10, so how much would that be without a conclusion?\r\n\r\nMan, I just realized that I have at least 2 wrong on math and 3 wrong on CR. I hope I can pull a 2200. :(",
"Solution_17": "Lol unless you guys are seniors, there's really no need to worry. Plenty of more opportunities.\r\n\r\nAs for the essay, note that some people just memorize a format and plug in the essay to an outline they've already decided on. Lol. It's very difficult to just see a question and start writing madly for 25 minutes unless you do that, especially if you're trying to get a full 2 pages at reasonable size.\r\n\r\nEssay scoring depends on the length of the essay as well. So unless you've written more than 3/2 pages, an 11-12 is out of the question and a 9-10 is pushing it. Otherwise, it just matters on how coherent your essay is and whether or not you've used correct syntax, good vocab, and supported your thesis. My two, unprofessional cents on that matter...",
"Solution_18": "I am a senior. :( \r\n\r\nAt least I know for sure that the 3 paragraphs I wrote this time are better than the ones I wrote on the March test. (Got a 6/12) :blush:",
"Solution_19": "[quote=\"Rushil\"]What about SAT II, how'd it go for ya!!![/quote]\r\n\r\nmath II C >> easy(answered every question) > should get 800\r\n\r\nphysics>> easy >> left 2 blank>> should get 800\r\n\r\nchemistry>> Super easy>> answered every question except one which i think was WRONG!! the choices were incorrect, that would be q 57 i think >> should get 800.\r\n\r\n\r\nthats it. :)",
"Solution_20": "Which question are you talking about in Chem (Sat II) that you think is wrong. PM me!!!",
"Solution_21": "[quote=\"paladin8\"]\n\nPhysics... was another story. I left 4 blank, which might kill my 800. I hope not, though, there's just too much material to study and formulas to memorize. The problems were easier than the AP test, but they don't give you formulas which is stupid; I forgot some of the waves/sound/optics ones.[/quote]\r\n\r\nwell in the AP test they don't give the formulas up to the FR section either.\r\n\r\nbut SAT physics is way easier than AP test (not that AP test is hard, cause its not!). for answering the MC problem all they ask you in the multiple choice problems is to know the basic knowledge of the subject, thats it. :)",
"Solution_22": "For people that took the Physics test, out of curiousity did you guys use a review book? If so, which one(s)? And were they any good?\r\n\r\nSame with chem.",
"Solution_23": "i took AP physics B, C and AP chem last year. and got 5's in all of them, so i didn't have anyproblem reviewing stuff, \r\nbut i went to the library and checked out the Kaplans SAT review for chem (which wasn't that good!), and also i looked at the SAT physics book by princeton hall the night before the test, that was pretty good, bettar than the Kaplan one.\r\n\r\noh i forgot to mention that the Kaplan book that i said, was for 03-04. so it was a bid too old, but i don't like Kaplan anyways.\r\n\r\n~amir",
"Solution_24": "for chem (which i took a couple years ago), i took the AP test in may (got a 5) and then took the SAT II in June. I was totally out of practice, didn't even go over the stuff til two days before the SAT II, and I still got a 760. i used princeton review, which isnt really the best review book, but i like the way theirs is organized. (you have to be very careful when going over math-type problems, because they most definitely WILL make mistakes).",
"Solution_25": "[quote=\"amirhtlusa\"][quote=\"paladin8\"]\n\nPhysics... was another story. I left 4 blank, which might kill my 800. I hope not, though, there's just too much material to study and formulas to memorize. The problems were easier than the AP test, but they don't give you formulas which is stupid; I forgot some of the waves/sound/optics ones.[/quote]\n\nwell in the AP test they don't give the formulas up to the FR section either.\n\nbut SAT physics is way easier than AP test (not that AP test is hard, cause its not!). for answering the MC problem all they ask you in the multiple choice problems is to know the basic knowledge of the subject, thats it. :)[/quote]\r\n\r\nTrue; in general the questions were a lot easier. But the AP test is graded on a ridiculous curve, so if you forget like 5 formulas it doesn't hurt your chances of pulling an easy 5. I don't know about the SAT, though. The curve is supposed to be generous, but how generous is the question.",
"Solution_26": "Kaplan = bad review books.\r\n\r\nI would suggets PR like most people, but I also like 5 Steps to a 5 for APs. Or barrons."
}
{
"Tag": [
"calculus",
"integration",
"geometry",
"derivative",
"function",
"geometric series",
"calculus computations"
],
"Problem": "Find the area under the curve $ f(x)\\equal{}4x^3 \\plus{} 3x^2 \\plus{} 2x \\plus{}1$ from $ x \\equal{} 0$ to $ x \\equal{} 2$.",
"Solution_1": "You need to tell us the value of $ f$ and its first derivative at some point otherwise the answer is not unique.",
"Solution_2": "Because of the constants and stuff that are canceled out when we find the derivative? I'll just change the problem. Sorry about that, I'm still learning all this calculus stuff.",
"Solution_3": "Do you know how to find the antiderivative of $ f$? If so, evaluate it at $ x \\equal{} 2$ and $ x \\equal{} 0$ and then subtract its value at $ x \\equal{} 0$ from its value at $ x \\equal{} 2$.",
"Solution_4": "[hide=\"Solution\"]\nFrom the rule $ f(x)\\equal{}x^{n}\\implies f'(x)\\equal{}nx^{n\\minus{}1}$, and noticing the pattern in the function $ f$ given ($ 4x^3$, etc), the antiderivative is pretty obvious.\n\n$ F(x)\\equal{}\\int f(x)\\; dx\\equal{}x^4\\plus{}x^3\\plus{}x^2\\plus{}x\\plus{}C$\n\nBy the fundamental theorem of calculus, the answer is $ F(2)\\minus{}F(0)\\equal{}\\frac{2^{5}\\minus{}1}{2\\minus{}1}\\minus{}1\\equal{}\\boxed{30\\text{ square units}}$.\n\n(That last step used the sum of a finite geometric series to skip a little calculation).\n[/hide]"
}
{
"Tag": [
"trigonometry",
"induction",
"LaTeX",
"linear algebra",
"matrix",
"geometry",
"ARML"
],
"Problem": "My hope is that we can get more interest in proofs by doing a proof marathon. The power question score for SC needs to come up. Anyways, here is the first item that needs to be proven. This is a quite famous identity. I will refrain from naming it in hopes that you guys will try to prove it on your own. Prove: \\[{ ( \\ cos\\theta + i\\sin}\\theta )^n = \\cos n\\theta + i\\sin n\\theta , n\\in {\\rm Z} \\]\r\n\r\n\r\n[hide=\"HINT\"] Show that the identity is true for positive integers first by using mathematical induction[/hide]",
"Solution_1": "[hide=\"Hint\"]\nHave you even tried induction yet?[/hide]",
"Solution_2": "i don't know if this attempt is strong enough to fully prove it, but it's proof by induction\r\n\r\nshow it works for n=1:\r\n$\\cos\\theta+\\i\\sin\\theta=\\cos\\theta+\\i\\sin\\theta$\r\nassume it works for some n=k:\r\n$(\\cos\\theta+\\i\\sin\\theta)^k=\\cos(k\\theta)+\\i\\sin(k\\theta)$\r\nshow it works for n=k+1:\r\n$(\\cos\\theta+\\i\\sin\\theta)^{k+1}=\\cos((k+1)\\theta)+\\i\\sin((k+1)\\theta)$\r\n$(\\cos(k\\theta)+\\i\\sin(k\\theta))(\\cos\\theta+\\i\\sin\\theta)=$\r\n$\\cos(k\\theta)*\\cos\\theta+i\\sin(k\\theta)*\\cos\\theta+i\\sin(\\theta)*\\cos(k\\theta)-\\sin(k\\theta*\\sin\\theta=$\r\ncombining real and imaginary parts and using trig identies:\r\n$\\cos(k\\theta+\\theta)+i\\sin( k\\theta+\\theta)=$\r\n$\\cos((k+1)\\theta)+i\\sin((k+1)\\theta)$\r\nwhich is what you would get by pluging in k+1 into the equation. this satisfies positive integers. \r\nto show negative integers, joe said to use (cis theta)^n where n is negative (i.e. n=-k for positive k) is the same as 1/(cis theta)^k but i don't feel like doing that right now. maybe shobhit will help me out. i'm $\\text{\\LaTeX}$ed out :(\r\nand if there are any errors in the proof, deal with it and assume i meant to put the correct thing :P",
"Solution_3": "I thought I told you *not* to use \\i and just put i? I don't even know what those \\i 's are :? \r\n\r\nWow...you posted exactly what i said over IM. If I knew that was going to be posted here I would have said it more clearly ;)",
"Solution_4": "$\\text{Problem 2}$\r\n\r\nLet $\\{F_i\\}_{i=1}^\\infty$ denote the Fibonacci sequence.\r\n\r\nProve that \\[ \\begin{pmatrix} 1 & 1 \\\\ 1 & 0 \\end{pmatrix}^n = \\begin{pmatrix} F_n & F_n\\\\ F_n & F_{n-1}\\end{pmatrix} \\]",
"Solution_5": "[quote=\"joml88\"]I thought I told you *not* to use \\i and just put i? I don't even know what those \\i 's are :? [/quote]\r\nwell, some of them came out ok :blush: \r\n\r\nand problem 2 is pretty cool. i did it recently so i'll let someone else tackle it (cough*shobhit*cough)",
"Solution_6": "Guys,\r\nFurious made a big \"no no\" in the world of proofs. You cannot start off by equating things you want to prove. You need to start with one side until it looks like the other side of the equation. Now, that is not to say that you can't figure out how to write the proof by equating things you want to show, but when you finally write the proof up, you need to work from one side of an equation until you can get to the other. Otherwise, you are assuming that the claim is already true when it may not be, e.g. Prove 1 = 2.",
"Solution_7": "He wrote what he wanted to prove first and then worked entirely on the LHS showing that it is equivalent to the RHS. So I think his proof is OK. It could use a lot of work on neatness/clarity/etc though.",
"Solution_8": "Yeah, \r\nI should have looked more closely. To be honest, I did not read the whole thing.",
"Solution_9": "[quote=\"furious\"]and problem 2 is pretty cool. i did it recently so i'll let someone else tackle it (cough*shobhit*cough)[/quote]\n[quote=\"furious\"] but i don't feel like doing that right now. maybe shobhit will help me out.[/quote]\r\nFor now, I will watch you guys solve these proofs and learn from your solutions. That is atleast how I improve in certain areas of math, I watch before I attempt! Give me about a month, and in April, a month before ARML, I'll start doing these ;)\r\n\r\nBut if you insist, here's how i would do the negative numbers part, let $x=-y$, so therefore, we have:\r\n\r\n$(cis\\theta)^x=(cis\\theta)^{-y}=((cis\\theta)^y)^{-1}$, if we assume demoivres theorem is true, we have $cisx\\theta=cis{-y\\theta}$ but since $x=-y$, then that holds true for negative numbers...is that good enough?",
"Solution_10": "[quote=\"Iversonfan2005\"]\nFor now, I will watch you guys solve these proofs and learn from your solutions. That is atleast how I improve in certain areas of math, I watch before I attempt! Give me about a month, and in April, a month before ARML, I'll start doing these ;)[/quote]\r\n\r\nYou should still start trying now. It's easier for us to tell you when you've done something wrong. Additionally, you learn much more from doing it yourself. Watching someone else prove something is far different from actually doing it yourself.",
"Solution_11": "Joe,\r\nI do not think the Fibonacci matrix equation is correct. Take a look, and you should see.",
"Solution_12": "Hey guys, Jim here. Mr. R. is finally forcing me to deal with computers. (AHHHH!)\r\n\r\nHere is the proof for problem 2 (note correction):\r\nP(n) = $\\begin{pmatrix} 1 & 1 \\\\ 1 & 0 \\end{pmatrix}^n = \\begin{pmatrix} F_{n+1} & F_n \\\\ F_n & F_{n-1}\\end{pmatrix}$\r\n\r\nWe will argue by induction that this holds true for integers $n>1$\r\nFirst, we show that is is true for $n=2$:\r\n\r\nP(2) = $\\begin{pmatrix} 1 & 1 \\\\ 1 & 0 \\end{pmatrix}^2 = \\begin{pmatrix} 2 & 1 \\\\ 1 & 1\\end{pmatrix} = \\begin{pmatrix} F_3 & F_2 \\\\ F_2 &{ F_1}\\end{pmatrix}$\r\n\r\nNow, assume this is true for $n=k$ i.e.\r\n\r\nP(k) = $\\begin{pmatrix} 1 & 1 \\\\ 1 & 0 \\end{pmatrix}^k = \\begin{pmatrix} F_{k+1} & F_k \\\\ F_k & F_{k-1}\\end{pmatrix}$\r\n\r\nNow we plug in $k+1$\r\n\r\nP(k+1) = $\\begin{pmatrix} 1 & 1 \\\\ 1 & 0 \\end{pmatrix}^{k+1} = \\begin{pmatrix} 1 & 1 \\\\ 1 & 0 \\end{pmatrix}^{k} \\begin{pmatrix} 1 & 1 \\\\ 1 & 0 \\end{pmatrix} = \\begin{pmatrix} F_{k+1} & F_k \\\\ F_k & F_{k-1}\\end{pmatrix} \\begin{pmatrix} 1 & 1 \\\\ 1 & 0 \\end{pmatrix} = \\begin{pmatrix} F_{k+1} + F_{k+1} & F_k \\\\ F_k + F_{k-1} & F_k\\end{pmatrix} = \\begin{pmatrix} F_{(k+1)+1} & F_{k+1} \\\\ F_{k+1} & F_{k}\\end{pmatrix}$\r\n\r\nSo $P(k) \\Rightarrow P(k+1)$ and hence, by PMI, the proposition is true for all integers $n>1$",
"Solution_13": "Here is Problem 3, sticking with matrices:\r\n\r\nProve:\r\n\r\n$\\begin{pmatrix} 2 & 1 \\\\ 0 & 1 \\end{pmatrix}^n = \\begin{pmatrix} {2^n} & 2^n-1 \\\\ 0 & 1 \\end{pmatrix}$\r\n\r\nfor all positive integer values of n.\r\n\r\nAlso, determine whether or not this holds true for $n=-1$ :ninja:",
"Solution_14": "[hide=\"Solution\"]\nWe begin with the base case, $n=1$:\n\n\\[ \\begin{bmatrix} 2 & 1 \\\\ 0 & 1\\end{bmatrix}^1 = \\begin{bmatrix} 2^1 & 2^1-1 \\\\ 0 & 1\\end{bmatrix} \\]\n\nwhich obviously holds.\n\nNow, we assume that the assertion is true for $n=k$:\n\n\\[ \\begin{bmatrix} 2 & 1 \\\\ 0 & 1 \\end{bmatrix}^{k} = \\begin{bmatrix} 2^k & 2^k-1\\\\ 0 & 1\\end{bmatrix}. \\]\n\nWe proceed to show that it holds for $n=k+1$:\n\n\\[ \\begin{eqnarray*} \\begin{bmatrix} 2 & 1 \\\\ 0 & 1\\end{bmatrix}^{k+1} &=& \\begin{bmatrix} 2 & 1 \\\\ 0 & 1\\end{bmatrix}^k\\begin{bmatrix} 2 & 1 \\\\ 0 & 1\\end{bmatrix}\\\\ &=& \\begin{bmatrix} 2^k & 2^k-1\\\\ 0 & 1 \\end{bmatrix} \\begin{bmatrix} 2 & 1\\\\ 0 & 1\\end{bmatrix}\\\\ &=& \\begin{bmatrix} 2^{k+1} & 2^{k+1}-1\\\\ 0 & 1\\end{bmatrix} \\]\n\nSo the assertion is true by the Principle of Mathematical Induction (PMI).[/hide]",
"Solution_15": "Let's try something other than induction....\r\n\r\n$\\text{Problem 4}$\r\n\r\nProve that $\\sqrt{11}$ is irrational.",
"Solution_16": "Let's say $\\sqrt{11}$ is rational, then it can be written in the form $11=\\frac{a^2}{b^2}$, so therefore $b^2 \\cdot 11=a^2$, because an even divided by an even results in an even number, both $a$ and $b$ are odd. But no odd square divides into another odd square unless they have they can be written with the same base. But in this case, this is impossible since the exponent will always be an even number, therefore, $\\sqrt{11}$ is irrational.....maybe.... :? $\\text{QED?}$",
"Solution_17": "An even divided by an even is always even? How about 6/2 or 40/8? That's a really good attempt for not having seen a problem like this before. Try again though, cause you're thinking in the right direction.",
"Solution_18": "[quote=\"joml88\"]An even divided by an even is always even? How about 6/2 or 40/8? [/quote]\r\n\r\nwow, :| :huh:\r\n\r\n\r\n\r\nok, just gotta think some more\r\n\r\nEDIT: John, why did u delete your post? i'm not gonna post the answer now cuz i'll just copy you word for word",
"Solution_19": "Sorry, I posted that before I saw all this extra stuff, so I was going to get rid of it and let you think more. Seeing as I usually post things really early in the morning, I'm not used to posting at the same time as other people. Anyway...\r\n\r\nSuppose $\\sqrt{11}=\\frac ab$ in simplest terms. Then $11b^2=a^2$ and $11|a$. Let $a=11c$; then $b^2=11c^2$ and $11|b$. Therefore, $\\frac ab$ is not in lowest terms, which is a contradiction. QED\r\n\r\n$\\text{Problem 5}$\r\nProve that e is irrational, assuming $e=1+\\frac 1{1!}+\\frac 1{2!}+\\frac 1{3!}+...$"
}
{
"Tag": [
"superior algebra",
"superior algebra unsolved"
],
"Problem": "This question appears in the section on transcendence bases in Hungerford's Algebra.\r\n\r\nIf $F = K(u_{1},\\ldots,u_{n})$ is a finitely generated extension of $K$ and $E$ is an intermediate field, then $E$ is a finitely generated extension of $K$.",
"Solution_1": "do you know german? :-D\r\n \r\nhttp://www.matheplanet.com/matheplanet/nuke/html/viewtopic.php?topic=49649",
"Solution_2": "Sorry but I'm afraid i don't know german."
}
{
"Tag": [
"geometry",
"trapezoid",
"rectangle"
],
"Problem": "[img]http://i165.photobucket.com/albums/u46/137456/polemath.png[/img]\r\n\r\nTwo poles with diameters 10 and 30 are bound together by a metal band. How could I find the length of the metal band?\r\n\r\nKnowing that tangents are perpendicular to the radiuses of the circles, I tried drawing trapezoids and such, but to no success. =(",
"Solution_1": "[img]http://ipicture.ru/uploads/080604/a4q1JcZ7Yi.png[/img]\r\nWe have $ AD\\equal{}5$, $ CE\\equal{}15$. Draw perpendicular from $ A$ to $ EC$. THen $ ADEF$ is rectangle and $ AD\\equal{}EF\\equal{}5$, so $ FC\\equal{}15\\minus{}5\\equal{}10$. $ A,B,C$ are colinear and $ AC\\equal{}5\\plus{}15\\equal{}20$. Then by Pythagora $ DE\\equal{}AF\\equal{}\\sqrt{AC^2\\minus{}CF^2}\\equal{}\\sqrt{400\\minus{}100}\\equal{}10\\sqrt{3}$",
"Solution_2": "Ah, I can't believe I never thought of that. From that triangle I was able to find the angles to also find what fraction of the circumfrences were being touched by the band and added it to 2(DE) to find the answer. Thanks!"
}
{
"Tag": [
"articles",
"geometry"
],
"Problem": "I have this idea.\r\n\r\nWhy don't we create a separate page in AoPs wiki for each of the books listed [url=http://www.artofproblemsolving.com/Wiki/index.php/Math_books]here[/url] and then one that separate page, we can let users give their in-depth review of it if they own the book and then give it a rating and maybe other stuff. Over time, more and more reviews will accumulate.\r\n\r\nI think this will be very helpful to those people looking to buy certain books especially if the reviews are from AoPSers. (and it will be very organized).",
"Solution_1": "I second that!",
"Solution_2": "If you make this I have no objection [i]as long as[/i] you make the page for the book and then make a separate page for reviews.\r\n\r\nFor example, the main article could be:\r\n\r\nChallenging Problems in Geometry\r\n\r\nand the review page would be:\r\n\r\nChallenging Problems in Geometry/Reviews\r\n\r\nA link to the second would be on the first, of course. Using this format should keep things neater.",
"Solution_3": "This sounds like a very good idea. . .",
"Solution_4": "I fifth that! (I think) :lol: \r\nI may be able to review some of the well-known books (AOPS vol.1 and 2, ACOPS, geometry revisited, etc) once I finsih some of them."
}
{
"Tag": [
"Divisor Functions"
],
"Problem": "Determine all positive integers $k$ such that \\[\\frac{d(n^{2})}{d(n)}= k\\] for some $n \\in \\mathbb{N}$.",
"Solution_1": "Let $ n\\equal{}\\prod p_{i}^{a_{i}}$, so $ d(n)\\equal{}\\prod (a_{i}\\plus{}1)$ and $ d(n^{2})\\equal{}\\prod (2a_{i}\\plus{}1)$. To have $ d(n^{2})\\equal{}kd(n)$, obviously $ k$ must be odd. \r\n\r\nWe will prove that for any odd $ k$ it works. Obviously $ k\\equal{}1$ works, so let $ w$ be the smallest odd positive integer for which it doesn't, and write $ w\\equal{}2^{m}k\\minus{}1$, with $ k 2, then p+1-2a is even so q is even so q=2, therefore b=1, so 2(a ^2 +1)=c ^2 +1=(p-a) ^2 +1=(a ^2 -a+1) ^2 +1. If a=1, that's impossible.If a=2 that's possible, and from here we obtain the solution (a,b,c)=(2,1,3) (and also (1,2,3), because we supposed that p>q).\r\nIf a>2 then a ^2 -a+1=a(a-1)+1>=2a+1, so (a ^2 -a+1) ^2 +1>=(2a+1) ^2 +1 > 2(a ^2 +1).\r\n\r\nTherefore (1,2,3) and (2,1,3) are the only solutions.",
"Solution_2": "The 2nd problem:\r\n\r\nwe presume that: m>n. Let x be their common solution of the two equations. Then x verifies x^n(x+1)(x^(m-n)-1)x=0 and x=-1 is not a solution. Let x be so x^(m-n)=1. We have x<> \\pm 1, x from C\\R, x satisfies: x^(m-1)=1, x^n*(x-1)=-1. We apply the module and get |x|=1 and |x-1|=1 => x cos(pi/3)+isin(pi/3) or x=cos(5pi/3)+i sin(5pi/3), which \r\nmeans x=w or x=w (w has a line on it) , where w^2-w+1=0, w is the 6th order root of unity.\r\nTaking modulo 6 of m and n we get that the only good cases are when m==1(mod 6) and n==1(mod6). So the common roots are given by:\r\n1) the eqs x^2-x+1=0 for m==n==1(mod 6).\r\n2) in other cases it doesn't exist!!!\r\n\r\ncheers mate!"
}
{
"Tag": [],
"Problem": "I come across one question and I do it in two ways , both ways \"sounds\" logical but there is something wrong in it ... here it goes\r\n\r\nQuestion : There are 5 couples , 4 person are chosen among them with the condition the must be at least one male and one female . In how many ways can this be done ?\r\n\r\n[b]First Method[/b] : \r\n\r\nAll ways - 4 male - 4 female = $10C4 - 5C4 - 5C4 = 200 $\r\n\r\n[b]Second Method[/b] :\r\n\r\nFirst we chose 1 male and 1 female which is $5C1\\times 5C1$ then for the remaining two vacancies , we can choose it randomly from the remaining 8 person which is $8C2$ . So total ways = $5C1\\times 5C1\\times 8C2 = 700 $ \r\n\r\nWell , second method seems to flow logically but we know it is not true . Where actually goes wrong ? :?",
"Solution_1": "to me the second methodmakes more sense; on the other hand:\r\n10C4=210.. ;) \r\n\r\nits 12:50am, i'm too sleepy to think, so i'm wrong",
"Solution_2": "your second solution is wrong because you count cases more than one.\r\ni.e: $f_1 m_1 f_2 m_2$ and $f_1 m_2 f_2 m_1$ and $f_2 m_1 f_1 m_2$ and $f_2 m_2 f_1 m_1$ so you count this case $4$ times!!! ;)",
"Solution_3": "[quote=\"a_vakilian\"]your second solution is wrong because you count cases more than one.\ni.e: $f_1 m_1 f_2 m_2$ and $f_1 m_2 f_2 m_1$ and $f_2 m_1 f_1 m_2$ and $f_2 m_2 f_1 m_1$ so you count this case $4$ times!!! ;)[/quote]\r\n\r\nHmm .... I guess that is why the second method gives us such big number :rotfl: .\r\nAnyway , could there be any way to get the same answer by using the concept like second method ? Like , could we divide it by some number(or combination) to cancel off the repeated part ? :?",
"Solution_4": "yes it is possible. you should divide the problem to $3$ case.when there is $2 woman ,2 man$ and $1 woman , 3 man$ and $3 woman, 1 man$ . ;)",
"Solution_5": "[quote=\"a_vakilian\"]yes it is possible. you should divide the problem to $3$ case.when there is $2 woman ,2 man$ and $1 woman , 3 man$ and $3 woman, 1 man$ . ;)[/quote]\r\n\r\nWell , what Im trying to ask is whether we can use the concept in second method , which is like setting a male and female first then the rest can be chosen at random . I know that we can divide it into 3 cases like you mention to solve it but what i want to know is whether the second method can be improve or not to make it right ?",
"Solution_6": "I think there is not such a solution that you want. or it is not worth knowing :? :oops:"
}
{
"Tag": [
"geometry",
"3D geometry",
"sphere",
"calculus",
"integration",
"HMMT",
"LaTeX"
],
"Problem": "A sphere with a radius of 5 is cut by a plane that is 4 away from the center of the sphere. What is the volume of the larger of the two resulting solids?",
"Solution_1": "[hide]I know that you could use calculus (take the equation for a circle, and use the cross sectional method and find the area beneath the graph from -5 to 4).\n\n$\\int_{-5}^{4}{\\pi(\\sqrt{25-x^{2}})^{2}dx}$\n$\\pi\\int_{-5}^{4}{(25-x^{2})dx}$\n$\\pi((25*4-\\frac{4^{3}}{3})-(25*-5-\\frac{(-5)^{3}}{3}))$\n$162\\pi$\n\nAnd you could imagine a hole with a radius of 3 being driven through the sphere. Then if you compare each cross section of that $h$ away from the center with each cross section of a sphere with a radius of 4, $h$ away from the center you notice they are equal. Then you add in the cyclinder that would fill up most of the hole and a sphere that is chopped off by two planes. \n\nSo end the end you would end up with $162\\pi$ like before.[/hide]\r\n\r\nEven if these answers aren't correct I think the process is correct. I just want to find a solution without using calculus or using a crazy idea.",
"Solution_2": "I don't believe a non-calculus formula exists for a sectoroid (a sector revolved about its axis) or a segmentoid although I may be mistaken\r\n\r\neven if an antiderivative existed to the integral for the volume of a segmentoid, it would most definitely not be very nice. Calculus is the best method here",
"Solution_3": "There is a non calculus way to find the volume of a sectoroid. \r\n\r\nhttp://web.mit.edu/hmmt/www/datafiles/problems/2003/pgeom03.pdf\r\n\r\nProblem #6 is not quite a sectoriod, but you can just add the cylinder in. The answer is in the solutions:\r\n\r\nhttp://web.mit.edu/hmmt/www/datafiles/solutions/2003/sgeom03.pdf\r\n\r\nAnd I am pretty sure I have posted both the calculus way and the noncalculus (from the links) way above. But I am guessing there really is not a way to find the volume without calculus or a great imagination so my question is answered.",
"Solution_4": "The solution is perfectly reasonable if you know the geometric derivation of the formula for a sphere.",
"Solution_5": "Yeah, I can't think of a non-calculus solution....",
"Solution_6": "Hint: Cavalieri's principle \r\n\r\nUse google to find out what it is.",
"Solution_7": "The solution Altheman is hinting at (which is essentially what is used in the HMMT solutions linked to above) begins like this:\r\n\r\nSit a cone of radius $r$ and height $r$ down next to a hemisphere of radius $r$ so that the vertex of the cone is in the same plane as the base of the hemisphere and the plane of the base of the cone is tangent to the hemisphere. (If the hemisphere is sitting on a table base-down, the cone is balanced on the same table on its tip.) Now consider the combined area of a cross-section of the two shapes at height $h \\leq r$ above the table. This is a nice way to get the volume of a sphere. It can also be used to get a nice formula for the \"cap\" of a sphere, as in the question here.\r\n\r\nSide note: this requires you to know the volume of a cone, but this can also be derived using a Cavalieri argument.",
"Solution_8": "o.O I am trying to force people to be independent and use google. This skill is very valuable. \r\n\r\nWell...I suppose if you were being all rigirous and stuff, you would need to derive the volume of a cone by putting it in correspondence with a pyriamid. But obviously surface area/volume of a cone fall within most first year geometry classes.",
"Solution_9": "[quote=\"Altheman\"]I am trying to force people to be independent and use google.[/quote] The effect you are having is of being cryptic and ensuring that most of your audience won't ever find out what you're talking about.\n\n[quote]But obviously surface area/volume of a cone fall within most first year geometry classes.[/quote] But the volume is usually asserted, not proved.",
"Solution_10": "[quote=\"JBL\"][quote=\"Altheman\"]I am trying to force people to be independent and use google.[/quote] The effect you are having is of being cryptic and ensuring that most of your audience won't ever find out what you're talking about.\n[/quote]\n\n[hide=\"@JBL\"]Okay. Lets consider an analogous situation. We are given a triangle concurrency problem. I suggest mass points [lets say in the HSB forum]. Then I should expect to get lots of stupid replies asking: \"what are mass points?\" \"where can I learn more about mass points?\" These are stupid questions because you can easiliy find the answer by a quick google search. I know you have this ability. ex. consider your response to mathfanatic in that global warming thread about following fallacies created by those against global warming. Anyway, this situation is similar. You can easily type: \"cavaleri principle sphere\" into google and obtain the link: [url]http://www.walter-fendt.de/m14e/volsphere.htm[/url]. This takes a few seconds and you get a diagram which I find much more helpful than any solution provided in this thread. While you do not get a complete solution in this page, it is enough of a hint to finish the problem. \n\nFurthermore, I think this type of hint is nicer than a full solution. This way the solver can still get a sense of accomplishment from solving the problem. I find that a fundamental part of my enjoyment of problem solving is when I come up with the solution on my own. By offering hints, I let the solver get started on the problem. The full solution should be left to those who have to struggle to solve the problem.\n\nIf I may predict your response, I think that you will think that this is a lot of ---- and I am just being lazy. I am not sure what position you think you are in so you can interpret my actions. I am in a better position than you are to I know what I am thinking.[/hide]\n\n[quote=\"JBL\"][quote]But obviously surface area/volume of a cone fall within most first year geometry classes.[/quote] But the volume is usually asserted, not proved.[/quote]\r\n\r\nReally? I am suprised about that. In freshman year geometry, [i]before I was interested in math[/i] [i.e. so I took not particular action to figure this out], I was aware of this proof:\r\ni) V=1/3 b*h where the base is a triangle; this comes from decomposing a parallelpiped [or w/e you call it]\r\nii) V=1/3 b*h where the base is a polygon; decompose the polygon into triangles and apply i)\r\niii) V=1/3 b*h where the base is a circle; a circle is a polygon with an infinite number of sides; apply ii)\r\n\r\nPerhaps I had a better geometry class than I thought; I thought that this was the standard \"non-calculus proof.\"\r\n\r\nI suppose that is not 100% rigirous since we are dealing with limits. But that is being pretty picky.",
"Solution_11": "[hide=\"@Altheman\"]You think saying, \"use mass points\" in the HSB forum is educational -- I think it is pedagogically unsound. Simply telling people to google things will lose most of your audience immediately -- they won't bother. Of those who do google it, they won't know what to google for, and a substantial portion of them will fail to find an appropriate introductory source. Far, far better (in my opinion) is to explain how to do something, and (if it's not obvious) show why it works. (Cavalieri's Principle is something where the \"why\" is really calculus, but the result itself is very intuitively clear.) In general, the general result is something best understood [i]after[/i] particular examples have been dealth with, and knowing the name (or the formula, or whatever) but not understanding it is worthless. Teachers should do more than simply hand their students a textbook for each question, and this is all the more true when the textbook in question is the entire internet.\n\nYou're probably right that I've misinterpreted your intent in the past, so I retroactively apologize. (That does not apply to my complaints about LaTeX non-usage, though :wink: )[/hide]\r\n\r\n[quote=\"JBL\"]i) V=1/3 b*h where the base is a triangle; this comes from decomposing a parallelpiped [[i]maybe prism? -- JBL[/i]]\nii) V=1/3 b*h where the base is a polygon; decompose the polygon into triangles and apply i)\niii) V=1/3 b*h where the base is a circle; a circle is a polygon with an infinite number of sides; apply ii)\n\nPerhaps I had a better geometry class than I thought; I thought that this was the standard \"non-calculus proof.\"\nI suppose that is not 100% rigirous since we are dealing with limits. But that is being pretty picky.[/quote] It could just be that my memory of geometry class is very rusty :). I'm not quite sure what you mean by (i), but it sounds plausible. I don't like (iii) very much, because I think Cavalieri's is a nicer way to go from a single pointy thing to all pointy things (in other words, there's no particular reason to go via (ii) -- one can spit out (ii) and (iii) together without ever using the words \"infinitely many\"). A lot has to do with how well something is explained, I guess.",
"Solution_12": "Well...I learn by reading the material first, then asking questions. And the lack of LaTex can be attributed to my procrastinating from school work...but w/e my posts have generally improved now that school has ended.\r\n\r\nAnyway, Here is euclid's proof for $\\frac{1}{3}bh$: http://aleph0.clarku.edu/~djoyce/java/elements/bookXII/propXII7.html\r\n\r\nI think that it is reasonable to consider a circle as a limiting form of a polygon with a large number of sides. If you are particularly picky, you can consider the inscribed and circumscribed regular polygons with n sides. Then you can apply the method of exhaustion [originally an idea from archimedes]. To show that the circle is inbetween these two volumes. This is quite rigirous actually. In the development of integral theory by Tom Apostol in \"Calculus,\" this is one of the 4 fundamental axioms that he takes for area. The analogue to volume is just as acceptable; if one object contains the other, then they have equal volume. Regardless, these are ideas that my dad would understand [and he does psychology]. I think they are sufficiently rigirous for say, the ancient greek geometers."
}
{
"Tag": [
"MATHCOUNTS",
"geometry",
"AMC",
"AIME",
"AMC 12",
"AMC 10"
],
"Problem": "Are they helpful? I am deciding on getting them and I want your thoughts on the books. What are their levels and what contests do they prepare you for? Are they worth the money? Thanks.",
"Solution_1": "They definitely are very helpful, at least in my opinion and what seems to be the majority of AoPSers. I haven't really started Volume 2 yet, but Volume 1 is extremely helpful for MathCounts and the AMCs, particularly the geometry section.",
"Solution_2": "They are very very very helpful! Vol 1 covers Mathcounts/AMC stuff, Vol 2 is more AIME/AMC/ARML stuff.\r\n\r\nVol 1 is recommended for middle school , Vol 2 is recommended for high school",
"Solution_3": "Just make sure that if you get them, you do a lot of the problems and actually learn math rather than being a passive reader.",
"Solution_4": "[quote=\"isabella2296\"]They definitely are very helpful, at least in my opinion and what seems to be the majority of AoPSers. I haven't really started Volume 2 yet, but Volume 1 is extremely helpful for MathCounts and the AMCs, particularly the geometry section.[/quote] did you make aime isabella",
"Solution_5": "If I was aiming for AIME via AMC12 (failed this year by a few mistakes, motivated to try AoPS now) should I just skip the first volume?",
"Solution_6": "Thanks to all. I will buy now.",
"Solution_7": "[quote=\"Ktk\"]If I was aiming for AIME via AMC12 (failed this year by a few mistakes, motivated to try AoPS now) should I just skip the first volume?[/quote]\r\nAoPS Vol.1 doesn't go into details about the stuff that's on the AMC 12 and not AMC 10, but it covers basics for the AMCs and just knowing all of that stuff really will help you more than knowing more advanced stuff from Vol. 2. So you should get Vol. 1."
}
{
"Tag": [
"\\/closed"
],
"Problem": "Recently, I've found that when I try to use the \"hide\" command, instead of applying the hide format, it will instead display the code. Was this an intentional modification, or a bug?",
"Solution_1": "Did you remember to close with (/hide)? (replace () with [])\r\n\r\n[hide=\"Hiding\"]...[/hide] works fine for me.",
"Solution_2": "Oops...\r\n\r\nAt least one time it was because I used the wrong slash. :oops: \r\n\r\n[hide=\"This is a test.\"] :maybe: [/hide]\r\n\r\nYep, that must have been it, because the above hide works just fine."
}
{
"Tag": [
"function",
"algebra proposed",
"algebra"
],
"Problem": "Prove that the elements of the set $ A\\equal{}\\{ (x,y)|x \\in \\mathbb{Z},y \\in \\mathbb{Z}, 2xy\\minus{}6x\\plus{}y\\equal{}0 \\}$ are on the graphic of the function $ f: \\mathbb{R} \\rightarrow \\mathbb{R}, f(2x\\minus{}3)\\equal{}8x\\minus{}7.$",
"Solution_1": "[quote=\"moldovan\"]Prove that the elements of the set $ A \\equal{} \\{ (x,y)|x \\in \\mathbb{Z},y \\in \\mathbb{Z}, 2xy \\minus{} 6x \\plus{} y \\equal{} 0 \\}$ are on the graphic of the function $ f: \\mathbb{R} \\rightarrow \\mathbb{R}, f(2x \\minus{} 3) \\equal{} 8x \\minus{} 7.$[/quote]\r\n\r\nUnfortunately, this is wrong : $ (\\minus{}1,6)\\in A$ but is not on the required graphic since $ f(\\minus{}1)\\equal{}1\\neq 6$ and $ f(6)\\equal{}29\\neq \\minus{}1$)"
}
{
"Tag": [
"AMC",
"AIME",
"USAMTS",
"USA(J)MO",
"USAMO"
],
"Problem": "Since this year seems to be easier than usual, what effect will that have on the cutoff? how far up will the cutoff move? Also, how is the cutoff determined? Is it the first score for which >x people got that score or above for some x?",
"Solution_1": "[quote=\"solafidefarms\"]Since this year seems to be easier than usual, what effect will that have on the cutoff? how far up will the cutoff move? Also, how is the cutoff determined? Is it the first score for which >x people got that score or above for some x?[/quote]\r\nI think that there are an x number of people who qualify for the AIME. Anyways, if the third round is uber hard than the qualifying score might be as low as 60-63.",
"Solution_2": "You start off with an index of 100 if you qualify through USAMTS, right?\r\n\r\nalso, people that qualify for the AIME through the USAMTS can qualify for the USAMO by both their index and the floor score, right?",
"Solution_3": "I think the index only matters in USAMO qualification is you are in 11th or 12th grade. Otherwise you go by floor which means if you score at least the score the lowest AIME score that the people that qualified by the index, you are in. So I don't think the USAMO index really does matter for us...\r\n\r\nBut I am interested in how qualifications to AIME is decided. It would feel good if I knew that even if I bombed the AMCs, I would still be in the AIME."
}
{
"Tag": [
"linear algebra",
"matrix",
"search",
"linear algebra unsolved"
],
"Problem": "DO any of u know what a \"[b][color=black][size=150]DIRAC MATRIX[/size][/color][/b]\" is?",
"Solution_1": "I have two words for you: [b]GOOGLE SEARCH[/b].",
"Solution_2": "THanks for that!!!!!\r\n\r\n\r\n@carcul- ia almost forgot that"
}
{
"Tag": [
"combinatorics unsolved",
"combinatorics"
],
"Problem": "There are n markers, each with one side white and the other side black,\r\naligned in a row so that their white sides are up. In each step, if possible, we\r\nchoose a marker with the white side up (but not one of the outermost markers),\r\nremove it and reverse the closest marker to the left and the closest marker to the\r\nright of it. Prove that one can achieve the state with only two markers remaining\r\nif and only if n - 1 is not divisible by 3.",
"Solution_1": "merged with http://www.mathlinks.ro/viewtopic.php?t=90046 .\r\n\r\n darij"
}
{
"Tag": [
"number theory proposed",
"number theory"
],
"Problem": "find all pairs of natural numbers $ (n,m)$ such that $ n!\\plus{}1\\equal{}m^2$\r\n\r\n\r\n$ (4,5),(5,11),(7,71)$ is this all ?",
"Solution_1": "This a very well known problem ( I think it is Brocard's problem) but unfortunately it hasn't been solved yet. I t seem that those are the only solutions but it is not known.\r\nIf someone can solve it I don't think he will post it here without publishing it :P :rotfl: \r\nBut I recommend you to try it, it's very nice\r\n\r\n\r\nDaniel"
}
{
"Tag": [
"calculus",
"derivative",
"real analysis",
"real analysis unsolved"
],
"Problem": "Show that $x^{n}+nx-1 =0 \\quad n\\geq 1$, has exactly one real root $a_{n}$. Does the series\r\n\\[\\sum_{n}a_{n}\\]\r\nconverge ?\r\n\r\n(I'm not sure about the problem. Think it should be \"one real [i]positive[/i] root\")",
"Solution_1": "It has at least one positive root by the Intermediate Value Theorem; its value at zero is negative, and it goes to $\\infty$ at $\\infty$. If $n$ is even, it has a negative root as well, so it should specify \"positive real root\".\r\nFor uniqueness, show that the derivative is positive for positive $x$; Rolle's theorem finishes the proof.\r\n\r\nThe series diverges: $\\frac1n \r\n\r\n[code]\n\\displaystyle{ {(\\sum_{i=1}^m (\\sum_{j=1}^n ({A_{i,j}}^s)^{r/s})}^s\n \\leq {(\\sum_{j=1}^n (\\sum_{i=1}^m ({A_{i,j}}^r)^{s/r})}^r }[/code]\r\nWhat would be the condition for equality?",
"Solution_1": "and i thought my latex skills were bad... :P",
"Solution_2": "Theorem \r\n\r\n0 M_s^n ( M_r^m A_{i,j} ) \\leq M_r^m (M_s^n A_{i,j} )\r\n\r\nwhere M_r^m {x_k} = (\\sum_{i=1}^m (x_i)^r)^{1/r) \r\n\r\nIn particulr when s= -\\infinity , r= +\\infinity we have the Minimax Inequality \r\n\r\nThe proof is already done by myself! :lol:",
"Solution_3": "Plus: Thanks for someone remind the inequality follows from Minkowski Inequality :roll:",
"Solution_4": "Note that the inequality can also be written into the form\r\n\r\n${01$. Prove that \\[\\frac{1}{2}+\\cdots+\\frac{1}{n}\\] is not an integer.",
"Solution_1": "[quote=\"nicetry007\"]Let $ 2^{k}\\;\\leq \\; n \\;<\\; 2^{k+1}$. \n\nIt is to be noted that every number $ m \\;\\neq\\; 2^{k}$ and $ m \\;\\leq\\;n$ has a power of $ 2$ strictly smaller than $ k$. \n\nSuppose not. Let $ m \\; = \\; 2^{l}\\cdot s$ where $ l \\;\\geq \\;k$ and $ s$ is odd and $ s \\;\\geq\\; 1$.\n\nWe have $ m \\; = \\; 2^{l}\\cdot s \\;\\leq \\;n\\;< \\;2^{k+1}\\;\\Rightarrow s \\;< \\;2^{k+1-l}\\;\\leq \\; 2\\;$ ( as $ l \\;\\geq\\; k$) $ \\Rightarrow s = 1$ (as $ s$ is odd ) $ \\Rightarrow m \\; = \\; 2^{l}\\;<\\; 2^{k+1}\\;\\Rightarrow \\; l \\;\\leq \\; k$.\n\nBut $ l\\;\\geq \\;k$. Hence, $ l \\; = \\; k \\;\\Rightarrow \\; m \\; = \\; 2^{k}$, which is a contradiction as $ m \\;\\neq\\; 2^{k}$.\n\nThus, we can conclude that every $ m \\; \\neq \\; 2^{k}$ and $ m \\;\\leq \\; n$ has a power of $ 2$ strictly smaller than $ k$.\n\nThe denominator of the sum is the $ lcm(2\\;,\\;3\\;,\\;\\cdots\\;,\\;n)$ which has $ 2^{k}$ as a factor. Thus, when we take the sum of all the unit fractions, the numerator of every fraction other than $ \\frac{1}{2^{k}}$ \ngets multiplied by a power of $ 2$ and the numerator of $ \\frac{1}{2^{k}}$ gets multiplied by an odd number. Hence, the numerator of the sum of all the fractions is an odd number as $ odd+even \\; = \\; odd$.\n\nTherefore, the sum can never be an integer as the numerator is odd and the denominator is even.[/quote]",
"Solution_2": "Sorry to revive this but I remember there was a proof using Bertrand's postulate and \nfactorials , but I can't remember the whole thing .Can anyone post it?",
"Solution_3": "[quote=\"aham\"]Sorry to revive this but I remember there was a proof using Bertrand's postulate and \nfactorials , but I can't remember the whole thing .Can anyone post it?[/quote]\n\nConsider \n\n$\\frac{1}{2}+\\frac{1}{3}+\\cdots +\\frac{1}{n}=\\frac{\\frac{n!}{1}+\\frac{n!}{2}+\\cdots+\\frac{n!}{n}}{n!}$\n\nBy Bertrand's postulate, there exists a prime $p$ such that $\\lfloor{\\frac{n}{2}}\\rfloor 0,00$ and $x+y+z=1$ prove that\\[x^2+y^2+z^2+3xyz\\ge\\frac{4}{9}\\].[/quote]\nLet $x\\ge y\\ge z$ , so $9\\sum x^2+27xyz-4=9 \\sum x \\sum x^2+27xyz-4(\\sum x)^3$\n\n$=\\frac{9}{4}(2x+2y-z)(x-y)^2+\\frac{1}{4}(2x+2y+5z)(x+y-2z)^2\\ge 0.$"
}
{
"Tag": [
"function",
"summer program",
"MathPath"
],
"Problem": "I recently read about proofs involving Ramsey numbers, and I understand a basic definition of Ramsey's Theorem, but I am not sure how to apply it. Could any of you please enlighten me as to where Ramsey Numbers become useful?\r\n\r\nPlease post some challenging problems, and tell me (as well as the entire AoPS community) what Ramsey Numbers exactly are. Post some interesting facts if you'd like :D",
"Solution_1": "Suppose you have a regular n-gon with all the diagonals drawn in. You want to color each diagonal with one of k colors so that you don't have an a1-gon with all diagonals in color 1, a regular a2-gon with all diagonals in color 2, etc. What is the largest possible value of n?\r\n\r\nThat is the standard Ramsey Theory problem. You can make variations on it (for example, what if you only want the sides and not the diagonals to be not monochromatic).\r\n\r\nRamsey Theory problems tend to be incredibly hard. Here's a seemingly very simple problem that is actually unsolved (at least, as of a few years ago, and I doubt anyone has solved it since):\r\n\r\nWhat is the smallest value of n so that if you color all the sides and diagonals of a regular n-gon red or blue, you are guaranteed to have a regular pentagon with all diagonals in either red or blue?",
"Solution_2": "In space, there are given p_n=[en!]+1 points, where [x] is the least integer function\r\nEach pair of points is connected by a line, and each line is colored with one of n colors. Prove that there is at least one triangle with sides of the same color",
"Solution_3": "Lol. Ramsey numbers...brings back some pretty weird memories.\r\n\r\nI remember at MathPath 2003 we were introduced to Ramsey numbers. One of my friends and I spent about a week trying to come up with some formula to express them (the dichromatic Ramsey numbers, anyway.) I remember we came up with some pretty fair approximations."
}
{
"Tag": [
"calculus"
],
"Problem": "So I skimmed through Mathematical Olympiad Treasures on Amazon and my jaw dropped. Could I get a recommendation on a textbook for harder Algebra problems such as the ones I saw in MOT book.",
"Solution_1": "Mathematical Olympiad Treasures is a problem-solving book. The subjects it handles are not meant to be approached in a \"textbook\" manner: you won't find any large theorems from which calculations can be done mechanically, such as in, say, differential calculus. \r\n\r\nThat having been said, [u]Problem-Solving Strategies[/u] by Arthur Engel is a good book that approaches Olympiad subjects in a somewhat more cohesive manner.",
"Solution_2": "[quote=\"t0rajir0u\"]Mathematical Olympiad Treasures is a problem-solving book. The subjects it handles are not meant to be approached in a \"textbook\" manner: you won't find any large theorems from which calculations can be done mechanically, such as in, say, differential calculus. \n\nThat having been said, [u]Problem-Solving Strategies[/u] by Arthur Engel is a good book that approaches Olympiad subjects in a somewhat more cohesive manner.[/quote]Aw, that's sad to know. I am currently in Calculus 2 but I find myself encountering Algebra problems (student tutor) that I've never seen before and usually I can solve them but some definitely give me a hard time and some I can't even solve. I just love problem-solving, thanks!",
"Solution_3": "Apparently, you already know some problem-solving, so I would recommend you Problem-Solving Strategies too... after all the book is written by a German, so it has to be great :P\r\n\r\n[/snobism]"
}
{
"Tag": [
"algebra unsolved",
"algebra"
],
"Problem": "Prove that if a sequence $ \\{G(n) \\}_{n\\equal{}0}^{\\infty}$ of integers satisfies: $ G(0)\\equal{}0, G(n)\\equal{}n\\minus{}G(G(n))\\quad (n\\equal{}1,2,3,...)$ then:\r\n$ (a)$ $ G(k) \\ge G(k\\minus{}1)$ for any positive integer $ k$\r\n$ (b)$ no integer $ k$ exists such that $ G(k\\minus{}1)\\equal{}G(k)\\equal{}G(k\\plus{}1)$.",
"Solution_1": "is easy to pove for all $ n > 0 G(n) < > 0$ because if not $ G(n) \\equal{} 0 \\equal{} n \\minus{} G(G(n)) \\equal{} n \\minus{} G(0) \\equal{} n$ so $ n \\equal{} 0$ absurde\r\n\r\nand for all $ n > 0 n G(n) \\equal{} n \\minus{} G(G(n)) \\le n \\minus{} 1$\r\n\r\n1) by recurence (fort) ( tha for all $ n$ $ G(n)\\geq G(n \\minus{} 1) \\geq ....\\geq G(0)$\r\n\r\nfor $ n \\equal{} 1$ $ G(1)\\in N$ so $ G(1) \\geq 0 \\equal{} G(0)$\r\n\r\nsuppose that is true for $ n$ and prove that is true for $ n \\plus{} 1$\r\n\r\n$ G(n \\plus{} 1) \\equal{} n \\plus{} 1 \\minus{} GG(n \\plus{} 1)$ you have $ G(n \\plus{} 1) \\le n$ so $ G(n) \\geq G(G(n \\plus{} 1))$ so \r\n\r\n$ G(n \\plus{} 1) \\equal{} n \\plus{} 1 \\minus{} GG(n \\plus{} 1) \\geq n \\plus{} 1 \\minus{} G(n) \\equal{} G(n) \\plus{} 1 > G(n)$\r\n\r\n2) if $ G(n) \\equal{} G(n \\minus{} 1)$ ===> $ n \\minus{} G(G(n)) \\equal{} n \\minus{} 1 \\minus{} G(G(n \\minus{} 1)) \\equal{} n \\minus{} 1 \\minus{} G(G(n))$ ===>$ 1 \\equal{} 0$ absurde so \r\n\r\n$ G(n)>G(n\\minus{}1)$",
"Solution_2": "[quote=\"mohamed-01-01\"] ... $ G(n \\plus{} 1) \\equal{} n \\plus{} 1 \\minus{} GG(n \\plus{} 1) \\geq n \\plus{} 1 \\minus{} G(n) \\equal{} G(n) \\plus{} 1 > G(n)$ ...[/quote]\r\nHello mohamed-01-01!\r\n\r\nWhy is \"$ n \\plus{} 1 \\minus{} G(n) \\equal{} G(n) \\plus{} 1$\" true ?",
"Solution_3": "[quote=\"pco\"][quote=\"mohamed-01-01\"] ... $ G(n \\plus{} 1) \\equal{} n \\plus{} 1 \\minus{} GG(n \\plus{} 1) \\geq n \\plus{} 1 \\minus{} G(n) \\equal{} G(n) \\plus{} 1 > G(n)$ ...[/quote]\nHello mohamed-01-01!\n\nWhy is \"$ n \\plus{} 1 \\minus{} G(n) \\equal{} G(n) \\plus{} 1$\" true ?[/quote]\r\n\r\noh sorry my solution is wrong but I think there is a mistake in this probleme\r\n\r\nbecause if $ G(n) \\geq G(n \\minus{} 1)$ \r\n\r\nand you have for all $ n$ $ G(n) < > G(n \\minus{} 1)$ because if $ G(n) \\equal{} G(n \\plus{} 1)$ ==> \r\n\r\n$ n \\plus{} 1 \\minus{} G(G(n \\plus{} 1)) \\equal{} n \\minus{} G(G(n)) \\equal{} n \\minus{} G(G(n \\plus{} 1))$ so $ 1 \\equal{} 0$\r\n\r\nso $ G(n \\plus{} 1) > G(n)$ ===>$ G(n) \\geq n$\r\n\r\nso $ G(n) \\equal{} n \\minus{} G(G(n)) \\le n \\minus{} G(n) \\le n \\minus{} n \\equal{} 0$ ???",
"Solution_4": "[quote=\"mohamed-01-01\"] oh sorry my solution is wrong but I think there is a mistake in this probleme\n\nbecause if $ G(n) \\geq G(n \\minus{} 1)$ \n\nand you have for all $ n$ $ G(n) < > G(n \\minus{} 1)$ because if $ G(n) \\equal{} G(n \\plus{} 1)$ ==> \n\n$ n \\plus{} 1 \\minus{} G(G(n \\plus{} 1)) \\equal{} n \\minus{} G(G(n)) \\equal{} n \\minus{} G(G(n \\plus{} 1))$ so $ 1 \\equal{} 0$\n\nso $ G(n \\plus{} 1) > G(n)$ ===>$ G(n) \\geq n$\n\nso $ G(n) \\equal{} n \\minus{} G(G(n)) \\le n \\minus{} G(n) \\le n \\minus{} n \\equal{} 0$ ???[/quote]\r\n\r\nThere is a little mistake in your demo but your conclusion seems correct to me : $ G(n)\\ne G(n\\plus{}1)$ only for $ n>0$, so maybe $ G(1)\\equal{}G(0)$ So, if the required non decreasing assertion is true, we have $ G(n)\\geq n\\minus{}1$ (and not $ G(n)\\geq n$). So $ G(G(n))\\geq n\\minus{}2$ and $ n\\equal{}n\\minus{}G(G(n))\\leq 2$ !\r\nAnd so it seems indeed that there is a mistake in this problem ... :huuh:"
}
{
"Tag": [
"geometry",
"incenter",
"circumcircle",
"conics",
"ellipse",
"perpendicular bisector",
"geometry unsolved"
],
"Problem": "Let AD be the altitude of a triangle ABC and E , F be the incenters of the triangle ABD and ACD , respectively. line EF meets AB and AC at K and L. prove tht AK=AL if and only if AB=AC or A=90",
"Solution_1": "Let I be the incenter of the triangle $\\triangle ABC$. We have to show that the bisector AI of the angle $\\angle A$ is perpendicular to EF iff either AB = AC or the angle $\\angle CAB = 90^\\circ$ is right. The bisectors $AI, BI \\equiv EI, CI \\equiv FI$ are concurent cevians of the triangle $\\triangle AEF$. Let $X \\in AF, Y \\in AE$ be the feet of EI, FI in this triangle.\r\n\r\n$\\angle EXA \\equiv \\angle BXA = 180^\\circ - \\frac{\\angle B}{2} - (90^\\circ - \\angle B) - \\frac{90^\\circ - \\angle C}{2} =$\r\n\r\n$= \\frac{90^\\circ + \\angle B + \\angle C}{2} = 90^\\circ + \\frac{90^\\circ - \\angle A}{2}$\r\n\r\n$\\angle FYA \\equiv \\angle CYA = 180^\\circ - \\frac{\\angle C}{2} - (90^\\circ - \\angle C) - \\frac{90^\\circ - \\angle B}{2} =$\r\n\r\n$= \\frac{90^\\circ + \\angle C + \\angle B}{2} = 90^\\circ + \\frac{90^\\circ - \\angle A}{2}$\r\n\r\nHence, $\\angle EXA = \\angle FYA$. If the angle $\\angle A = 90^\\circ$ is right, EX, FY are 2 altitudes of the triangle $\\triangle AEF$, I its orthocenter and $AI \\perp EF$ its remaining altitude. Assume now that the angle $\\angle A \\neq 90^\\circ$ is not right and that we still have $AI \\perp EF$, i.e., AI is the A-altitude of the triangle $\\triangle AEF$, but I is no longer its orthocenter. Since $\\angle EXA = \\angle FYA \\neq 90^\\circ$, then also $\\angle EXF = \\angle FYE \\neq 90^\\circ$, i.e., the quadrilateral EFXY is cyclic. Let (P) be its circumcircle, which is centered on the perpendicular bisector of the segment EF. Using parallel projection, we can project the triangle $\\triangle AEF$ into a triangle $\\triangle A'EF$, so that the point I is projected into its orthocenter I'. Then the cevians EX, FY are projected into its altitudes EX', FY', i.e., the quadrilateral EFX'Y' is also cyclic, with the circumcircle (O') centered at the midpoint O' of the segment EF. Thus the points X, Y lie both on the circle (P) and on an ellipse $o$ with the main axis EF (which is projected into the circle (O') in our parallel projection), not identical with the circle (P). Since both the ellipse $o$ and the circle (P) are symmetrical with respect to the perpendicular bisector of the segment EF, so are their intersections X, Y, i.e. EX = FY and EY = FX. Hence, the triangles $\\triangle EFX \\cong \\triangle FEY$ are congruent and it immediately follows that the triangle $\\triangle AEF$ is isosceles with AE = AF. This and $AD \\perp BC$ implies that the incircles $(E) \\cong (F)$ are congruent, which means that the triangle $\\triangle ABC$ itself is isosceles with AB = AC."
}
{
"Tag": [
"trigonometry"
],
"Problem": "How did you guys do?",
"Solution_1": "Just a reminder that the problems should be not posted or discussed before they are officially posted on the website, since other cities may write the exam at a later date;",
"Solution_2": "Out of curiousity, do you all take Senior A?",
"Solution_3": "yes, I took senior A.",
"Solution_4": "Any idea why they're written at so many times of the year? I mean, Calgary doesn't write them for another month or so.",
"Solution_5": "What is this? Have a link? Thanks.\r\n\r\n-The Living in Closet all life person",
"Solution_6": "[quote=\"billzhao\"]Just a reminder that the problems should be not posted or discussed before they are officially posted on the website, since other cities may write the exam at a later date;[/quote]\r\n\r\nI have no idea when Edmonton is writing this. :? Does anyone know? :D",
"Solution_7": "Oh, William, you've never heard of Tournament of Town? I guess they don't hold it at Waterloo. Here is link for T of T in Toronto, [url]http://www.math.toronto.edu/oz/turgor/news.php[/url]. It's basically a 4/5 hour contest primarily organized by some Russian math commitee and hold in a few cities in Europe and North America. I think the contest is rather an informal one, but in a sense, make Olypiad level problems more approachable to beginner like me. The contest is fairly nice. O-Level is not that challenging, A-Level is a bit hard for me...\r\n\r\nBTW, Alex, I did horrible today. I solved only two problems and BSed for couple others. I wrote 3 pages of trigs for #5, and couldn't prove it in the end.... Was you there today? I didn't even see you.",
"Solution_8": "Is this the same thing as mathbattle?\r\n\r\nEither way, I'm looking for activities for our math club to join as a team. Maybe this, mathbattle, and MCTIC (CARML) would be good candidates?\r\n\r\nBTW Alan good to hear from you again! Haven't seen you in so long :)",
"Solution_9": "yes i was there; an hour into the end i left because i knew i couldnt break a max-O-level score which I am sure that i could get later anyway.\r\n\r\nbtw, solving 2 problems and bsing #5 is very good. Isn't this close to a perfect score? (assuming \"solving 2 problems\" doesn't include #1)\r\n\r\nI had very strong ideas on how to solve all, but couldn't bring them out.",
"Solution_10": "[quote=\"TheDreamer\"]Is this the same thing as mathbattle?\n\nEither way, I'm looking for activities for our math club to join as a team. Maybe this, mathbattle, and MCTIC (CARML) would be good candidates?\n\nBTW Alan good to hear from you again! Haven't seen you in so long :)[/quote]\r\n\r\nI'm not sure what exactly mathbattle is, but I'm quite sure that it's different.\r\n This competition gives you 5~7 problems(I don't remember the exact number) and 4~5 hours(I don't remember this either) to solve them. Each problem is weighted differently and they only take the best three(right?) problems that you solved. There are two rounds in a year, fall and spring round. There are also two levels, O level for grade 10 and below and A level for grade 11 and above(but they have some overlapping problems). This is an individual competition. There are no prizes(except for a certificate for doing ok), but the problems are [i][b]very[/b][/i] beautiful.",
"Solution_11": "[quote=\"lightrhee\"]There are also two levels, O level for grade 10 and below and A level for grade 11 and above(but they have some overlapping problems).[/quote]\r\n\r\nYou got it confused there. There are 2 levels, Junior for grade 10 and under, and Senior for grade 11 and over. There are two rounds, O-Level written first, the easier of the two, and A-Level which is harder.",
"Solution_12": "Yea, this is different from mathbattle. This is more or less in a format similar to \r\nCMO(although much longer). T of T is individual contest, whereas mathbattle is a team competition. MathBattle is a lot more fun though. It's like you have a set of problems and your team will work on them for like 2.5 hrs. And you and your opposing team take turn in showing solution to problems by the other team. You will have rebuttals, judges too. It's strategic sometimes too, not always depend on how many prolbems you solved. There are more info. on the website I provided in the above post. Oh, we get free pizza and pop too. So free lunch, if that is an incentive for anyone to come.\r\n\r\nBTW, William you are going to have a team from your school come to Toronto to join the MathBattle? Well, Waterloo is not that far from Toronto after all. I would be glad to see you there. I knew couple others people from your school also.",
"Solution_13": "Hmm that would be really cool. I'll see how much interest there is in our math club. We wouldn't really stand a chance though :s, so it may not be such a good idea, making ppl lose confidence lol",
"Solution_14": "How large is your Math Club? We only have 10 members at most.",
"Solution_15": "We had 25 the first time, with another 25 who said they'll come next time. Although prolly some of them will just keep procrastinating and won't come at all, so i'm guessing in the end we'll have around 30, which is about right i think"
}
{
"Tag": [
"geometry",
"perimeter",
"trigonometry",
"function",
"complex analysis",
"parameterization",
"parametric equation"
],
"Problem": "A mark is placed on the perimeter of a wheel where it touches the road. The wheel is rolled along the road without skidding though less than half a rotation. The mark has advanced horizontally from its starting position by x. Given x and the radius of the wheel (r) what is the height (y) of the mark above the road?\r\n\r\n\r\nIt seems a simple problem but as far as I can gather there is no solution to this question that does not involve numerical aproximation methods.",
"Solution_1": "Imagine that the wheel was squeaky clean at the beginning, and now a part of it (i.e. a circular arc) is covered with road dirt. The length of this arc is $ x$, therefore ... \r\n\r\nAlso, this is not complex analysis, and not even calculus.",
"Solution_2": "[quote=\"mlok\"]Imagine that the wheel was squeaky clean at the beginning, and now a part of it (i.e. a circular arc) is covered with road dirt. The length of this arc is $ x$, therefore ... \n\nAlso, this is not complex analysis, and not even calculus.[/quote]\r\n\r\nYou have mis-interpreted the question.\r\n\r\nThe dusty arc represents the distance the wheel the has rolled along the road and this distance (call it z) is given by $ R\\theta$ wher $ \\theta$ is the angle the wheel has turned through. The x in the question is the distance the mark has traveled along the x axis and this is given by $ x\\equal{}r(\\theta\\minus{}\\sin(\\theta))$ or $ x\\equal{}z\\minus{}\\sin(\\theta)$ and is not the same as the distance the wheel has rolled. (The mark traces out a cycloid). The parametric equation for a cycloid defines y as $ R(1\\minus{}\\cos(\\theta))$ but I am looking for y in terms of x when $ \\theta$ is not given. \r\n\r\nIf you think that is an easy question I'll give $ \\$$50 to a nominated charity or to whoever answers the question (if that is allowed) if you provide an equation for y in terms of x where the angle the the wheel has turned through $ (\\theta)$ is not given.\r\n\r\nRules:\r\n\r\nThe answer must be given in the form of a cartesian equation with y and only y on the left hand side and where $ \\theta$ or y do not appear on the right hand side of cartesian equation. \r\n\r\nBy the way, if you know y then there is a cartesian equation for x that does not include $ \\theta$:\r\n\r\n$ x\\equal{}\\pm\\arccos\\left({R\\minus{}y \\over R}\\right)R\\pm\\sqrt{2Ry\\minus{}y^2}$\r\n\r\nAll you have to do is solve that equation for y ;)\r\n\r\nWould have been nice if you had actually read the question properly before moving it.",
"Solution_3": "Don't waste your time looking. The inverse function $ y(x)$ is not an elementary function. There is no reason to expect transcendental equations to be solvable in elementary terms.\r\n\r\nIn practice, you can do anything you want with the parametric form. Numerical methods also work well here."
}
{
"Tag": [
"real analysis",
"modular arithmetic",
"function",
"real analysis unsolved"
],
"Problem": "A cute result :):\r\n\r\nLet $T$ be a measure preserving invertible transformation on the unit interval $I$ (with the usual Lebesgue measure), such that for almost all $x\\in I,\\ T^{n}x=x$, and $x,\\ Tx,\\ \\ldots,T^{n-1}x$ are distinct. \r\n\r\nProve that there is a measurable subset $E$ of $I$ with measure $\\frac{1}{n}$ such that $E,\\ T(E),\\ \\ldots,T^{n-1}(E)$ are pairwise disjoint.",
"Solution_1": "While we're all trying to prove this, let me state, trivially, that the topic is not vacuous. It is possible to construct such a $T.$\r\n\r\nThe simplest such example I can think of is to let $T(x)\\equiv x+\\frac1n\\pmod{1}.$\r\n\r\n(And in that case, $E=\\left[0,\\frac1n\\right)$ can be the set we want.)",
"Solution_2": "Here's an attempt. Let $J$ be the set of points $x$ that have period $n$. By the hypothesis, $J$ has measure 1.\r\n\r\nSay that a point $x$ is [i]early[/i] if $x \\le T^{k}x$ for all integers $k$. Let $E$ be the set of early points in $J$. Then $E$ is measurable because, um, well, I don't really know why. (How do I even show that the set of $x$ such that $x \\le Tx$ is a measurable set?) But I will assume that $E$ is measurable.\r\n\r\nBy the given property, it is easy to see that $E$, $TE$, $\\ldots$, $T^{n-1}E$ form a partition of $J$. All these $n$ sets have the same measure because $T$ is measure-preserving. Thus $E$ has measure $\\frac{1}{n}$.",
"Solution_3": "Neat :). My proof was a bit more involved. Anyway, regarding your question:\r\n\r\nOne possible definition for the product $\\sigma$-algebra of Lebesgue measurable subsets of $I\\times I$ is: the smallest $\\sigma$-algebra that will make both projections $\\pi_{i},\\ i=1,2$ from $I\\times I$ onto $I$ (i.e. the projections on the two factors) measurable. This implies that a map $S: I\\to I\\times I$ is measurable iff $\\pi_{i}\\circ S$ are both measurable self-maps of $I$. \r\n\r\nNow let $Sx=(x,Tx)$. By the above characterization of measurable functions from $I$ to $I\\times I,\\ S$ is measurable. Your set $\\{x\\ |\\ x\\le Tx\\}$ is the preimage of the (measurable, of course) triangle $\\{(x,y)\\ |\\ x\\le y\\}$ through $S$, and is thus measurable. And when you do this for every $T^{k}$ and take intersections as $k$ goes from $1$ to $n-1$, you find the set of early points to be measurable, as desired.",
"Solution_4": "Thanks for the help on measurability."
}
{
"Tag": [
"modular arithmetic",
"polynomial",
"number theory unsolved",
"number theory"
],
"Problem": "If $ a^2\\plus{}ab\\plus{}b^2$ is a prime$ (a,b \\in Z)$,prove that $ a^2\\plus{}ab\\plus{}b^2\\equiv 1 \\pmod 3$",
"Solution_1": "Try $ p \\equal{} 3$.\r\n\r\n$ p \\equal{} a^2 \\plus{} ab \\plus{} b^2 \\Leftrightarrow$\r\n$ 4p \\equal{} (2a \\plus{} b)^2 \\plus{} 3b^2 \\implies$\r\n$ (2a \\plus{} b)^2 \\equiv \\minus{} 3b^2 \\bmod p \\implies$\r\n$ \\left( \\frac {2a \\plus{} b}{b} \\right)^2 \\equiv \\minus{} 3 \\bmod p$\r\n\r\nwhich is possible if and only if $ p \\equal{} 3$ or $ p \\equiv 1 \\bmod 3$. (In the other direction: $ p \\equiv 1 \\bmod 3 \\implies \\exists x : p | x^2 \\plus{} 3 \\equal{} (x \\plus{} \\sqrt { \\minus{} 3})(x \\minus{} \\sqrt { \\minus{} 3})$, which implies that $ p$ is not prime in the [url=http://mathworld.wolfram.com/EisensteinInteger.html]Eisenstein integers[/url].)",
"Solution_2": ":arrow: If $ 3|a \\equal{}> a\\equal{}3$.\r\nWe have $ A \\equal{} b^2\\plus{}3b\\plus{}9$\r\nIf $ b \\vdots 3 (b\\equal{}3)$, $ A\\equal{}27$, which is not a prime.\r\nSo b is not divisible by 3, then $ b^2 \\equiv 1 (mod 3) \\equal{}> A \\equiv 1 (mod 3).$\r\n\r\n :arrow: If a and b are not divisible by 3, $ a^2\\plus{}b^2 \\equiv 1\\plus{}1\\equal{}2 (mod 3).$\r\n$ ab \\equiv 1 or 2 (mod 3)$\r\nThen, $ A \\equiv 0 or 1 (mod 3)$.\r\nBecause A > 3, $ A \\equiv 1 (mod 3)$.\r\n\r\nHence, $ A \\equiv 1 (mod 3)$ if A is a prime.",
"Solution_3": "Another problem\r\n\r\nIf $ 3k\\plus{}1$ is a prime , prove that it can be a form like \"$ a^2\\plus{}ab\\plus{}b^2(a,b\\in Z)$\" \r\n\r\n\r\n\r\nl'm sorry.My English is poor,so it's hard for me to translate the problems from Chinese into English. :fool:",
"Solution_4": "Already solved that problem in the last line. See also my comments on the corresponding theorem for $ a^2 \\plus{} b^2$ [url=http://www.artofproblemsolving.com/Forum/weblog_entry.php?t=219017]here[/url].",
"Solution_5": "[quote=\"skywalkerJ.L.\"]If $ a^2 \\plus{} ab \\plus{} b^2$ is a prime$ (a,b \\in Z)$,prove that $ a^2 \\plus{} ab \\plus{} b^2\\equiv 1 \\pmod 3$[/quote]\r\n\r\nif $ a\\equal{}b\\equal{}1$ ? then $ a^2\\plus{}b^2\\plus{}ab\\equal{}3$ !!!!!!!!"
}
{
"Tag": [
"USAMTS",
"search"
],
"Problem": "So, what are some formulas that are usefull?",
"Solution_1": "The reason nobody's responding to your post is that there aren't just a few formulas that are useful. Anyway, formulas are not the important thing. They really don't help much when solving problems, except for the formulas that we all know and use commonly.",
"Solution_2": "Search Mathworld if you want to know something. It's much easier than memorizing obscure formulae."
}
{
"Tag": [
"Gauss",
"linear algebra",
"matrix"
],
"Problem": "There is something about matrices that I have found out... and I attempted to see if my idea has ever been thought of before, and I found nothing.\r\nI need someone to help me find out if this idea was pre-existant and I just came up with something that already existed.\r\n\r\nWhat its about: Matrices\r\n\r\nUses principles presented in: Guassian Elimination, Gauss-Jordan Elimnation",
"Solution_1": "Here it is... Im posting it up\r\n\r\nBasic idea:\r\n\r\nGiven any sets of three matrices where AX=B, one may perform any identical Gaussian elimination operations to both A and B and the equation will remain equal.\r\n\r\nWhat this can be used for:\r\n\r\nInstead of needing to find the inverse of Matrix A, and multiply both sides by A, one can simply manipulate both sides until A is the identity. :P",
"Solution_2": "Yes, this is well known. If you look at it closely you will see that it's just a slight extension of the Gauss-Jordan method for solving systems of equations.\r\n\r\nThe good thing about it is that it doesn't just work for equations of 2x2 or nxn size, but also for matrix equations of any size (something that Cramer's Rule doesn't do). It also ultimately turns out to be the fastest and most efficient way of solving matrix equations for large matrices.\r\n\r\nYou've re-discovered a really great thing here! See if you can use it to try and solve equations of non-square matrices."
}
{
"Tag": [
"inequalities",
"induction",
"rearrangement inequality",
"inequalities proposed"
],
"Problem": "Let $ 0 < x_{1}\\leq\\frac {x_{2}}{2}\\leq\\cdots\\leq\\frac {x_{n}}{n}, 0 < y_{n}\\leq y_{n \\minus{} 1}\\leq\\cdots\\leq y_{1},$ Prove that $ (\\sum_{k \\equal{} 1}^{n}x_{k}y_{k})^2\\leq(\\sum_{k \\equal{} 1}^{n}y_{k})(\\sum_{k \\equal{} 1}^{n}(x_{k}^2 \\minus{} \\frac {1}{4}x_{k}x_{k \\minus{} 1})y_{k}).$ where $ x_{0} \\equal{} 0.$",
"Solution_1": "The inequality to prove rewrites as $ A = (\\sum x_k^2y_k)(\\sum y_k) - (\\sum x_ky_k)^2\\ge B = \\frac14(\\sum y_k)(\\sum y_kx_kx_{k - 1})$. From the identity $ (\\sum a_i^2)(\\sum b_i^2) - (\\sum a_ib_i)^2 = \\displaystyle\\sum_{i < j}(a_ib_j - a_jb_i)^2$, we have $ A = \\sum_{i < j}y_iy_j(x_i - x_j)^2$. We are going to resort now to induction.\r\n\r\nFor $ n = 1,2$ the inequality can be verified easily. For the step of the induction, it is sufficient to prove that \r\n$ y_n\\displaystyle\\sum_{k = 1}^{n - 1}y_k(x_n - x_k)^2\\ge \\frac {y_nx_nx_{n - 1}(y_1 + \\ldots + y_{n - 1} + y_n)}4 +$ $ y_n\\cdot\\frac {y_2x_2x_1 + \\ldots + y_{n - 1}x_{n - 1}x_{n - 2}}4$. \r\n\r\nSince $ x_k\\le x_n$, it is enough to prove the last inequality for $ x_1 = \\frac {x_2}2 = \\ldots = \\frac {x_n}n$. Actually we can assume $ x_i = i$, $ i = 1,2,\\ldots,n$. Hence we are to prove \r\n$ \\displaystyle\\boxed{\\sum_{k = 1}^{n - 1}y_k(n - k)^2\\ge\\frac {n(n - 1)(y_1 + \\ldots + y_n)}4 + \\frac {y_2(2^2 - 2) + \\ldots + y_{n - 1}[(n - 1)^2 - (n - 1)]}4}$. \r\nLet ${ Y = y_1 + \\ldots + y_{n - 1}}$.\r\nFrom $ y_n\\le y_{n - 1}\\le\\ldots\\le y_1$ and rearrangement inequality we deduce the following:\r\n\r\n[b]A.[/b] $ y_1 + \\ldots + y_n\\le \\frac n{n - 1}Y$.\r\n\r\n[b]B.[/b] $ y_2 + \\ldots + y_{n - 1}\\le \\frac {n - 2}{n - 1}Y$.\r\n\r\n[b]C.[/b] $ \\displaystyle\\sum_{k = 1}^{n - 1}y_k(n - k)^2\\ge \\frac {Y[1^2 + 2^2 + \\ldots + (n - 1)^2]}{n - 1} = \\frac {n(2n - 1)}6Y$.\r\n\r\n[b]D.[/b] $ E = y_2(2^2 - 2) + \\ldots + y_{n - 1}[(n - 1)^2 - (n - 1)]\\le$ $ \\frac {(y_2 + \\ldots + y_{n - 1})[1^2 - 1 + 2^2 - 2 + \\ldots + (n - 1)^2 - (n - 1)]}{n - 2}$.\r\n\r\n[b]E.[/b] $ E\\le\\frac Y{n - 1}[\\frac {(n - 1)n(2n - 1) - 3n(n - 1)}6]$.\r\n\r\nHence, it would be enough to have that $ \\frac {n(2n - 1)}6Y = \\frac {n^2}4Y + Y[\\frac {n(2n - 1) - 3n}{24}]$, which is an identity."
}
{
"Tag": [
"logarithms",
"induction",
"algebra unsolved",
"algebra"
],
"Problem": "given the array $ x_{n}$: that:\r\n$ x_{1} \\equal{} c$ ,c is an integer\r\n$ x_{n \\plus{} 1} \\equal{} cx_{n} \\plus{} \\sqrt {(c^2 \\minus{} 1)(x_{n}^2 \\minus{} 1)}$\r\nprove that $ x_{n}$ is the array of integers",
"Solution_1": "Let $ c\\equal{}ch(a)$, were $ a\\equal{}\\ln {(c\\plus{}\\sqrt{c^2\\minus{}1})}$. Then by induction $ x_n\\equal{}ch(na)\\equal{}P_n(ch(a)$, were \r\n\\[ P_n(x)\\equal{}\\sum_{k\\equal{}0}^{[n/2]}C_n^{2k}x^{n\\minus{}2k}(x^2\\minus{}1)^k\\]- polinom with integer coefficients.\r\nTherefore \\[ x_n\\equal{}\\sum_{k\\equal{}0}^{[n/2]}C_n^{2k}c^{n\\minus{}2k}(c^2\\minus{}1)^k\\] - is integer.",
"Solution_2": "thanks Rust\r\nbut ,I need a nice solution\r\n :) any idea???",
"Solution_3": "If $ c\\le \\minus{} 1$, the sequence is periodic $ c,2c^2 \\minus{} 1,c,2c^2 \\minus{} 1,\\cdots$.\r\n\r\nIf $ c \\equal{} 0$, the sequence is periodic $ 0,1,0,1,\\cdots$\r\n\r\nIf $ c\\ge 1$, the sequence is $ x_{n \\plus{} 1} \\equal{} 2cx_n \\minus{} x_{n \\minus{} 1}$. It can be shown by induction.",
"Solution_4": "[quote=\"xxp2000\"]If $ c\\le \\minus{} 1$, the sequence is periodic $ c,2c^2 \\minus{} 1,c,2c^2 \\minus{} 1,\\cdots$.\n\nIf $ c \\equal{} 0$, the sequence is periodic $ 0,1,0,1,\\cdots$\n\nIf $ c\\ge 1$, the sequence is $ x_{n \\plus{} 1} \\equal{} 2cx_n \\minus{} x_{n \\minus{} 1}$. It can be shown by induction.[/quote]\r\nthanks\r\n :)"
}
{
"Tag": [
"inequalities unsolved",
"inequalities"
],
"Problem": "let $ a,d\\geq 0$,$ b,c>0$, such that $ a\\plus{}d\\leq b\\plus{}c$\r\n\r\nfind the minimum of \r\n\r\n$ \\frac{b}{c\\plus{}d}\\plus{}\\frac{c}{a\\plus{}b}$",
"Solution_1": "hello, after my conjecture is ${ \\frac{b}{c+d}+\\frac{c}{a+b}\\geq\\frac{1}{2}(2\\sqrt{2}-1)}$. The equal sign will obtain for $ a=\\frac{1}{2}+\\frac{\\sqrt{2}}{2},b=\\frac{1}{2}(\\sqrt{2}-1),c=1,d=0$.\r\nSonnhard."
}
{
"Tag": [
"function",
"algebra unsolved",
"algebra"
],
"Problem": "prove that doesnot exist the function f: R -> R satisfy f(f(x)) = x^2 - 2008",
"Solution_1": "[quote=\"caothujjj\"]prove that doesnot exist the function f: N -> N satisfy f(f(x)) = x^2 - 2008[/quote]\r\n\r\n$ x\\equal{}1$ implies $ f(f(1))\\equal{}\\minus{}2007<0$, which is impossible, since $ f(n)$ is from $ \\mathbb{N}$ in $ \\mathbb{N}$",
"Solution_2": "[quote=\"pco\"][quote=\"caothujjj\"]prove that doesnot exist the function f: N -> N satisfy f(f(x)) = x^2 - 2008[/quote]\n\n$ x \\equal{} 1$ implies $ f(f(1)) \\equal{} \\minus{} 2007 < 0$, which is impossible, since $ f(n)$ is from $ \\mathbb{N}$ in $ \\mathbb{N}$[/quote]\r\n\r\n sorry, i have repaired it"
}
{
"Tag": [
"inequalities proposed",
"inequalities"
],
"Problem": "Let $ a,b,c$ be positive number. Prove: \r\n$ \\frac{(a\\plus{}b)^2}{a\\plus{}b\\plus{}2c} \\plus{} \\frac{(b\\plus{}c)^2}{b\\plus{}c\\plus{}2a} \\plus{} \\frac{(c\\plus{}a)^2}{c\\plus{}a\\plus{}2b} \\ge \\sqrt{3(a^{2}\\plus{}b^{2}\\plus{}c^{2})}$",
"Solution_1": "[quote=\"......\"]Let $ a,b,c$ be positive number. Prove: \n$ \\frac {(a \\plus{} b)^2}{a \\plus{} b \\plus{} 2c} \\plus{} \\frac {(b \\plus{} c)^2}{b \\plus{} c \\plus{} 2a} \\plus{} \\frac {(c \\plus{} a)^2}{c \\plus{} a \\plus{} 2b} \\ge \\sqrt {3(a^{2} \\plus{} b^{2} \\plus{} c^{2})}$[/quote]\r\nEasy SOS:\r\n$ \\sum_{cyc}\\frac {(a \\plus{} b)^2}{a \\plus{} b \\plus{} 2c}\\geq\\sqrt {3(a^{2} \\plus{} b^{2} \\plus{} c^{2})}\\Leftrightarrow$\r\n$ \\Leftrightarrow\\sum_{cyc}\\left(\\frac {(a \\plus{} b)^2}{a \\plus{} b \\plus{} 2c} \\minus{} c\\right)\\geq\\sqrt {3(a^{2} \\plus{} b^{2} \\plus{} c^{2})} \\minus{} a \\minus{} b \\minus{} c\\Leftrightarrow$\r\n$ \\Leftrightarrow\\sum_{cyc}\\frac {(a \\plus{} b)^2 \\minus{} c^2 \\minus{} c(a \\plus{} b \\plus{} c)}{a \\plus{} b \\plus{} 2c}\\geq\\sum_{cyc}\\frac {(a \\minus{} b)^2}{a \\plus{} b \\plus{} c \\plus{} \\sqrt {3(a^2 \\plus{} b^2 \\plus{} c^2)}}\\Leftrightarrow$\r\n$ \\Leftrightarrow\\sum_{cyc}\\frac {(a \\plus{} b \\plus{} c)(a \\plus{} b \\minus{} 2c)}{a \\plus{} b \\plus{} 2c}\\geq\\sum_{cyc}\\frac {(a \\minus{} b)^2}{a \\plus{} b \\plus{} c \\plus{} \\sqrt {3(a^2 \\plus{} b^2 \\plus{} c^2)}}\\Leftrightarrow$\r\n$ \\Leftrightarrow\\sum_{cyc}(a \\plus{} b \\plus{} c)\\left(\\frac {b \\minus{} c}{a \\plus{} b \\plus{} 2c} \\minus{} \\frac {c \\minus{} a}{a \\plus{} b \\plus{} 2c}\\right)\\geq$\r\n$ \\geq\\sum_{cyc}\\frac {(a \\minus{} b)^2}{a \\plus{} b \\plus{} c \\plus{} \\sqrt {3(a^2 \\plus{} b^2 \\plus{} c^2)}}\\Leftrightarrow$\r\n$ \\Leftrightarrow\\sum_{cyc}(a \\plus{} b \\plus{} c)\\left(\\frac {a \\minus{} b}{a \\plus{} c \\plus{} 2b} \\minus{} \\frac {a \\minus{} b}{b \\plus{} c \\plus{} 2a}\\right)\\geq$\r\n$ \\geq\\sum_{cyc}\\frac {(a \\minus{} b)^2}{a \\plus{} b \\plus{} c \\plus{} \\sqrt {3(a^2 \\plus{} b^2 \\plus{} c^2)}}\\Leftrightarrow$\r\n$ \\Leftrightarrow\\sum_{cyc}(a \\minus{} b)^2\\left(\\frac {a \\plus{} b \\plus{} c}{(a \\plus{} c \\plus{} 2b)(b \\plus{} c \\plus{} 2a)} \\minus{} \\frac {1}{a \\plus{} b \\plus{} c \\plus{} \\sqrt {3(a^2 \\plus{} b^2 \\plus{} c^2)}}\\right)\\geq0.$\r\nLet $ a\\geq b\\geq c.$ Hence, \r\n$ \\sum_{cyc}(a \\minus{} b)^2\\left(\\frac {a \\plus{} b \\plus{} c}{(a \\plus{} c \\plus{} 2b)(b \\plus{} c \\plus{} 2a)} \\minus{} \\frac {1}{a \\plus{} b \\plus{} c \\plus{} \\sqrt {3(a^2 \\plus{} b^2 \\plus{} c^2)}}\\right)\\geq$\r\n$ \\geq\\sum_{cyc}(a \\minus{} b)^2\\left(\\frac {a \\plus{} b \\plus{} c}{(a \\plus{} c \\plus{} 2b)(b \\plus{} c \\plus{} 2a)} \\minus{} \\frac {1}{2(a \\plus{} b \\plus{} c)}\\right) \\equal{}$\r\n$ \\equal{} \\sum_{cyc}\\frac {(a \\minus{} b)^2(c^2 \\plus{} ac \\plus{} bc \\minus{} ab)(a \\plus{} b \\plus{} 2c)}{2(a \\plus{} b \\plus{} c)(b \\plus{} c \\plus{} 2a)(a \\plus{} c \\plus{} 2b)(a \\plus{} b \\plus{} 2c)}\\geq$\r\n$ \\geq\\frac {(a \\minus{} b)^2(c^2 \\plus{} ac \\plus{} bc \\minus{} ab)(a \\plus{} b \\plus{} 2c)}{2(a \\plus{} b \\plus{} c)(b \\plus{} c \\plus{} 2a)(a \\plus{} c \\plus{} 2b)(a \\plus{} b \\plus{} 2c)} \\plus{}$\r\n$ \\plus{} \\frac {(a \\minus{} c)^2(b^2 \\plus{} ab \\plus{} bc \\minus{} ac)(a \\plus{} c \\plus{} 2b)}{2(a \\plus{} b \\plus{} c)(b \\plus{} c \\plus{} 2a)(a \\plus{} c \\plus{} 2b)(a \\plus{} b \\plus{} 2c)}\\geq$\r\n$ \\geq\\frac {(a \\minus{} b)^2((c^2 \\plus{} ac \\plus{} bc \\minus{} ab)(a \\plus{} b \\plus{} 2c) \\plus{} (b^2 \\plus{} ab \\plus{} bc \\minus{} ac)(a \\plus{} c \\plus{} 2b))}{2(a \\plus{} b \\plus{} c)(b \\plus{} c \\plus{} 2a)(a \\plus{} c \\plus{} 2b)(a \\plus{} b \\plus{} 2c)} \\equal{}$\r\n$ \\equal{} \\frac {(a \\minus{} b)^2(b^3 \\plus{} c^3 \\plus{} b^2a \\plus{} c^2a \\plus{} 2b^2c \\plus{} 2c^2b)}{(a \\plus{} b \\plus{} c)(b \\plus{} c \\plus{} 2a)(a \\plus{} c \\plus{} 2b)(a \\plus{} b \\plus{} 2c)}\\geq0.$",
"Solution_2": ":( you said it easy...For such terrible caculation...\r\nAny nice method to this problem?",
"Solution_3": "We can solve it easily by Am-Gm, my friend :)",
"Solution_4": "Can you post your solution for this problem, nguoivn ?",
"Solution_5": "Using the well-known result: $ \\sum\\ \\frac {x^2}{y \\plus{} z} \\geq\\ \\frac {5\\sum\\ x^2 \\minus{} 2\\sum\\ xy}{2(x\\plus{}y\\plus{}z)}$\r\n(we can prove easily this result by SOS).\r\nLet $ x \\equal{} a \\plus{} b$;... we obtain:\r\n$ \\sum\\ \\frac {(a \\plus{} b)^2}{a \\plus{} b \\plus{} 2c} \\geq\\ \\frac {2(a^2 \\plus{} b^2 \\plus{} c^2) \\plus{} ab \\plus{} bc \\plus{} ca}{a \\plus{} b \\plus{} c} \\equal{}$\r\n$ \\equal{} \\frac {1}{2}[a \\plus{} b \\plus{} c \\plus{} \\frac {3(a^2 \\plus{} b^2 \\plus{} c^2)}{a \\plus{} b \\plus{} c}] \\geq\\ \\sqrt {3(a^2 \\plus{} b^2 \\plus{} c^2)}$ (by Am-Gm)\r\nWe have done :)"
}
{
"Tag": [
"floor function",
"irrational number",
"algebra proposed",
"algebra"
],
"Problem": "Here is something for everyone: part (i) --- for beginners; part (ii) --- for intermediate; part (iii) --- for advanced problem-solvers (I haven't yet found a solution for (iii); may be very difficult, but may as well be trivial, in case I missed something obvious).\r\n\r\n[b](i)[/b] (easy; many would perceive it as a joke :) ). Let $\\alpha > 1$ be a real number such that $n \\mid \\lfloor n\\alpha^n \\rfloor$ for all $n \\in \\mathbb{N}$. Must $\\alpha$ be an integer?\r\n[b](ii)[/b] (intermediate) Does there exist an irrational $\\alpha > 1$ such that $n \\mid \\lfloor \\alpha^n \\rfloor$ for infinitely many $n$?\r\n[b](iii)*[/b] (very difficult, I think; may be an open problem) Does there exist an $\\alpha > 1$ such that $n \\mid \\lfloor \\alpha^n \\rfloor$ for [b]all[/b] $n \\in \\mathbb{N}$?\r\n\r\nPerhaps a little surprisingly, I think that the answer to (iii) should be 'no.' I'd find quite unnormal the existence of an integer sequence $\\{a_n\\}$ such that $\\big( (a_{n+1}-1)^{\\frac{1}{n+1}}, (a_{n+1} + 1)^{\\frac{1}{n+1}} \\big) \\subset \\big( (a_n-1)^{\\frac{1}{n}}, (a_n + 1)^{\\frac{1}{n}} \\big)$ AND $n \\mid a_n$, because the length of the interval $\\big( (a_n-1)^{\\frac{n+1}{n}} + 1, (a_n + 1)^{\\frac{n+1}{n}} - 1 \\big)$ (from which we have to choose $a_{n+1}$) is $o(n)$ (if I did the estimates correctly). Any ideas are welcome! :lol: \r\n\r\nBesides, enjoy :D !\r\n\r\n--Vesselin",
"Solution_1": "(i) Hmm... $\\alpha = \\frac 1 2 $ so that $0 \\leq n \\alpha^n < 1$ for all $n$.\r\nI like problems I can solve in 10 seconds :P \r\n\r\nPierre.",
"Solution_2": "What exactly does that mean? :? He mentioned $\\alpha>1$ in all three problems.",
"Solution_3": "Aaargh...10 seconds is not enough for me to understand the problem :blush: \r\n\r\nPierre.",
"Solution_4": "(i) Hint: Construct [b]explicitly[/b] an irrational number $\\alpha > 1$ such that $\\lfloor n\\alpha^n \\rfloor = n \\lfloor \\alpha^n \\rfloor$ for all $n$ :lol: . This is why I said the problem is a 'joke,' but it's not [i]absolutely[/i] trivial --- though this hint itself is possibly the most non-trivial part :) ."
}
{
"Tag": [],
"Problem": "Suppose that $ a$ and $ b$ are positive real numbers such that:\r\n\r\n$ a^{3}\\equal{} a\\plus{}1$ and $ b^{6}\\equal{} b\\plus{}3a$\r\n\r\nShow that $ a > b$.",
"Solution_1": "Look at [url=http://www.mathlinks.ro/Forum/viewtopic.php?t=79648]here[/url]."
}
{
"Tag": [
"algorithm",
"function",
"algebra",
"polynomial",
"rational function",
"complex analysis"
],
"Problem": "Hello everyone! I am beginner, I have a questions(may be easy). Help me please.\r\n\r\nLet $ P(z)\\equal{}\\displaystyle\\sum_{k\\equal{}1}^{n}a_kz^k$ be a polynomial.\r\ni)Express $ P(z)$ as a power series around some point $ u\\neq 0$. \r\nii)Express the following rational map explicitly as a power series around $ z\\equal{}2$: $ f(z)\\equal{}\\frac{3z^4\\plus{}z^3\\plus{}2z^2\\plus{}7}{z^2\\minus{}z}$.\r\nii)Give an algorithm of expressing as a power series an arbitrary rational function $ f\\equal{}\\frac{P(z)}{Q(z)}$.",
"Solution_1": "Step 1: Substitute $ w\\equal{}z\\minus{}u$ everywhere and expand. It's just easier to see things that way.\r\nIn the rational function cases, you should definitely do the long division to separate out the polynomial part. For the proper rational part, there are two basic approaches:\r\n- Build the coefficients recursively, getting the equations by clearing the denominator and matching coefficients. You don't usually get an explicit closed form this way, but it's pretty easy to calculate as far as you want.\r\n- Partial fractions. $ \\frac1{(1\\minus{}az)^n}\\equal{}\\sum_{k\\equal{}0}^{\\infty}\\binom{n\\plus{}k\\minus{}1}{k}a^k z^k$; use these for an explicit closed form for the coefficients."
}
{
"Tag": [
"linear algebra",
"matrix",
"function",
"Euler",
"algebra",
"polynomial",
"linear algebra unsolved"
],
"Problem": "Let $ \\mathbf{P}$ denote the \"infinite matrix\"\r\n\r\n$ \\left[ \\begin{array}{ccccc} 1 & 0 & 0 & 0 & \\hdots \\\\\r\n1 & 1 & 0 & 0 & \\hdots \\\\\r\n1 & 2 & 1 & 0 & \\hdots \\\\\r\n1 & 3 & 3 & 1 & \\hdots \\\\\r\n\\vdots & \\vdots & \\vdots & \\vdots & \\ddots \\\\\r\n\\end{array} \\right]$\r\n\r\nwith entries $ \\mathbf{P}_{ij} \\equal{} {i \\minus{} 1 \\choose j \\minus{} 1}$ and let $ \\mathbf{I}$ denote the \"infinite identity matrix.\" Compute the inverse of $ \\mathbf{P} \\plus{} \\mathbf{I}$.\r\n\r\n[hide=\"Remark\"] The computation of this inverse has apparently been the subject of several published papers, one of which has the thread title, even though it is relatively straightforward if looked at from the proper perspective. [/hide]\n\n[hide=\"Hint 1\"] $ \\mathbf{P}$ acts on $ \\mathbb{C}[[x]]$ (regarded as the ring of formal exponential generating functions over $ \\mathbb{C}$) by multiplication by $ e^x$. [/hide] \n\n[hide=\"Hint 2\"] Generalize to multiplication by $ e^{tx}$. [/hide]",
"Solution_1": "[hide=\"Hint 3\"] It suffices to compute the power series of $ \\frac{1}{1 \\plus{} e^x}$. This computation can be done using [url=http://mathworld.wolfram.com/EulerPolynomial.html]Euler polynomials[/url] or you can write it as $ \\frac{1}{2} \\cdot \\frac{1}{1 \\plus{} \\frac{e^x \\minus{} 1}{2}}$ and expand directly. [/hide]"
}
{
"Tag": [
"geometry",
"rectangle"
],
"Problem": "Find a recursion for $ a_{n}$, where $ a_{n}$ is the number of ways you can tile a $ 2\\times n$ rectangle with $ 1\\times 1$ squares and L-trominoes.\r\n\r\nI know problems like this have been posted, but for some reason my answer doesn't agree with JBL's from a post of his in 2004. \r\n\r\n[hide=\"My solution\"]\nThe standard procedure is to consider tiling the leftmost side. Using only $ 1\\times 1$ squares, there is 1 way. Using only L-trominoes, there are 2 ways.\n\n$ a_{n}=a_{n-1}+2b_{n-1}$,\n\nwhere $ b_{n-1}$ is the number of ways to tile a $ 2\\times (n-1)$ rectangle with a corner missing. \n\n$ b_{n-1}=a_{n-2}+a_{n-3}$.\n\nSo $ a_{n}=a_{n-1}+2a_{n-2}+2a_{n-3}$. \n\nJBL's answer was $ a_{n}= a_{n-1}+4a_{n-2}+2a_{n-3}$.[/hide]",
"Solution_1": "I calculated the first four cases and got $ a_{1}= 1, a_{2}= 5, a_{3}= 11, a_{4}= 58$, which fits JBL's recursion.\r\n\r\nBut your solution makes sense too so I don't know what happened.",
"Solution_2": "Oh I just realized I might have forgotten the case where you use 1 $ 1\\times 1$ square and 1 L-trominoe. \r\n\r\nSo $ a_{n}=a_{n-1}+2b_{n-1}+2a_{n-2}$.\r\n\r\nThen $ b_{n-1}=a_{n-2}+a_{n-3}$.\r\n\r\n$ a_{n}=a_{n-1}+4a_{n-2}+2a_{n-3}$, like JBL's answer."
}
{
"Tag": [
"algebra",
"polynomial",
"function",
"LaTeX",
"calculus"
],
"Problem": "Hey, this is a total shot in the dark, but... I just joined because I need help with a math problem [i]now[/i]. I have been working on it all night long and my brain is finally shot. I can't seem to do anything else with it. If you could show me how to solve for x I'd be forever indebted to you, but if you can't, I'll live and still thank you for reading this.\r\n\r\n[921,600x^10+1,105,920rt(3)x^8-2,525,184x^6-2,112,306rt(3)x^4+4,357,416.96x^2-1,901,076x^(-2)+268,738.56x^(-6)+597,196.8rt(3)]/[146,966,400x^4+62,985,600rt(5)x^4]=-(2/3)x^2+(36/25)x^(-2)-(6rt(3)/15)\r\n\r\nrt=square root, and I know it's long and messy, but I hope you could follow it...\r\n\r\nHow can I solve for x?",
"Solution_1": ":o \r\n\r\nWhere on Earth is this problem from?",
"Solution_2": "Well, just for screams and giggles, I'll translate this into latex and you can tell me if I got it right. Ok?\r\n$\\frac{[921600x^{10}+1105920\\sqrt{3}x^{8}-2525184x^{6}-2112306\\sqrt{3}x^{4}+4357416.96x^{2}-1901076x^{-2}+268738.56x^{-6}+597196.8\\sqrt{3}]}{146966400x^{4}+62985600\\sqrt{5}x^{4}}=$\r\n$=-\\frac{2}{3}\\cdot x^{2}+\\frac{36}{25}\\cdot x^{-2}-(\\frac{6\\sqrt{3}}{15})$\r\n\r\nIs that right?\r\n\r\nWell, unless you happen to have mathematica, I think you're hopelessly screwed. However, it's late at night, and this might be one of those trick questions that work out nicely. However, like Torajirou said... Where did this come from???",
"Solution_3": "Yep, that's it. The problem came from my own awful mind. I had to create a story problem for AP Calculus, and I had about six pages of work to get to that point where I just totally blanked and couldn't solve anymore. I guess I'll just have to make an easier problem before third period today."
}
{
"Tag": [
"Support",
"geometry",
"percent",
"articles",
"analytic geometry",
"absolute value"
],
"Problem": "My tributes to the man who did not bow to the US verdicts till the end.\r\nHe had his faults, but he is far better than tha people who hanged him.\r\n\r\nThe Iraqis and the Americans will pay for this. The fact that papa Bush now sits comfortably somewhere in his son's kingdom is especially stinging. :mad:",
"Solution_1": "Yeah Saddam Hussein's defiance to the last minute is worthy of respect.",
"Solution_2": "Killing people is not nice.\r\n\r\nInterpret the above as you wish...",
"Solution_3": "look who's talking!\r\nwho told me once 'i have an instant urge to stab you'?",
"Solution_4": "[quote=\"beta\"]Yeah Saddam Hussein's defiance to the last minute is worthy of respect.[/quote]\n\nIf non-defiance would have changed his situation in any aspect, this would make sense.\n\n[quote=\"bubka\"]He had his faults, but he is far better than tha people who hanged him.[/quote]\r\n\r\nPlease do specify the criteria for \"good\" you are using. Unless the post is meant just as another provocation to start a flamewar/spam topic.\r\n\r\n darij",
"Solution_5": "[quote=\"beta\"]Yeah Saddam Hussein's defiance to the last minute is worthy of respect.[/quote]\r\nNothing about that man is worthy of respect. Have you seen the atrocities he and his son committed?\r\n\r\nThat said, killing people is still [i]not nice.[/i]\r\n\r\nEDIT: And what's up with Bubka subtitling this 'the martyr'? Honestly, this is a new low, even for Bubka.",
"Solution_6": "we already had a discusion about the death penalty.\r\nsure there were some other ways than killing him...",
"Solution_7": "He's no martyr. He committed genocide; he was a brutal tyrant and a dictator and in no way worthy of anyone's respect.\r\n\r\nI think the Iraq war was a bad idea from the start, and I oppose the death penalty (I think life imprisonment would be a better sentance), but I find it hard to get upset over his death, or feel any sympathy for him.",
"Solution_8": "I have this question, how many of the people in this forum had heard about Sadam's crimes [i]before[/i] the begginning of hostilities that finally led to Iraq's war and Sadam's capture?\r\nThe question is relevant to me because I hear all these opinions about Sadam's being a brutal tyrant and all which I'm sure he was but were you people aware [i]before[/i] the western world campaign against Sadam started.\r\n\r\nThat trial was a fraud, pretty conveniently orchestrated from the US and of course taking place in Iraq where law allows these quick executions. Had the trial taken place in the US, there was no way you could convict Sadam and some days later execute him. That, besides the fact that the conviction came right on time for the elections, curious, pity it didn't do the republicans any good.\r\n\r\nAnd the Iraq war is for oil. The idea of US coming to save the world of the opression of the tyrant is pretty but c'mon!. The accusations made to Iraq prior to the war had nothing to do with the killing of many shii'te or kurds and were false and I don't buy they didn't know they were. And that's is despicable.\r\n\r\nAs for Sadam's hanging, I truly hope someday we won't have DP anymore. As other people have said I'm just against DP, whoever it is, I don't see the point. And it does not fall in the way of making our society better.\r\n\r\nAlso it is no joke to add the martyr thing in the title of this thread. That is also an insult to many people who suffered Sadam's regime. No matter [b]bubka[/b] what your political views on the subject are, which btw you can expose rightfully in the thread, I can't figure out the point of putting it in the title so it is probably only to make people angry about it. That is a very childish behaviour\r\n\r\nUrsula",
"Solution_9": "[quote=\"deimos\"]we already had a discusion about the death penalty.\nsure there were some other ways than killing him...[/quote]\nYes but the Iraqi law did allow executions and there have been executions of ordinary criminals since the fall of the regime. So it would make no sense to pardon him and not those others. \n\n[quote=\"ursula\"]I have this question, how many of the people in this forum had heard about Sadam's crimes [i]before[/i] the begginning of hostilities that finally led to Iraq's war and Sadam's capture?\nThe question is relevant to me because I hear all these opinions about Sadam's being a brutal tyrant and all which I'm sure he was but were you people aware [i]before[/i] the western world campaign against Sadam started.[/quote]\nWell I had, but only after the first Gulf War erupted.\n\n[quote]That trial was a fraud, pretty conveniently orchestrated from the US and of course taking place in Iraq where law allows these quick executions. Had the trial taken place in the US, there was no way you could convict Sadam and some days later execute him. [/quote]\nThere was a proposal to let the trial take place in Sweden too, because they knew if he would have been sentenced to death, Sweden would have never given him back.\n\n[quote]That, besides the fact that the conviction came right on time for the elections, curious, pity it didn't do the republicans any good.[/quote]\nThat was silly.\n\n[quote]And the Iraq war is for oil. The idea of US coming to save the world of the opression of the tyrant is pretty but c'mon!. The accusations made to Iraq prior to the war had nothing to do with the killing of many shii'te or kurds and were false and I don't buy they didn't know they were. And that's is despicable.[/quote]\nIt's amazing how many people have forgotten that back in 1988,the Bush administration tried to blame Iran for the Halabja incident... :| \n\n\n[quote]Also it is no joke to add the martyr thing in the title of this thread. That is also an insult to many people who suffered Sadam's regime. No matter [b]bubka[/b] what your political views on the subject are, which btw you can expose rightfully in the thread, I can't figure out the point of putting it in the title so it is probably only to make people angry about it. That is a very childish behaviour[/quote]\r\nIt's the same kind of behavior as Chavez's, hanging out with other people who don't like the USA (he hangs with Lukashenko, the Iranian president, Castro,..)\r\nBubka's ideas tend to frighten me. He seems to defend communism at all costs, including democracy and human rights. And anti-USA sentiment, at all costs.",
"Solution_10": "Okay here is a simple question:\r\nbetween saddam and papa bush, who deserves more to be hanged? (On second thoughts, I'll add drawn and quartered.)\r\n\r\nI'll come to the 'martyr' part later.",
"Solution_11": "There are many ways for a man to be dead.\r\n\r\n\r\nJust think about it...\r\n\r\n\r\n\r\nIt doesn't mean just hanged or executed.\r\n\r\nHe could be killed in many other ways, not violently or with any weapons....\r\n\r\n\r\nmake him ashamed..",
"Solution_12": "[quote=\"bubka\"]Okay here is a simple question:\nbetween saddam and papa bush, who deserves more to be hanged? (On second thoughts, I'll add drawn and quartered.)\n[/quote]\r\n\r\nI'd still have to say Saddam- Bush hasn't committed any sins that Saddam hasn't committed many times over.\r\n\r\nHowever I oppose the death penalty on general principle, and I would have to agree with bubka on the martyrdom thing- martyrdom is in the eyes of the beholder, and to be honest, Saddam's defiance, and his call for nonviolence at the end of his life will probably bring him to the status of martyr in the eyes of many.\r\n\r\nIn other words, the government probably shouldn't have given him the death penalty- life in prison would have been far more successful at erasing his influence.",
"Solution_13": "oh dear. poor bush. mother's icicle. \r\n\r\nanyway what about donald ramsfeld who ordered the torture in abu ghraib?",
"Solution_14": "[quote=\"bubka\"]oh dear. poor bush. mother's icicle. \n\nanyway what about donald ramsfeld who ordered the torture in abu ghraib?[/quote]\r\n\r\nCan you find evidence for that statement?",
"Solution_15": "[quote=\"Go Around the Tree\"][quote=\"kstan013\"]In a land using the Code of Hammurabi, Saddam was treated, er, justly.\n@bubka What if I said when that the tsunami hit southern India, I was very happy although some Americans died. And the more frightening thing about 9/11 was the TERROR. Living in the US is not as happy jolly as you might think. :wink:[/quote]\n\n\nI quite agree. TERRORISTS ARE TERRIBLE.\n\nSADDAM WAS A TERRORIST. SADDAM=TERRIBLE\n\n\nGet the picture?\n\n\nI don't see why they hanged him so soon though. :maybe: \n\n\nwere you happy, guys, that the WTC was knocked down? \n\n\nYou support Saddam? \n\n\nWHYYY???? \n\nSADDAM IS A TERRORIST. ALTHOUGH BUSH IS A BAD PRESIDENT IN MY OPINION, SADDAM IS STILL MORE EVIL .[/quote]\nDoes Saddam have anything to do with the attacks on the World Trade Center?\n\n\n[quote]NUMBER OF PEOPLE KILLED ON THE 9/11 ATTACKS \n\n2973 \n\nNow... \n\nDarfur conflict, Sudan(ongoing): ~400 000(mostly civilians) \nSecond Sudanese Civil War, Sudan (1983-2005): ~2 000 000 deaths \nTutsis-Hutus conflict, Rwanda (three months of 1994): ~1 000 000 deaths (that's 10 000 per day) \nIraqi conflict, Iraq(ongoing): 3000+ US soldiers, the number of Iraqi civilian casualties differ greatly, from 30 000 to 600 000 deaths \nChechenyan War, Chechenya(1994-1996): 30 000 ~ 100 000 deaths many more thousands in the second conflict. \nGulf War, Kuwait: 100 000 Iraqi deaths \nKosovo War: ~12 000 deaths, NATO bombing in 1999, 2000 civilian deaths \nCroatian Independence War: ~15 000 deaths \nEritrea Ethiopia war (1998-2000): 70 000~170 000 deaths \nAlgerian Civil War, (1992-2001): 100 000 ~ 200 000 deaths \nSierra Leone Civil War(1991-2001): ~75 000 deaths [/quote]\r\n\r\nYou left out the Second Congo War(1998-2003) : more than 3 500 000 civilian casualties :ninja: \r\n\r\nIt's time we simply accept that people have double standards. Posters here are doing their best to express grief for the people who died in Manhattan but seem to forget that action like those are a consequence of US policy and MORE IMPORTANTLY : will lead to much more of the same suffering among others (the Iraqi people first)\r\nPeople get bombed everyday, I suggest you watch Fahrenheit 9/11 if you wanna see real wounded peopple, not the clean version. \r\nAnd now this has led to retaliations in Spain and London...",
"Solution_16": "[quote=\"fredbel6\"][quote=\"Go Around the Tree\"][quote=\"kstan013\"]In a land using the Code of Hammurabi, Saddam was treated, er, justly.\n@bubka What if I said when that the tsunami hit southern India, I was very happy although some Americans died. And the more frightening thing about 9/11 was the TERROR. Living in the US is not as happy jolly as you might think. :wink:[/quote]\n\n\nI quite agree. TERRORISTS ARE TERRIBLE.\n\nSADDAM WAS A TERRORIST. SADDAM=TERRIBLE\n\n\nGet the picture?\n\n\nI don't see why they hanged him so soon though. :maybe: \n\n\nwere you happy, guys, that the WTC was knocked down? \n\n\nYou support Saddam? \n\n\nWHYYY???? \n\nSADDAM IS A TERRORIST. ALTHOUGH BUSH IS A BAD PRESIDENT IN MY OPINION, SADDAM IS STILL MORE EVIL .[/quote]\nDoes Saddam have anything to do with the attacks on the World Trade Center?\n\n\n[quote]NUMBER OF PEOPLE KILLED ON THE 9/11 ATTACKS \n\n2973 \n\nNow... \n\nDarfur conflict, Sudan(ongoing): ~400 000(mostly civilians) \nSecond Sudanese Civil War, Sudan (1983-2005): ~2 000 000 deaths \nTutsis-Hutus conflict, Rwanda (three months of 1994): ~1 000 000 deaths (that's 10 000 per day) \nIraqi conflict, Iraq(ongoing): 3000+ US soldiers, the number of Iraqi civilian casualties differ greatly, from 30 000 to 600 000 deaths \nChechenyan War, Chechenya(1994-1996): 30 000 ~ 100 000 deaths many more thousands in the second conflict. \nGulf War, Kuwait: 100 000 Iraqi deaths \nKosovo War: ~12 000 deaths, NATO bombing in 1999, 2000 civilian deaths \nCroatian Independence War: ~15 000 deaths \nEritrea Ethiopia war (1998-2000): 70 000~170 000 deaths \nAlgerian Civil War, (1992-2001): 100 000 ~ 200 000 deaths \nSierra Leone Civil War(1991-2001): ~75 000 deaths [/quote]\n\nYou left out the Second Congo War(1998-2003) : more than 3 500 000 civilian casualties :ninja: \n\nIt's time we simply accept that people have double standards. Posters here are doing their best to express grief for the people who died in Manhattan but seem to forget that action like those are a consequence of US policy and MORE IMPORTANTLY : will lead to much more of the same suffering among others (the Iraqi people first)\nPeople get bombed everyday, I suggest you watch Fahrenheit 9/11 if you wanna see real wounded peopple, not the clean version. \nAnd now this has led to retaliations in Spain and London...[/quote]\r\n\r\n\r\n\r\nBush really didn't have a GOOD reason. He just associated Terrorists with WTC with Saddam.\r\n\r\n\r\nEven though Saddam didn't cause WTC and wasn't involved, Bush uses the tone that makes us believe he is in with the same group and that is used as the reason why we are in Iraq.",
"Solution_17": "I agree that the media takes what they think will sell the most papers and blows it out of perspective. Yet, despite this, many Americans are becoming aware of other problems, such as the Darfur conflicts. Recently, that has become a big thing here, and I have even seen a few disaster relief fund commercials.\r\n\r\nAlso, on a separate note, even as a conservative Republican, I do not support the war in Iraq, nor do I agree with many of Bush's policies. What a lot of foreign people don't get is that a strong majority of Americans do not support the war in Iraq. America is around 50% Republican (Bush's party) and 50% Democratic. However, Bush's approval rating dipped below 40% at the end of last month, and less than 35% of Americans approve of the war in Iraq.",
"Solution_18": "[quote=\"fredbel6\"][\n\nYou left out the Second Congo War(1998-2003) : more than 3 500 000 civilian casualties :ninja: \n[/quote]\r\n\r\nYou are quite right. I firstly wrote the conflicts I remembered and then went through Internet looking for the death tolls I didn't knew about. But I knew that you were going to be there if I missed any :)\r\n\r\nUrsula",
"Solution_19": "[quote=\"ursula\"][quote=\"fredbel6\"][\n\nYou left out the Second Congo War(1998-2003) : more than 3 500 000 civilian casualties :ninja: \n[/quote]\n\nYou are quite right. I firstly wrote the conflicts I remembered and then went through Internet looking for the death tolls I didn't knew about. But I knew that you were going to be there if I missed any :)\n\nUrsula[/quote]\r\n\r\nYou missed the toll in Bosnia too (150.000-200.000)... :!:",
"Solution_20": "[quote=\"hsiljak\"][quote=\"ursula\"][quote=\"fredbel6\"][\n\nYou left out the Second Congo War(1998-2003) : more than 3 500 000 civilian casualties :ninja: \n[/quote]\n\nYou are quite right. I firstly wrote the conflicts I remembered and then went through Internet looking for the death tolls I didn't knew about. But I knew that you were going to be there if I missed any :)\n\nUrsula[/quote]\n\nYou missed the toll in Bosnia too (150.000-200.000)... :!:[/quote]\r\n\r\nOK thanks, :) I'm editing the post, so it is more complete. \r\n\r\nUrsula",
"Solution_21": "THE AMERICAN WAY OF LIFE\r\naccording to bubka\r\n\r\nEveryone goes to the beach every day,\r\nnobody ever has to work,\r\nwe all sit in recumbent lawn chairs all day sipping pina coladas out of coconut shells,\r\noil is flowing out of our ears and we burn it for fun,\r\nwe worship Bush and never ever disagree with him,\r\nwe kill Iraqis in our free time\r\n\r\nBubka, we donated millions of dollars in relief efforts to help the tsunami victims and what did we get for 9/11? Not a dime.\r\n\r\nI realize that the Iraqi way of life was bad, don't get me wrong, but when I said [quote=\"kstan013\"]Living in the US is not as happy jolly as you might think. :wink: [/quote]I was referring to how you think Americans live.",
"Solution_22": "[quote=\"kstan013\"]Bubka, we donated millions of dollars in relief efforts to help the tsunami victims and what did we get for 9/11? Not a dime.\n[/quote]\r\n\r\nI'm sorry my friend but that argument is ridiculous.\r\n1) A lot of developed countries and not developed gave help to the tsunami victims one way or another. I don't know about the actual numbers but certainly the US weren't the only good souls in the world. I'm not sure about this one but it tends to happen that the help the US gives is absurd compared with what other countries do.\r\n\r\n2) There are not a lot of countries who can presume of being able to give something to the US that they can not proveide themselves. If furthermore such countries observe that after 9/11 you rather spend the money in multiple wars - and that's a lot of money -, money that could be spent in reconstruction or in helping the families of the victims, or the families of the firemen now sick, or en fin preventing Katrina, in such a case, you question the point of helping the US.\r\n\r\n3) I remember you that a long time ago there was this UN agreement about donating 0.7 percent of the GIP of all developed countries to the developing ones. The greatest GIP of the world, that is, the US, set an example by donating about 0.15 percent some years. I don't know how the thing is now, but your behavior towards the developing countries is full of stains.\r\n\r\nUrsula",
"Solution_23": "I don't understand why you all find a necessity to make juxtapositions with other items that have no implication on the absolute value of a deed.\r\n\r\nBush vs. Saddam\r\nHurricane deaths vs. trade center\r\n\r\nSimply put Bubka, let's take things for face value. 1 death due to evil is wrong. That's it. There is no need to compare it to a natural disaster or what Americans did or so on. Looking at everything from an individualistic standpoint ellicits a neutral position on the subject that is unbiased by the influence of needless comparisons. This is the most logical way of handling it.\r\n\r\nI am no judge, but if you consider yourself a good person you will admit that evil is evil at face value.",
"Solution_24": "[quote=\"Potato Theory\"]\nI am no judge, but if you consider yourself a good person you will admit that evil is evil at face value.[/quote]\r\n\r\n[b]Yay for Ad Hominem arguments![/b]\r\n\r\nThough I am still not sure why this thread hasn't undergone serious deletion by mods.\r\n\r\nAt any rate, it would be kinda cool for people, instead of accusing everyone else of making stuff up, to give us credible sources that back their arguments (by credible I mean actually credible, if you don't know what I mean, well, at the very least don't cite random news articles or something that are all going to be obviously biased).",
"Solution_25": "[quote=\"JSteinhardt\"]\nThough I am still not sure why this thread hasn't undergone serious deletion by mods.\n[/quote]\r\n\r\nWell, it is a serious discussion, although totally off topic thanks to our friend bubka (and me too for responding to him, I guess).",
"Solution_26": "[quote=\"JSteinhardt\"]\n[b]Yay for Ad Hominem arguments![/b]\n[/quote]\r\n\r\nThe best kind! :wink:",
"Solution_27": "[quote=\"ursula\"][quote=\"kstan013\"]Bubka, we donated millions of dollars in relief efforts to help the tsunami victims and what did we get for 9/11? Not a dime.\n[/quote]\n\nI'm sorry my friend but that argument is ridiculous.\n1) A lot of developed countries and not developed gave help to the tsunami victims one way or another. I don't know about the actual numbers but certainly the US weren't the only good souls in the world. I'm not sure about this one but it tends to happen that the help the US gives is absurd compared with what other countries do.[/quote]\n\n[url=http://en.wikipedia.org/wiki/Humanitarian_response_to_the_2004_Indian_Ocean_earthquake#List_of_Donors]link[/url]\n\n[quote]2) There are not a lot of countries who can presume of being able to give something to the US that they can not proveide themselves. If furthermore such countries observe that after 9/11 you rather spend the money in multiple wars - and that's a lot of money -, money that could be spent in reconstruction or in helping the families of the victims, or the families of the firemen now sick, or en fin preventing Katrina, in such a case, you question the point of helping the US.[/quote]\r\nTrue, after 9/11 we heard the usual \"stand as one\" talk... but what does it really mean? Lots of firemen are now sick and unable to provide for themselves. In the mean time 3000 US soldiers have died in Iraq in a war that has cost more than 350 $10^{9}$ US dollars.\r\n\r\nBut what you say about helping the US is not entirely correct. Even Afghanistan sent some money after Katrina even though that was probably symbolic. But Belgium sent some soldiers to Louisiana to help them coordinate the relief efforts... even though we were quite surprised that a superpower able to wage several costly wars at the same time was not able to help its own citizens in its backyard... The Dutch, masters when it comes to manipulating water, have also offered help reconstructing the levees.",
"Solution_28": "EDIT: post removed...\r\n\r\nOne thing I would like to mention is that those of you who want to discuss \"hot\" issues like this should try to do so in a less abrasive manner. While I hate to say it, I have learned one thing from my learning argumentation in english. What you have to say is less important than how you say it. This is why I suggest [note I claim no authority] that people, like Bubka, consider rogerian argumentation. That way, you don't look like a troll who just wants to start a flame war [not only this topic, but the topic about the cost of WOOT].",
"Solution_29": "[quote=\"fredbel6\"]But what you say about helping the US is not entirely correct. Even Afghanistan sent some money after Katrina even though that was probably symbolic. But Belgium sent some soldiers to Louisiana to help them coordinate the relief efforts... even though we were quite surprised that a superpower able to wage several costly wars at the same time was not able to help its own citizens in its backyard... The Dutch, masters when it comes to manipulating water, have also offered help reconstructing the levees.[/quote]\n\n[quote=\"fredbel6\"]But what you say about helping the US is not entirely correct. Even Afghanistan sent some money after Katrina even though that was probably symbolic. But Belgium sent some soldiers to Louisiana to help them coordinate the relief efforts... even though we were quite surprised that a superpower able to wage several costly wars at the same time was not able to help its own citizens in its backyard... The Dutch, masters when it comes to manipulating water, have also offered help reconstructing the levees.[/quote]\r\n\r\nI didn't actually meant the US didn't receive any help for Katrina :). Actually even [url=http://www.cnn.com/2005/WEATHER/09/03/katrina.castro/]Cuba[/url] proposed to send help to the US. But I see it can be misunderstood.\r\nOne point more in my argument could be that compared to the tsunami, 9/11 is a minor catastrophe in terms of human lives lost aswell as of economical damage.\r\n\r\nAlso [url=http://www.unmillenniumproject.org/involved/action07.htm]here you have[/url] the link about the 0.7 percent the rich countries agreed to donate to the developing ones. There you have data for year 2005. You see that the US is along with Portugal the developed country who gives [i]the least[/i], you can also see that is one of the few who have NOT set a timetable for doing it. Draw your own conclusions\r\n\r\nUrsula"
}
{
"Tag": [
"ARML",
"Support"
],
"Problem": "Here's another change to the ARML rules for 2009:\r\n\r\n[quote]In order to keep the proportions of points as they were prior to the addition of the fifth individual round, the scoring structure will be altered as follows.\n\n * Individual Rounds: 1 pt each (total of 10 points possible per person, 150 points possible per team)\n\n\n * Team Round: 5 pts each (total of 50 possible points per team)\n\n\n * Power Round: 50 points possible points per team\n\n\n * Relay Rounds: correct answer submitted in 3 minutes 3 pts, correct answer submitted in 6 minutes 2 pts. (15 points possible per team for Relay 1, 15 points possible per team for Relay 2)[/quote]\r\n\r\nSo instead of 120/40/40/40, it's going to be 150/50/50/30. It doesn't actually keep the proportions of points the same, but I guess they only meant that to apply to individual, team, and power. Comments?\r\n\r\nI guess I like it. Relays always seem kind of unpredictable, and it's too easy to blow a big lead with a bad relay.\r\n\r\nBy the way, NYSML has made the same change, except relays are worth 50 points (5 for 3 minutes, 2 for 6 minutes).",
"Solution_1": "I was under the impression that in order to both preserve proportions for the events and make the relay scoring gentler on B teams, the values were going to change to 5 pts / 3 minutes and 3 pts / 6 minutes.\r\n\r\nThe scoring Silas mentions indeed makes the 6 minutes less penalizing than before, since it is now worth more than half the previous scoring. I know that relays seem to elicit the most contentious opinions from ARML students and coaches, so I'm curious how reducing the overall impact of the relays will be met. \r\n\r\nI guess it may depend somewhat on how seriously you take the competition. I see ARML as more \"for fun\" than anything else at this level, so I don't mind the random element that the relays seem to impart. From a coaching standpoint, the possibility of a massive swing in the standings heading into the final event is a good motivator for my students... no matter how bad a day they might have had up to that point, they can still jump several spots if they buckle down and do a good job.",
"Solution_2": "I think of all the events, the relays is the one that is hardest to \"carry,\" with a couple good people, that is, you need many smart people to score well on relays, whereas for team and power, a couple strong people (2 in the case of my ARML team last year) can lead the team to 30+ scores.",
"Solution_3": "Yeah, I would have thought 5/3 also. I would also support that scoring. I would object to 3/1 or 5/2, because I've always thought 6-minute answers should be worth at least half as much as 3-minute answers.\r\n\r\nGoing to 5/3 relays would also give a way to make a rough comparison between new-format and old-format scores.",
"Solution_4": "An excellent change! I am completely in favor of it.",
"Solution_5": "I emailed the head writer and I think indeed that there was a miscommunication regarding the relay scoring rules. Hopefully this will be clarified soon :lol:",
"Solution_6": "I heard from Dr. Merryfield that the relay score won't be changing at all... so not 40 to 30, but just 40 still.",
"Solution_7": "Brut3Forc3, my source was this thread, which I seem to have misread. Believe what you see here.",
"Solution_8": "A lot of ARML officials have been emailing each other voicing their opinions on the relay scoring change. It is sounding like the 3/2 scoring rubric sent out earlier was the result of some miscommunication, and that the intended scoring was 5/3. Hopefully that will be made official soon :)",
"Solution_9": "Apparently things have been taken care of. It will in deed be 3 points for a 6 minute answer, 5 points for a 3 minute answer, and 23 points for a 36-second answer.",
"Solution_10": "[quote=\"generating\"]Apparently things have been taken care of. It will in deed be 3 points for a 6 minute answer, 5 points for a 3 minute answer, [b]and 23 points for a 36-second answer.[/b][/quote]\r\n\r\nHuh? I hope you were kidding.",
"Solution_11": "Note the date.",
"Solution_12": "Which is why I was going to wait until tomorrow to ask him about the first part of that sentence."
}
{
"Tag": [],
"Problem": "hi, I'm new and I thought I would post a problem... at least to see what happens :D \r\n\r\nHere it is\r\n\r\nLaura is 3 times as old as Sara was when Laura was as old as Sara is now. In 2 years Laura will be twice as old as Sara was 2 years ago. How old are they now?",
"Solution_1": "Well, what happens is people like me respond with a answer and a solution.\r\n[hide]We can deduct that Laura has to be a multiple of six, because her age is a multiple of 3 and has to be able to be divisible by two because an odd +2 is always odd.\nSo plug in 6 and see if it works, which it doesn't\nThen try 12 and see if it works, and it does, so \n[b]12[/b] is the answer[/hide]\r\nEDIT- I mean plug in 6 and 12 for the age of Laura.",
"Solution_2": "me too! :) \r\n[hide]i found that for every 3 years Laura goes up, Sara goes up by two. so, i estimated and trial and error two or three times, and i got Laura = 18 and Sara = 12.[/hide]",
"Solution_3": "[hide=\"an albegraic solution :)\"]\nCall Laura's age L\nSara's age S\n\nFor the second statement, it is easy to decode.\n$L+2=2(S-2)$\n$L+2=2S-4$\n$2S-L=6$\nFor the first, \nIf the difference in ages is $d$,\nthen when Laura was Sara's age, Sara was $S-d=S-(L-S)=2S-L$\nThus $L=3(2S-L)$\n$L=6S-3L$\n$4L-6S=0$\n$2L-3S=0$\n\nNow we have to solve.\nChange the first equation to \n$-2L+4L=12$\n$2L-3S=0$\n\nAdd\n$L=12$\n\nSubstitute back in to get $S=8$[/hide]"
}
{
"Tag": [
"LaTeX",
"USAMTS"
],
"Problem": "for one of my usamts files i have 8 syntax errors but i cant find anyof them and the doc is only like 2 paragraphs long wut do i do i no i cant post it for people to look at because of the honor code\r\nalso is there a way to get a spell check in latex?",
"Solution_1": "Usually, it's one error that causes several errors. For Example (not breaking honor code)...\r\n\r\nblablabla\r\n\\ begin*{eqnarray}\r\n\r\nblabla...\r\n\r\n\\end*{eqnarray}\r\n\r\nwill create about 100 errors while\r\n\r\nblablabla\r\n\\ begin*{eqnarray}\r\nblabla...\r\n\\end*{eqnarray}\r\n\r\nwill create none.",
"Solution_2": "Usually, when you have a string or two of LaTeX, it's hard to determine where the error is. So I recommend deleting like the second row and see if the problem persists. If it does, then delete more until you have just the code that shows up right and try to fix from there. I used to look at entire string but what happens is sometimes, you actually make correct code to be wrong, thinking it was the wrong code and therefore, make the problem even more difficult.",
"Solution_3": "are you saying that u dont put a spare line after \\begin{document}\r\nand before \\end{document} because i did\r\nill try removing the lines",
"Solution_4": "i just found a section of the forum for latex so ill stop posting latex stuff here"
}
{
"Tag": [
"number theory unsolved",
"number theory"
],
"Problem": "Solve in $ \\mathbb{P}$ the equation $ 2^{x\\plus{}1}\\plus{}y^2\\equal{}z^2$.",
"Solution_1": "(z+y)(z-y)=2^(x+1). As z+y > z-y, z+y is a multiple of z-y\r\n\r\nAll primes greater than 3 can be expressed as 6k+1or 6k-1. So, either z+y is a multiple of 3 or \r\nz-y is a multiple of 3. So, not possible for y>3.\r\nFor y=3,\r\nz+3 is a multiple of z-3. So, z<7. The only possible prime value for z=5\r\nFor y=2,\r\nz<5. So, no possible primes for this case.\r\n\r\n(2,3,5) is the only solution in P.",
"Solution_2": "i have a question\r\nwhy you said\r\n$ x\\plus{}y>x\\minus{}y\\Longrightarrow x\\minus{}y\\mid x\\plus{}y$\r\ni didnt understand :blush:",
"Solution_3": "[quote=\"pelao_malo\"]i have a question\nwhy you said\n$ x \\plus{} y > x \\minus{} y\\Longrightarrow x \\minus{} y\\mid x \\plus{} y$\ni didnt understand :blush:[/quote]\r\nbecause x+y and x-y are both powers of 2, which means x+y is a greater power of 2 than x-y"
}
{
"Tag": [
"calculus",
"integration",
"limit",
"real analysis",
"function",
"calculus computations"
],
"Problem": "What condition is necessary so that:\r\n\r\n$ \\lim_{x\\to\\infty}\\left(\\int_{a}^{b}f(x,y) \\ dy\\right) \\equal{} \\int_{a}^{b}\\lim_{x\\to\\infty}f(x,y) \\ dy$\r\n\r\n?",
"Solution_1": "I don't want to get into asking about necessary conditions; I suspect there really isn't anything nice you can say. In other words, sometimes you \"shouldn't\" be able to interchange the limit and integral, but it works anyway.\r\n\r\nThere are several reasonable sufficient conditions.\r\n\r\n1. If $ f(x,y)\\to g(y)$ [b]uniformly[/b] as $ x\\to\\infty,$ then this is justified. That is, $ \\forall\\,\\epsilon>0\\,\\exists\\,N\\equal{}N(\\epsilon)$ such that if $ x>N,$ then $ |f(x,y)\\minus{}g(y)|<\\epsilon.$\r\n\r\n2. If there exists $ h(y)$ defined almost everywhere on $ [a,b]$ such that $ |f(x,y)|\\le h(y)$ and $ \\int_a^bh(y)\\,dy<\\infty,$ then this is justified. Of course, this is simply the Lebesgue Dominated Convergence Theorem, with some minor stuff about turning limits on arbitrary sequences into limits at infinity.",
"Solution_2": "If the functions in question are non-negative, the interval is finite, the limit exists a.e. or in measure, and the integral of the limit function is finite, then eventual equiintegrability (or whatever the right term in English is) is necessary and sufficient. The property is just that for every $ \\varepsilon>0$ there exists $ \\delta>0$ such that whenever the measure of a set $ E\\subset[a,b]$ is less then $ \\delta$, one has $ \\int_E f(x,y)\\,dy<\\varepsilon$ for all sufficiently large $ x$. Another way to state the same is to say that there exists a function $ \\Psi: [0,\\plus{}\\infty)\\to[0,\\plus{}\\infty)$ such that $ \\lim_{t\\to\\plus{}\\infty}\\frac{\\Psi(t)}{t}\\equal{}\\plus{}\\infty$ and $ \\limsup_{x\\to\\infty}\\int_a^b \\Psi(f(x,y))\\,dy<\\plus{}\\infty$. For sign changing functions it is as close to a necessary and sufficient condition as you can possibly wish but, of course, as Kent said there is nothing that is absolutely necessary."
}
{
"Tag": [
"number theory unsolved",
"number theory"
],
"Problem": "Find all pairs of positive integers $ (a, b)$ such that\r\n\\[ ab \\equal{} gcd(a, b) \\plus{} lcm(a, b).\r\n\\]",
"Solution_1": "[quote=\"STARS\"]Find all pairs of positive integers $ (a, b)$ such that\n\\[ ab \\equal{} gcd(a, b) \\plus{} lcm(a, b).\n\\]\n[/quote]\r\n\r\n\r\n\r\n$ (a;b)\\equal{}k$----->$ a\\equal{}nk;b\\equal{}mk$-----$ (m,n)\\equal{}1$---->$ [a;b]\\equal{}nmk$----->$ ab \\equal{} gcd(a, b) \\plus{} lcm(a, b)\\equal{}(a;b)\\plus{}[a;b]$----->\r\n\r\n\r\n$ nmk^{2}\\equal{}k\\plus{}mnk$-----> $ mnk\\equal{}1\\plus{}mn$----->$ k\\equal{}\\frac{1}{mn}\\plus{}1$---->$ mn\\equal{}1$-----> $ m\\equal{}n\\equal{}1$ and $ k\\equal{}2$----> $ a\\equal{}b\\equal{}2$",
"Solution_2": "It is well known that $ ab \\equal{} \\gcd(a,b)lcm(a,b)$, so\r\n\r\n$ \\gcd(a,b)lcm(a,b) \\equal{} \\gcd(a,b) \\plus{} lcm(a,b)$\r\n$ (\\gcd(a,b) \\minus{} 1)(lcm(a,b) \\minus{} 1) \\equal{} 1$\r\n\r\n$ \\gcd(a,b) \\equal{} lcm(a,b) \\equal{} 2$.\r\n\r\n$ (a,b) \\equal{} (2,2)$"
}
{
"Tag": [],
"Problem": "Four people think of different positive integers. They add them and their sum is 10. What is the greatest integer?",
"Solution_1": "Lets say biggest integer is more then 5. Then other 3 integer have to sum up to equal or less then $ 5$.\r\nSmallest sum of different positive integer is $ 1 \\plus{} 2 \\plus{} 3 \\equal{} 6$ Therefore biggest integere has to be $ 4$ or less.\r\nWe can easily see that integers are $ 1,2,3,$ and $ 4$ Therefore $ 4$ is biggest integer"
}
{
"Tag": [
"function",
"algebra unsolved",
"algebra"
],
"Problem": "Determine all function $f: \\mathbb N \\to \\mathbb N$ such that\n\\[f(m+n)f(m-n) = f(m^{2}),\\]\nfor all $m,n \\in \\mathbb N$.",
"Solution_1": "[quote=\"hoangclub\"]Denote $ Z^{+}$ is set of all positive integer numbers. \n determine all function $ f$:$ Z^{+}$ ->$ Z^{+}$ such that\n $ f(m+n)f(m-n)$=$ f(m^{2})$ for all m,n $ \\in$ $ Z^{+}$.[/quote]\r\n\r\n${ f(n)f(n+4)=f((n+2)^{2}})=f(n+1)f(n+3)$ and so $ \\frac{f(n+4)}{f(n+3)}=\\frac{f(n+1)}{f(n)}$ and so It exists $ u,a,b,c$ such that :\r\n$ f(3p+1)=ua^{p}b^{p}c^{p}$\r\n$ f(3p+2)=ua^{p+1}b^{p}c^{p}$\r\n$ f(3p+3)=ua^{p+1}b^{p+1}c^{p}$\r\n\r\nThen :\r\n$ f(1)f(3)=f(4)$ $ \\implies$ $ u uab=uabc$ and $ u=c$\r\n$ f(1)f(5)=f(9)$ $ \\implies$ $ u ua^{2}bc = ua^{3}b^{3}c^{2}$ and $ u=ab^{2}c$\r\n$ f(1)f(7)=f(16)$ $ \\implies$ $ u ua^{2}b^{2}c^{2}= ua^{5}b^{5}c^{5}$ and $ u=a^{3}b^{3}c^{3}$\r\n$ f(1)f(9)=f(25)$ $ \\implies$ $ u ua^{3}b^{3}c^{2}= ua^{8}b^{8}c^{8}$ and $ u=a^{5}b^{5}c^{6}$\r\n\r\nAnd so $ a=b=c=u=1$\r\n\r\nAnd $ f(n)=1$ $ \\forall n$"
}
{
"Tag": [
"inequalities",
"induction",
"inequalities proposed"
],
"Problem": "Prove that for all positive integers $00$ and $a^2+b^2+c^2+d^2>0$, in order to show that this is composite we must show that $a+d=b+c+1$ is not possible. Suppose it is possible. Then squaring yields \\[a^2+2ad+d^2=b^2+2bc+c^2+2b+2c+1\\implies a^2+ad+d^2=bc+2b+2c+1.\\] Using the fact that $ad=b^2+bc+c^2$ again gives \\[a^2+b^2+c^2+d^2+ad+bc=ad+bc+2b+2c+1\\implies a^2+(b-1)^2+(c-1)^2+d^2=1.\\] However, since $a^2+d^2\\geq 2$ we have a contradiction. Therefore $a+d-b-c\\geq 2$ and $a^2+b^2+c^2+d^2$ is composite, as desired. $\\blacksquare$",
"Solution_4": "Note that \n\\[(a+d)^2 - (b+c)^2 = a^2+d^2 + 2(ad-bc) - b^2-c^2 = a^2+d^2 + 2(b^2+c^2)-b^2-c^2 = a^2+b^2+c^2+d^2\\]\nThus, $a^2+b^2+c^2+d^2 = (a+d-b-c)\\cdot (a+d+b+c)$. Thus, it suffices to show that $a+d\\geq b+c+2$. AFTSOC that $a+d\\leq b+c+1$, then \n\\[b^2+bc+c^2=ad\\leq \\frac{(a+d)^2}{4} \\leq \\frac{(b+c+1)^2}{4}\\]\nSo, \n\\[4b^2+4bc+4c^2\\leq b^2+c^2+2bc+2b+2c+1\\]\nThen,\n\\[3b^2+2bc+3c^2\\leq 2b+2c+1\\]\nbut since $b,c$ are positive, we have $3b^2>2b, 2bc>1, 3c^2>2c$, so this is absurd. Thus, $(a+d-b-c)\\geq 2$, so $a^2+b^2+c^2+d^2$ can be written as the product of two integers $\\geq 2$ and we're done."
}
{
"Tag": [
"induction",
"trigonometry",
"algebra unsolved",
"algebra"
],
"Problem": "How to prove the identity\r\n\r\n${{\\sin^2(n+1)x}\\over {\\sin x}} = \\sin x+\\sin 2x +\\sin 3x+\\ldots +\\sin (2n+1)x$\r\n\r\nby induction?",
"Solution_1": "I don't think it's correct :? (so we can't prove it by induction when assuming consistency of our axioms :P ).\r\nTake $n=1$ to get $\\frac{\\sin(2x)^2}{\\sin(x)} = \\sin(x)+\\sin(2x)+\\sin(3x)$, which is wrong for all $x \\neq \\frac{n\\pi}{2}$.",
"Solution_2": "The correct formula should be\r\n$\\sin\\alpha+\\sin 2\\alpha+\\sin 3\\alpha+\\cdots +\\sin n\\alpha=\\frac{\\sin\\frac{n\\alpha}{2}\\sin\\frac{(n+1)\\alpha}{2}}{\\sin\\frac{\\alpha}{2}}$.",
"Solution_3": "Indeed, and your formula can be easily proven using complex numbers ;) (if we don't know the RHS of course, else, induction!)",
"Solution_4": "The correct identity should be:\r\n\r\n${{\\sin^2(n+1)x}\\over {\\sin x}}=\\sinh+\\sin3x +\\ldots+\\sin (2n+1)x$\r\n\r\nFor $n=1$:\r\n\r\nLHS${= {{\\sin^2 2x}\\over {\\sin x}}={{(2\\sin x\\cos x)^2}\\over {\\sin x}} ={{4\\sin^2 x\\cos^2 x}\\over {\\sin x}}=4\\sin x\\cos^2 x}$\r\n\r\n\r\nRHS$=\\sin x+\\sin3x =2\\sin{{3x+x}\\over 2}\\cos{{3x-x}\\over 2} =2\\sin 2x\\cos x=4\\sin x\\cos^2 x=$LHS"
}
{
"Tag": [],
"Problem": "1. Find all values of $ x,y \\in\\mathbb{N}$ that satisfy\r\n$ (x\\plus{}y)\\plus{}(x\\minus{}y)\\plus{}xy\\plus{}\\frac2{y}\\equal{}2009$\r\n\r\n2. Find the value of $ x$ if\r\n$ x^2\\minus{}2\\sqrt{x\\minus{}1\\minus{}\\frac1{x}\\plus{}\\frac1{x^2}}\\minus{}x\\plus{}\\frac2{x}\\equal{}1$, $ x\\neq 0$\r\n\r\n3. It is given that $ x\\equal{}\\frac1{2\\minus{}\\sqrt{3}}$. Find the value of\r\n$ x^6\\minus{}2\\sqrt{3}x^5\\minus{}x^4\\plus{}x^3\\minus{}4x^2\\plus{}2x\\minus{}\\sqrt{3}$\r\n\r\nthx",
"Solution_1": "[quote=\"IW@IT\"]1. Find all values of $ x,y \\in\\mathbb{N}$ that satisfy\n$ (x \\plus{} y) \\plus{} (x \\minus{} y) \\plus{} xy \\plus{} \\frac2{y} \\equal{} 2009$\n[/quote]\r\n\r\n[hide=\"1.\"]\n$ \\because y\\in\\mathbb{N}$ then $ y\\in \\left\\{1,2\\right\\}$\n\nIf $ y\\equal{}1$ then $ 2x\\plus{}x\\plus{}2\\equal{}2009 \\implies x\\equal{}667$\n\nIf $ y\\equal{}2$ then $ 2x\\plus{}2x\\plus{}1\\equal{}2009 \\implies x\\equal{}502$\n\n$ \\therefore (x,y)\\in \\left\\{(667,1),(502,2)\\right\\}$\n[/hide]",
"Solution_2": "[quote=\"IW@IT\"]\n2. Find the value of $ x$ if\n$ x^2 - 2\\sqrt {x - 1 - \\frac1{x} + \\frac1{x^2}} - x + \\frac2{x} = 1$, $ x\\neq 0$\n[/quote]\r\n\r\n[hide=\"2.\"]\n$ x^2 - x + \\frac2{x}-1- 2\\sqrt {\\frac{x^2(x-1)-(x-1)}{x^2}}=0$\n\n$ \\implies \\frac{x^3-x^2-x+2}{x}=\\frac{2(x-1)\\sqrt{x+1}}{x}$\n\n${ \\implies x^3-x^2-x+2=2(x-1)\\sqrt{x+1}}$\n\n${ \\implies (x^3-x^2-x+1)+1=2(x-1)\\sqrt{x+1}}$\n\n${ \\implies (x^2-1)(x-1)+1=2(x-1)\\sqrt{x+1}}$\n\n${ \\implies (x^2-1)(x-1)+1=2(x-1)\\sqrt{x+1}}$\n\n${ \\implies (x-1)^2(x+1)-2(x-1)\\sqrt{x+1}}+1=0$\n\n$ \\implies [(x-1)\\sqrt{x+1}-1]^2=0$\n\n$ \\implies (x-1)\\sqrt{x+1}-1=0$\n\n$ \\implies (x-1)^2(x+1)=1$\n\n$ \\implies x^3-x^2-x=0 \\implies x^2-x-1=0 \\implies x=\\frac{1\\pm\\sqrt{5}}{2}$\n\n[/hide] :P",
"Solution_3": "[quote=\"IW@IT\"]\n3. It is given that $ x = \\frac1{2 - \\sqrt {3}}$. Find the value of\n$ x^6 - 2\\sqrt {3}x^5 - x^4 + x^3 - 4x^2 + 2x - \\sqrt {3}$\n[/quote]\r\n\r\n[hide=\"3.\"]\n$ x = \\frac1{2 - \\sqrt {3}}=\\frac1{2 - \\sqrt {3}}.\\frac{2+\\sqrt3}{2+\\sqrt3}=2+\\sqrt 3$\n\n\n$ x^6 - 2\\sqrt {3}x^5 - x^4 \\\\\n=x^5(x-2\\sqrt {3})-x^4 \\\\\n=x^5(2-\\sqrt 3)-x^4=x^4(2+\\sqrt 3)(2-\\sqrt 3)-x^4=x^4-x^4=0$\n\n$ x^3 - 4x^2 + 2x - \\sqrt {3}\\\\\n=x^2(x-4)+2x-\\sqrt {3}\\\\\n=x^2(2+\\sqrt 3-4)+2x-\\sqrt {3}\\\\\n=x^2(\\sqrt 3-2)+2x-\\sqrt {3}\\\\\n=x(\\sqrt 3+2)(\\sqrt 3-2)+2x-\\sqrt {3}\\\\\n=x(3-4)+2x -\\sqrt {3}\\\\\n=x-\\sqrt {3}\\\\\n=2+\\sqrt 3-\\sqrt {3}=\\boxed{2}$ :P \n\n\n\n[/hide]",
"Solution_4": "[quote=\"IW@IT\"]\n$ x^6 \\minus{} 2\\sqrt {3}x^5 \\minus{} x^4 \\plus{} x^3 \\minus{} 4x^2 \\plus{} 2x \\minus{} \\sqrt {3}$\n[/quote]\r\n\r\nNinja'd by vinskman! ;)\r\n\r\n[hide=\"3\"]\n$ x \\equal{} 2 \\plus{} \\sqrt {3}$\n\nWe have $ x^5(x \\minus{} 2\\sqrt {3}) \\equal{} x^6 \\minus{} 2\\sqrt {3}x^5 \\equal{} x^4$.\n\nTherefore the expression is equivalent to $ x^4 \\minus{} x^4 \\plus{} x^3 \\minus{} 4x^2 \\plus{} 2x \\minus{} \\sqrt {3} \\equal{} x^3 \\minus{} 4x^2 \\plus{} 2x \\minus{} \\sqrt {3}$\n\nThis is not too hard to calculate, it's $ 26 \\plus{} 15\\sqrt {3} \\minus{} 28 \\minus{} 16\\sqrt {3} \\plus{} 4 \\plus{} 2\\sqrt {3} \\minus{} \\sqrt {3} \\equal{} 2$. \n[/hide]",
"Solution_5": "[quote=\"ffao\"][quote=\"IW@IT\"]\n$ x^6 \\minus{} 2\\sqrt {3}x^5 \\minus{} x^4 \\plus{} x^3 \\minus{} 4x^2 \\plus{} 2x \\minus{} \\sqrt {3}$\n[/quote]\n\nNinja'd by vinskman! ;)\n\n[hide=\"3\"]\n$ x \\equal{} 2 \\plus{} \\sqrt {3}$\n\nWe have $ x^5(x \\minus{} 2\\sqrt {3}) \\equal{} x^6 \\minus{} 2\\sqrt {3}x^5 \\equal{} x^4$.\n\nTherefore the expression is equivalent to $ x^4 \\minus{} x^4 \\plus{} x^3 \\minus{} 4x^2 \\plus{} 2x \\minus{} \\sqrt {3} \\equal{} x^3 \\minus{} 4x^2 \\plus{} 2x \\minus{} \\sqrt {3}$\n\nThis is not too hard to calculate, it's $ 26 \\plus{} 15\\sqrt {3} \\minus{} 28 \\minus{} 16\\sqrt {3} \\plus{} 4 \\plus{} 2\\sqrt {3} \\minus{} \\sqrt {3} \\equal{} 2$. \n[/hide][/quote]\r\n\r\nHello , my guy :lol: what means \"Ninja'd by vinskman\"? would you tell me?",
"Solution_6": "It means that you posted exactly what I would post, right before I posted.",
"Solution_7": "[quote=\"ffao\"]It means that you posted exactly what I would post, right before I posted.[/quote]\r\n\r\n\r\nThanks.\r\n\r\nNo worries.You are welcome. :D :P",
"Solution_8": "[quote=\"ffao\"]\nTherefore the expression is equivalent to $ x^4 \\minus{} x^4 \\plus{} x^3 \\minus{} 4x^2 \\plus{} 2x \\minus{} \\sqrt {3} \\equal{} x^3 \\minus{} 4x^2 \\plus{} 2x \\minus{} \\sqrt {3}$\n[/quote]\r\n[hide]$ 2\\plus{}\\sqrt{3}$ is a root of $ x^2\\minus{}4x\\plus{}1\\equal{}0$ so $ x^3 \\minus{} 4x^2 \\plus{} 2x \\minus{} \\sqrt {3}\\equal{}x(x^2\\minus{}4x\\plus{}1)\\plus{}x\\minus{}\\sqrt{3}\\equal{}x\\minus{}\\sqrt{3}\\equal{}2$[/hide]"
}
{
"Tag": [
"number theory",
"prime numbers",
"number theory unsolved"
],
"Problem": "Find all prime numbers $ p$ such that $ 1\\plus{}p\\plus{}p^2\\plus{}...\\plus{}p^{p\\minus{}1}$ and $ 1\\minus{}p\\plus{}p^2\\minus{}p^3\\plus{}....\\plus{}p^{p\\minus{}1}$ are prime numbers",
"Solution_1": "http://www.mathlinks.ro/viewtopic.php?t=229499"
}
{
"Tag": [
"geometry",
"incenter",
"circumcircle",
"Euler",
"geometric transformation",
"reflection"
],
"Problem": "Dear Mathlinkers,\r\nlet ABC be a triangle,\r\n1 the incircle , I the incenter, X the pointy of contact of 1 with BC, X' the antipode of X wrt 1, A\"B\"C\" the I-circumtriangle, U the point of intersection of B\"C\" and AI, and Fe the Feuerbach' s point of ABC.\r\nProve that Fe, U and X' are collinear.\r\nSincerely\r\nJean-Louis",
"Solution_1": "Dear Jean-Louis\r\nWhat is \"$ I$-circumtriangle\"?",
"Solution_2": "Dear Zhang Fangyu,\r\nthe I-curcumtriangle is the triangle forwicg the vertex are the second intersection od the cevians AI, BI, CI with the circumcircle of ABC.\r\nSincerely\r\nJean-Louis",
"Solution_3": "A1,A2 are the projections of A onto BI,CI\r\n\u25b3AA1A2\u223d\u25b3ABC A1A2\u2225BC \r\nFrom here we (http://www.mathlinks.ro/viewtopic.php?t=219598) we know X\u2019Fe is the Euler Line of \u25b3AA1A2 \r\nSo now our goal is to prove U is on Euler Line of \u25b3AA1A2 \r\nNote that AA\u2019\u2019\u22a5B\u2019\u2019C\u2019\u2019 => U,A,C\u2019\u2019,A2 are concyclic\r\n\u2220UAA2=UA2A=\u2220CC\u2019\u2019B\u2019\u2019=\u2220AC\u2019\u2019B\u2019\u2019=1/2\u2220B\r\nWe have \u25b3AA2I is a right triangle , thus U is midpoint of IA => U is the circumcenter of AA2IA1 => U lies on the Euler Line of \u25b3AA1A2 => X\u2019, U , Fe are collinear[/url]",
"Solution_4": "[quote]A1,A2 are the projections of A onto BI,CI\n\u25b3AA1A2\u223d\u25b3ABC A1A2\u2225BC\nFrom here we (http://www.mathlinks.ro/viewtopic.php?t=219598) we know X\u2019Fe is the Euler Line of \u25b3AA1A2\nSo now our goal is to prove U is on Euler Line of \u25b3AA1A2\nNote that AA\u2019\u2019\u22a5B\u2019\u2019C\u2019\u2019 => U,A,C\u2019\u2019,A2 are concyclic\n\u2220UAA2=UA2A=\u2220CC\u2019\u2019B\u2019\u2019=\u2220AC\u2019\u2019B\u2019\u2019=1/2\u2220B\nWe have \u25b3AA2I is a right triangle , thus U is midpoint of IA => U is the circumcenter of AA2IA1 => U lies on the Euler Line of \u25b3AA1A2 => X\u2019, U , Fe are collinear.[/quote]\r\n\r\nIndeed plane geometry, the ideas from previous messages keep coming back. \r\nYour short and good solution is even still a bit too long, because $ U$ can be immediately proven to be the desired circumcenter:\r\n\r\nSince $ I$ is the orthocenter of $ \\Delta A''B''C''$, it follows that $ U$ is the midpoint of $ A$ and $ I$, and hence the circumcenter of $ \\Delta AA_1A_2$. Since $ X'$ is on the Euler line of this triangle, the problem now follows from Hatzipolakis' theorem.\r\n\r\nBut anyway, your solution is good. I wonder if we can avoid all those overkill far-fetched theorems and find an independent olympiad solution. I bet we can use Monge-d'Alembert with some additional arguments or something. I don't have time right now to investigate further.",
"Solution_5": "Could anyone show me why X\u2019Fe is the Euler Line of \u25b3AA1A2 :) \r\nI searched http://www.mathlinks.ro/viewtopic.php?t=219598 but I didn't find a specific proof\r\nThank you very much!",
"Solution_6": "Dear Mathlinkers,\r\nsee for example :\r\nVonk J., The Feuerbach Point and Reflections of the Euler Line, Forum Geometricorum, Volume 9 (2009) 47\u201355 ;\r\nhttp://forumgeom.fau.edu/FG2009volume9/FG200905.pdf\r\nSincerely\r\nJean-Louis",
"Solution_7": "Thank you so much :lol:",
"Solution_8": "Dear jayme\uff0cyour problem actually is that $ Fe$ is on the $ Line\\minus{}OH$ what it the Euler-line of \u0394$ DPQ$ .\r\nBut your description prompt another way to prove the Problem.",
"Solution_9": "Dear Lym and Mathlinkers,\r\nif I am not wrong, ithink that this way concerns another problem post on Mathlinks recently.\r\nYour observation is nice\r\nSincerely\r\nJean-Louis",
"Solution_10": "jayme,I read the proof in your excellent article :) But I find that it used a tough property of $ X_{80}$\r\nIt states that $ X_{80}$ is the reflection of incenter wrt feuerbach's.I spent a whole afternoon to solve it,and I create a solution using inversion and a bit long calculation(if necessary I will post here).Do you have or know a syntheic proof for it?I am eager to see it :)",
"Solution_11": "Dear Mathlinkers,\r\nit is not my proof but these of Jan Vonk.\r\nSincerely\r\nJean-Louis",
"Solution_12": "Can anyone provide a synthetic proof :) \r\nMy solution is very long but I like it very much.",
"Solution_13": "Dear Mathlinkers,\r\nfor more explanation, see for example\r\nhttp://perso.orange.fr/jl.ayme vol. 4 Symetriques de OI par rapport aux triangles de contact et m\u00e9dian p. 21\r\nSincerely\r\nJean-Louis"
}
{
"Tag": [],
"Problem": "Prove:\r\n$\\frac{1989}{2}-\\frac{1988}{3}+\\frac{1987}{4}-...+\\frac{1}{1990}= \\frac{1}{996}+\\frac{3}{997}+\\frac{5}{998}+...+\\frac{1989}{1990}$",
"Solution_1": "Using a nice identity of the harmonic numbers:\r\n\r\n[hide=\"Solution\"]\n$h_{n}= \\sum_{i = 1}^{n}\\frac1i$\n$\\frac12 h_{n}= \\sum_{i = 1}^{n}\\frac1{2i}$\n$h_{2n}-\\frac12 h_{n}= \\sum_{i = 1}^{n}\\frac1{2i-1}$\n$h_{2n}-h_{n}=-\\sum_{i = 1}^{2n}\\frac{(-1)^{n}}{i}$\n\n$\\sum_{i = 2}^{2n}(-1)^{i}\\frac{2n-i+1}{i}=-1+\\sum_{i = 2}^{2n}(-1)^{i}\\frac{2n+1}{i}=-1+(2n+1)(h_{n}-h_{2n}+1)$\n\n$\\sum_{i =1}^{n}\\frac{2i-1}{n+i}= \\sum_{i =1}^{n}2-\\frac{2n+1}{n+i}= 2n-(2n+1)(h_{2n}-h_{n})$\n[/hide]"
}
{
"Tag": [
"parameterization",
"function",
"calculus",
"derivative",
"conics",
"ellipse",
"limit"
],
"Problem": "For which real $p$ has system of equations:\r\n$(x-y)^2+z^2=(z-y)^2+x^2=p^2$\r\n$xyz=p^3$\r\nat least two solutions $(x,y,z)$?",
"Solution_1": "Subtracting the first 2 equations immediately yields $y(z - x) = 0$. But if $y = 0$, then from the 2nd equation $p = 0$ and substituting this to the first 2 equations leads to a single solution $x = y = z = 0$. Hence, it must be $z = x$ and we actually have only 2 equations:\r\n\r\n$2x^2 - 2xy + y^2 = p^2$\r\n$yx^2 = p^3$\r\n\r\nObviously, this set of 2 equations always has the solution $x = y = p$ for any $p \\neq 0$. The second equation represents the function $y = \\frac {p^3}{x^2}$, which has derivative at $x = p$ equal to\r\n\r\n$y' = -2\\frac{p^3}{x^3}, \\ y'(p) = -2$\r\n\r\nThe first equation represents an ellipse centered at the origin $[0, 0]$ (because the equation is symmetrical WRT the exchange $[x, y] \\leftrightarrow [-x, -y]$) and passing through the points $[-p, -p], [0, -p], [0, p], [p, p]$. Solving the ellipse equation for $y$ and differentiating:\r\n \r\n$y = x \\pm \\sqrt{p^2 - x^2}$\r\n$y' = 1 \\mp \\frac {x}\\sqrt{p^2 - x^2}$ $, \\ \\lim_{x\\to \\pm p}{y'} = \\infty$\r\n\r\nHence, this ellipse has vertical tangents at the points $[-p, -p], [p, p]$. This is sufficient to conclude that the ellipse intersects the graph of the function $y = \\frac {p^3}{x^2}$ at exactly one more point different from the point $[p, p]$ for any $p \\neq 0$.\r\n\r\nEdit:\r\n$x = y = -p$ is not a solution, error fixed.\r\n$p\\ \\#\\ 0$ replaced by $p \\neq 0$.\r\nThanks.\r\n\r\n...",
"Solution_2": "correct me if i'm wrong, but it seems to be that $x=y=z=-p$ is not a solution?",
"Solution_3": "You are right, I will edit out the error. Tx.",
"Solution_4": "yetti you can use \\neq (not equal) command to make $p \\neq 0$ if you want ;)"
}
{
"Tag": [
"inequalities",
"inequalities proposed"
],
"Problem": "(***)\r\n59. Let $ x,y$ be distinct positive reals. Determine the minimum value of\r\n\\[ \\frac {1}{(x \\minus{} y)^2} \\plus{} \\frac {16}{xy} \\plus{} 6(x \\plus{} y)\r\n\\]\r\nand determine when that minimum occurs.\r\n\r\n[hide=\"Comment:\"] A proof exists with only Cauchy and AM-GM.[/hide]",
"Solution_1": "[quote=\"Altheman\"](***)\n59. Let $ x,y$ be distinct positive reals. Determine the minimum value of\n\\[ \\frac {1}{(x \\minus{} y)^2} \\plus{} \\frac {16}{xy} \\plus{} 6(x \\plus{} y)\n\\]\nand determine when that minimum occurs.\n\n[hide=\"Comment:\"] A proof exists with only Cauchy and AM-GM.[/hide][/quote]\r\nLet $ x \\ge y$ then : $ LHS \\ge (\\frac {1}{(x \\minus{} y)^2}\\plus{}(x\\minus{}y)\\plus{}(x\\minus{}y))\\plus{}(4x\\plus{}8y\\plus{}\\frac {16}{xy} ) \\ge (3)\\plus{}(24)\\equal{}27$;( by AM-GM )\r\nequality is for $ x\\equal{}2,y\\equal{}1;$ :)",
"Solution_2": "Hey, I like your solution, it is very nice!!\r\n\r\nHere is my solution:\r\n\r\n$ \\frac{1}{(x\\minus{}y)^2}\\plus{}\\frac{64}{4xy}\\plus{}6(x\\plus{}y)\\ge \\frac{(1\\plus{}8)^2}{(x\\minus{}y)^2 \\plus{} 4xy}\\plus{}6(x\\plus{}y)$\r\n\r\n$ \\equal{}\\frac{81}{(x\\plus{}y)^2}\\plus{}3(x\\plus{}y)\\plus{}3(x\\plus{}y)\\ge 3\\sqrt[3]{81*3*3}\\equal{}27$.\r\n\r\nEquality occurs iff $ \\frac{1}{(x\\minus{}y)^2}\\equal{}\\frac{8}{4xy}$ and $ \\frac{81}{(x\\plus{}y)^2}\\equal{}3(x\\plus{}y)$. The first one factors as $ (x\\minus{}2y)(2x\\minus{}y)\\equal{}0$. The second equation is $ x\\plus{}y\\equal{}3$. So $ (x,y)\\equal{}(1,2),(2,1)$."
}
{
"Tag": [],
"Problem": "What is the maximum number of bishops that can be placed on any $8\\times 8$ chessboard such that at most three bishops lie on any diagonal?\r\n\r\nHere are my thoughts. Can they be made more rigorous? \r\n[hide=\"Approach\"]Consider all white squares. There are two 1 square diagonals, two 2 square diagonals, ... , and two 7 square diagonals. There is also one 8 square diagonal, for a total of 15 diagonals.\n\nEach bishop is contained in exactly two diagonals, and each diagonal (except the 1 and 2 squares) can contain 3 bishops.\n\nHence, the bishops can theoretically occupy $3(15)-4-2=39$ spots on these 15 diagonals (subtract 4 and 2 since there are two diagonals of length 1 and two of length 2), where one bishop occupies \"two spots\" (one on each diagonal). However, since one bishop occupies two spots at a time, there can only be $38$ occupied, so $19$ bishops on white squares. \n\nSimilarly, $19$ bishops on black squares, for a total of $38.$\n[/hide]\nIf this method works, I believe this generalization will folllow\n[hide=\"Generalization?\"]For each color, $3(2n-1)-4-2$ spots. Only $6n-10$ can be used, so $3n-5$ bishops on each color.\n\nIn total $6n-10$ bishops.[/hide]\r\nNot sure how to show that the maximum value is attainable though.",
"Solution_1": "aren't you 4everwise?"
}
{
"Tag": [
"ratio",
"vector",
"geometry",
"trapezoid"
],
"Problem": "Prove that two medians in a triangle are congruent iff the sides which they bisect are congruent.",
"Solution_1": "[hide]I don't have a diagram, but...\n\nSince two sides of a triangle are congruent, the triangle, say ABC, is isosceles, and the medians are coming from the 2 base angles B and C. The medians intersect sides AB and AC at M and N, respectively.\n\nSince $ AB \\equal{} AC$, $ MB \\equal{} NC$, and since $ \\angle B \\equal{} \\angle C$ and $ BC \\equal{} BC$, $ \\triangle BMC\\cong\\triangle CNB$.\nThus, $ CM \\equal{} BN$\n\nNow, for the \"only if\" part:\n\nConsider triangle ABC, with medians CM and BN, with CM=BN. Denote the intersection of CM and BN as P. Because the intersection of the medians divide the medians into a ratio of 2:1, and since CM=BN, PM=PN and PB=PC. Since $ \\angle MPB \\equal{} \\angle NPC$, $ \\triangle MPB\\cong\\triangle NPC$.\n\nThus, $ \\angle ABN \\equal{} \\angle ACM$. Since $ \\angle A \\equal{} \\angle A$, and $ BN \\equal{} MC$, $ \\triangle AMC\\cong\\triangle ANB$, and thus AB=AC.\n\nQED[/hide]\r\n\r\nEDIT: Yeah, didn't notice it sad \"iff\", but added it now.",
"Solution_2": "What about the other direction?",
"Solution_3": "Let $ A$ be the origin, and let $ 2y$, $ 2z$ be vectors that denote points $ B$ and $ C$, respectively. Let $ N$ and $ P$ be the midpoints of $ AC$ and $ AB$, respectively. Then \\[ BN\\equal{}CP\\iff BN^2\\equal{}CP^2\\iff (2y\\minus{}z)\\cdot (2y\\minus{}z)\\equal{}(2z\\minus{}y)\\cdot (2z\\minus{}y)\\]\r\n\\[ \\iff 2y\\cdot 2y\\minus{}4y\\cdot z\\plus{}z\\cdot z\\equal{}2z\\cdot 2z\\minus{}4z\\cdot y\\plus{}y\\cdot y\\equal{}y\\cdot y\\equal{}z\\cdot z\\iff AB\\equal{}AC\\]\r\n\r\n\r\nyay",
"Solution_4": "Let $ AA'$ and $ BB'$ be two medians in the triangle $ ABC$\r\n$ AA'\\equal{}BB'$,$ AG\\equal{}BG$, $ AB'A'B$ is a trapezoid therefore $ AB'\\equal{}BA'$ $ \\implies$ $ BC\\equal{}AC$",
"Solution_5": "[hide] Let ABC be an isosceles triangle with AB=AC\n\nand BD and CE are the medians\n\nclearly AD=AE and \\equal{} 0$ to satisfy the equation(since $ 0 \\equal{} < {\\theta} \\equal{} < pi$) and $ cos{\\theta} \\equal{} 0$ is not a solution to this equation!\r\nSo, we just need to care about $ cos{\\theta} > 0$\r\nNow, dividing both sides by $ cos{\\theta}$ we get: $ \\frac {1}{cos^2{\\theta}} \\equal{} 2004(tan{\\theta} \\minus{} 1)$.\r\nThis equation is equivalent to: $ 1 \\plus{} tan^2{\\theta} \\equal{} 2004tan{\\theta} \\minus{} 2004$. \r\nThis is the quadratic equation, so we easily get: $ tan{\\theta} \\equal{} 2005$ or $ tan{\\theta} \\equal{} \\minus{} 1$. Since $ sin{\\theta}$ and $ cos{\\theta} > 0$, we eliminate all negative values of $ tan{\\theta}$. Finally, the sum of $ tan{\\theta} \\equal{} 2005$(Q.E.D)\r\nHmm... The 2nd one seems to be tougher than this one even though it looks familiar to me.I'll post the solution to it later.",
"Solution_3": "whoops, forgot to post :huh: \r\n\r\nghjk: In the first question, $ 0 \\le \\theta < \\pi$, so $ \\tan \\theta$ can take any real. :wink: \r\n[hide=\"1\"]\nDividing both sides by $ \\cos^2 \\theta$ (clearly $ \\cos \\theta \\neq 0$), we have $ \\sec^2 \\theta = 1 + \\tan^2 \\theta = 2004 \\tan \\theta - 2004$, and by vieta's [each value occurs once] the sum is $ 2004$. \n[/hide]\n[hide=\"2\"]\nRepeatedly applying the product to sum formula yields\n\\begin{align*}8\\cos \\alpha \\cos 3\\alpha \\cos 5\\alpha \\cos 7\\alpha & = 2 \\left[\\cos 8 \\alpha + \\cos 2\\alpha\\right]\\left[\\cos 8\\alpha + \\cos 6 \\alpha \\right] \\\\\n& = 2\\left[\\cos^2 8\\alpha + \\cos 8\\alpha \\cos 6\\alpha + \\cos 8\\alpha \\cos 2\\alpha + \\cos 6\\alpha \\cos 2\\alpha\\right] \\\\\n& = \\left[\\cos 2\\alpha + \\cos 4\\alpha + \\cdots + \\cos 14\\alpha \\right] - \\cos 12\\alpha.\\end{align*}\nThus, we have $ \\frac {1}{2} = \\sum_{n = 1}^{7} \\cos 2n\\alpha = \\text{Re}\\left(\\sum_{n = 1}^7 e^{i2n\\alpha}\\right) = \\text{Re}\\left(\\frac {e^{i16 \\alpha} - 1}{e^{i2\\alpha} - 1}\\right) = \\frac {\\sin 7\\alpha \\cos 8\\alpha}{\\sin \\alpha}$. Then $ \\sin \\alpha = 2\\sin 7\\alpha \\cos 8\\alpha \\cdot \\frac {\\sin 8\\alpha}{\\sin 8\\alpha} = \\frac {\\sin 7 \\alpha \\sin 16\\alpha}{\\sin 8 \\alpha}$ $ \\Longrightarrow 2\\cos 9\\alpha = \\cos 23\\alpha + \\cos 7\\alpha$ $ \\Longrightarrow \\cos 9\\alpha = \\cos 15\\alpha \\cos 8 \\alpha$ .. shouldn't be too much harder from here ..\n[/hide]",
"Solution_4": "[quote=\"azjps\"]whoops, forgot to post :huh: \n\nghjk: In the first question, $ 0 \\le \\theta < \\pi$, so $ \\tan \\theta$ can take any real. :wink: \n[/quote]\r\nHmm... Did you read my argument above? I already showed that $ cos{\\theta}>0$(I leave its proof for you to think of it first!). Therefore, $ tan{\\theta}>0$ with $ 0 \\le\\theta <\\pi$ :)",
"Solution_5": "Edit: Actually, I'm right :lol: its $ \\tan^2 \\theta \\minus{} 2004\\tan \\theta \\plus{} 2005 \\equal{} 0$, the roots are both positive.",
"Solution_6": "[quote=\"azjps\"]Edit: Actually, I'm right :lol: its $ \\tan^2 \\theta \\minus{} 2004\\tan \\theta \\plus{} 2005 \\equal{} 0$, the roots are both positive.[/quote]\r\nMy equation is right, but I made a dumb factorization. :o Since the quadratic equation gives two positive roots for tan, the sum is indeed 2004(you're lucky since it gives both positive roots, aren't u?) :oops:",
"Solution_7": "[quote=\"ghjk\"]Beautiful problem! Where did you get this problem, my friend??\nI hope you like my solution.[/quote]\r\n\r\n :D I really like your solution!",
"Solution_8": "[quote=\"azjps\"]\n[hide=\"2\"]\nRepeatedly applying the product to sum formula yields\n\\begin{align*}8\\cos \\alpha \\cos 3\\alpha \\cos 5\\alpha \\cos 7\\alpha & = 2 \\left[\\cos 8 \\alpha + \\cos 2\\alpha\\right]\\left[\\cos 8\\alpha + \\cos 6 \\alpha \\right] \\\\\n& = 2\\left[\\cos^2 8\\alpha + \\cos 8\\alpha \\cos 6\\alpha + \\cos 8\\alpha \\cos 2\\alpha + \\cos 6\\alpha \\cos 2\\alpha\\right] \\\\\n& = \\left[\\cos 2\\alpha + \\cos 4\\alpha + \\cdots + \\cos 14\\alpha \\right] - \\cos 12\\alpha.\\end{align*}\nThus, we have $ \\frac {1}{2} = \\sum_{n = 1}^{7} \\cos 2n\\alpha = \\text{Re}\\left(\\sum_{n = 1}^7 e^{i2n\\alpha}\\right) = \\text{Re}\\left(\\frac {e^{i16 \\alpha} - 1}{e^{i2\\alpha} - 1}\\right) = \\frac {\\sin 7\\alpha \\cos 8\\alpha}{\\sin \\alpha}$. Then $ \\sin \\alpha = 2\\sin 7\\alpha \\cos 8\\alpha \\cdot \\frac {\\sin 8\\alpha}{\\sin 8\\alpha} = \\frac {\\sin 7 \\alpha \\sin 16\\alpha}{\\sin 8 \\alpha}$ $ \\Longrightarrow 2\\cos 9\\alpha = \\cos 23\\alpha + \\cos 7\\alpha$ $ \\Longrightarrow \\cos 9\\alpha = \\cos 15\\alpha \\cos 8 \\alpha$ .. shouldn't be too much harder from here ..\n[/hide][/quote]\r\n\r\n\r\nIs there a better solution for this?",
"Solution_9": "Lets solve the system $ a \\equal{} x(y \\minus{} x)$ $ x^2 \\plus{} y^2 \\equal{} 1$ where $ a$ is given.\r\n\r\nNaturally, we homogenize in $ x,y$ to get $ a(x^2 \\plus{} y^2) \\equal{} x (y \\minus{} x)$ and we easily solve for $ x: y$ by the quadratic formula and we are done.\r\n\r\n\r\nThis motivates the other solution that was given...\r\n\r\n\r\nThis works for any system in the form $ A \\equal{} (Bx \\plus{} Cy)(Dx \\plus{} Ey)$ and $ x^2 \\plus{} y^2 \\equal{} 1$...",
"Solution_10": "[quote=\"Altheman\"]Lets solve the system $ a \\equal{} x(y \\minus{} x)$ $ x^2 \\plus{} y^2 \\equal{} 1$ where $ a$ is given.\n\nNaturally, we homogenize in $ x,y$ to get $ a(x^2 \\plus{} y^2) \\equal{} x (y \\minus{} x)$ and we easily solve for $ x: y$ by the quadratic formula and we are done.\n\n\nThis motivates the other solution that was given...\n\n\nThis works for any system in the form $ A \\equal{} (Bx \\plus{} Cy)(Dx \\plus{} Ey)$ and $ x^2 \\plus{} y^2 \\equal{} 1$...[/quote]\r\n\r\n\r\nWhat in the world does this have to do with trig?",
"Solution_11": "He was just explaining the motivation for the solution to #1, namely homogenizing. (there should be a comma between $ a \\equal{} x(y\\minus{}x)$ and $ x^2 \\plus{} y^2 \\equal{} 1$, and now look at the substitution $ x \\equal{} \\cos \\theta, y \\equal{} \\sin \\theta$)."
}
{
"Tag": [
"LaTeX",
"advanced fields",
"advanced fields theorems"
],
"Problem": "What is it? \r\nHow to prove things with it?\r\nFor example, how to proof that (in this method)\r\nA -> B |-- (C V A) -> (C V B)\r\n\r\n(I can prove it with true-table only...)\r\n\r\nThank you",
"Solution_1": "Please $ \\text{\\LaTeX}$ your formula. Is it $ (A \\rightarrow B) \\wedge (C \\vee A) \\rightarrow C \\vee B$ ?",
"Solution_2": "I believe it's: $ A \\Rightarrow B \\vdash (C \\vee A) \\Rightarrow (C \\vee B)$\r\n\r\nThe symbol \"$ p, q, r, \\ldots \\vdash c$\" means \"given $ p, q, r, \\ldots$, prove $ c$\".",
"Solution_3": "$ A \\Rightarrow B \\vdash (C \\vee A) \\Rightarrow (C \\vee B)$ Yes. \r\n\r\n(I am sorry, I relatively new user and I am not so good in Latex)"
}
{
"Tag": [
"group theory",
"abstract algebra",
"geometry",
"geometric transformation",
"rotation",
"superior algebra",
"superior algebra solved"
],
"Problem": "Without any computation show $S_{4}$ has a subgroup isomorphic to $Z_{4}$ and a subgroup isomorphic to $K_{4}$ (Klein four group). Extend this for $S_{n}.$",
"Solution_1": "$S_{n}$ always contains an $n$-cycle $(1,2,3,...,n)$ clearly having order $n$.\r\nAnd $K_{4}$ is the group of permuations of two stacks of two elements without perturbation, thus also contained.\r\n$S_{n}$ also always contains the dihedral group $D_{n}$ since it is a group of permutations on $n$ objects. By this, the rotations give another way of getting an element of order $n$.",
"Solution_2": "[quote=\"ZetaX\"]$S_{n}$ always contains an $n$-cycle $(1,2,3,...,n)$ clearly having order $n$.\nAnd $K_{4}$ is the group of permuations of two stacks of two elements without perturbation, thus also contained.\n$S_{n}$ also always contains the dihedral group $D_{n}$ since it is a group of permutations on $n$ objects. By this, the rotations give another way of getting an element of order $n$.[/quote]\r\n\r\nYou are right, but note that I said that without any computation!! What you say about this problem:\r\n\r\nWithout any computation show $S_{8}$ has subgroups isomorphic to $Z_{8}, Z_{2}\\times Z_{4},Z_{2}\\times Z_{2}\\times Z_{2}, Q$ and $D$ where $Q$ is the quaternion group of order 8 and $D$ dihedral group of order 8.",
"Solution_3": "Where did I use computations (even for the cycle I gave a pure combi-algebraical proof).\r\nAnd the generalisation is simply that $S_{n}$ contains every group that is a permutation group on $\\leq n$ element set, so especially contains any group of order $\\leq n$ (Cayley's theorem).",
"Solution_4": "Ok, this is my solution exactly.\r\n\r\nThanks"
}
{
"Tag": [],
"Problem": "Jest dany trojkat $ABC$ z $\\angle ABC = 3\\angle CAB$. Punkty $M$ i $N$ sa wybrane na boku $AC$ z $N$ pomiedzy $A$ i $M$ tak, ze $\\angle CBM =\\angle MBN = \\angle NBA$. Niech $L$ bedzie dowolnym punktem wewnetrznym odcinka $BN$, zas $K$ punktem na $BM$ takim, ze $LK || AC$. Udowodnij, ze proste $AL$,$NK$ i $BC$ przecinaja sie w jednym punkcie.\r\n\r\nP.S. reanimacja trwa :P",
"Solution_1": "Najwyzszy czas podac rozwiazanie :P Niech $ NK\\cap BC\\equal{}P$, $ LK\\cap AC\\equal{}Q$ oraz $ LK\\cap AB\\equal{}R$. Z tw. Menelausa mamy $ \\frac{BP\\cdot QK\\cdot CN}{PQ\\cdot KL\\cdot BN}\\equal{}1\\iff \\frac{BP\\cdot BC\\cdot QC}{PQ\\cdot BN\\cdot BC}\\equal{}1\\iff \\frac{BP}{PQ}\\equal{}\\frac{BN}{QC}$. Teraz mamy $ \\frac{BN\\cdot QL\\cdot AR}{PQ\\cdot RL\\cdot AB}\\equal{}\\frac{BN\\cdot CN\\cdot CQ}{CQ\\cdot AN\\cdot BL}\\equal{}1$ qed",
"Solution_2": "Ca\u0142kiem nie\u017ale: po blisko 3 latach rozwi\u0105zanie (nie sprawdza\u0142em czy poprawne...) \r\n\r\n :)",
"Solution_3": "Sa literowki. Ma byc\r\n$ LK\\cap BC \\equal{} Q$ i w ostatniej rownosci$ \\frac{BP\\cdot QL\\cdot AR}{PQ\\cdot RL\\cdot AB}\\equal{}\\frac{BN\\cdot CN\\cdot CQ}{CQ\\cdot AN\\cdot BC}\\equal{}1$"
}
{
"Tag": [
"algebra",
"polynomial",
"calculus",
"derivative",
"induction",
"algebra proposed"
],
"Problem": "If $b_{1}, b_{2}, \\ldots, b_{n}$ are non-negative reals not all zero, then prove that the polynomial \\[x^{n}-b_{1}x^{n-1}-b_{2}x^{n-2}-\\ldots-b_{n}=0\\] has only one positive root $p$, which is simple. Moreover prove that any root of the polynomial does not exceed $p$ in absolute value.",
"Solution_1": "[quote=\"iura\"]If $b_{1}, b_{2}, \\ldots, b_{n}$ are non-negative reals not all zero, then prove that the polynomial\n\\[x^{n}-b_{1}x^{n-1}-b_{2}x^{n-2}-\\ldots-b_{n}=0 \\]\nhas only one positive root $p$, which is simple. [/quote]\nIt is obviosly, consider \n(*) $1-b_{1}y-b_{2}y^{2}-...-b_{n}y^{n}, \\ y=1/x$.\n[quote]Moreover prove that any root of the polynomial does not exceed $p$ in absolute value.[/quote]\r\nIt followed from (*).",
"Solution_2": "Part 1: Use Descartes' Sign Rule .\r\nPart 2: If $x_{0}$ is a real root, then $|x_{0}|^{n}\\leq b_{1}|x_{0}|^{n-1}+...+b_{n}$. And we're done.",
"Solution_3": "In book \"Polynomials\" by Prasolov, this is stated as a theorem (Cauchy's)",
"Solution_4": "[quote=\"Goblin\"]In book \"Polynomials\" by Prasolov, this is stated as a theorem (Cauchy's)[/quote]\r\nYes, theorem 1.3, page 11. But prove in that not as of mine :P",
"Solution_5": "can you please post an entire solution?",
"Solution_6": "[quote=\"iura\"]If $b_{1}, b_{2}, \\ldots, b_{n}$ are non-negative reals not all zero, then prove that the polynomial\n\\[f(x)=x^{n}-b_{1}x^{n-1}-b_{2}x^{n-2}-\\ldots-b_{n}=0 \\]\nhas only one positive root $p$, which is simple. Moreover prove that any root of the polynomial does not exceed $p$ in absolute value.[/quote]\r\nThis is prove in that book.\r\n\r\nPut $F(x)=-\\frac{f(x)}{x^{n}}=\\frac{b_{1}}{x}+\\frac{b_{2}}{x^{2}}+...+\\frac{b_{n}}{x^{n}}-1.$ If $x\\not = 0$ then $f(x)=0$ is equivalent to $F(x)=0$. Now if $x\\to+\\infty$ then $F(x)\\to-1$ and if $x\\to 0^{+}$ then $F(x)\\to+\\infty$, by $F$ is continuous and monotone on $(0,+\\infty)$ we have $f(x)$ has only a positive root, denote it by $p$. We also have $-\\frac{f'(p)}{p^{n}}=F'(p)=-\\frac{b_{1}}{p^{2}}-...-\\frac{nb_{n}}{p^{n+1}}<0$, so $p$ is simple.\r\n\r\nAssume that $x_{0}$ is a root of $f$. We have $q=|x_{0}|\\leq p$. In fact, if $q>p$ we have $F(q)<0$ and so $f(q)>0$. But from $x_{0}^{n}=b_{1}x_{0}^{n-1}+...+b_{n}$ we have $q^{n}\\leq b_{1}q^{n-1}+...+b_{n}$ or $f(q)\\leq 0$. Contradiction!",
"Solution_7": "You could use derivatives from the beginning:\r\nUse induction on $n$. If $f$ is the polynomial in the problem, then $f^\\prime$ satisfies more or less the induction hypothesis (you should watch out for the special case $x^{n}-\\alpha$). Let $x_{0}$ be the positive root of $f^\\prime$. Then, we have that $f$ is strictly decreasing on $[0,x_{0}]$ and strictly increasing on $\\left[ x_{0}, \\infty \\right)$. But $f(0) \\leq 0$, so $f$ has a unique positive root.",
"Solution_8": "You must remember that we need prove $p$ is SIMPLE!",
"Solution_9": "I thought that was clear. We have that $f \\left( x_{0}\\right) < 0$, so the positive root $u$ of $f$ must satisfy $u > x_{0}\\, \\Longrightarrow \\, f^\\prime (u) > 0$."
}
{
"Tag": [],
"Problem": "I cant seem to comprehend this type of questions. Thorough explanation would help me a lot.\r\n\r\n2. A flare is dropped from a plane flying over level ground at a velocity of $70 m/s$ in the\r\nhorizontal direction. At the instant the flare is released, the plane begins to accelerate horizontally\r\nat $0.75 m/s^{2}$. The flare takes $4.0 s$ to reach the ground. Assume air resistance is negligible.\r\nRelative to a spot directly under the flare at release, where does the flare land?",
"Solution_1": "I don't see why the acceleration of the plane is important... The time the flare needs to reach the ground is $t$, so the distance it covers in the horizontal direction is $vt$, since there are no forces acting on it in that direction. That should be the answer.",
"Solution_2": "thanks, I appreciate the help. :)"
}
{
"Tag": [],
"Problem": "find the last two digits of\r\n$2^{2004},3^{2004},4^{2004}$\r\nhow would you find the first two digits of the same numbers",
"Solution_1": "the last two digits is basically:\r\n\r\n$\\varphi(100)=(2-1)\\cdot2^{1}(5-1)\\cdot5^{1}=40$\r\n\r\n$2^{2004}\\bmod100\\equiv2^{4}\\bmod100=16$\r\n$3^{2004}\\bmod100\\equiv3^{4}\\bmod100=81$\r\n$(2^{2004})^{2}\\bmod100\\equiv(2^{4})^{2}\\bmod100=56$\r\n\r\nthx..",
"Solution_2": "SM4RT, Euler's totient theorem is only valid when the base is relatively prime to the modulus.",
"Solution_3": "But it gets the correct answer for each problem, lol. \r\n\r\n[hide=\"$2^{2004}$\"]\nWe have that\n\\[16^{ \\varphi(25)}=16^{20}\\equiv 1 \\bmod 25 \\ \\ \\wedge \\ \\ 16^{20}\\equiv 0 \\bmod 4 \\stackrel{\\text{CRT}}{\\implies }16^{20}\\equiv 76 \\bmod 100 \\]\nso\n\\[2^{2004}=16^{501}\\equiv 16 \\cdot (76)^{25}\\bmod 100 \\]\nBut, notice that\n\\[76 \\bmod 100 \\equiv 76^{2}\\bmod 100 \\equiv 76^{3}\\mod 100 \\equiv \\ldots \\]\nso our expression is equivalent to:\n\\[16 \\cdot (76) \\bmod 100 \\equiv \\boxed{16 \\bmod 100}\\]\n[/hide]\n\n[hide=\"$3^{2004}$\"]\nWe have\n\\[(3,100)=1 \\implies 3^{ \\varphi(100)}=3^{40}\\equiv 1 \\bmod 100 \\]\n\n\\[\\implies 3^{2004}\\equiv 3^{4}\\equiv \\boxed{81 \\bmod 100}\\]\n[/hide]\n\n[hide=\"$4^{2004}$\"]\nFrom the first problem, we know that $2^{2004}\\equiv 16 \\mod 100$. But,\n\\[4^{2004}=\\left( 2^{2004}\\right)^{2}\\equiv 16^{2}\\bmod 100 \\equiv \\boxed{ 56 \\bmod 100}\\]\n[/hide]\r\n(Click on each expression to see the solution. :D )",
"Solution_4": "What is CRT and how does it apply?"
}
{
"Tag": [],
"Problem": "How would you produce formic acid from acetic acid?\r\n\r\n[hide=\"Is this OK?\"]$ \\ce{CH3COOH}\\plus{}\\ce{NaOH}\\longrightarrow\\ce{CH3COONa}\\plus{}\\ce{H2O}\\\\\n\\ce{CH3COONa}\\plus{}\\ce{NaOH}\\,(\\ce{CaO},\\Delta )\\longrightarrow\\ce{CH4}\\plus{}\\ce{Na2CO3}\\\\\n\\ce{CH4}\\plus{}\\ce{I2}\\longrightarrow\\ce{CH3I}\\plus{}\\ce{HI}\\\\\n\\ce{CH3I}\\plus{}\\ce{NaOH}\\,(\\text{aq})\\longrightarrow\\ce{CH3OH}\\plus{}\\ce{NaI}\\\\\n\\ce{CH3OH}\\plus{}2[\\ce{O}]\\longrightarrow\\ce{HCOOH}\\plus{}\\ce{H2O}$[/hide]",
"Solution_1": "2 mistakes in step 3 itself.\r\n\r\n[hide=\"1\"]$ \\ce {HI}$ is a reducing agent and you will get back the alkane, meaning to say, the reaction is not completely towards the right. Hence one must use an oxidising agent also, say $ \\ce {HIO_3}$ or $ \\ce {HNO_3}$[/hide]\n\n[hide=\"2\"]Even still, you cannot gaurantee a 100% pure product. You will get a mixture of products. ($ \\ce {CH_2I_2}$, $ \\ce {CHI_3}$ etc will also be formed.)[/hide]",
"Solution_2": "May I ask what in God's name might be the purpose of preparing formic acid from acetic acid? If you want to have practice in organic reactions there are much more instructive examples to work with.\r\n\r\nHere's the method I would use:\r\n\r\n1) Sell a bottle of acetic acid.\r\n\r\n2) With the money earned in 1), buy a bottle of formic acid.",
"Solution_3": "[b]Side question:[/b]\r\n\r\nWell I was working on Schmidt reaction but I cant proceed further: \r\n\r\n+$ N_3H$ (Schmidt reaction) to give $ CH_3NH_2$\r\n\r\nHow can we go on to produce acid from an amine.",
"Solution_4": "amine to amide\r\nthen hydrolysis of amide formed",
"Solution_5": "Amine -> Amide how? Note that I want to form HCOOH",
"Solution_6": "Thanks all for your help. A friend of mine suggested creating $ \\ce{Ca}$ salt and decomposing it to get propanone. Then create $ \\ce{HCOONa}$ and iodoform. But I can't understand the last step. Could anyone please help?\r\n\r\n@hell_ever: Yes I thought about the second mistake, but I couldn't find any other way of doing it. Could you please help fix the mistake or find another way of doing this?\r\n\r\n[quote=\"Carcul\"]May I ask what in God's name might be the purpose of preparing formic acid from acetic acid? If you want to have practice in organic reactions there are much more instructive examples to work with.\n\nHere's the method I would use:\n\n1) Sell a bottle of acetic acid.\n\n2) With the money earned in 1), buy a bottle of formic acid.[/quote]\r\n\r\nYes I'm pretty amateur in organic chemistry and I've been trying to learn organic reactions. But from what you say above, I don't think I should learn anymore about these, I can go to a store and buy whatever I need. You helped me lose my interest in this subject and now I think it's nothing but memorizing a stupid bunch of reactions. Thank you very much for making me realize this.",
"Solution_7": "[quote=\"nayel\"]i think it's nothing but memorizing a stupid bunch of reactions[/quote]\natleast organic is a lot more than this\n\nwat if suddenly a fault appears and the starting product used by most of industries disappear....sm1 will hav 2 divise new method :oops: \n\n[hide][quote=\"nayel\"]i think it's nothing but memorizing a stupid bunch of reactions[/quote]\nactually i also think so :rotfl: :P [/hide]",
"Solution_8": "Nayel, I apologize if my post above looked a bit harsh, but, as you said, you are \"pretty amateur in organic chemistry\". Therefore, according to my experience, this doesn't look like a problem with pedagogic value. Even for starters, the problem BanishedTraitor has posted about protoanemonin is an excellent one for learning several important reactions, their mechanisms, and to start thinking like a synthetic organic chemist. I advise you to look at it.\r\n\r\nAlso, there was nothing im my post above that implies Organic Chemistry \"it's nothing but memorizing a stupid bunch of reactions\". It's much more than that, specially in the laboratory.\r\n\r\nFinally, don't forget you can always prove me wrong by providing a nice synthesis of formic acid from acetic acid.",
"Solution_9": "[hide=\"@ Carcul\"]I understand; I also apologize for my comments. But as a friend once told me: we are here to help one another, so please make sure no one gets upset. :) [/hide]\r\n\r\nWhat if we reduce the acetic acid (with for example $ \\ce{LiAlH4}$) to ethanol, and dehydrate it to ethylene. Then ozonolysis of the ethylene in presence of water should produce formic acid.\r\n\r\nShouldn't it work? :maybe:",
"Solution_10": "Actually, ozonolysis in the presence of water should produce formaldehyde. In the presence of hydrogen peroxide (or direct treatment with hot potassium permanganate) would, in principle, yield formic acid."
}
{
"Tag": [
"LaTeX"
],
"Problem": "If you consider the problem:\r\n\r\nHow many base-4 numbers with $ k$ digits have at least one digit a \"2\" AND have no \"0\"s before that \"2\"? (Note the number doesn't have to have a \"0\")?\r\n\r\nThis is the answer written recursively:\r\n\r\n[hide]\n$ k_1 \\equal{} 1$\n$ k_n \\equal{} 4^{n\\minus{}1} \\plus{} 2*k_{n\\minus{}1}$\nSo, for $ k\\equal{}1,2,3,4$, you get $ k_n \\equal{} 1, 6, 28, 120$\n[/hide]\n\nMy question is\n\n[hide]\nWhy are the solutions perfect numbers? What is the relationship (an algebraic one would be fine, but if anyone knows an intuitive relationship, that would be better)\n[/hide]",
"Solution_1": "[quote=\"magixter\"]Why are the solutions perfect numbers?[/quote]\r\nThey are not all perfect numbers. For example, $ k_6 \\equal{} 2016$ is not a perfect number. :lol:\r\n\r\nThere is a theorem that states that an integer is an even perfect number if and only if it is of the form $ 2^{p \\minus{} 1}M_p$ for some prime $ p$ and Mersenne prime $ M_p \\equal{} 2^p \\minus{} 1.$ From the recursive definition, it is easy to easy why some of the $ k_i$ should be of this form and hence are even perfect numbers. ;)",
"Solution_2": "[quote=\"ValentineA\"][quote=\"magixter\"]Why are the solutions perfect numbers?[/quote]\nThey are not all perfect numbers. For example, $ k_6 \\equal{} 2016$ is not a perfect number. :lol:[/quote] Or, uh, $ k_4 \\equal{} 120$ ;) To slightly restate what ValentineA wrote, you can solve that recurrence to get $ k_n \\equal{} 2^{n \\minus{} 1} \\cdot (2^n \\minus{} 1)$, so it follows from the theorem ValentineA quoted that every even number is of the form $ k_p$ for some $ p$.\r\n\r\nBy the way, a LaTeX suggestion for the O.P.: \\cdot makes a much nicer multiplication dot than the asterix. If you want a multiplication cross, you can use \\times.",
"Solution_3": "Heh, I missed $ k_4.$ :blush:"
}
{
"Tag": [],
"Problem": "Prove that for all positive integers $n$,\\[0<\\sum^n_{k=1}\\frac{g(k)}{k}-\\frac{2n}{3}<\\frac{2}{3}\\]where $g(k)$ denotes the greatest odd divisor of $k$",
"Solution_1": "can anyone solve this problem?",
"Solution_2": "are you sur that this in\u00e9galit\u00e9 it's true ?? :huh:",
"Solution_3": "Do you have a counterexample???",
"Solution_4": "Well indeed is not dificult at all...first let\u00b4s see that $\\displaystyle 0<\\sum^n_{k=1}\\frac{g(k)}{k}-\\frac{2n}{3}<\\frac{2}{3}$ is iff ${\\displaystyle \\frac{2n}{3}<\\sum^n_{k=1}\\frac{1}{p(k)}<\\frac{2n}{3}+\\frac{2}{3}}$ where $p(k)$ is the largest power of 2 which divides k. Now lets call $Q(n)=\\sum^n_{k=1}\\frac{1}{p(k)}$ then lets see that if n fit in the equality then 2n also...that is because:\r\n\r\n$Q(2n)=\\sum^{n}_{x=1}\\frac{1}{p(2x-1)}+\\sum^{n}_{x=1}\\frac{1}{p(2x)}$ now see that in the first sume there are just 1\u00b4s...\r\n\r\n$Q(2n)=n+\\sum^{n}_{x=1}\\frac{1}{p(2x)}=n+\\frac{1}{2}\\sum^{n}_{x=1}\\frac{1}{p(x)}=n+\\frac{1}{2}Q(n)$\r\nand if n fit in the equality then $\\frac{2(2n)}{3}+\\frac{2}{3}>n+\\frac{n}{3}+\\frac{1}{3}>n+\\frac{1}{2}Q(n)>n+\\frac{n}{3}=\\frac{2(2n)}{3}$\r\n so 2n fits...\r\n\r\nalso y $2n$ fits then $2n+1$ and that is because $Q(2n+1)=Q(n)+\\frac{1}{p(2x+1)}=Q(2n)+1$\r\nand that make 2n+1 fits in the equality...just check i have to leave...but lets see that $Q(1)=1$ and it fits so every other natural fits...just multiply by 2 and sum 1"
}
{
"Tag": [
"inequalities",
"search",
"inequalities open"
],
"Problem": ":rotfl: :) :o \r\nplease prove this inequlity",
"Solution_1": "I think it maybe classify number theory.\r\n\r\nSee this\r\nhttp://www.mathlinks.ro/Forum/viewtopic.php?search_id=605111911&t=6233"
}
{
"Tag": [
"inequalities",
"linear algebra",
"matrix"
],
"Problem": "Problem. Give $ A,B \\in M_n(k)$. Prove that:\r\n1.\r\n$ rank(A \\plus{} B) \\leq rank(A) \\plus{} rank(B)$\r\n2.\r\n$ rank(A) \\plus{} rank(B) \\minus{} n \\leq rank (AB) \\leq min(rank(A),rank(B))$",
"Solution_1": "1. More generally, let $ A,B \\in M_{m \\times n} (\\mathbb{K})$, and let $ T: V \\to W$, $ L: V \\to W$ (with $ \\dim V \\equal{} n$, $ \\dim W \\equal{} m$) be linear transformations represented by matrices $ A, B$, respectively. Then, $ A \\plus{} B$ represents the linear transformation $ T \\plus{} L$. Now, since $ Im(T\\plus{}L), Im(T), Im(L)$ are subspaces of $ W$ and $ Im(T\\plus{}L) \\subseteq Im(T) \\plus{} Im(L)$, then $ Im(T\\plus{}L)$ is a subspace of $ Im(T) \\plus{} Im(L)$, and so\r\n\r\n$ \\dim Im(T\\plus{}L) \\leq \\dim [Im(T) \\plus{} Im(L)]$\r\n\r\n$ \\equal{} \\dim Im(T) \\plus{} \\dim Im(L) \\minus{} \\dim [Im(T) \\cap Im(L)]$\r\n\r\n$ \\leq \\dim Im(T) \\plus{} \\dim Im(L)$.\r\n\r\nSince for any linear transformation F, the dimension of its image equals the rank of any matrix that represents it, we conclude from the previous inequality that\r\n\r\n$ rank(A\\plus{}B) \\leq rank(A) \\plus{} rank(B)$. $ \\Diamond$",
"Solution_2": "2. Let A,B be matrices such that the product AB is defined. Since the columns of AB are linear combinations of the columns of A, we have $ rank(AB) \\leq rank (A)$. On the other hand, the rows of AB are linear combinations of the rows of B, and so $ rank(AB) \\leq rank B$. Hence, $ rank(AB) \\leq min\\,\\,[rank(A),rank(B)]$. The other inequality, $ rank(A) \\plus{} rank(B) \\leq rank(AB) \\plus{} n$, is Sylvester's inequality, which can be proved directly or as a special case of the Frobenius inequality. In any case, you can also use the concept of linear transformation, and I will leave that to you.",
"Solution_3": "[quote=\"vipCD_A1\"]Problem. Give $ A,B \\in M_n(k)$. Prove that:\n\n2.\n$ rank(A) \\plus{} rank(B) \\minus{} n \\leq rank (AB)$[/quote]\r\n\r\nCan you detail solve?\r\nI have thought about it but I can't solve it. Please, carcul!",
"Solution_4": "For instance, start by assuming that $ A \\equal{} diag \\{I_{r(A)}, O\\}$, where $ r(A)$ is the rank of A. Then show that $ r(AB) \\geq r(A) \\plus{} r(B) \\minus{} n$. Next, if A does not have the form above, there are invertible matrices P,Q such that $ D \\equal{} diag \\{I_{r(A)}, O\\} \\equal{} PAQ$. Using the previous result we have $ r(DB) \\equal{} r(PAQB) \\geq r(D) \\plus{} r(B) \\minus{} n \\equal{} r(A) \\plus{} r(B) \\minus{} n$. Now show that $ r(PAQB) \\equal{} r(AB)$."
}
{
"Tag": [
"pigeonhole principle"
],
"Problem": "Im new to the pigeonhole prinicple and i think i understand the theory but i cant solve tough problems...e.g.\r\n\r\n(1)If {a1,a2,a3...a1995} (numbers are subscripts) be a sequence of positive integers whose sum is 3989, show that there is a block of r sucessive ai's (i is subscripts here) whose sum is 95.\r\n\r\n(2)A Chessmaster who has 11 weeks to prepare for a tournament decides to play at least one game everyday, but no more than 12 games during a week.Show that there exists a succession of days during which the chessmaster played exactly 21 games.\r\n\r\nCan someone please give me advice on how to attack such problems.....also it would be nice if someone posts the solution to the to probs.......",
"Solution_1": "i was going to post something one this or might have already. but yeah"
}
{
"Tag": [
"inequalities proposed",
"inequalities"
],
"Problem": "A,b,c,d are real nunbers .a^2+b^2+c^2+d^2=1\r\nProve that ab+ac+ad+bc+bd+cd<=4abcd+5/4",
"Solution_1": "[quote=\"stevenwang1992\"]A,b,c,d are real nunbers .a^2+b^2+c^2+d^2=1\nProve that $ ab\\plus{}ac\\plus{}ad\\plus{}bc\\plus{}bd\\plus{}cd\\leq4abcd\\plus{}\\frac{5}{4}$[/quote]\r\nLet $ a\\equal{}\\min\\{a,b,c,d\\}.$\r\n$ ab\\plus{}ac\\plus{}ad\\plus{}bc\\plus{}bd\\plus{}cd\\leq4abcd\\plus{}\\frac{5}{4}\\Leftrightarrow$\r\n$ 5(a^2\\plus{}b^2\\plus{}c^2\\plus{}d^2)^2\\plus{}16abcd\\geq4(ab\\plus{}ac\\plus{}ad\\plus{}bc\\plus{}bd\\plus{}cd)(a^2\\plus{}b^2\\plus{}c^2\\plus{}d^2),$ \r\nwhich is easy to check after substitution $ b\\equal{}a\\plus{}x,$ $ c\\equal{}a\\plus{}y$ and $ d\\equal{}a\\plus{}z.$ :wink:"
}
{
"Tag": [
"inequalities",
"pigeonhole principle"
],
"Problem": "Taken a point A in a regular polygon A1...An. Prove that there exists at least one angle A_i O A_j verifying the inequalities:\r\n\r\n pi(1-1/n) < A_i O A_j < pi\r\n[/hide]",
"Solution_1": "it is pigeonhole on the n wells $(\\frac{k}{n}, \\frac{k+1}{n})$ for $1\\leq k\\leq n$"
}
{
"Tag": [
"number theory open",
"number theory"
],
"Problem": "S={1,2,...,3^{2}-1}\r\n3^{2} | 1+2+...+3^{2}-1\r\n3^{1} | 1*2+2*3+...+(3^{2}-3)*(3^{2}-1) (just sum of 2-element combination of S)\r\n \r\nIf change S={1,2,...,3^{2}-1} to S={1,2,...,3^{3}-1}\r\n3^{3} | 1+2+...+3^{3}-1\r\n3^{2} | 1*2+2*3+...+(3^{3}-3)*(3^{3}-1) (just sum of 2-element combination of S)\r\n3^{1} | 1*2*3+2*3*4+...+(3^{3}-4)*(3^{3}-3)*(3^{3}-1) (just sum of 3-element combination of S)\r\n\r\nand so on\r\n\r\nIn the set S, changing 3 will not make diff situation\r\n\r\nFrom some iteration, above must be true, but why?\r\nplease let me know this simple thing...",
"Solution_1": "$1+2+3+...+n=\\sum k$\r\n$1\\cdot 2+2\\cdot 3+...+n\\cdot (n+1)=\\sum k(k+1)=\\sum k^{2}+\\sum k$\r\n$1\\cdot 2\\cdot 3+2\\cdot 3\\cdot 4+...+n\\cdot (n+1) \\cdot (n+2)=\\sum k(k+1)(k+2)=\\sum k^{3}+3\\sum k^{2}+2\\sum k$\r\nAll the sums from the $RHS$ are known, and divisibility should give easy.",
"Solution_2": "hmm\r\nsorry, my representation is somewhat wrong.\r\n\r\nexactly writting is following\r\n3^{2} | 1*2+1*3+...+2*3+...+(3^{3}-3)*(3^{3}-1) (just sum of 2-element combination of S) \r\n\r\nactually problem can be written like this...(hmm above rep was somewhat stupid :blush: )\r\n3^{3} | (x+1)(x+2)...(x+(3^{3}-1))'s 2th-coeff\r\n3^{2} | (x+1)(x+2)...(x+(3^{3}-1))'s 3th-coeff\r\n3^{1} | (x+1)(x+2)...(x+(3^{3}-1))'s 4th-coeff"
}
{
"Tag": [
"geometry"
],
"Problem": "Given that the largest triangle is equilateral and all the smaller triangles are formed by connecting the midpoints of the sides of the larger triangle that contains, them, how many equilateral triangles are there?\n[asy]import olympiad; import geometry_dev; import graph; size(100); defaultpen(linewidth(0.8));\n void drawEqTriangle(real side, pair blCorner, real degrees = 0){\n pair c1 = blCorner;\n pair c2 = c1 + side*dir(degrees);\n pair c3 = c2 + side*dir(120 + degrees);\n draw(c1--c2--c3--cycle);\n }\n drawEqTriangle(1,origin); drawEqTriangle(1,(1,0)); drawEqTriangle(1,(0.5,sqrt(3)/2));\n for(int i = 1; i <=3; ++i){\n for(int j = 0; j <i; ++j){\n drawEqTriangle(1/4, (1 - i/8 + 1/4*j, sqrt(3)/8*i));\n }\n }[/asy]",
"Solution_1": "From smallest triangles to largest: 16+7+3+4+1=31"
}
{
"Tag": [
"function",
"real analysis",
"real analysis unsolved"
],
"Problem": "Show that if $ f: R \\rightarrow R$ is a monotone increasing function, then the set where $ f$ is discontinuous has measure zero.",
"Solution_1": "The set where f is dicontinuous is countable, because they are all jump discontinuities. Uncountably many jump discontinuities of positive size on some interval $ (a,b)$ would make $ f$ increase more than $ f(b) \\minus{} f(a)$ on the interval",
"Solution_2": "in fact, I saw what you said, but I don't know how to prove that [i]uncountably many jump discontinuities of positive size on some interval $ (a,b)$ would make increase $ f$ more than on the interval $ (f(a),f(b))$.[/i]",
"Solution_3": "I was just loosely saying that an \"uncountable sum\" of positive numbers is infinite. Index all the discontinuities in $ (a,b)$ as $ x_j$ where $ j$ runs through some index, and let the corresponding jump size at $ x_j$ be $ K_j$. Then the set of $ x_j$ such that $ K_j \\geq 1/n$ is finite. The set of all $ x_j$ is the countable union of the former sets as $ n$ runs through the positive integers. The countable union of finite sets is countable, so the number of $ x_j$ is countable.",
"Solution_4": "[quote=\"Kalle\"]I was just loosely saying that an \"uncountable sum\" of positive numbers is infinite. Index all the discontinuities in $ (a,b)$ as $ x_j$ where $ j$ runs through some index, and let the corresponding jump size at $ x_j$ be $ K_j$. Then the set of $ x_j$ such that $ K_j \\geq 1/n$ is finite. The set of all $ x_j$ is the countable union of the former sets as $ n$ runs through the positive integers. The countable union of finite sets is countable, so the number of $ x_j$ is countable.[/quote]\r\n\r\nThank you Kalle !",
"Solution_5": "[quote=\"Kalle\"]I was just loosely saying that an \"uncountable sum\" of positive numbers is infinite. Index all the discontinuities in $ (a,b)$ as $ x_j$ where $ j$ runs through some index, and let the corresponding jump size at $ x_j$ be $ K_j$. Then the set of $ x_j$ such that $ K_j \\geq 1/n$ is finite. The set of all $ x_j$ is the countable union of the former sets as $ n$ runs through the positive integers. The countable union of finite sets is countable, so the number of $ x_j$ is countable.[/quote]\r\n\r\nwhy is the set of $ x_j$ s.t. $ K_j \\geq \\frac{1}{n}$ finite?"
}
{
"Tag": [],
"Problem": "For natural numbers $m, n$, set $a = (n+1)^{m}-n$ and $b = (n+1)^{m+3}-n$.\r\n\r\n(a) Prove that $a$ and $b$ are coprime if $3 \\nmid m$.\r\n\r\n(b) Find all numbers $m, n$ for which $a$ and $b$ are not coprime.",
"Solution_1": "This is solve at\r\nhttp://www.mathlinks.ro/Forum/viewtopic.php?t=116060"
}
{
"Tag": [
"geometry",
"3D geometry",
"sphere",
"Pythagorean Theorem"
],
"Problem": "Given two balls. The larger ball of radius 9 leans against the wall of a room, and the smaller ball, or radius 4, rests upon the larger ball. Each ball is tangent to the wall, and the larger ball is tangent to the floor. What is the distance from the floor to the point of tangency of the smaller ball?\r\n\r\nThe solution given is 21, but I'm getting 22.",
"Solution_1": "The solution uses the picture.\r\n\r\n[hide=\"Agree with the book\"]By the Pythagorean Theorem, $ GE = 12$. $ 9 = DA = EB$. Hence, the length is $ 12+9 = 21$[/hide]",
"Solution_2": "the correct answer is $ 21$.\r\n\r\n[hide=\"solution\"]\n[asy]size(200); draw((0,0)--(100,0)); draw((0,0)--(0,100)); pair A,B,C,D,R,F,M,P,Q,O; O=origin; A=(0,27); B=(27,0); C=(54,27); D=(0,63); R=(24,63); F=(12,51); draw(circumcircle(A,B,C)); draw(circumcircle(D,R,F)); label(\"D\",D,W); label(\"A\",A,W); P=(27,27); Q=(12,63); label(\"P\",P,E); label(\"Q\",Q,E); dot(P); dot(Q); draw(P--Q); M=(12,27); label(\"M\",M,S); dot(M); draw(A--P); draw(Q--M); label(\"O\",O,W);[/asy]\nfrom the diagram, we get using pythagorean theorem, $ PQ=9+4=13=\\sqrt{QM^{2}+(9-4)^{2}}$ thus $ QM=AD=12$.\nso $ OD=AD+AO=12+9=21$.\n[/hide]\r\n\r\nEDIT: sorry vishalarul, didn't see your post there.",
"Solution_3": "Thanks for your help, I see my error! :lol:"
}
{
"Tag": [
"trigonometry",
"geometry",
"function",
"probability",
"AMC"
],
"Problem": "This is a stupid question and can be deleted at soon as there is one accurate response. How much time do you get for the AMC 10/12? I've heard 75 and I've heard 90.\r\n\r\nEdit: Perhaps I'll substantiate this post slightly by asking, what tips/strategies people have for maximizing productivity under limited and straining time conditions. Also, how can one reach the later level questions quickly (as they require more time than the earlier questions), without making silly careless errors?",
"Solution_1": "AMC 10/12 has a 75 minute time limit. The old AHSME had a 90 minute time limit (AHSME turned into AMC).\r\n\r\nTips: \r\n\r\nDon't waste too much time on a problem. If, for example, there is a trig problem but you just learned trig last week in your precalc class, skip it (unless you are confident you can do it in a timely fashion). \r\n\r\nTry to stay organized. You don't want to be unable to correctly read what you wrote. It also helps to have organized work in case you have the chance to go back and check your work.\r\n\r\nThere aren't really any tips I can give for you to get through the 'easier' problems quicker. That mostly comes with practice (the more times you've seen a certain type of problem, the faster you'll recognize how to approach, and so forth).",
"Solution_2": "Why should you skip a problem that you just learned how to do? That seems like it would be one of the problems you would want to make sure you solve.",
"Solution_3": "He means skip it, since you were just recently introduced to the concept, so that you don't spend a disproportionate amount of time on it. Obviously, if you've finished all the problems that you can do easily, go back to that one and try your hand at it, but don't go after it when you get to it (unless it's at the end of the test, obviously :P). As joml said, if you're sure you can do it quickly, go for it, but make sure you assess your capabilities in that subject realistically.\r\n\r\nWow, I just used a whole lot of big words. :what?:",
"Solution_4": "More tips:\r\n\r\n1. Always check as you go. Of course it's nice to check back at the end, especially if you've organized your work well, but often you don't have time anyway. Also, you might have forgotten some special cases that you noticed the first time. Being meticulous the first time is usually the most effective method.\r\n\r\n2. Don't be too hasty. Actually I'm not really sure if this helps, but a lot of times I dive into a problem because I immediately see a way, when in fact there's an easier way that I could have used to save up to a minute or so if I had just spent a few seconds longer before diving in.",
"Solution_5": "After 3 minutes and you've gotten no where on a problem, it's not the end of the world, just skip it and come back to it after doing like 2 or 3 other problems.",
"Solution_6": "i would do all the problems that i am good at (geometry, nt, functions,etc) and skip all but the easiest probability (probability and combinatorics make me mad :mad: ), then if i have time, go back to the problems i am bad at, also most of the early amc problems can be done with 2 sentences of explaination if you do them correctly, so don't attack a tedious solution, find a better one, or move on, if you find the best solution, very little work is needed, and don't hesitate to use lots of paper, writing big helps you check better",
"Solution_7": "I'm certainly no expert but if you look at a problem and it looks hard to you dont waste time, do the easy problems first, and then investigate those problems..",
"Solution_8": "I think the key is to focus correctly.\r\n\r\nFocus on reading every word of the problem, and taking in the meaning.\r\nThen just get everything clear in your mind exactly what you need to do.\r\nOrganize your work and write out as much as you can (without going too slowly). Continue to keep your mind clear as to what you're doing and what you need to do. Especially, don't go through sloppy calculations or thoughts in your head; write things down.\r\nOnce you have the answer just write it in and go on to the next problem.\r\nStill remember not to take too much time on each problem.\r\n\r\nOne example is that in a math competition, I had a factoring/geometry problem. I got three solutions, 0, 8, and 60. I quickly thought through in my head that 0 didn't work, and didn't bother to check 8 and 60, thinking they worked because they were positive. As it turned out, 8 didn't work. Only 60 did.\r\n\r\nAlthough that particular problem might not apply to AMC 12 directly, the idea behind it does. Rather than quickly thinking through in my head, I should have paid attention to each answer individually, doing the full process of checking each answer.\r\nAnyway, checking the answers should have only taken a few seconds apiece. One can still focus in this way and do a problem quickly."
}
{
"Tag": [],
"Problem": "Joyce wants to share six identical pieces of licorice equally among four people. What is the mnimum number of pieces that must be cut?",
"Solution_1": "2\r\n :P :P",
"Solution_2": "2 is correct, but if you don't see why, I will explain.\r\n\r\nSharing 6 pieces amoung 4 people means each person gets $\\frac64=1+\\frac12$. Give each person a stick, and you have 2 sticks remaining. Now, cut them in half so there are 4 halfs to give to the people. You only have to cut 2 of the sticks."
}
{
"Tag": [
"number theory proposed",
"number theory"
],
"Problem": "Find all $ x,y\\in Z^\\plus{}$ such that $ 3^x\\equal{}2^x.y\\plus{}1$\r\nIf post before,please give links :)",
"Solution_1": "Let $ x = m.2^k$, then\r\n${ 3^x - 1 = 3^{m.2^k} - 1 = (3^{2^k} - 1)(3^{2k.(m - 1}} + . . . + 1)$.\r\n${ (3^{2k.(m - 1}} + . . . + 1)$ is sum of $ m$ integers odd, so it's odd.\r\n$ 3^{2^k} - 1 = (3 - 1)(3 + 1)(3^2 + 1)(3^{2^2} + 1) . . . (3^{2^{k - 1}} + 1) \\parallel{} 2^{k + 1}$.\r\nSo $ m.2^k\\leq k + 1$.\r\n$ \\rightarrow m = 1,k = 0,1$.\r\n$ (x,y) = (1,1);(2,2)$.",
"Solution_2": "Also, refer to the exponent lifting lemma"
}
{
"Tag": [
"algebra proposed",
"algebra"
],
"Problem": "Let $ m, n \\in N$ and $ \\frac{m^{n}-1}{n-1}+\\frac{n^{m}-1}{m-1}\\in Z$\r\nshow that $ \\frac{m^{n}-1}{n-1}\\in Z$ and $ \\frac{n^{m}-1}{m-1}\\in Z$",
"Solution_1": "$ n\\neq{1}$and$ m\\neq{1}$\r\nSuppose by absurd $ \\frac{m^{n}-1}{n-1}\\notin{Z}$ and $ \\frac{n^{m}-1}{m-1}\\notin{Z}$ then \r\n$ (m-1)\\frac{(m^{n}-1)}{n-1}\\in{Z}$ and $ (n-1)\\frac{n^{m}-1}{m-1}\\in{Z}$\r\n$ \\Leftrightarrow \\frac{m-1}{n-1}\\in{Z}$ and $ \\frac{n-1}{m-1}\\in{Z}\\Leftrightarrow{m=n}$absurd"
}
{
"Tag": [
"trigonometry"
],
"Problem": "If $ \\sin x\\plus{}a \\cos x\\equal{}b$, express $ |a \\sin x \\minus{}\\cos x|$ in terms of $ a,b$.",
"Solution_1": "Let's calculate $ (\\sin x \\plus{} a \\cos x)^2\\plus{}|a \\sin x \\minus{} \\cos x|^2.$"
}
{
"Tag": [
"superior algebra",
"superior algebra unsolved"
],
"Problem": "Show that $C$ has infinitely many field automorphisms. What is the cardinality of the set of these automorphisms?",
"Solution_1": "There are as many as possible: $2^{2^{\\aleph_{0}}}$. This is because any permutation of a transcendence basis of $\\mathbb C$ over $\\mathbb Q$ (and all such bases have cardinality $2^{\\aleph_{0}}$) extends to an automorphism of $\\mathbb C$.",
"Solution_2": "Grobber can you calrify ?"
}
{
"Tag": [
"calculus",
"integration",
"limit",
"logarithms",
"geometry",
"rectangle",
"inequalities"
],
"Problem": "Let $ n$ be natural number. Find the limit value of ${ \\lim_{n\\to\\infty} \\frac{1}{n}(\\frac{1}{\\sqrt{2}}+\\frac{2}{\\sqrt{5}}}+\\cdots\\cdots +\\frac{n}{\\sqrt{n^2+1}}).$",
"Solution_1": "hello kunny, we get\r\n$ \\lim_{n\\to \\infty}\\frac{1}{n}\\left(\\sum_{i\\equal{}1}^{n}\\frac{i}{\\sqrt{i^2\\plus{}1}}\\right)\\equal{}1$\r\nSonnhard.",
"Solution_2": "That's correct. How did you get it? :)",
"Solution_3": "[quote=\"Dr Sonnhard Graubner\"]hello kunny, we get\n$ \\lim_{n\\to \\infty}\\frac {1}{n}\\left(\\sum_{i \\equal{} 1}^{n}\\frac {i}{\\sqrt {i^2 \\plus{} 1}}\\right) \\equal{} 1$\nSonnhard.[/quote]\r\nYour answer is circular answer.\r\nYou are the same as answering by feeling.\r\n\r\n[b]My Solution[/b]\r\nLet $ f(n) \\equal{} \\frac{n}{\\sqrt{n^2\\plus{}1}}$.\r\nSo,\r\n$ f(1) \\plus{} \\int_1^n f(x)dx \\le \\sum_{i \\equal{} 1}^{n} f(i) \\le \\int_1^n f(x)dx \\plus{} f(n)$\r\n$ \\iff \\frac{1}{\\sqrt{2}} \\plus{} \\sqrt{n^2\\plus{}1} \\minus{} \\sqrt{2} \\le \\sum_{i \\equal{} 1}^{n} f(i) \\le \\sqrt{1\\plus{}n^2} \\minus{} \\sqrt{2} \\plus{} \\frac {n}{\\sqrt {n^2\\plus{}1}}$\r\n$ \\iff \\sqrt{n^2\\plus{}1} \\minus{} \\frac{1}{\\sqrt{2}} \\le \\sum_{i \\equal{} 1}^{n} f(i) \\le \\frac {n^2\\plus{}n\\plus{}1}{\\sqrt {n^2\\plus{}1}} \\minus{} \\sqrt{2}$\r\n$ \\iff \\sqrt{1\\plus{}\\frac{1}{n^2}} \\minus{} \\frac{1}{n\\sqrt{2}} \\le \\frac{1}{n}\\left(\\sum_{i \\equal{} 1}^{n} f(i)\\right) \\le \\frac {1\\plus{}\\frac{1}{n}\\plus{}\\frac{1}{n^2}}{\\sqrt {1\\plus{}\\frac{1}{n^2}}} \\minus{} \\frac{\\sqrt{2}}{n}$\r\n$ \\Rightarrow 1 \\le \\frac{1}{n}\\left(\\sum_{i \\equal{} 1}^{n} f(i)\\right) \\le 1 \\ \\ \\ \\ \\ (n \\to \\infty)$\r\n\r\nThus, $ \\lim_{n \\to \\infty} \\frac{1}{n}\\left(\\sum_{i \\equal{} 1}^{n} f(i)\\right) \\equal{} \\boxed{1}$",
"Solution_4": "We have:\r\n$ 1 \\geq \\frac{1}{n}\\sum_{i\\equal{}1}^{n}\\frac{i}{\\sqrt{i^2}} \\geq \\frac{1}{n}\\sum_{i\\equal{}1}^{n}\\frac{i}{\\sqrt{i^2 \\plus{} 1}} \\geq \\left(\\prod_{i\\equal{}1}^{n} \\frac{i}{\\sqrt{i^2 \\plus{} 1}}\\right)^{\\frac{1}{n}} \\geq \\left(\\prod_{i\\equal{}1}^{n} \\frac{i}{\\sqrt{\\left(i \\plus{} 1\\right)^2}}\\right)^{\\frac{1}{n}} \\equal{} \\left(\\prod_{i\\equal{}1}^{n} \\frac{i}{i \\plus{} 1}\\right)^{\\frac{1}{n}} \\equal{} \\left(1 \\plus{} n\\right)^{\\frac{1}{n}}$\r\n\r\nApplying the sandwich method we get the limit to be 1.",
"Solution_5": "[quote=\"akech\"]We have:\n$ 1 \\geq \\frac {1}{n}\\sum_{i \\equal{} 1}^{n}\\frac {i}{\\sqrt {i^2}} \\geq \\frac {1}{n}\\sum_{i \\equal{} 1}^{n}\\frac {i}{\\sqrt {i^2 \\plus{} 1}} \\geq \\left(\\prod_{i \\equal{} 1}^{n} \\frac {i}{\\sqrt {i^2 \\plus{} 1}}\\right)^{\\frac {1}{n}} \\geq \\left(\\prod_{i \\equal{} 1}^{n} \\frac {i}{\\sqrt {\\left(i \\plus{} 1\\right)^2}}\\right)^{\\frac {1}{n}} \\equal{} \\left(\\prod_{i \\equal{} 1}^{n} \\frac {i}{i \\plus{} 1}\\right)^{\\frac {1}{n}} \\equal{} \\left(1 \\plus{} n\\right)^{\\frac {1}{n}}$\n\nApplying the sandwich method we get the limit to be 1.[/quote]\r\n\r\nyour solution and Kouichi Nakagawa's both are excellent. I do agree with Kouichi Nakagawa that Dr Sonnhard Graubner's answer is probably based on answering by feeling. \r\n\r\nI think instead of $ \\left(1 \\plus{} n\\right)^{\\frac {1}{n}}$\r\nwrite $ \\frac{1}{\\left(1 \\plus{} n\\right)^{\\frac {1}{n}}}$\r\n\r\nand $ \\lim_{n\\to\\infty}\\left(1 \\plus{} n\\right)^{\\frac {1}{n}}\\equal{}1$",
"Solution_6": "[color=blue]\nMy solution as follows:\n\nAs $ \\ \\frac {r}{\\sqrt {(r\\plus{}1)^2}}<\\frac {r}{\\sqrt {r^2\\plus{}1}}<\\frac {r}{\\sqrt {r^2}}\\ \\forall \\ r \\equal{} 1,\\ 2,\\ 3,\\ \\dots$\n\n$ \\implies\\frac {r}{r\\plus{}1}<\\frac {r}{\\sqrt {r^2\\plus{}1}}<\\frac {r}{r}$\n\n$ \\implies1\\minus{}\\frac {1}{r\\plus{}1}<\\frac {r}{\\sqrt {r^2\\plus{}1}}<1$\n\n$ \\implies \\sum_{r \\equal{} 1}^{n}\\left(1\\minus{}\\frac {1}{r\\plus{}1}\\right)<\\sum_{r \\equal{} 1}^{n}\\frac {r}{\\sqrt {r^2\\plus{}1}}<\\sum_{r \\equal{} 1}^{n}1$\n\n$ \\implies n\\minus{}\\sum_{r \\equal{} 2}^{n\\plus{}1}\\frac {1}{r}<\\sum_{r \\equal{} 1}^{n}\\frac {r}{\\sqrt {r^2\\plus{}1}}< n$\n \nAs $ \\ \\int_{1}^{n\\plus{}1}\\frac{1}{x}\\ dx \\plus{} \\frac {1}{n\\plus{}1}\\ge \\sum_{r \\equal{} 2}^{n\\plus{}1}\\frac {1}{r}$\n\n$ \\implies n\\minus{}\\int_{1}^{n\\plus{}1}\\frac{1}{x}\\ dx \\minus{} \\frac {1}{n\\plus{}1}<\\sum_{r \\equal{} 1}^{n}\\frac {r}{\\sqrt {r^2\\plus{}1}}< n$\n\n$ \\implies 1\\minus{}\\frac{\\ln (n\\plus{}1)}{n} \\minus{} \\frac {1}{n(n\\plus{}1)}<\\frac{1}{n}\\sum_{r \\equal{} 1}^{n}\\frac {r}{\\sqrt {r^2\\plus{}1}}< 1$\n\nAs $ \\ \\lim_{n\\to\\infty}\\frac{\\ln (n\\plus{}1)}{n}\\equal{}0$ and $ \\ \\lim_{n\\to\\infty}\\frac{1}{n(n\\plus{}1)}\\equal{}0, $\n\nBy applying sandwich theorem we get,\n\n$ \\boxed {\\boxed {\\lim_{n\\to\\infty}\\frac{1}{n}\\sum_{r \\equal{} 1}^{n}\\frac {r}{\\sqrt {r^2\\plus{}1}}\\equal{}1}}$\n\n[/color]",
"Solution_7": "Another approach: let $ a_n \\equal{} \\frac{n}{\\sqrt{n^2 \\plus{} 1}}$. As $ n$ approaches infinity, $ a_n$ approaches 1. So by Cesaro's theorem, $ \\frac{1}{n} \\sum_{i\\equal{}1}^n a_i$ approaches 1 too.",
"Solution_8": "[quote=\"Ravi B\"]Another approach: let $ a_n \\equal{} \\frac {n}{\\sqrt {n^2 \\plus{} 1}}$. As $ n$ approaches infinity, $ a_n$ approaches 1. So by Cesaro's theorem, $ \\frac {1}{n} \\sum_{i \\equal{} 1}^n a_i$ approaches 1 too.[/quote]\r\n\r\nCan u explain Cesaro's theorem in detail",
"Solution_9": "If you have a sequence of numbers $ a_n$ that converges to $ L$, then the sequence of partial averages $ \\frac{1}{n} \\sum_{i\\equal{}1}^n a_i$ also converges to $ L$.",
"Solution_10": "Note that the sum is equivalent to the area of the rectangles above the curve $ y = \\frac {x}{\\sqrt {x^2 + 1}}$ on $ [0,n]$, where each rectangle has width $ 1$. Note also that each term of the sum is less than $ 1$, so these rectangles all lie below $ y=1$. Hence, we get the inequality\r\n\r\n$ \\int_0^n \\! \\frac {x}{\\sqrt {x^2 + 1}} \\, dx \\le \\sum_{i = 1}^{n}\\frac {i}{\\sqrt {i^{2} + 1}} \\le \\int_0^n \\! dx$\r\n\r\nand subsequently,\r\n\r\n$ \\frac {\\sqrt {n^2 + 1} - 1}{n} \\le \\frac {1}{n} \\sum_{i = 1}^{n}\\frac {i}{\\sqrt {i^{2} + 1}} \\le 1$\r\n\r\nSince $ \\lim_{n\\to\\infty}\\!\\frac {\\sqrt {n^2 + 1} - 1}{n}\\, = 1$ and $ \\lim_{n\\to\\infty}1 = 1$, by the Squeeze Theorem, we get\r\n\r\n$ {\\lim_{n\\to\\infty}\\frac {1}{n}\\sum_{r = 1}^{n}\\frac {r}{\\sqrt {r^{2} + 1}} = 1}$",
"Solution_11": "Thank you for your replies! :lol: \r\n\r\nKouichi Nalagawa and Elixir Solution are a standard for Japanese high school students.\r\n\r\nMy solution is almost same as Elixir 's, but I write $ \\int_0^n f(x)\\ dx < \\sum_{k \\equal{} 1}^n \\frac {k}{\\sqrt {k^2 \\plus{} 1}} < \\boxed{\\int_1^{n \\plus{} 1}}\\ f(x)\\ dx$\r\n\r\nCesaro's theorem by Ravi B is very useful, but it usually appeares in University mathematics in Japan.",
"Solution_12": "I am rather pleased to see that Japanese high school* students are taught some interesting theory and asked questions that actually require some thought - a \"hard\" question in the UK would be probably be off a STEP paper;\r\n\r\nhttp://www.admissionstests.cambridgeassessment.org.uk/adt/step\r\n\r\nI just remmbered this is a medical entrance exam - in the UK a medical student would also hopelessly struggle as mathematics is not a requirement for entrance into medical schools."
}
{
"Tag": [
"inequalities",
"linear algebra",
"matrix",
"linear algebra unsolved"
],
"Problem": "Let $A_i \\in {\\mathbb C}^{n \\times n},i=1,2,...,n$. If there is at least a $k \\in \\{1,2,...,n\\}$ such that $A_k = O_n$ then show that the following inequality holds: $\\sum_{i=1}^{n} rank(A_i) \\leq n(n-1)$.\r\n :ninja:",
"Solution_1": ":huh:\r\nIs $O_n$ the zero matrix (then it's trivial), or is it an orthogonal matrix (then it's false) ?",
"Solution_2": "$O_n$ is zero matrix ;)"
}
{
"Tag": [
"probability",
"AMC",
"AIME",
"counting",
"distinguishability",
"summer program",
"Mathcamp"
],
"Problem": "What do ppl mean when they say u need to know the balls-and-urns argument in order to solve many of the probability problems in AIME? Can someone tell me what that argument is. :D",
"Solution_1": "If you have $b$ indistinguishable balls and $u$ urns, the number of ways you can put $b$ balls in $u$ urns is $\\dbinom{b+u-1}{u-1}$. If there is the additional restriction that there must be at least one ball in each urn, the number of ways is $\\dbinom{b-1}{u-1}$.",
"Solution_2": "What if u have different types of balls? Then does the same argument still hold or is it somewhat different?",
"Solution_3": "Oh, I forgot, sorry, they have to be indistinguishable balls. Distinguishable balls have different methods.",
"Solution_4": "A specific question: If you have $6$ indistinguishable balls that you need to pout into $4$ distinguishable urns, how many ways can you put the balls into the urns?\r\n\r\nAnswer:\r\nWe make a bijection. Instead of putting balls into urns, let us imagine that the balls are arranged in a straight line and that we are putting dividers between the balls. There will be 3 dividers.\r\n\r\nFor example, oo|o||ooo means put two balls in the first urn, one ball in the second urn, zero balls in the third urn, and 3 balls into the fourth urn.\r\n\r\nEach permutation of oooooo||| represents a different way to put the balls into the urns, and each way the balls can be put into urns can be represented as a permutation of oooooo|||. However, it is easy to count the number of permutations of oooooo|||, which is $\\binom{9}{3}$.\r\n\r\n\r\nIn general with b balls and u urns, you get what davidyko has.",
"Solution_5": "woah.. i was about to ask the same question... So where did everyone learn the balls and urns argument from? Was it in Aops?1 or 2?",
"Solution_6": "I learned in MOP, I think it's in AoPS, and the teacher in my school's honors math class even talked about it! This concept is amazingly useful. For example:\r\n\r\nHow many 4-digit numbers have the property that no digit is less than the digit to its left?",
"Solution_7": "[quote=\"turtlecloud\"]woah.. i was about to ask the same question... So where did everyone learn the balls and urns argument from? Was it in Aops?1 or 2?[/quote]\r\nIt's in intro to C&P and AoPS vol. 2.",
"Solution_8": "[quote=\"randomdragoon\"]I learned in MOP, I think it's in AoPS, and the teacher in my school's honors math class even talked about it! This concept is amazingly useful. For example:\n\nHow many 4-digit numbers have the property that no digit is less than the digit to its left?[/quote]\r\n\r\n[hide=\"hint\"]\n\nI interpret it as, \"How many way are there to pick 4 digits from 9 if we allow repetition?\"\n\n[/hide]",
"Solution_9": "Where is it found in aops 2? i looked through the book but i still cant find it.",
"Solution_10": "[quote=\"Kamil Witek\"]Where is it found in aops 2? i looked through the book but i still cant find it.[/quote]\r\nEh, in the counting in the twighlight zone section I think.\r\n\r\nNot really sure.",
"Solution_11": "Meh, I don't think it is in there. Oh well, I learned it from these forums. :P",
"Solution_12": "[quote=\"davidyko\"]Meh, I don't think it is in there. Oh well, I learned it from these forums. :P[/quote]\r\nI'm almost positive it's in there.\r\nIt was a problem about a person buying 8 german dogs with three breeds to choose from.",
"Solution_13": "Never mind, you're right. :roll: \r\nChapter 17.2, Clever Correspondences, the first problem about 8 dogs and three breeds, just in case anyone wanted to know.",
"Solution_14": "What about indistinguishable urns? How would you do that?",
"Solution_15": "[quote=\"123s\"]What about indistinguishable urns? How would you do that?[/quote]\r\n\r\nI believe, correct me if I'm wrong, that that's counting the number of partitions, which has an extremely extremely complicated formula (Bell numbers) that I think was on the back of some mathcamp shirts and went all the way across.",
"Solution_16": "For that you could be 1337 and use generating functions, then plug into mathematica...\r\n\r\ncoefficient of $x^{n}$ in $(1+x+x^{2}+\\cdots)^{\\infty}$ is the number of partitions of $n$.",
"Solution_17": "[quote=\"13375P34K43V312\"]For that you could be 1337 and use generating functions, then plug into mathematica...\n\ncoefficient of $x^{n}$ in $(1+x+x^{2}+\\cdots)^{\\infty}$ is the number of partitions of $n$.[/quote]\r\n\r\nHeh Heh no.\r\n\r\nThat expression you gave is always $\\infty$. (look at the expansion for the coefficient of x).\r\n\r\nThe correct generating function would be\r\n$(1+x+x^{2}+\\cdots)(1+x^{2}+x^{4}+\\cdots)(1+x^{3}+x^{6}+\\cdots)(\\cdots)$\r\n\r\nwhich is equal to\r\n\r\n$\\prod_{n\\in\\mathbb{N}}\\frac{1}{1-x^{n}}$",
"Solution_18": "[quote=\"randomdragoon\"]I learned in MOP, I think it's in AoPS, and the teacher in my school's honors math class even talked about it! This concept is amazingly useful. For example:\n\nHow many 4-digit numbers have the property that no digit is less than the digit to its left?[/quote]\r\n\r\nis the answer 12 choose 4? (which is 495)",
"Solution_19": "It's also called stars and bars.",
"Solution_20": "[quote=\"machack\"][quote=\"randomdragoon\"]I learned in MOP, I think it's in AoPS, and the teacher in my school's honors math class even talked about it! This concept is amazingly useful. For example:\n\nHow many 4-digit numbers have the property that no digit is less than the digit to its left?[/quote]\n\nis the answer 12 choose 4? (which is 495)[/quote]\r\n12 choose 3 is the answer. Remember that if you have n urns you will only have n-1 divider bars!\r\n\r\n\r\nEDIT: This post is wrong, see below",
"Solution_21": "I'm feeling incredibly stupid after posting about balls-and-urns...\r\nWill someone enlighten me why it's [b]12[/b] choose 3?",
"Solution_22": "I get 12C4 as well...isn't it analogous to the situation of having to give out 4 apples to 9 kids (we label them from 1 to 9) with no restrictions?",
"Solution_23": "In that case, it would be 12C3.\r\nBut why 12? Why 9 kids? I think I'm missing something big here.",
"Solution_24": "Perhaps what you are missing is that the first digit can't be zero. Since no digit is less than the one on its left, this means that no digit can be a zero. :roll:",
"Solution_25": "i really think its 12 c 4 because it isn't exactly the concept of ball and urn (the numbers don't have to add up to a ceertain sum. and the \"dividing\" concept don't work in this example.\r\nAnyone agree with me?",
"Solution_26": "[quote=\"randomdragoon\"]\n\nFor example, oo|o||ooo means put two balls in the first urn, one ball in the second urn, zero balls in the third urn, and 3 balls into the fourth urn.\n\nEach permutation of oooooo||| represents a different way to put the balls into the urns, and each way the balls can be put into urns can be represented as a permutation of oooooo|||. However, it is easy to count the number of permutations of oooooo|||, which is $\\binom{9}{3}$.\n[/quote]\r\n\r\ni thought that was the method behind hockey-stick identity? :huh:",
"Solution_27": "Never mind, got it. Yep, it's $\\dbinom{12}{3}$.\r\nA good night's sleep can work wonders.",
"Solution_28": "I don't see the conection between the 4-digit number problem and the balls-in-urns formula",
"Solution_29": "Sorry it is $\\binom{12}{4}$. I needed a night's sleep as well. :oops: \r\n\r\n\r\nExplanation: There aren't 4 urns, there are 9 urns, and 4 balls."
}
{
"Tag": [
"calculus",
"integration",
"algebra",
"function",
"domain",
"superior algebra",
"superior algebra unsolved"
],
"Problem": "If $R$ is an integral domain which has only finitely many ideals, then show that $R$ is a field.\r\n\r\n Darij",
"Solution_1": "Solution without thinking: finitely many ideals => artinian => zero-dimensional, and zero-dimensional + domain => field. owk",
"Solution_2": "Is it the Krull dimension you are talking about? In fact, I am not that far in Atiyah-Macdonald...\r\n\r\n[hide=\"Solution without looking up\"][i]Solution.[/i] Let $ a$ be a nonzero element of $ R$. Denote the (finitely many) ideals of $ R$ (including both $ R$ itself and $ 0$) by $ I_{1}$, $ I_{2}$, ..., $ I_{n}$ such that $ I_{1} \\equal{} R$ and such that $ I_{i}\\neq I_{j}$ for any two integers $ i$ and $ j$ from the set $ \\left\\{1,2,...,n\\right\\}$ satisfying $ i\\neq j$. Then, $ aI_{1}$, $ aI_{2}$, ..., $ aI_{n}$ are also ideals of $ R$.\n\nAssume that $ aI_{i} \\equal{} aI_{j}$ holds for two integers $ i$ and $ j$ from the set $ \\left\\{1,2,...,n\\right\\}$. Then, for every $ s_{i}\\in I_{i}$, there exists some $ s_{j}\\in I_{j}$ satisfying $ as_{i} \\equal{} as_{j}$ (because $ as_{i}\\in aI_{i} \\equal{} aI_{j}$); this becomes $ a\\left(s_{i} \\minus{} s_{j}\\right) \\equal{} 0$, what yields $ s_{i} \\minus{} s_{j} \\equal{} 0$ because $ a$ is nonzero (and $ R$ is an integral domain). Hence, $ s_{i} \\equal{} s_{j}$, so that $ s_{i}\\in I_{j}$. Since this holds for every $ s_{i}\\in I_{i}$, we thus get $ I_{i}\\subseteq I_{j}$. Similarly, $ I_{j}\\subseteq I_{i}$. Thus, $ I_{i} \\equal{} I_{j}$, what yields $ i \\equal{} j$ (since we else would have $ I_{i}\\neq I_{j}$).\n\nSo we see that if $ i$ and $ j$ are two integers from the set $ \\left\\{1,2,...,n\\right\\}$, then $ aI_{i} \\equal{} aI_{j}$ holds only if $ i \\equal{} j$. On the other hand, $ aI_{1}$, $ aI_{2}$, ..., $ aI_{n}$ are ideals of $ R$ and thus elements of the set $ \\left\\{I_{1},I_{2},...,I_{n}\\right\\}$. Thus, $ aI_{1}$, $ aI_{2}$, ..., $ aI_{n}$ are $ n$ pairwise distinct elements of the set $ \\left\\{I_{1},I_{2},...,I_{n}\\right\\}$. This yields that the set $ \\left\\{aI_{1},aI_{2},...,aI_{n}\\right\\}$ is a permutation of the set $ \\left\\{I_{1},I_{2},...,I_{n}\\right\\}$. Thus, in particular, there exists some $ k\\in\\left\\{1,2,...,n\\right\\}$ such that $ I_{1} \\equal{} aI_{k}$. Since $ 1\\in R \\equal{} I_{1}$, this yields $ 1\\in aI_{k}$, so that there exists some $ b\\in I_{k}$ satisfying $ 1 \\equal{} ab$. Hence, $ a$ has a multiplicative inverse in $ R$. Since $ a$ can be any arbitrary nonzero element of $ R$, we have thus shown that every nonzero element of $ R$ has a multiplicative inverse, and thus $ R$ is a field, qed.[/hide]\r\n\r\n darij",
"Solution_3": "just consider $(x) \\supseteq (x^{2}) \\supseteq ...$, and you're done. this also shows domain + artinian = field.",
"Solution_4": "[quote=\"darij grinberg\"]Is it the Krull dimension you are talking about? In fact, I am not that far in Atiyah Macdonald...\n[/quote]\r\n\r\nYes, but zero-dimensional just means that all prime ideals are maximal. owk",
"Solution_5": "[quote=\"darij grinberg\"]If $ R$ is an integral domain which has only finitely many ideals, then show that $ R$ is a field. [/quote]\r\n\r\nThe nonzero principal ideals form a group, being a finite cancellative monoid."
}
{
"Tag": [
"number theory"
],
"Problem": "Can anyone suggest me some good books for preparing for the RMO . i am currently in 12th and i need some advice on this",
"Solution_1": "The book Excursion in Mathematics cover all the topics that could ever come in RMO.For the depth on those topics you can try books on a particular topic like \r\nBurton's Number Theory,Balakrishnan's Combinatorics and Sharygin's Geometry(Though out of print).If you are looking for a problem book then arthur engel is a unique collection of problems.But most problems are very difficult.There are other olympiad problem books like titu's Mathematical Olympiad Challenges but their coverage is incomplete.",
"Solution_2": "Is Challenge and Thrill of Pre-College Mathematics by V. Krishnamurthy, C. R. Pranesachar, K. N. Ranganathan and B. J. Venkatachala(New Age International Publishers) good?Well;I find the questions pretty tough!",
"Solution_3": "i had to bear much difficulties before ensuring myself a copy of'challenge & thrills.......'\r\nthe book is a nice one............",
"Solution_4": "I want it to! Asking a friend in Delhi to mail it :P",
"Solution_5": "u may also contact the publication office...",
"Solution_6": "u may also contact the publication office...",
"Solution_7": "My parents went to Delhi for 2 days, so they got it for me :)",
"Solution_8": "I bought my copy at the Kolkata Book fair",
"Solution_9": "I miss Kol :(",
"Solution_10": "Are u from kol? :o",
"Solution_11": "lol\r\nu guys shud be in pune , people complete CAT in their 8th :P",
"Solution_12": "I am happier in kgp, :) \r\nover there,I would have suffered from a worse inferiority complex! :(",
"Solution_13": "[quote=\"Scary math\"]Are u from kol? :o[/quote]\r\nLived there from 4-10 years of age."
}
{
"Tag": [
"abstract algebra",
"calculus",
"integration",
"group theory",
"Ring Theory",
"superior algebra",
"superior algebra unsolved"
],
"Problem": "Assume R is commutative. Prove that the set of prime ideals in R has a minimal element with respect to inclusion. (by Zorn's lemma)",
"Solution_1": "the cut of a chain in the set of prime ideals is a obviously a lower bound for it, so zorn's lemma (applied to the \"dual\" inclusion relation) yields the result.",
"Solution_2": "\"the cut of a chain in the set of prime ideals is a obviously a lower bound\"\r\n\r\n...can you explain a little bit more? I have no idea on how to construct a chain in it.....",
"Solution_3": "zorn's lemma advises you to take an arbitrary chain (totally ordered subset) - it must not be constructed - and find a lower bound ..",
"Solution_4": "related to this question..\r\nme and friend were wondering: is the proposition still true if you ask for [i]non-zero[/i] minimal prime ideals?\r\nof course, forget the case of $R$ field, or maybe (just in order to let the statement have a sense) consider also the whole field as a prime ideal.\r\nor maybe reformulate the statement for ideals which are non-zero primes or the whole field :P\r\n\r\nthe question reduces to this: does it exist a (non-noetherian, integral) ring with an infinite descending chain of non-zero prime ideals with trivial intersection?\r\nactually, i have some problem in finding a ring with an infinite descending chain of prime ideals :blush: :maybe: ...",
"Solution_5": "Look what happened in [url=http://www.artofproblemsolving.com/Forum/viewtopic.php?t=125503]this[/url] thread: the algebraic question was transferred, via valuation rings, to a question on totally ordered abelian groups, and then the problem consisted simply of finding a totally ordered abelian group with certain properties. We can do the same thing here.\r\n\r\nLet $\\nu: K^{*}\\to\\Gamma$ be, just as in the other thread, a valuation of the field $K$ onto the totally ordered abelian group $\\Gamma$, and let $A$ be the valuation ring corresponding to $\\nu$, i.e. $A=\\{a\\in K\\ |\\ \\nu(a)\\ge 0\\}$. A subgroup $\\Delta\\le\\Gamma$ is called [i]isolated[/i] if whenever $0\\le\\beta\\le\\alpha$ and $\\alpha\\in\\Delta$, we also have $\\beta\\in\\Delta$. If $\\frak p$ is a prime ideal of $A$, then it can be shown that the set $\\nu(A\\setminus\\frak p)$ is the set of non-negative elements of an isolated subgroup of $\\Gamma$, and that this gives a bijection between prime ideals and isolated subgroups of $\\Gamma$. Now, the question of finding infinite descending chains of prime ideals in $A$ becomes, through this correspondence, equivalent to finding infinite ascending chains of isolated subgroups of $\\Gamma$.\r\n\r\nWe can choose $\\Gamma$ to be any totally ordered abelian group we want. Take it to be the set of infinite sequences $(\\alpha_{n})_{n\\in\\mathbb Z}$ (infinite in both directions) of integers such that $\\alpha_{n}$ is zero for small enough $n$, with componentwise addition. We order this set lexicographically, i.e. $(\\alpha_{n})_{n}<(\\beta_{n})_{n}$ if $\\alpha_{n}<\\beta_{n}$ for the smallest $n$ where the sequences differ. Now define $\\Delta_{n},\\ n\\le 0$ to be the set of elements in $\\Gamma$ whose components with index smaller than $n$ is zero. It is easy to check that $\\Delta_{n}$ for an infinite ascending chain of isolated subgroups of $\\Gamma$, so we\u2019re done. Moreover, this example can be modified to give descending chains of non-zero prime ideals of any cardinality we want."
}
{
"Tag": [
"MATHCOUNTS"
],
"Problem": "MATHCOUNTS 2004-2005 Warm-Up 13\r\n\r\nA two-pan balance scale comes with a collection of weights. Each weight weighs a whole number of grams. Weights can be put in either or both pans during a weighing. To ensure any whole number of grams up to 100 grams can be measured, what is the minimum number of weights needed in the collection?\r\n\r\n[hide=\"Answer\"]Here is how I did and wondering if there is a better way:\n\nWe start with 1 and 3 to cover number 1-4.\nTo get 5, we need 4+5=9, so 1, 3 and 9 can cover number 5-13.\nTo get 14, we need 13+14=27, so 1, 3, 9 and 27 can cover number 14-40.\nTo get 41, we need 40+41=81, so 1, 3, 9, 27 and 81 can cover number 41-121.\n\nSo, the answer is [b]5[/b].[/hide]",
"Solution_1": "If you think of the famous base 2 numbers magic trick, you'll realize that you just need the powers of two, including 1. The answer is the number of powers of 2 less than 100, which are 1,2,4,8,16,32, and 64. That gives you the answer of 7. Also you could do log(2)64, which also equals 7.",
"Solution_2": "[quote]\nIf you think of the famous base 2 numbers magic trick, you'll realize that you just need the powers of two, including 1.[/quote]\r\n\r\nBut that doesn't give you the [b]minimum[/b] number of weights, because you are allowed to put weights on both sides of the scale.\r\n\r\nYakko"
}
{
"Tag": [
"logarithms",
"function",
"Euler",
"complex analysis"
],
"Problem": "[b]\nDetermine is this Conv. Or Div. and Is there any effect if $ z\\in \\mathbb {C}$ OR $ z\\in \\mathbb{R}$.\n\n$ \\prod\\limits_{n = 1}^\\infty {\\left( {1 - \\frac {z}{n}} \\right)} e^{\\frac {z}{n}}$\n\n\nI need details enough to understand why?\n\nMy thinking , it's not Con. , since no any factor of it's equal to 0 yes Or not ?\n\n[/b]",
"Solution_1": "If one of the factors is $ 0$, it certainly converges to $ 0$. Otherwise, take logarithms and reduce the problem to that about convergence of a series (you need to know that $ \\log(1 \\minus{} w) \\equal{} \\minus{} w \\plus{} O(|w|^2)$ for small $ w$).\r\n\r\nBy the way, [b]entire posts in bold are slightly irritating[/b], so, please, use normal fonts switching to [b]bold[/b], [i]italic[/i], [color=red]color[/color], etc. only when you need to emphasize something :).\r\n\r\nAlso, please spell the words in the topic title according to English standards!",
"Solution_2": "I'm sorry :blush: about Bold and my spelling . I'll use Standard English In titles and bold in something need to emphasize about it.\r\n\r\nThank u about Your Polite way. :) \r\n\r\nAnyway,\r\n\r\nCould you do more steps about my question please ? I think I still have warping about it.\r\n\r\n\r\nThank you.",
"Solution_3": "Let's see. There are two possibilities for an infinite product $ \\prod_n a_n$ to converge.\r\n1) One of the factors is $ 0$. Then the entire product obviously converges to $ 0$.\r\n2) All $ a_n\\ne 0$, and $ \\sum_n \\log a_n$ converges. Then the limit of the partial products is just the value of the exponential function at the limit of the partial sums of the logarithms.\r\n\r\nSo, assume that none of the factors in your product is $ 0$. When $ n$ is much larger than $ |z|$, you can write \r\n\\[ \\log(1\\minus{}\\tfrac zn)\\plus{}\\frac zn\\equal{}\\minus{}\\frac zn\\plus{}O(n^{\\minus{}2})\\plus{}\\frac zn\\equal{}O(n^{\\minus{}2})\\]\r\nso the series of logarithms is dominated by the positive series $ \\sum_N \\frac 1{n^2}$, which converges. I hope that the rest is clear :)",
"Solution_4": "That's clear dude.\r\n\r\nI'm So thankful.\r\n\r\n :D :wink:",
"Solution_5": "[quote=\"fedja\"]\n1) One of the factors is $ 0$. Then the entire product obviously converges to $ 0$.\n[/quote]\r\nI'd just like to point out that typically the product is not considered to converge just because one of the factors is 0. This is because if you allow this for convergence, then you cannot say anything about the terms. As an example, you'd often like to know if the product converges to a continuous or differentiable functions, and in such discussion it becomes apparent that when factors are 0 you need to throw out the zero factors and look at the rest of the factors.\r\n\r\n(fedja knows this, others might not :wink: )",
"Solution_6": "I'm going to go ahead and move this topic into the complex analysis forum. The standards for convergence in Kalle's post are those typically elaborated in a complex analysis course. And by those standards, this product converges as nicely as we could possibly ask for: uniformly on each compact subset of $ \\mathbb{C}.$ Hence, the function defined by this product is an entire function, and it has a simple zero at each positive integer.\r\n\r\nNow let me add a question: what does it converge to? Can we recognize and identify this entire function? Can we at least find some useful algebraic properties of this entire function?",
"Solution_7": "Well, applying the logarithm to the function and doing some basic manipulation, we can get th expression:\r\n\r\n$ \\prod^\\infty_{n\\equal{}1}\\left(1\\minus{}\\frac{z}{n}\\right)e^{\\frac{z}{n}}\\equal{}\\frac{e^{z\\, \\gamma}}{\\Gamma(1\\minus{}z)}$\r\n\r\nwhere $ \\gamma$ is the Euler-Mascheroni constant."
}
{
"Tag": [
"calculus",
"calculus computations"
],
"Problem": "Write an equation for the tangent line at (c,f(c)).\r\n\r\nf(x) =sqrt(x); c=4",
"Solution_1": "Since I can flip open my calculus textbook and find many homework exercises (in a fairly early chapter) that look exactly like this, I'll have to ask: what have you done so far?",
"Solution_2": "y - x/4 - 1 = 0",
"Solution_3": "Yes, that's a correct answer."
}
{
"Tag": [
"trigonometry",
"inequalities",
"geometry proposed",
"geometry"
],
"Problem": "Prove that\r\n\r\n$ a^4\\plus{}b^4\\plus{}c^4\\geq16s(s\\minus{}a)(s\\minus{}b)(s\\minus{}c)(\\cot A\\cot B \\plus{} \\cot B \\cot C \\plus{} \\cot C \\cot A)$",
"Solution_1": "Same form like previous problems\r\nUse $ 4R\\equal{}\\frac{abc}{S}, 2R\\equal{}\\frac{a}{\\sin A}, \\cos A \\equal{}\\frac{b^{2}\\plus{}c^{2}\\minus{}a^{2}}{2bc}$\r\nThen it goes to obvious.",
"Solution_2": "[quote=\"Heebeen, Yang\"]Same form like previous problems\nUse $ 4R \\equal{} \\frac {abc}{S}, 2R \\equal{} \\frac {a}{\\sin A}, \\cos A \\equal{} \\frac {b^{2} \\plus{} c^{2} \\minus{} a^{2}}{2bc}$\nThen it goes to obvious.[/quote]\r\n\r\n\r\nDear [color=red][b]Yang[/b][/color] can you describe better your solution? Thanks",
"Solution_3": "If we use them,\r\n$ \\cot A\\equal{}\\frac{2R(b^{2}\\plus{}c^{2}\\minus{}a^{2})}{2abc}$and $ \\cot B, \\cot C$ defined similar.\r\nThen, above inequality is equivalent to \r\n$ a^{4}\\plus{}b^{4}\\plus{}c^{4} \\geq \\sum_{cyc}(b^{2}\\plus{}c^{2}\\minus{}a^{2})(c^{2}\\plus{}a^{2}\\minus{}b^{2})$\r\n<=>\r\n$ a^{4}\\plus{}b^{4}\\plus{}c^{4} \\geq a^{2}b^{2}\\plus{}b^{2}c^{2}\\plus{}c^{2}a^{2}$",
"Solution_4": "$ 16S^2\\equal{}16s(s \\minus{} a)(s \\minus{} b)(s \\minus{} c) \\equal{} \\sum (a^2\\plus{}b^2\\minus{}c^2)(a^2\\plus{}c^2\\minus{}b^2)\\equal{} 2\\sum b^2c^2 \\minus{} \\sum a^4$ \r\n\r\nand $ \\sum\\cot B\\cot C \\equal{} 1$ . [b]Simply ![/b] The proposed inequality becomes $ \\sum a^4\\ge \\sum b^2c^2$ what is evidently.",
"Solution_5": "Well.... That's right wow.\r\nI've forgottend that property.\r\nThank you for your advice ^^",
"Solution_6": "[quote=\"Virgil Nicula\"]$ 16S^2 \\equal{} 16s(s \\minus{} a)(s \\minus{} b)(s \\minus{} c) \\equal{} \\sum (a^2 \\plus{} b^2 \\minus{} c^2)(a^2 \\plus{} c^2 \\minus{} b^2) \\equal{} 2\\sum b^2c^2 \\minus{} \\sum a^4$ \n\nand $ \\sum\\cot B\\cot C \\equal{} 1$ . [b]Simply ![/b] The proposed inequality becomes $ \\sum a^4\\ge \\sum b^2c^2$ what is evidently.[/quote]\r\n\r\nVery Nice my Dear Friend.. Thanks :)",
"Solution_7": "[quote=\"Heebeen, Yang\"]If we use them,\n$ \\cot A \\equal{} \\frac {2R(b^{2} \\plus{} c^{2} \\minus{} a^{2})}{2abc}$and $ \\cot B, \\cot C$ defined similar.\nThen, above inequality is equivalent to \n$ a^{4} \\plus{} b^{4} \\plus{} c^{4} \\geq \\sum_{cyc}(b^{2} \\plus{} c^{2} \\minus{} a^{2})(c^{2} \\plus{} a^{2} \\minus{} b^{2})$\n<=>\n$ a^{4} \\plus{} b^{4} \\plus{} c^{4} \\geq a^{2}b^{2} \\plus{} b^{2}c^{2} \\plus{} c^{2}a^{2}$[/quote]\r\n\r\nThanks Dear [color=red][b]Yang[/b][/color]...."
}
{
"Tag": [
"quadratics",
"LaTeX",
"trigonometry",
"limit",
"algebra unsolved",
"algebra"
],
"Problem": "Solve:\r\n$ e^{x\\minus{}y}\\equal{}\\frac{sinx}{siny}$\r\n$ 10\\sqrt{x^6\\plus{}1}\\equal{}3(y^4\\plus{}2)$\r\n$ \\pi